Sunteți pe pagina 1din 448

fiziks

Institute for NET/JRF, GATE, IIT‐JAM, JEST, TIFR and GRE in PHYSICAL SCIENCES 
 

fiziks

Forum for CSIR-UGC JRF/NET, GATE, IIT-JAM/IISc,


M.Sc. Entrance, JEST, TIFR and GRE in PHYSICS

Previous Year’s Papers & Solutions

(CSIR-JRF/NET/GATE)

                                                                                
Head office  Branch office 
 
fiziks, H.No. 40 D, G.F, Jia Sarai,  Anand Institute of Mathematics, 
 
Near IIT, Hauz Khas, New Delhi‐16  28‐B/6, Jia Sarai, Near IIT 
 
Phone: 011‐26865455/+91‐9871145498 Hauz Khas, New Delhi‐16 
                                                   
                                             Website: www.physicsbyfiziks.com                                                                                          
                                                            Email: fiziks.physics@gmail.com                                                                      
fiziks
Institute for NET/JRF, GATE, IIT‐JAM, JEST, TIFR and GRE in PHYSICAL SCIENCES 
 
MATEMATICAL PHYSICS SOLUTIONS
NET/JRF (JUNE-2011)
Q1. The value of the integral ∫ dz z 2 e z , where C is an open contour in the complex z -plane as
C
lm z
shown in the figure below, is:
5 5
(a) +e (b) e − (0,1)
e e
5 5 C
(c) −e (d) − −e
e e
Re z
Ans: (c) − (1,0) (1,0)
Solution: If we complete the contour, then by Cauchy integral theorem

∫ dzz e + ∫ dzz e = 0 ⇒ ∫ dzz e = − ∫ dzz e = − [z e − 2 ze + 2e ] −1 =


1 1
1 5
2 z 2 z 2 z 2 z 2 z z z
−e
−1 C C −1
e

Q2. Which of the following matrices is an element of the group SU (2 ) ?

⎛1+ i −1 ⎞
⎜ ⎟
⎛ 1 1⎞
(a) ⎜⎜ ⎟⎟ (b) ⎜ 3 3⎟
⎝ 0 1⎠ ⎜ 1 1− i ⎟
⎜ ⎟
⎝ 3 3⎠

⎛ 1 3⎞
⎜ ⎟
⎛2 + i i ⎞
(c) ⎜⎜ ⎟⎟ (d) ⎜ 2 2 ⎟
⎝ 3 1+ i⎠ ⎜ 3 1 ⎟
⎜ ⎟
⎝ 2 2 ⎠
Ans: (b)
⎧⎛ α −β⎞ ⎫
Solution: SU (2 ) is a group defined as following: SU (2 ) = ⎨⎜⎜ ⎟⎟ : α , β ∈ C ; α + β = 1⎬
2 2

⎩⎝ β α ⎠ ⎭
1+ i 1− i
clearly (b) hold the property of SU (2 ) . α =
1 1
,β = and α = ,β = .
3 3 3 3
Note: SU (2 ) has wide applications in electroweak interaction covered in standard model
of particle physics.

                                                                                
Head office  Branch office 
 
fiziks, H.No. 40 D, G.F, Jia Sarai,  Anand Institute of Mathematics, 
 
Near IIT, Hauz Khas, New Delhi‐16  28‐B/6, Jia Sarai, Near IIT 
 
Phone: 011‐26865455/+91‐9871145498 Hauz Khas, New Delhi‐16 
                                                   
                                             Website: www.physicsbyfiziks.com                                                                                          
                                                            Email: fiziks.physics@gmail.com                                                                   1 
fiziks
Institute for NET/JRF, GATE, IIT‐JAM, JEST, TIFR and GRE in PHYSICAL SCIENCES 
 
Q3. Let a and b be two distinct three dimensional vectors. Then the component of b that is
perpendicular to a is given by

(a)
a× b×a( ) (b)
(
b × a×b ) (c)
(a ⋅ b )b (d)
(b ⋅ a )a
a2 b2 b 2
a2
Ans: (a)
Solution: a × b = ab sin θ nˆ where n̂ is perpendicular to plane containing

a and b and pointing upwards. b b sin θ kˆ


θ
( )
a × a × b = ab sin θ (a × nˆ ) = − a 2 b sin θ kˆ

b sin θ kˆ =
− a × a ×b
⇒ b
(sin θ
)
ˆ = a× b×a .
k
( ) a

a2 a2
Q4. Let p n ( x ) (where n = 0,1, 2, ...... ) be a polynomial of degree n with real coefficients,
4
defined in the interval 2 ≤ n ≤ 4 . If ∫ pn ( x ) pm ( x )dx = δ nm , then
2

(a) p 0 (x ) = and p1 ( x ) = (− 3 − x ) (b) p0 ( x ) = and p1 ( x ) = 3 (3 + x )


1 3 1
2 2 2

(c) p0 ( x ) = and p1 ( x ) = (3 − x ) (d) p 0 ( x ) = and p1 ( x ) = (3 − x )


1 3 1 3
2 2 2 2
Ans: (d)
Solution: For n not equal to m kroneker delta become zero. One positive and one negative term

can make integral zero. So answer may be (c) or (d). Now take n = m = 0 so p0 ( x ) =
1
2
and then integrate. (d) is correct option because it satisfies the equation Check by
integration and by orthogonal property of Legendre polynomial also.

                                                                                
Head office  Branch office 
 
fiziks, H.No. 40 D, G.F, Jia Sarai,  Anand Institute of Mathematics, 
 
Near IIT, Hauz Khas, New Delhi‐16  28‐B/6, Jia Sarai, Near IIT 
 
Phone: 011‐26865455/+91‐9871145498 Hauz Khas, New Delhi‐16 
                                                   
                                             Website: www.physicsbyfiziks.com                                                                                          
                                                            Email: fiziks.physics@gmail.com                                                                   2 
fiziks
Institute for NET/JRF, GATE, IIT‐JAM, JEST, TIFR and GRE in PHYSICAL SCIENCES 
 
Q5. Which of the following is an analytic function of the complex variable z = x + iy in the

domain z < 2 ?

(a) (3 + x − iy ) (b) (1 + x + iy ) (7 − x − iy )
7 4 3

(c) (1 − x − iy ) ( 7 − x + iy ) (d) (x + iy − 1)
4 3 1/ 2

Ans: (b)
Solution: Put z = x + iy . If z = x − iy appears in any of the expressions then that expression is
1
non-analytic. For option (d) we have a branch point singularity as the power is which
2
is fractional. Hence only option (b) is analytic.
⎛ 1 1 1⎞
Q6. Consider the matrix M = ⎜1 1 1⎟
⎜ 1 1 1⎟
⎝ ⎠
A. The eigenvalues of M are
(a) 0, 1, 2 (b) 0, 0, 3 (c) 1, 1, 1 (d) – 1, 1, 3
Ans: (b)
⎡1 − λ 1 1 ⎤

Solution: For eigen values ⎢ 1 1− λ 1 ⎥⎥ = 0
⎢⎣ 1 1 1 − λ ⎥⎦

(1 − λ )((1 − λ )2 − 1) − (1 − λ − 1) + 1(1 − (1 − λ )) = 0
(1 − λ )(1 + λ2 − 2λ − 1) + λ + λ = 0 ⇒ λ2 − 2λ − λ3 + 2λ2 + 2λ = 0
λ3 − 3λ2 = 0 ⇒ λ2 (λ − 3) = 0 ⇒ λ = 0, 0, 3
For any n × n matrix having all elements unity eigenvalues are 0, 0, 0,..., n .
B. The exponential of M simplifies to (I is the 3 × 3 identity matrix)
⎛ e3 − 1 ⎞ M2
(a) e M = I + ⎜⎜ ⎟⎟ M (b) e M = I + M +
⎝ 3 ⎠ 2!

(c) e M = I + 33 M (d) e M = (e − 1)M


Ans: (a)

                                                                                
Head office  Branch office 
 
fiziks, H.No. 40 D, G.F, Jia Sarai,  Anand Institute of Mathematics, 
 
Near IIT, Hauz Khas, New Delhi‐16  28‐B/6, Jia Sarai, Near IIT 
 
Phone: 011‐26865455/+91‐9871145498 Hauz Khas, New Delhi‐16 
                                                   
                                             Website: www.physicsbyfiziks.com                                                                                          
                                                            Email: fiziks.physics@gmail.com                                                                   3 
fiziks
Institute for NET/JRF, GATE, IIT‐JAM, JEST, TIFR and GRE in PHYSICAL SCIENCES 
 
M
Solution: For e let us try to diagonalize matrix M using similarity transformation.

⎡− 2 1 1 ⎤ ⎡ x1 ⎤ ⎡0⎤
For λ = 3 , ⎢ 1 − 2 1 ⎥⎥ ⎢⎢ x 2 ⎥⎥ = ⎢⎢0⎥⎥

⎢⎣ 1 1 − 2⎥⎦ ⎢⎣ x3 ⎥⎦ ⎢⎣0⎥⎦

⇒ − 2 x1 + x2 + x 3 = 0 , x1 − 2 x 2 + x3 = 0 , x1 + x 2 − 2 x3 = 0

⇒ −3x 2 + 3x3 = 0 or x2 = x3 ⇒ x1 = x 2 = x3 = k .

⎡1⎤
Eigen vector is 1 3 ⎢1⎥ where k = 1 .
⎣⎢1⎦⎥
For λ = 0 ,
⎡1 1 1⎤ ⎡ x1 ⎤ ⎡0⎤
⎢1 1 1⎥ ⎢ x ⎥ ⎢0⎥ ⇒ x + x + x = 0
⎢ ⎥⎢ 2 ⎥⎢ ⎥ 1 2 3

⎢⎣1 1 1⎥⎦ ⎢⎣ x3 ⎥⎦ ⎢⎣0⎥⎦

⎡ k1 ⎤ ⎡1⎤
Let x1 = k1 , x 2 = k 2 and x3 = k1 + k 2 . Eigen vector is ⎢ k 2 ⎥ = 1 / 2 ⎢⎢− 1⎥⎥
⎢ ⎥ where
⎢⎣(k 1+ k 2 )⎥⎦ ⎢⎣ 1 ⎥⎦

k1 = k 2 = 1 .

⎡1⎤
Let x1 = k1 , x 2 = k 2 and x3 = −(k1 + k 2 ) . Other Eigen vector 1 / 2 ⎢⎢ 0 ⎥⎥ where
⎢⎣− 1⎥⎦

k1 = 1, k 2 = −1 .

⎡ 0 1 1⎤ ⎡1 − 2 1 ⎤
S = ⎢− 1 0 1⎥ ⇒ S = ⎢⎢2 − 1 − 1⎥⎥ ⇒ D = S −1 MS , M = SDS −1 .
⎢ ⎥ −1

⎣⎢ 1 − 1 1⎥⎦ ⎢⎣1 − 1 1 ⎦⎥

⎡1 0 0 ⎤
eM = Se D S −1 ⇒ e D = ⎢⎢0 1 0 ⎥⎥ ⇒ e M = 1 +
e3 − 1 M ( )
3
⎢⎣0 0 e 3 ⎥⎦

                                                                                
Head office  Branch office 
 
fiziks, H.No. 40 D, G.F, Jia Sarai,  Anand Institute of Mathematics, 
 
Near IIT, Hauz Khas, New Delhi‐16  28‐B/6, Jia Sarai, Near IIT 
 
Phone: 011‐26865455/+91‐9871145498 Hauz Khas, New Delhi‐16 
                                                   
                                             Website: www.physicsbyfiziks.com                                                                                          
                                                            Email: fiziks.physics@gmail.com                                                                   4 
fiziks
Institute for NET/JRF, GATE, IIT‐JAM, JEST, TIFR and GRE in PHYSICAL SCIENCES 
 
NET/JRF (DEC-2011)
Q7. An unbiased dice is thrown three times successively. The probability that the numbers of
dots on the uppermost surface add up to 16 is
1 1 1 1
(a) (b) (c) (d)
16 36 108 216
Ans: (b)
Solution: We can get sum of dice as 16 in total six ways i.e. three ways (6, 5, 5) and three ways
(6, 6, 4).
Total number of ways for 3 dice having six faces = 6 × 6 × 6
6 1
= =
6 × 6 × 6 36

Q8. The generating function F (x, t ) = ∑ Pn ( x )t n for the Legendre polynomials Pn ( x )
n =0

(
is F (x, t ) = 1 − 2 xt + t 2 )−1
2
. The value of P3 (− 1) is

(a) 5 / 2 (b) 3 / 2 (c) + 1 (d) − 1


Ans: (d)

Solution: P3 =
1
2
( ) 1
2
3 1
2
(
5 x 3 − 3x ⇒ P3 (− 1) = 5(− 1) − 3(− 1) = [− 5 + 3] = −1 )
Q9. The equation of the plane that is tangent to the surface xyz = 8 at the point (1, 2, 4 ) is
(a) x + 2 y + 4 z = 12 (b) 4 x + 2 y + z = 12
(c) x + 4 y + 2 = 0 (d) x + y + z = 7
Ans: (b)
Solution: To get a normal at the surface lets take the gradient

∇( xyx ) = yziˆ + zxˆj + kˆxy = 8iˆ + 4 ˆj + 2kˆ

We want a plane perpendicular to this so: r − r0 ⋅ ( ) (8iˆ64+ +4 16ˆj ++2k4ˆ) = 0 .


[(x −1)iˆ + ( y − 2) ˆj + (z − 4)kˆ]⋅ [8iˆ + 4 ˆj + 2kˆ] = 0 ⇒ 4 x + 2 y + z = 12 .
                                                                                
Head office  Branch office 
 
fiziks, H.No. 40 D, G.F, Jia Sarai,  Anand Institute of Mathematics, 
 
Near IIT, Hauz Khas, New Delhi‐16  28‐B/6, Jia Sarai, Near IIT 
 
Phone: 011‐26865455/+91‐9871145498 Hauz Khas, New Delhi‐16 
                                                   
                                             Website: www.physicsbyfiziks.com                                                                                          
                                                            Email: fiziks.physics@gmail.com                                                                   5 
fiziks
Institute for NET/JRF, GATE, IIT‐JAM, JEST, TIFR and GRE in PHYSICAL SCIENCES 
 
Q10. [ ]
A 3 × 3 matrix M has Tr [M ] = 6, Tr M = 26 and Tr M 3 = 90 . Which of the following
2
[ ]
can be a possible set of eigenvalues of M ?
(a) {1,1, 4} (b) {− 1, 0, 7} (c) {− 1, 3, 4} (d) {2, 2, 2}
Ans: (c)
[ ]
Solution: Tr M 2 = (− 1) + (3) + (4) also Tr M 3 = (− 1) + (3) + (4) = 90 .
2 2 2
[ ] 3 3 3

Q11. Let x1 (t ) and x2 (t ) be two linearly independent solutions of the differential equation

d 2x dx (t ) dx (t )
+ 2 + f (t )x = 0 and let w(t ) = x1 (t ) 2 − x2 (t ) 1 . If w(0 ) = 1, then w(1) is
dx
2
dt dt dt dt
given by
(a) 1 (b) e 2 (c) 1 / e (d) 1 / e 2
Ans: (d)
Solution: W (t ) is Wronskian of D.E.

W = e ∫ = e −2t ⇒ W (1) = e −2 since P = 2 .


− Pdt

⎧1 for 2n ≤ x ≤ 2n + 1
Q12. The graph of the function f ( x ) = ⎨
⎩0 for 2n + 1 ≤ x ≤ 2n + 2

where n = (0,1, 2,......) is shown below. Its Laplace transform f (s ) is


~

f (x )
1 + e−s 1 − e−s
(a) (b)
s s 1
1 1 x
s (1 + e − s ) s (1 − e − s )
(c) (d) 0 1 2 3 4 5

Ans: (c)
∞ 1 2 3
Solution: L( f ( x )) = ∫ e − sx
f ( x ) dx = ∫ e − sx
⋅ 1dx + ∫ e − sx
⋅ 0dx + ∫ e − sx ⋅ 1dx + ......
0 0 1 2

1 3
⎡ e − sx ⎤ ⎡ e − sx ⎤
=⎢ ⎥ + 0 + ⎢ ⎥ + ...... =
1 −s
e −1 +
1 −3 s
[
e − e − 2 s + ...... ] [ ]
⎣ − s ⎦0 ⎣ − s ⎦2 −s −s

                                                                                
Head office  Branch office 
 
fiziks, H.No. 40 D, G.F, Jia Sarai,  Anand Institute of Mathematics, 
 
Near IIT, Hauz Khas, New Delhi‐16  28‐B/6, Jia Sarai, Near IIT 
 
Phone: 011‐26865455/+91‐9871145498 Hauz Khas, New Delhi‐16 
                                                   
                                             Website: www.physicsbyfiziks.com                                                                                          
                                                            Email: fiziks.physics@gmail.com                                                                   6 
fiziks
Institute for NET/JRF, GATE, IIT‐JAM, JEST, TIFR and GRE in PHYSICAL SCIENCES 
 
=
1
−s
[ 1
] [
− 1 + e − s − e − 2 s + e −3 s + ........ = 1 − e − s + e − 2 s − e −3s + ....
s
]
a 1⎡ 1 ⎤
Since S ∞ = where r = −e − s and a = 1 ⇒ S ∞ = ⎢
s ⎣ (1 + e − s )⎥⎦
.
1− r

Q13. The first few terms in the Taylor series expansion of the function f ( x ) = sin x around
π
x= are:
4

1 ⎡ ⎛ π ⎞ 1⎛ π ⎞ 1⎛ π ⎞ ⎤
2 3

(a) ⎢1 + ⎜ x − ⎟ + ⎜ x − ⎟ + ⎜ x − ⎟ .....⎥
2 ⎢⎣ ⎝ 4 ⎠ 2! ⎝ 4 ⎠ 3! ⎝ 4⎠ ⎥⎦

1 ⎡ ⎛ π⎞ 1⎛ π⎞ 1⎛ π⎞ ⎤
2 3

(b) ⎢1 + ⎜ x − ⎟ − ⎜ x − ⎟ − ⎜ x − ⎟ .....⎥
2 ⎢⎣ ⎝ 4 ⎠ 2! ⎝ 4 ⎠ 3! ⎝ 4⎠ ⎥⎦

⎡⎛ π ⎞ 1⎛ π ⎞ ⎤
3

(c) ⎢⎜ x − ⎟ − ⎜ x − ⎟ .....⎥
⎢⎣⎝ 4 ⎠ 3! ⎝ 4⎠ ⎥⎦

1 ⎡ x 2 x3 ⎤
(d) ⎢1 − x + − .....⎥
2 ⎣ 2! 3! ⎦

Ans: (c)
Solution: f ( x ) = sin x

⎛π ⎞ 1
f ⎜ ⎟=
⎝4⎠ 2

⎛π ⎞ π 1
f ′ ⎜ ⎟ = cos =
⎝4⎠ 4 2

⎛π ⎞ π 1
f ′′ ⎜ ⎟ = − sin = −
⎝4⎠ 4 2
So Taylor’s series is given by

1 ⎡ ⎛ π⎞ 1⎛ π⎞ 1⎛
2
π⎞
3

⎢1 + ⎜ x − ⎟ − ⎜ x − ⎟ − ⎜ x − ⎟ .....⎥
2 ⎢⎣ ⎝ 4 ⎠ 2! ⎝ 4 ⎠ 3! ⎝ 4⎠ ⎥⎦

                                                                                
Head office  Branch office 
 
fiziks, H.No. 40 D, G.F, Jia Sarai,  Anand Institute of Mathematics, 
 
Near IIT, Hauz Khas, New Delhi‐16  28‐B/6, Jia Sarai, Near IIT 
 
Phone: 011‐26865455/+91‐9871145498 Hauz Khas, New Delhi‐16 
                                                   
                                             Website: www.physicsbyfiziks.com                                                                                          
                                                            Email: fiziks.physics@gmail.com                                                                   7 
fiziks
Institute for NET/JRF, GATE, IIT‐JAM, JEST, TIFR and GRE in PHYSICAL SCIENCES 
 
NET/JRF (JUNE-2012)

Q14. A vector perpendicular to any vector that lies on the plane defined by x + y + z = 5 , is

(a) iˆ + ˆj (b) ˆj + kˆ (c) iˆ + ˆj + kˆ (d) 2iˆ + 3 ˆj + 5kˆ


Ans: (c)
⎛ ∂ ∂ ∂⎞
Solution: Let φ = x + y + z − 5 ⇒ ∇φ = ⎜⎜ iˆ + ˆj + kˆ ⎟⎟( x + y + z − 5) = iˆ + ˆj + kˆ .
⎝ ∂x ∂y ∂z ⎠

⎛1 2 3 ⎞
⎜ ⎟
Q15. The eigen values of the matrix A = ⎜ 2 4 6 ⎟ are
⎜3 6 9 ⎟
⎝ ⎠
(a) (1, 4, 9 ) (b) (0, 7, 7 ) (c) (0,1,13) (d) (0, 0,14 )
Ans: (d)
⎡1 − λ 2 3 ⎤
Solution: For eigenvalues A − λI = 0 ⇒ ⎢⎢ 2 4−λ 6 ⎥⎥ = 0
⎢⎣ 3 6 9 − λ ⎥⎦

(1 − λ )[(4 − λ )(9 − λ ) − 36] − 2[2(9 − λ ) − 18] + 3[12 − 3(4 − λ )] = 0


(1 − λ )(4 − λ )(9 − λ ) − 36(1 − λ ) − 4(9 − λ ) + 36 + 9λ = 0
λ3 − 14λ2 = 0 ⇒ λ2 (λ − 14) = 0 ⇒ λ = 0, 0, 14 .
1
Q16. The first few terms in the Laurent series for in the region 1 ≤ z ≤ 2 and
(z − 1)(z − 2)
around z = 1 is

[1 + z + z 2 + ....]⎡⎢1 + z + z + z + ....⎤⎥
2 3
+ z − (1 − z ) + (1 − z ) + ....
1 1 2 3
(a) (b)
2 ⎣ 2 4 8 ⎦ 1− z

⎡ 1 1 ⎤⎡ 2 4 ⎤
(d) 2( z − 1) + 5( z − 1) + 7(z − 1) + ....
1
⎢⎣1 + z + z 2 + ....⎥⎦ ⎢⎣1 + z + z 2 + ....⎥⎦
2 3
(c)
z2
Ans:

                                                                                
Head office  Branch office 
 
fiziks, H.No. 40 D, G.F, Jia Sarai,  Anand Institute of Mathematics, 
 
Near IIT, Hauz Khas, New Delhi‐16  28‐B/6, Jia Sarai, Near IIT 
 
Phone: 011‐26865455/+91‐9871145498 Hauz Khas, New Delhi‐16 
                                                   
                                             Website: www.physicsbyfiziks.com                                                                                          
                                                            Email: fiziks.physics@gmail.com                                                                   8 
fiziks
Institute for NET/JRF, GATE, IIT‐JAM, JEST, TIFR and GRE in PHYSICAL SCIENCES 
 
− (1 + (1 − z ))
1 1 1 1 1 1 −1
Solution: = − = + =
(z − 1)(z − 2) z − 2 z − 1 1 − z (z − 1) − 1 1 − z
=
1 ⎡
− ⎢1 − (1 − z ) +
(− 1)(− 2) (1 − z )2 + (− 1)(− 2)(− 3) (1 − z )3 ...⎤
1− z ⎣ 21 31 ⎥⎦

=
1
1− z
[
− z + (1 − z ) − (1 − z ) + ....
2 3
]
Q17. Let u ( x, y ) = x + (x − y 2 ) be the real part of analytic function f (z ) of the complex
1 2
2
variable z = x + i y . The imaginary part of f ( z ) is

(a) y + xy (b) xy (c) y (d) y 2 − x 2


Ans: (a)

Solution: u ( x, y ) = x + ( )
x − y 2 , v ( x, y ) = ?
1 2
2
∂u ∂v ∂u ∂v
Check = and =− .
∂x ∂y ∂y ∂x
∂u ∂v ∂v
⇒ = , = 1+ x v = y + xy + f ( x )
∂x ∂y ∂y
∂u ∂v ∂v
=− ⇒ = +y v = yx + f ( y )
∂y ∂x ∂x

y + xy + f ( x ) = yx + f ( y )

f ( x) = 0 f ( y) = y
V = xy + y
Q18. Let y ( x ) be a continuous real function in the range 0 and 2π , satisfying the

d2y dy ⎛ π⎞
inhomogeneous differential equation: sin x 2 + cos x = δ⎜x − ⎟
dx dx ⎝ 2⎠
The value of dyldx at the point x = π / 2
(a) is continuous (b) has a discontinuity of 3
(c) has a discontinuity of 1/3 (d) has a discontinuity of 1

                                                                                
Head office  Branch office 
 
fiziks, H.No. 40 D, G.F, Jia Sarai,  Anand Institute of Mathematics, 
 
Near IIT, Hauz Khas, New Delhi‐16  28‐B/6, Jia Sarai, Near IIT 
 
Phone: 011‐26865455/+91‐9871145498 Hauz Khas, New Delhi‐16 
                                                   
                                             Website: www.physicsbyfiziks.com                                                                                          
                                                            Email: fiziks.physics@gmail.com                                                                   9 
fiziks
Institute for NET/JRF, GATE, IIT‐JAM, JEST, TIFR and GRE in PHYSICAL SCIENCES 
 
Ans: (d)
d2y dy ⎛ x⎞
Solution: After dividing by sin x , + cot x = cosec 2δ ⎜ x − ⎟
⎝ 2⎠
2
dx dx

dy ⎛ dy ⎞ ⎛ π⎞
Integrating both sides, + ∫ cot x⎜ ⎟dx = ∫ cosec xδ ⎜ x − ⎟dx
dx ⎝ dx ⎠ ⎝ 2⎠
dy
+ cot x ⋅ y − ∫ cosec x ⋅ ydx = 1
dx
Using Dirac delta property: ∫ f (x )δ (x − x ) = f (x ) (it lies with the limit).
0 0

dy cos x
+ y⋅ + ∫ y sin 2 xdx = 1 , at x = π ; sin x = 0 . So this is point of discontinuity.
dx sin x
Q19. A ball is picked at random from one of two boxes that contain 2 black and 3 white and 3
black and 4white balls respectively. What is the probability that it is white?
(a) 34 / 70 (b) 41 / 70 (c) 36 / 70 (d) 29 / 70
Ans: (b)
Solution: Probability of picking white ball
2 B 3W 3B 4W
3 4
From box I = and from box II =
5 7 Ι ΙΙ
1 ⎡ 3 4 ⎤ 41
Probability of picking a white ball from either of the two boxes is = + =
2 ⎢⎣ 5 7 ⎥⎦ 70
Q20. The eigenvalues of the antisymmetric matrix,
⎛ 0 − n3 n2 ⎞
⎜ ⎟
A = ⎜ n3 0 − n1 ⎟
⎜ − n2 0 ⎟⎠
⎝ n1

where n1 , n 2 and n3 are the components of a unit vector, are


(a) 0, i, − i (b) 0,1, − 1
(c) 0,1 + i, − 1, − i (d) 0, 0, 0
Ans: (a)

                                                                                
Head office  Branch office 
 
fiziks, H.No. 40 D, G.F, Jia Sarai,  Anand Institute of Mathematics, 
 
Near IIT, Hauz Khas, New Delhi‐16  28‐B/6, Jia Sarai, Near IIT 
 
Phone: 011‐26865455/+91‐9871145498 Hauz Khas, New Delhi‐16 
                                                   
                                             Website: www.physicsbyfiziks.com                                                                                          
                                                            Email: fiziks.physics@gmail.com                                                                   10 
fiziks
Institute for NET/JRF, GATE, IIT‐JAM, JEST, TIFR and GRE in PHYSICAL SCIENCES 
 
⎡ 0 − n3 n 2 ⎤ ⎡ 0 − n3 n 2 ⎤

Solution: A = ⎢ n3 0 ⎥ ⎢
− n1 ⎥ ⇒ − A = ⎢ n3
T
0 − n1 ⎥⎥
⎢⎣− n 2 n1 0 ⎥⎦ ⎢⎣− n 2 n1 0 ⎥⎦

⎡ 0 − n3 n2 ⎤
⎢n 0 −n1 ⎥
⎢ 3 ⎥
⎢⎣ − n2 n1 0 ⎥⎦

⇒ λ1 = 0 ⇒ λ2 = − − n12 − n22 − n32 ⇒ λ3 = −n12 − n22 − n32

but n12 + n22 + n32 = 1

so, λ1 = 0 , λ2 = L , λ3 = − L

A = − AT (Antisymmetric). Eigenvalues are either zero or purely imaginary.


Q21. Which of the following limits exists?
⎛ N 1 ⎞ ⎛ N 1 ⎞
(a) lim ⎜ ∑ + ln N ⎟ (b) lim ⎜ ∑ − ln N ⎟
N →∞
⎝ m =1 m ⎠ N →∞
⎝ m =1 m ⎠
⎛ N
1 ⎞ N
1
(c) lim ⎜⎜ ∑ − ln N ⎟⎟ (d) lim ∑
N →∞
⎝ m =1 m ⎠ N →∞
m =1 m

Ans: (b)
Q22. A bag contains many balls, each with a number painted on it. There are exactly n balls
which have the number n (namely one ball with 1, two balls with 2, and so on until N on
them). An experiment consists of choosing a ball at random, noting the number on it and
returning it to the bag. If the experiment is repeated a large number of times, the average
value the number will tend to
2N + 1 N N +1 N ( N + 1)
(a) (b) (c) (d)
3 2 2 2
Ans: (a)
N ( N + 1)
Solution: Total number of balls 1 + 2 + 3 + 4 + ..... + N =
2
k
The probability for choosing a k th ball at random =
N ( N + 1)
2
                                                                                
Head office  Branch office 
 
fiziks, H.No. 40 D, G.F, Jia Sarai,  Anand Institute of Mathematics, 
 
Near IIT, Hauz Khas, New Delhi‐16  28‐B/6, Jia Sarai, Near IIT 
 
Phone: 011‐26865455/+91‐9871145498 Hauz Khas, New Delhi‐16 
                                                   
                                             Website: www.physicsbyfiziks.com                                                                                          
                                                            Email: fiziks.physics@gmail.com                                                                   11 
fiziks
Institute for NET/JRF, GATE, IIT‐JAM, JEST, TIFR and GRE in PHYSICAL SCIENCES 
 
2Σk 2 2 N ( N + 1)(2 N + 1)
Average of it is given by k = Σk ⋅ P = = ⋅
N ( N + 1) N (N + 1) 6
2N +1 N ( N + 1)(2 N + 1)
= where Σk 2 = .
3 6
Q23. Consider a sinusoidal waveform of amplitude 1V and frequency f 0 . Starting from an
1
arbitrary initial time, the waveform is sampled at intervals of . If the corresponding
2 f0

Fourier spectrum peaks at a frequency f and an amplitude A , them

(a) f = 2 f 0 and A = 1V (b) f = 2 f 0 and 0 ≤ A ≤ 1 A


f0 1
(c) f = 0 and A = 1V (d) f = and A = V
2 2
Ans: (b)
Solution: y = 1sin (2π f 0t ) . y
The fourier transform is:
IV
F (y) =
1
[δ ( f + f 0 )] − δ [ f − f 0 ]
2
t
1 0 T
In Fourier domain f = f 0 , A = .
2

NET/JRF (DEC-2012)
⎡ a b c ⎤
Q24. The unit normal vector of the point ⎢ , , ⎥ on the surface of the ellipsoid
⎣ 3 3 3⎦
x2 y 2 z 2
+ + = 1 is
a2 b2 c2

bciˆ + caˆj + abkˆ aiˆ + bˆj + ckˆ


(a) (b)
a2 + b2 + c2 a2 + b2 + c2

biˆ + cˆj + akˆ iˆ + ˆj + kˆ


(c) (d)
a2 + b2 + c2 3

                                                                                
Head office  Branch office 
 
fiziks, H.No. 40 D, G.F, Jia Sarai,  Anand Institute of Mathematics, 
 
Near IIT, Hauz Khas, New Delhi‐16  28‐B/6, Jia Sarai, Near IIT 
 
Phone: 011‐26865455/+91‐9871145498 Hauz Khas, New Delhi‐16 
                                                   
                                             Website: www.physicsbyfiziks.com                                                                                          
                                                            Email: fiziks.physics@gmail.com                                                                   12 
fiziks
Institute for NET/JRF, GATE, IIT‐JAM, JEST, TIFR and GRE in PHYSICAL SCIENCES 
 
Ans: All the options given are incorrect.
x2 y2 z2
Solution: Here φ = + + −1.
a2 b2 c2

∇φ
Unit normal vector is .
∇φ

⎛ ∂ ∂ ∂ ⎞ ⎛ x2 y 2 z 2 ⎞ 2 xiˆ 2 yˆj 2 zkˆ


So, ∇φ = ⎜ i + j
⎜ ˆ + k ⎟⎟ ⋅ ⎜⎜ 2 + 2 + 2 − 1⎟⎟ = 2 + 2 + 2
ˆ
⎝ ∂x ∂y ∂z ⎠ ⎝ a b c ⎠ a b c

2 ˆ 2 ˆ 2 ˆ
∇φ ⎛ a b c ⎞
= i+ j+ k
⎜⎜ , , ⎟⎟ a 3 b 3 c 3
⎝ 3 3 3⎠

4 4 4 2 b 2c 2 + a 2c 2 + a 2c 2
∇φ = + + =
3a 2 3b 2 3c 2 3 a 2b 2 c 2
2 ˆ 2 ˆ 2 ˆ
i+ j+ k
∇φ a 3 b 3 c 3 bciˆ + caˆj + abkˆ
= =
∇φ ⎛ a b c ⎞
2 b 2c 2 + c 2 a 2 + a 2b 2 b 2c 2 + c 2 a 2 + a 2b 2
⎜⎜ , , ⎟⎟
⎝ 3 3 3⎠ 3 abc

Q25. Given a 2 × 2 unitary matrix U satisfying U †U = UU † = 1 with det U = e iϕ , one can


construct a unitary matrix V (V †V = VV † = 1) with det V = 1 from it by

(a) multiplying U by e −iϕ / 2


(b) multiplying any single element of U by e − iϕ
(c) multiplying any row or column of U by e −iϕ / 2
(d) multiplying U by e − iϕ
Ans: (a)

                                                                                
Head office  Branch office 
 
fiziks, H.No. 40 D, G.F, Jia Sarai,  Anand Institute of Mathematics, 
 
Near IIT, Hauz Khas, New Delhi‐16  28‐B/6, Jia Sarai, Near IIT 
 
Phone: 011‐26865455/+91‐9871145498 Hauz Khas, New Delhi‐16 
                                                   
                                             Website: www.physicsbyfiziks.com                                                                                          
                                                            Email: fiziks.physics@gmail.com                                                                   13 
fiziks
Institute for NET/JRF, GATE, IIT‐JAM, JEST, TIFR and GRE in PHYSICAL SCIENCES 
 
Q26. The graph of the function f (x ) shown below is best described by
1.00
(a) The Bessel function J 0 ( x )
0.25
(b) cos x 0.50
0.25

f (x)
(c) e − x cos x 0.00
1 − 0.25
(d) cos x
x − 0.50
− 0.75
Ans: (a) − 1.00
0 1 2 3 4 5 6 7 8 9 10
x
Q27. In a series of five Cricket matches, one of the captains calls “Heads” every time when the
toss is taken. The probability that he will win 3 times and lose 2 times is
(a) 1 / 8 (b) 5 / 8 (c) 3 / 16 (d) 5 / 16
Ans: (d)
3 5−3 2
⎛1⎞ ⎛ 1⎞ 5! 1 ⎛1⎞ 5!
Solution: P = ⎜ ⎟ ⎜1 − ⎟ = ×⎜ ⎟ .
⎝2⎠ ⎝ 2⎠ 3!(5 − 3)! 8 ⎝ 2 ⎠ 3!(5 − 3)!
1 5 × 4 × 3! 20 5 5
= ⋅ = = =
32 3!× 2! 32 8 × 2 16
The probability of getting exactly k successes in n trials is given by probability mass

p k ⋅ (1 − p ) , k = successes, n = trials.
n! n−k
function =
k !(n − k )!

Q28. The Taylor expansion of the function ln (cosh x ) , where x is real, about the point x = 0
starts with the following terms:
1 2 1 4 1 2 1 4
(a) − x + x + .... (b) x − x + ....
2 12 2 12
1 2 1 4 1 2 1 4
(c) − x + x + .... (d) x + x + ....
2 6 2 6
Ans: (b)
e x + e−x
Solution: cosh x = .Tailor’s series expansion of f ( x ) about x = a
2

                                                                                
Head office  Branch office 
 
fiziks, H.No. 40 D, G.F, Jia Sarai,  Anand Institute of Mathematics, 
 
Near IIT, Hauz Khas, New Delhi‐16  28‐B/6, Jia Sarai, Near IIT 
 
Phone: 011‐26865455/+91‐9871145498 Hauz Khas, New Delhi‐16 
                                                   
                                             Website: www.physicsbyfiziks.com                                                                                          
                                                            Email: fiziks.physics@gmail.com                                                                   14 
fiziks
Institute for NET/JRF, GATE, IIT‐JAM, JEST, TIFR and GRE in PHYSICAL SCIENCES 
 
f ′(a )
f ( x ) = f (a ) + (x − a ) + f ' ' (a ) (x − a )2 + + f ' ' ' (x − a )3 + ... . Here a = 0 .
1! 2! 3!

⎡e x + e−x ⎤ e x − e−x ex − ex
f ( x ) = log ⎢ ⎥ = 0 , f ′( x ) =
1
⋅ = = tanh x = 0
⎣ 2 ⎦ x =0
x =0
e x + e−x 2 e x + e −x
2

f ' ' (x ) =
(e x
)( ) (
+ e−x e x + e−x − e x − e−x e x − e−x )( ) = (e x
+ e−x ) − (e
2 x
− e−x )
2

= 1 − tanh 2 x
(e x
+e )
−x 2
(e x
+e )
−x 2

At x = 0, f ' ' ( x ) = 1, f ' ' ' ( x ) = −2

⇒ f (x ) =
1 2 1 4
x − x + .......
2 12
z 3 dz
Q29. The value of the integral ∫ , where C is a closed contour defined by the
C z 2 − 5z + 6

equation 2 z − 5 = 0, traversed in the anti-clockwise direction, is

(a) − 16π i (b) 16π i (c) 8π i (d) 2π i


Ans: (a)
Solution: z 2 − 5 z + 6 = 0 ⇒ z 2 − 2 z − 3z + 6 = 0 ⇒ z ( z − 2 ) − 3( z − 2) = 0 ⇒ z = 3, 2

2 z = 5 ⇒ z = 2.5 , only 2 will be inside.

z3 z 3 dz
Residue = ( z − 2 ) = 2π i (− 8) = −16π i
8
= = −8 ⇒ ∫ 2
(z − 3)(z − 2) z = 2 2 − 3 c z − 5z + 6

NET/JRF (JUNE-2013)


tn
∑ H n (x ) = e −t + 2tx
2
Q30. Given that
n =0 n!
the value of H 4 (0 ) is
(a) 12 (b) 6 (c) 24 (d) – 6
Ans: (a)

                                                                                
Head office  Branch office 
 
fiziks, H.No. 40 D, G.F, Jia Sarai,  Anand Institute of Mathematics, 
 
Near IIT, Hauz Khas, New Delhi‐16  28‐B/6, Jia Sarai, Near IIT 
 
Phone: 011‐26865455/+91‐9871145498 Hauz Khas, New Delhi‐16 
                                                   
                                             Website: www.physicsbyfiziks.com                                                                                          
                                                            Email: fiziks.physics@gmail.com                                                                   15 
fiziks
Institute for NET/JRF, GATE, IIT‐JAM, JEST, TIFR and GRE in PHYSICAL SCIENCES 
 
∞ n ∞ n
t4 t6
Solution: ∑ H n ( x ) = e −t + 2tx ⇒ ∑ H n (0 ) = e −t = 1 − t 2 + −
t 2 t 2

n =0 n! n =0 n! 2! 3!

H 4 (0 ) 4 t 4
t = ⇒ H 4 (0) = = 12 .
4!

4! 2! 2!
Q31. A unit vector n̂ on the xy -plane is at an angle of 120 o with respect to iˆ . The angle

between the vectors u = a iˆ + b nˆ and v = anˆ + b iˆ will be 60 o if

(a) b = 3a / 2 (b) b = 2a / 3 (c) b = a / 2 (d) b = a


Ans: (c)

Solution: u = aiˆ + bnˆ , v = anˆ + biˆ

( )( )
⇒ u ⋅ v = aiˆ + bnˆ ⋅ anˆ + biˆ ⇒ u v cos 60 = a 2 iˆ ⋅ nˆ + ab + ba + b 2 nˆ.iˆ

(a 2
+ b 2 + 2ab cos120 ) ⋅ cos 60 = a
2
2
cos120 + 2ab + b 2 cos120

⎜ a + b − 2ab × ⎟ ⋅ cos 60 = − (a + b ) + 2ab = (a + b ) − = − (a 2 + b 2 ) + 2ab


⎛ 2 2 1⎞ 1 2 2 1 2 2 ab 1
⎝ 2⎠ 2 2 2 2
5ab a
⇒ a2 + b2 = ⇒b= .
2 2
Q32. With z = x + iy, which of the following functions f ( x, y ) is NOT a (complex) analytic
function of z ?

(a) f ( x, y ) = ( x + iy − 8) 4 + x 2 − y 2 + 2ixy
3
( )
7

(b) f ( x, y ) = ( x + iy ) (1 − x − iy )
7 3

(
(c) f ( x, y ) = x 2 − y 2 + 2ixy − 3 )
5

(d) f ( x, y ) = (1 − x + iy ) (2 + x + iy )
4 6

Ans: (d)
Solution: f ( x, y ) = (1 − x + iy ) (2 + x + iy )
4 6

= {1 − ( x − iy )} (2 + x + iy )
4 6

Due to present of z = ( x − iy )

                                                                                
Head office  Branch office 
 
fiziks, H.No. 40 D, G.F, Jia Sarai,  Anand Institute of Mathematics, 
 
Near IIT, Hauz Khas, New Delhi‐16  28‐B/6, Jia Sarai, Near IIT 
 
Phone: 011‐26865455/+91‐9871145498 Hauz Khas, New Delhi‐16 
                                                   
                                             Website: www.physicsbyfiziks.com                                                                                          
                                                            Email: fiziks.physics@gmail.com                                                                   16 
fiziks
Institute for NET/JRF, GATE, IIT‐JAM, JEST, TIFR and GRE in PHYSICAL SCIENCES 
 
Q33. The solution of the partial differential equation
∂2 ∂2
u ( x , t ) − u ( x, t ) = 0
∂t 2 ∂x 2
satisfying the boundary conditions u (0, t ) = 0 = u (L, t ) and initial conditions

u ( x,0 ) = sin (πx / L ) and u (x, t ) t =0 = sin (2πx / L ) is
∂t

(a) sin (π x / L ) cos(π t / L ) + sin (2πx / L ) cos(2π t / L )


L

(b) 2 sin (πx / L ) cos(π t / L ) − sin (πx / L ) cos(2π t / L )

(c) sin (πx / L ) cos(2π t / L ) + sin (2πx / L )sin (π t / L )


L
π

(d) sin (πx / L ) cos(π t / L ) + sin (2πx / L )sin (2π t / L )


L

Ans: (d)
∂ 2u ∂ 2u πx ∂u 2π x
Solution: − 2 = 0 , u ( x,0 ) = sin and = sin
∂t 2
∂x L ∂t L
This is a wave equation
⎛ anπ t an π t ⎞
So solution is given by u ( x, t ) = ∑ ⎜ An cos + Bn sin ⎟
n ⎝ L L ⎠

nπx nπx
L L
f (x )sin g (n )sin
2 2
with An =
L0∫ L
dx, Bn = ∫
anπ 0 L
dx

∂ 2u ∂ 2u πx 2π x
Comparing a 2 = 2 , We have a = 1 and f ( x ) = sin , g (n ) = sin ,
∂t 2
∂x L L
⎛ 2πx ⎞
L ⎜ 1 − cos ⎟
πx nπ x π
L L
2 2 x 2 L ⎟dx = 2 ⋅ L = 1 (let n = 1 )
An = ∫ sin sin dx = ∫ sin 2 dx = ∫ ⎜
L0 L L L0 L L 0⎜ 2 ⎟ L 2
⎜ ⎟
⎝ ⎠
Putting n = 2

                                                                                
Head office  Branch office 
 
fiziks, H.No. 40 D, G.F, Jia Sarai,  Anand Institute of Mathematics, 
 
Near IIT, Hauz Khas, New Delhi‐16  28‐B/6, Jia Sarai, Near IIT 
 
Phone: 011‐26865455/+91‐9871145498 Hauz Khas, New Delhi‐16 
                                                   
                                             Website: www.physicsbyfiziks.com                                                                                          
                                                            Email: fiziks.physics@gmail.com                                                                   17 
fiziks
Institute for NET/JRF, GATE, IIT‐JAM, JEST, TIFR and GRE in PHYSICAL SCIENCES 
 
⎛ 4πx ⎞
L ⎜ 1 − cos ⎟
2πx nπx 2 2πx
L L
2 2 2 ⎜ L ⎟dx = 2 ⋅ L = L
anπ ∫0 2π ∫0 2π ∫0 ⎜
Bn = sin ⋅ sin dx = sin dx =
L L L 2 ⎟ 2π 2 2π
⎜ ⎟
⎝ ⎠
Q34. The solution of the differential equation
dx
= x2
dt
with the initial condition x(0 ) = 1 will blow up as t tends to
(a) 1 (b) 2 (c) ½ (d) ∞
Ans: (a)
dx dx x −2+1 −1
Solution: = x 2 ⇒ ∫ 2 = ∫ dt ⇒ =t +C ⇒ =t +C
dt x − 2 +1 x
−1 −1
⇒ x(0) = 1 ⇒
1
= 0 + C ⇒ C = −1 ⇒ = t −1 ⇒ x = as t → 1 x blows up
1 x 1− t
1
Q35. The inverse Laplace transforms of is
s (s + 1)
2

1 2 −t 1 2
(a) t e (b) t + 1 − e −t
2 2

(c) t −1 + e −t (d)
1 2
2
(
t 1 − e −t )
Ans: (c)
⎡ 1 ⎤ t −t
Solution: f (s ) =
1
⇒ f (t ) = e −t ⇒ L−1 ⎢ ⎥ = ∫ e dt = − e −t ( ) = (− e
t −t
+ 1)
s +1 ⎣ s(s + 1) ⎦ 0
0

⎡ 1 ⎤ t
⇒L ⎢ 2 −1 −t −t
( t −t
)
⎥ = ∫ − e + 1 dt = e + t 0 = e + t − 1 . ( )
⎣ s (s + 1) ⎦ 0
Q36. The approximation cos θ ≈ 1 is valid up to 3 decimal places as long as θ is less than:

(take 180 o / π ≈ 57.29 o )


(a) 1.28° (b) 1.81° (c) 3.28° (d) 4.01°
Ans: (b)

                                                                                
Head office  Branch office 
 
fiziks, H.No. 40 D, G.F, Jia Sarai,  Anand Institute of Mathematics, 
 
Near IIT, Hauz Khas, New Delhi‐16  28‐B/6, Jia Sarai, Near IIT 
 
Phone: 011‐26865455/+91‐9871145498 Hauz Khas, New Delhi‐16 
                                                   
                                             Website: www.physicsbyfiziks.com                                                                                          
                                                            Email: fiziks.physics@gmail.com                                                                   18 
fiziks
Institute for NET/JRF, GATE, IIT‐JAM, JEST, TIFR and GRE in PHYSICAL SCIENCES 
 
θ2 θ2 θ2
Solution: cos θ = 1 − + − ....... ≈ 1 −
2! 4! 2!
π
cos θ ≈ 1 when θ = 1.81o ≈ = .0314
100

JRF/NET-(DEC-2013)

Q37. If A = iˆyz + ˆjxz + kˆxy , then the integral ∫ A ⋅ dl


C
(where C is along the perimeter of a

rectangular area bounded by x = 0, x = a and y = 0, y = b ) is

(a)
1 3
2
(
a + b3 ) (
(b) π ab 2 + a 2 b ) (
(c) π a 3 + b 3 ) (d) 0

Ans: (d)

∫ A ⋅ d l = ∫ (∇ × A).d a = 0 since ∇ × A = 0 .
C S

Q38. If A, B and C are non-zero Hermitian operators, which of the following relations must
be false?
(a) [A, B ] = C (b) AB + BA = C (c) ABA = C (d) A + B = C
Ans: (a)
Solution: [ A, B ] = C ⇒ AB − BA = C ⇒ ( AB − BA)† = C †

(( AB)† − ( BA)† ) = C † ⇒ ( B † A† ) − ( A† B † ) = C †
Hence A,B and C are hermitian then
BA − AB = C ≠ [A, B ] = C
Q39. Which of the following functions cannot be the real part of a complex analytic function
of z = x + iy ?

(a) x 2 y (b) x 2 − y 2 (c) x 3 − 3 xy 2 (d) 3x 2 y − y − y 3


Ans: (a)

                                                                                
Head office  Branch office 
 
fiziks, H.No. 40 D, G.F, Jia Sarai,  Anand Institute of Mathematics, 
 
Near IIT, Hauz Khas, New Delhi‐16  28‐B/6, Jia Sarai, Near IIT 
 
Phone: 011‐26865455/+91‐9871145498 Hauz Khas, New Delhi‐16 
                                                   
                                             Website: www.physicsbyfiziks.com                                                                                          
                                                            Email: fiziks.physics@gmail.com                                                                   19 
fiziks
Institute for NET/JRF, GATE, IIT‐JAM, JEST, TIFR and GRE in PHYSICAL SCIENCES 
 
2
Solution: Let x y be real part of a complex function. Use Milne Thomson’s method to write
analytic complex function. The real part of that function should be (1) but that is not the
case. So this cannot be real part of an analytic function. Also,
z 2 = (x + iy ) = x 2 − y 2 + 2ixy , Real part option (2)
2

z 3 = ( x + iy ) = x 3 − iy 3 + 3ixy ( x + iy )
3

= x 3 − iy 3 + 3ix 2 y − 3xy 2 , Real part option (3)


Q40. The expression
⎛ ∂2 ∂2 ∂2 ∂2 ⎞ 1
⎜ 2 + 2 + 2 + 2 ⎟ 2
⎜ ∂x
⎝ 1 ∂x 2 ∂x3 ∂x 4 ⎠ 1 (
⎟ x + x2 + x2 + x2
2 3 4 )
is proportional to
(a) δ ( x1 + x 2 + x3 + x 4 ) (b) δ ( x1 )δ (x 2 )δ (x3 )δ ( x 4 )

(
(c) x12 + x 22 + x32 + x 42 )−3 / 2
(
(d) x12 + x 22 + x32 + x 42 )−2

Ans: (b)
⎡ ∂ ⎛ 1 ⎞⎤ − 2 x1
Solution: ⎢ ⎜ 2 ⎟⎥ =
⎠⎥⎦ (x1 + x2 + x3 + x4 )
⎜ ⎟
⎢⎣ ∂x1 ⎝ x1 + x 2 + x3 + x 4 2 2
2 2 2 2 2 2

∂2 ⎡ (x 2 + x 2 + x 2 + x 2 )2 ⋅ 1 − 2 ⋅ 2 x ⋅ x (x 2 + x 2 + x 2 + x 2 )⎤
= −2⎢ 1 2 3 4 1 1 1 2 3 4

∂x12 ⎢⎣ (x1 + x2 + x3 + x4 )
2 2 2 2 4
⎥⎦

( 2 3 4 1 )
⎡ x 2 + x 2 + x 2 + x 2 2 − 4 x 2 ⎤ 8x 2 − 2 x 2 + x 2 + x 2 + x 2
= −2 ⎢ 1 ⎥= 1 1 2 3 4 ( )
⎣⎢ (
x1 + x 2 + x3 + x 4
2 2 2 2 3
⎦⎥ )
x1 + x 2 + x3 + x 4
2 2 2 2 3
( )
Now similarly solving all and add up then we get
⎛ ∂2 ∂2 ∂2 ∂ 2 ⎞⎛ 1 1 1 1 ⎞
⎜ 2 + 2 + 2 + 2 ⎟⎜ 2 + 2 + 2 + 2 ⎟
⎜ ∂x ⎟⎜ ⎟
⎝ 1 ∂x 2 ∂x3 ∂x 4 ⎠⎝ x1 x 2 x3 x 4 ⎠

8 ( x12 + x22 + x32 + x42 ) − 8 ( x12 + x22 + x32 + x42 )


= =0
(x + x22 + x32 + x42 )
2 3
1

also if all x1 , x2 , x3 , x4 becomes zero it should be infinity.

                                                                                
Head office  Branch office 
 
fiziks, H.No. 40 D, G.F, Jia Sarai,  Anand Institute of Mathematics, 
 
Near IIT, Hauz Khas, New Delhi‐16  28‐B/6, Jia Sarai, Near IIT 
 
Phone: 011‐26865455/+91‐9871145498 Hauz Khas, New Delhi‐16 
                                                   
                                             Website: www.physicsbyfiziks.com                                                                                          
                                                            Email: fiziks.physics@gmail.com                                                                   20 
fiziks
Institute for NET/JRF, GATE, IIT‐JAM, JEST, TIFR and GRE in PHYSICAL SCIENCES 
 
⎛∂ 2
∂ 2
∂ 2
∂ ⎞
2
= δ ( x1 ) ⋅ δ ( x 2 ) ⋅ δ ( x3 ) ⋅ δ ( x 4 )
1
So ⎜⎜ 2 + 2 + 2 + 2 ⎟⎟ 2

⎝ 1x ∂ x 2 ∂ x 3 ∂x 4 ⎠ (x1 + x 2
2 + x 2
3 + x 2
4 )
∞ ∞
dx π dx
Q41. Given that the integral ∫ 2
0 y + x
2
=
2y
, the value of ∫ (y
0
2
+ x2 )
2
is

π π π π
(a) (b) 3
(c) (d)
y 3
4y 8y 3
2 y3
Ans: (b)
∞ ∞

Solution: ∫ (y
dx 1
= ∫
dx
, pole is of 2nd order at x = iy , residue = 1/ 4iy 3 ( )
+ x2 ) ( )
2 2
0
2 2 −∞ y + x
2 2

Integral = 1/ 2* 2π i *1/ (4iy 3 ) = π / (4 y 3 )


Q42. The Fourier transform of the derivative of the Dirac δ - function, namely δ ′( x ) , is
proportional to
(a) 0 (b) 1 (c) sin k (d) ik
Ans: (d)
Solution: Fourier transform of δ ′( x )

H (K ) = ∫ δ ′(x )e ikx dx = ike (k ⋅0 ) = ik
−∞

Q43. Consider an n × n(n > 1) matrix A , in which Aij is the product of the indices i and j

(namely Aij = ij ). The matrix A

(a) has one degenerate eigevalue with degeneracy (n − 1)


(b) has two degenerate eigenvalues with degeneracies 2 and (n − 2 )
(c) has one degenerate eigenvalue with degeneracy n
(d) does not have any degenerate eigenvalue
Ans: (a)
⎛ 1 2⎞
Solution: If matrix is 2 × 2 let ⎜ ⎟ then eigen value is given by
⎝ 2 4⎠

                                                                                
Head office  Branch office 
 
fiziks, H.No. 40 D, G.F, Jia Sarai,  Anand Institute of Mathematics, 
 
Near IIT, Hauz Khas, New Delhi‐16  28‐B/6, Jia Sarai, Near IIT 
 
Phone: 011‐26865455/+91‐9871145498 Hauz Khas, New Delhi‐16 
                                                   
                                             Website: www.physicsbyfiziks.com                                                                                          
                                                            Email: fiziks.physics@gmail.com                                                                   21 
fiziks
Institute for NET/JRF, GATE, IIT‐JAM, JEST, TIFR and GRE in PHYSICAL SCIENCES 
 
⎛1 − λ 2 ⎞
⎜ ⎟ = 0 ⇒ (1 − λ )(4 − λ ) − 4 = 0 ⇒ λ = 0,5
⎝ 2 4−λ⎠

⎛ 1 2 3⎞
⎜ ⎟
If If matrix is 3 × 3 let ⎜ 2 4 6 ⎟ then eigen value is given by
⎜ 3 6 9⎟
⎝ ⎠

⎛1 − λ 2 3 ⎞
⎜ ⎟
⎜ 2 4−λ 6 ⎟=0
⎜ 3 9 − λ ⎟⎠
⎝ 6

(1 − λ )[(4 − λ )(9 − λ ) − 36] + 2[18 − 2(9 − λ )] + 3[12 − 3(4 − λ )]


(1 − λ )[λ2 − 13λ + 36 − 36] + 2[18 − 18 + 2λ ] + 3[12 − 12 + 3λ ] = 0
λ2 − 13λ − λ3 + 13λ2 + 13λ = 0 ⇒ λ3 − 14λ2 = 0 ⇒ λ = 0, 0, λ = 14
i.e. has one degenerate eigenvalue with degeneracy 2.
Thus one can generalized that for n dimensional matrix has one degenerate eigevalue
with degeneracy (n − 1) .
Q44. Three sets of data A, B and C from an experiment, represented by × , and Ο , are
plotted on a log-log scale. Each of these are fitted with straight lines as shown in the
figure. 1000

100
C
B
10

1 ×
×
× A
×
0.1 1 10 100 1000
0.1

The functional dependence y ( x ) for the sets A, B and C are respectively


x 1 1
(a) x , x and x 2 (b) − , x and 2 x (c) , x and x 2 (d) , x and x 2
2 x2 x
Ans: (d)

                                                                                
Head office  Branch office 
 
fiziks, H.No. 40 D, G.F, Jia Sarai,  Anand Institute of Mathematics, 
 
Near IIT, Hauz Khas, New Delhi‐16  28‐B/6, Jia Sarai, Near IIT 
 
Phone: 011‐26865455/+91‐9871145498 Hauz Khas, New Delhi‐16 
                                                   
                                             Website: www.physicsbyfiziks.com                                                                                          
                                                            Email: fiziks.physics@gmail.com                                                                   22 
fiziks
Institute for NET/JRF, GATE, IIT‐JAM, JEST, TIFR and GRE in PHYSICAL SCIENCES 
 
JRF/NET-(JUNE-2014)
Q45. Consider the differential equation
d 2x dx
2
+2 +x=0
dt dt
with the initial conditions x(0 ) = 0 and x(0 ) = 1 . The solution x(t ) attains its maximum
value when t is
(a) 1/2 (b) 1 (c) 2 (d) ∞
Ans: (b)
d2x dx
+ 2 + x = 0 ⇒ m 2 + 2m + 1 = 0 ⇒ ( m + 1) = 0 ⇒ m = −1, 1
2
Solution: 2
dt dt
⇒ x = ( c1 + c2 t ) e − t since x ( 0 ) = 0 ⇒ 0 = c1 ⇒ x = c2 te −t 0

⇒ x = c2 ⎡⎣ −te−t + e− t ⎤⎦

Since x ( 0 ) = 1 ⇒ 1 = c2 ⇒ x = te−t

For maxima or minima x = 0 ⇒ x = −te− t + e −t = 0 ⇒ x = e −t (1 − t )

⇒ e− t = 0, 1 − t = 0 ⇒ t = ∞, t = 1
x = e − t ( −1) + (1 − t ) e − t ( −1) = −e − t + ( t − 1) e − t
⇒ x (1) = −e −1 + 0e − t < 0
Q46. Consider the matrix
⎛ 0 2i 3i ⎞
⎜ ⎟
M = ⎜ − 2i 0 6i ⎟
⎜ − 3i − 6i 0 ⎟
⎝ ⎠
The eigenvalues of M are
(a) − 5, − 2, 7 (b) − 7, 0, 7 (c) − 4i, 2i, 2i (d) 2, 3, 6
Ans: (b)
⎛ 0 2i 3i ⎞ ⎛ 0 2i 3i ⎞
⎜ ⎟ ⎜ ⎟
Solution: M = ⎜ −2i 0 6i ⎟ , M + = ⎜ −2i 0 6i ⎟
⎜ −3i −6i 0 ⎟ ⎜ −3i −6i 0 ⎟
⎝ ⎠ ⎝ ⎠
                                                                                
Head office  Branch office 
 
fiziks, H.No. 40 D, G.F, Jia Sarai,  Anand Institute of Mathematics, 
 
Near IIT, Hauz Khas, New Delhi‐16  28‐B/6, Jia Sarai, Near IIT 
 
Phone: 011‐26865455/+91‐9871145498 Hauz Khas, New Delhi‐16 
                                                   
                                             Website: www.physicsbyfiziks.com                                                                                          
                                                            Email: fiziks.physics@gmail.com                                                                   23 
fiziks
Institute for NET/JRF, GATE, IIT‐JAM, JEST, TIFR and GRE in PHYSICAL SCIENCES 
 
+
M =M
Matrix is Hermitian so roots are real and trace = 0.
λ1 + λ2 + λ3 = 0, λ1 ⋅ λ2 ⋅ λ3 = 0 ⇒ λ = −7, 0, 7
1
Q47. If C is the contour defined by z = , the value of the integral
2
dz
∫ C sin 2 z

is
(a) ∞ (b) 2π i (c) 0 (d) π i
Ans: (c)
1 ⎛ 1⎞
Solution: f ( z ) = ⎜z = ⎟
sin z ⎝
2
2⎠
3 5
z z 1 1
sin z = z − + .... ⇒ 2
= 2
3 5 sin z ⎛ z3 z5 ⎞
⎜ z − + .... ⎟
⎝ 3 5 ⎠
−2
1 1 ⎡ z2 z4 ⎤ dz
⇒ = ⎢1 − + ....⎥ ⇒
sin 2 z z 2 ⎣ 3 5 ⎦ ∫ C sin 2 z
=0

∑ Pn ( x )t n = (1 − 2 xt + t 2 ) , for t < 1 , the value of P5 (− 1) is


∞ −1 / 2
Q48. Given n =0

(a) 0.26 (b) 1 (c) 0.5 (d) –1


Ans: (d)
Pn ( −1) = −1 if n is odd ⇒ P5 ( −1) = −1

                                                                                
Head office  Branch office 
 
fiziks, H.No. 40 D, G.F, Jia Sarai,  Anand Institute of Mathematics, 
 
Near IIT, Hauz Khas, New Delhi‐16  28‐B/6, Jia Sarai, Near IIT 
 
Phone: 011‐26865455/+91‐9871145498 Hauz Khas, New Delhi‐16 
                                                   
                                             Website: www.physicsbyfiziks.com                                                                                          
                                                            Email: fiziks.physics@gmail.com                                                                   24 
fiziks
Institute for NET/JRF, GATE, IIT‐JAM, JEST, TIFR and GRE in PHYSICAL SCIENCES 
 
Q49. The graph of a real periodic function f (x ) for the range [− ∞, ∞ ] is shown below
f (x )

Which of the following graphs represents the real part of its Fourier transform?

Re f (k )
(a) (b)
Re f (k )

k k

(c) Re f (k ) (d) Re f (k )

k k

Ans: (b)
Solution: This is cosine function
A
f ( x ) = A cos x ⇒ F ( k ) = ⎡δ ( k − k0 ) + δ ( k + k0 ) ⎤⎦
2⎣
NET/JRF (DEC-2014)
Q50. Let r denote the position vector of any point in three-dimensional space, and r = r .

Then
(a) ∇ ⋅ r = 0 and ∇ × r = r / r (b) ∇ ⋅ r = 0 and ∇ 2 r = 0

(c) ∇ ⋅ r = 3 and ∇ 2 r = r / r 2 (d) ∇ ⋅ r = 3 and ∇ × r = 0


Ans: (d)
Solution: r = xxˆ + yyˆ + zzˆ
                                                                                
Head office  Branch office 
 
fiziks, H.No. 40 D, G.F, Jia Sarai,  Anand Institute of Mathematics, 
 
Near IIT, Hauz Khas, New Delhi‐16  28‐B/6, Jia Sarai, Near IIT 
 
Phone: 011‐26865455/+91‐9871145498 Hauz Khas, New Delhi‐16 
                                                   
                                             Website: www.physicsbyfiziks.com                                                                                          
                                                            Email: fiziks.physics@gmail.com                                                                   25 
fiziks
Institute for NET/JRF, GATE, IIT‐JAM, JEST, TIFR and GRE in PHYSICAL SCIENCES 
 
∂x ∂y ∂z
∇ ⋅ r = = + + = 1+1+1 = 3
∂x ∂y ∂z

xˆ yˆ zˆ
⎛ ∂z ∂y ⎞ ⎛ ∂x ∂z ⎞ ⎛ ∂y ∂x ⎞
∇ × r = ∂ / ∂x ∂ / ∂y ∂ / ∂z = xˆ ⎜ − ⎟ + yˆ ⎜ − ⎟ + zˆ ⎜ − ⎟ = 0
x y z
⎝ ∂y ∂z ⎠ ⎝ ∂z ∂x ⎠ ⎝ ∂x ∂y ⎠

⎛a⎞ ⎛0 0 1⎞
⎜ ⎟ ⎜ ⎟
Q51. The column vector ⎜ b ⎟ is a simultaneous eigenvector of A = ⎜ 0 1 0 ⎟ and
⎜a⎟ ⎜1 0 0⎟
⎝ ⎠ ⎝ ⎠

⎛0 1 1⎞
⎜ ⎟
B = ⎜ 1 0 1 ⎟ if
⎜1 1 0⎟
⎝ ⎠
(a) b = 0 or a = 0 (b) b = a or b = −2a
(c) b = 2a or b = −a (d) b = a / 2 or b = −a / 2
Ans: (b)
Solution: Let b = a

⎛ 0 0 1 ⎞⎛ a ⎞ ⎛ a ⎞ ⎛0 1 1⎞⎛ a⎞ ⎛ a⎞ ⎛ a⎞
⎜ ⎟⎜ ⎟ ⎜ ⎟ ⎜ ⎟⎜ ⎟ ⎜ ⎟ ⎜ ⎟
⎜ 0 1 0 ⎟⎜ a ⎟ = ⎜ a ⎟ and ⎜ 1 0 1 ⎟ ⎜ a ⎟ = ⎜ a ⎟ = ⎜ a ⎟
⎜ 1 0 0 ⎟⎜ a ⎟ ⎜ a ⎟ ⎜1 1 0⎟⎜ a ⎟ ⎜ a ⎟ ⎜ a ⎟
⎝ ⎠⎝ ⎠ ⎝ ⎠ ⎝ ⎠⎝ ⎠ ⎝ ⎠ ⎝ ⎠
Let b = −2a

⎛0 0 1⎞⎛ a ⎞ ⎛ a ⎞ ⎛ 0 1 1 ⎞ ⎛ a ⎞ ⎛ −a ⎞ ⎛ a ⎞
⎜ ⎟⎜ ⎟ ⎜ ⎟ ⎜ ⎟⎜ ⎟ ⎜ ⎟ ⎜ ⎟
⎜ 0 1 0 ⎟ ⎜ −2a ⎟ = ⎜ −2a ⎟ and ⎜ 1 0 1 ⎟ ⎜ −2a ⎟ = ⎜ 2a ⎟ = −1⎜ −2a ⎟
⎜1 0 0⎟⎜ a ⎟ ⎜ a ⎟ ⎜ 1 1 0 ⎟ ⎜ a ⎟ ⎜ −a ⎟ ⎜ a ⎟
⎝ ⎠⎝ ⎠ ⎝ ⎠ ⎝ ⎠⎝ ⎠ ⎝ ⎠ ⎝ ⎠
For other combination above relation is not possible.
∞ sin (2 x )
Q52. The principal value of the integral ∫ −∞ x3
dx is

(a) − 2π (b) − π (c) π (d) 2π


Ans: (a)

                                                                                
Head office  Branch office 
 
fiziks, H.No. 40 D, G.F, Jia Sarai,  Anand Institute of Mathematics, 
 
Near IIT, Hauz Khas, New Delhi‐16  28‐B/6, Jia Sarai, Near IIT 
 
Phone: 011‐26865455/+91‐9871145498 Hauz Khas, New Delhi‐16 
                                                   
                                             Website: www.physicsbyfiziks.com                                                                                          
                                                            Email: fiziks.physics@gmail.com                                                                   26 
fiziks
Institute for NET/JRF, GATE, IIT‐JAM, JEST, TIFR and GRE in PHYSICAL SCIENCES 
 
i2z
e
Solution: Let f ( z ) = 3
z
i2z
3 e
lim z →0 ( z − 0 ) f ( z ) = lim z →0 ( z − 0 ) 3 = 1( finite and ≠ 0 ) ⇒ z = 0 is pole of order 3.
3

z
1 d ⎡
2
3 e ⎤
iz

2 ⎢(
Residue R = lim z − 0 ) 3 ⎥ = −2
2! z → 0 dz ⎣ z ⎦
∞ ∞
⇒∫ f ( x ) dx = π iΣR = π i ( −2 ) = −2π i ⇒ Im. Part = −2π ⇒ ∫ f ( x ) dx = − 2π
−∞ −∞

Q53. The Laurent series expansion of the function f ( z ) = e z + e1 / z about z = 0 is given by

zn ⎛ n 1 ⎞1
∑n=−∞ n! for all z < ∞ ∑
∞ ∞
(a) (b) ⎜z + n ⎟
n=0
only if 0 < z < 1
⎝ z ⎠ n!
⎛ n 1 ⎞1 zn
∑ (d) ∑n = −∞
∞ ∞
(c) n=0
⎜z + n ⎟ for all 0 < z < ∞ only if z < 1
⎝ z ⎠ n! n!
Ans: (c)
⎛ z2 ⎞ ∞ z
n
1 1 1 1
Solution: e z = ⎜1 + z + + .... ⎟ = ∑ n =0 + .... = ∑ n =0 n

and e1/ z = 1 + + 2
⎝ 2! ⎠ n! z 2! z z n!
∞ ⎛ 1 ⎞ 1
⇒ f ( z ) = e z + e1/ z = ∑ n =0 ⎜ z n + n ⎟ for all 0 < z < ∞
⎝ z ⎠ n !
Q54. Two independent random variables m and n , which can take the integer values
0, 1, 2, ..., ∞ , follow the Poisson distribution, with distinct mean values μ and ν
respectively. Then
(a) the probability distribution of the random variable l = m + n is a binomial
distribution.
(b) the probability distribution of the random variable r = m − n is also a Poisson
distribution.
(c) the variance of the random variable l = m + n is equal to μ + ν
(d) the mean value of the random variable r = m − n is equal to 0.
Ans: (c)
Solution: σ l2 = σ m2 + σ n2 = μ + ν

                                                                                
Head office  Branch office 
 
fiziks, H.No. 40 D, G.F, Jia Sarai,  Anand Institute of Mathematics, 
 
Near IIT, Hauz Khas, New Delhi‐16  28‐B/6, Jia Sarai, Near IIT 
 
Phone: 011‐26865455/+91‐9871145498 Hauz Khas, New Delhi‐16 
                                                   
                                             Website: www.physicsbyfiziks.com                                                                                          
                                                            Email: fiziks.physics@gmail.com                                                                   27 
fiziks
Institute for NET/JRF, GATE, IIT‐JAM, JEST, TIFR and GRE in PHYSICAL SCIENCES 
 
f ( z ) = ln(1 − z )
1
Q55. Consider the function of a complex variable
z
z = re iθ (r ≥ 0, −∞ < θ < ∞ ) . The singularities of f ( z ) are as follows:
(a) branch points at z = 1 and z = ∞ ; and a pole at z = 0 only for 0 ≤ θ < 2π
(b) branch points at z = 1 and z = ∞ ; and a pole at z = 0 for all θ other than 0 ≤ θ < 2π
(c) branch points at z = 1 and z = ∞ ; and a pole at z = 0 for all θ
(d) branch points at z = 0, z = 1 and z = ∞ .
Ans: None of the above is correct

1 1⎛ z 2 z3 ⎞ z z2
Solution: For f ( z ) = ln (1 − z ) = ⎜ − z − − − ..... ⎟ = −1 − − − .....
z z⎝ 2 3 ⎠ 2 3
There is no principal part and when z → 0 , f ( z ) = −1 . So there is removable singularity

at z = 0 . Also z = 1 and z = ∞ is Branch point.

The function f ( x ) = ∑n =0
∞ (− 1)n ⎛ x ⎞ 2 n+1
Q56. ⎜ ⎟ satisfies the differential equation
n !(n + 1)! ⎝ 2 ⎠

+ (x 2 + 1) f = 0 + (x 2 − 1) f = 0
d2 f df d2 f df
(a) x 2 2
+x (b) x 2 2
+ 2x
dx dx dx dx

+ (x 2 − 1) f = 0 + (x 2 − 1) f = 0
d2 f df d2 f df
(c) x 2 2
+x (d) x 2 2
−x
dx dx dx dx
Ans: (c)

f ( x ) = ∑n =0
∞ (− 1) ⎛ x ⎞
n 2 n +1

Solution: ⎜ ⎟ is generating function (Bessel Function of first kind)


n !(n + 1)! ⎝ 2 ⎠
d2 f
dx 2
df
+ x + x 2 − n 2 f = 0 , put n = 1 .
which satisfies the differential equation x 2
dx
( )
Q57. Let α and β be complex numbers. Which of the following sets of matrices forms a
group under matrix multiplication?
⎛α β ⎞ ⎛1 α⎞
(a) ⎜⎜ ⎟⎟ (b) ⎜⎜ ⎟⎟ , where αβ ≠ 1
⎝0 0⎠ ⎝β 1 ⎠
⎛α α * ⎞ ⎛ α β⎞
(c) ⎜⎜ ⎟ , where αβ * is real (d) ⎜⎜ ⎟ , where α + β =1
2 2
*⎟ *⎟
⎝β β ⎠ ⎝− β α ⎠
*

Ans: (d)

                                                                                
Head office  Branch office 
 
fiziks, H.No. 40 D, G.F, Jia Sarai,  Anand Institute of Mathematics, 
 
Near IIT, Hauz Khas, New Delhi‐16  28‐B/6, Jia Sarai, Near IIT 
 
Phone: 011‐26865455/+91‐9871145498 Hauz Khas, New Delhi‐16 
                                                   
                                             Website: www.physicsbyfiziks.com                                                                                          
                                                            Email: fiziks.physics@gmail.com                                                                   28 
fiziks
Institute for NET/JRF, GATE, IIT‐JAM, JEST, TIFR and GRE in PHYSICAL SCIENCES 
 
α β
= α + β =1
2 2
Solution: ∵
−β α
* *

∑∈ {x , {p , Lk }} (where ∈i j k is the Levi-Civita symbol, x , p, L are


3
Q58. The expression i jk i j
i , j , k =1

the position, momentum and angular momentum respectively, and {A, B} represents the
Poisson Bracket of A and B ) simplifies to
(a) 0 (b) 6 (
(c) x , p × L ) (d) x × p
Ans: (b)
NET/JRF (JUNE-2015)
∞ dx
Q59. The value of integral ∫−∞ 1 + x4
π π
(a) (b) (c) 2π (d) 2π
2 2
Ans. (a)
∞ dz
Solution: ∫−∞ 1 + z 4
∵z =R

π
( 2 n +1)
Now pole ⇒ z=e 4


1 1 −1 1
n = 0, ⇒ z0 = e = 4
+i , n = 2 ⇒ z2 = −i
2 2 2 2
i 3π
−1 1 1 1
n = 1 ⇒ z1 = e 4
= +i , n = 3 ⇒ z3 = + −i
2 2 2 2
only z0 and z1 lies in contour

1⎛ 1 1 ⎞
i.e., residue at z = e 4 = ⎜ − −i ⎟
4⎝ 2 2⎠
i 3π
1⎛ 1 1 ⎞
residue at z = e 4
= ⎜ −i ⎟
4⎝ 2 2⎠
∞ dx π
now ∫
−∞ x +1
4
= 2π iε Re s =
2

                                                                                
Head office  Branch office 
 
fiziks, H.No. 40 D, G.F, Jia Sarai,  Anand Institute of Mathematics, 
 
Near IIT, Hauz Khas, New Delhi‐16  28‐B/6, Jia Sarai, Near IIT 
 
Phone: 011‐26865455/+91‐9871145498 Hauz Khas, New Delhi‐16 
                                                   
                                             Website: www.physicsbyfiziks.com                                                                                          
                                                            Email: fiziks.physics@gmail.com                                                                   29 
fiziks
Institute for NET/JRF, GATE, IIT‐JAM, JEST, TIFR and GRE in PHYSICAL SCIENCES 
 
2
d x dx
Q60. Consider the differential equation 2
− 3 + 2 x = 0 . If x = 0 at t = 0 and x = 1 at
dt dt
t = 1 , the value of x at t = 2 is
(a) e2 + 1 (b) e2 + e (c) e + 2 (d) 2e
Ans. (b)
Solution: D 2 − 3D + 2 = 0
( D − 1)( D − 2 ) = 0 ⇒ D = 1, 2 ⇒ x = c1e 2t + c2 et

using boundary condition x = 0, t = 0


c1 = c2

again using boundary condition x = 1, t = 1

1 1 e2t 1 t
c2 = , c1 = 2 ⇒x= 2 + e
e−e 2
e −e e − e e − e2
again using t = 2 then x = e 2 + e
Q61. The Laplace transform of 6t 3 + 3sin 4t is
36 12 36 12
(a) + 2 (b) + 2
s 4
s + 16 s 4
s − 16
18 12 36 12
(c) + 2 (d) + 2
s 4
s − 16 s 3
s + 16
Ans. (a)
n +1
Solution: L ⎡⎣ 6t 3 + 3sin 4t ⎤⎦ ∵ L ⎡⎣t n ⎤⎦ =
S n +1
a
∵ L sin at =
s + a2
2

6 × 4 3× 4 36 12
⇒ + 2 ⇒ 4 + 2
s 4
s + 16 s s + 16

                                                                                
Head office  Branch office 
 
fiziks, H.No. 40 D, G.F, Jia Sarai,  Anand Institute of Mathematics, 
 
Near IIT, Hauz Khas, New Delhi‐16  28‐B/6, Jia Sarai, Near IIT 
 
Phone: 011‐26865455/+91‐9871145498 Hauz Khas, New Delhi‐16 
                                                   
                                             Website: www.physicsbyfiziks.com                                                                                          
                                                            Email: fiziks.physics@gmail.com                                                                   30 
fiziks
Institute for NET/JRF, GATE, IIT‐JAM, JEST, TIFR and GRE in PHYSICAL SCIENCES 
 
∂2 f 2 ∂ f
2
Q62. Let f ( x, t ) be a solution of the wave equation = v in 1-dimension. If at
∂t 2 ∂x 2
∂f
t = 0, f ( x, 0 ) = e − x and ( x, 0 ) = 0 for all x , then f ( x, t ) for all future times t > 0 is
2

∂t
described by
(
− x 2 − v 2t 2 )
(b) e−( x −vt )
2
(a) e
1 −( x −vt )2 3 −( x + vt )2 1 ⎡ −( x −vt )2 −( x + vt ) ⎤
2
(c) e + e (d) e + e
4 4 2 ⎣⎢ ⎦⎥
Ans. (d)
∂2 f 2 ∂ f
2
Solution: For = v
∂t 2 ∂x 2
∂f
( x, 0 ) = 0 and f ( x, 0 ) = e− x
2

∂t
1
f = ⎡ f ( x + vt ) + f ( x − vt ) ⎤⎦
2⎣

(
1 −( x −vt )
)
2
+ e −( x + vt )
2
therefore, solution is f = e
2

NET/JRF (DEC-2015)
Q63. In the scattering of some elementary particles, the scattering cross-section σ is found to
depend on the total energy E and the fundamental constants h (Planck’s constant) and c
(the speed of light in vacuum). Using dimensional analysis, the dependence of σ on
these quantities is given by
2
hc hc ⎛ hc ⎞ hc
(a) (b) 3 / 2 (c) ⎜ ⎟ (d)
E E ⎝E⎠ E
Ans.: (c)
Solution: The dimension of σ is dimension of “Area”
h = Joul − sec
c = m / sec

                                                                                
Head office  Branch office 
 
fiziks, H.No. 40 D, G.F, Jia Sarai,  Anand Institute of Mathematics, 
 
Near IIT, Hauz Khas, New Delhi‐16  28‐B/6, Jia Sarai, Near IIT 
 
Phone: 011‐26865455/+91‐9871145498 Hauz Khas, New Delhi‐16 
                                                   
                                             Website: www.physicsbyfiziks.com                                                                                          
                                                            Email: fiziks.physics@gmail.com                                                                   31 
fiziks
Institute for NET/JRF, GATE, IIT‐JAM, JEST, TIFR and GRE in PHYSICAL SCIENCES 
 
E = Joul
2
⎛ hc ⎞
⎜ ⎟ = m dimension of area
2

⎝ ⎠
E
1
Q64. If y = , then x is
tanh ( x )

⎛ y +1 ⎞ ⎛ y −1 ⎞
(a) ln ⎜ ⎟ (b) ln ⎜ ⎟
⎝ y −1 ⎠ ⎝ y +1 ⎠

y −1 y +1
(c) ln (d) ln
y +1 y −1
Ans.: (d)
1
Solution: y =
tanh x
e x + e− x e2 x + 1
y= =
e x − e− x e2 x − 1
ye 2 x − y = e2 x + 1 ⇒ ye2 x − e2 x = 1 + y ⇒ e 2 x ( y − 1) = 1 + y

⎛ y +1 ⎞
2 x = ln ⎜ ⎟
⎝ y −1 ⎠
1
1 ⎛ y +1 ⎞ ⎛ y + 1 ⎞2
x = ln ⎜ ⎟ = ln ⎜ ⎟
2 ⎝ y −1 ⎠ ⎝ y −1⎠
z
Q65. The function of a complex variable z has
sin π z 2
(a) a simple pole at 0 and poles of order 2 at ± n for n = 1, 2,3...

(b) a simple pole at 0 and poles of order 2 at ± n and ±i n for n = 1, 2,3...

(c) poles of order 2 at ± n , n = 0,1, 2,3...


(d) poles of order 2 at ± n , n = 0,1, 2,3...

                                                                                
Head office  Branch office 
 
fiziks, H.No. 40 D, G.F, Jia Sarai,  Anand Institute of Mathematics, 
 
Near IIT, Hauz Khas, New Delhi‐16  28‐B/6, Jia Sarai, Near IIT 
 
Phone: 011‐26865455/+91‐9871145498 Hauz Khas, New Delhi‐16 
                                                   
                                             Website: www.physicsbyfiziks.com                                                                                          
                                                            Email: fiziks.physics@gmail.com                                                                   32 
fiziks
Institute for NET/JRF, GATE, IIT‐JAM, JEST, TIFR and GRE in PHYSICAL SCIENCES 
 
Ans. : (b)
z z
Solution: f ( z ) = =
sin π z 2
sin π z 2
π z2
π z2
sin π z 2
at z = 0 these is a simple pole since lim =1
z →0 π z2

( ) z
2
Also, sin π z 2 = sin nπ lim z − n .
z→ n sin π z 2
π z 2 = ± nπ z = ± n , ±i n
exists so its pole of order 2 .
+∞
Q66. The Fourier transform of f ( x ) is f ( k ) = ∫ dxeikx f ( x ) .
−∞

If f ( x ) = αδ ( x ) + βδ ′ ( x ) + γδ ′′ ( x ) , where δ ( x ) is the Dirac delta-function (and prime

denotes derivative), what is f ( k ) ?

(a) α + i β k + iγ k 2 (b) α + β k − γ k 2

(c) α − i β k − γ k 2 (d) iα + β k − iγ k 2
Ans.: (c)

Solution: f ( k ) = ∫ dx e (αδ ( x ) + βδ ′ ( x ) + γδ ′′ ( x ) )
ikx

−∞

∫ αδ ( x ) e dx = α
ikx

−∞


⎡ ∞


∫ βδ ′ ( x ) eikx dx = β ⎢ eikxδ ( x ) −∞ − ∫ ike δ ( x ) dx ⎥⎦ = −iβ k
ikx

−∞ ⎣ −∞

∫ γδ ′′ ( x ) e dx = −γ k 2
ikx

−∞

                                                                                
Head office  Branch office 
 
fiziks, H.No. 40 D, G.F, Jia Sarai,  Anand Institute of Mathematics, 
 
Near IIT, Hauz Khas, New Delhi‐16  28‐B/6, Jia Sarai, Near IIT 
 
Phone: 011‐26865455/+91‐9871145498 Hauz Khas, New Delhi‐16 
                                                   
                                             Website: www.physicsbyfiziks.com                                                                                          
                                                            Email: fiziks.physics@gmail.com                                                                   33 
fiziks
Institute for NET/JRF, GATE, IIT‐JAM, JEST, TIFR and GRE in PHYSICAL SCIENCES 
 
dx
Q67. The solution of the differential equation = 2 1 − x 2 , with initial condition x = 0 at
dt
t = 0 is

⎧ π ⎧ π
⎪⎪ sin 2t , 0 ≤ t < 4 ⎪⎪sin 2t , 0 ≤ t < 2
(a) x = ⎨ (b) x = ⎨
⎪sinh 2t , π ⎪ 1, π
t≥ t≥
⎪⎩ 4 ⎪⎩ 2

⎧ π
⎪⎪sin 2t , 0 ≤ t < 4
(c) x = ⎨ (d) x = 1 − cos 2t , t ≥ 0
⎪ 1, π
t≥
⎪⎩ 4
Ans.: (c)
dx
Solution: = 2 1 − x2
dt
dx
= 2dt sin −1 x = 2t + c
1− x 2

x = 0, t = 0 so, c = 0
x = sin 2t
x should not be greater than 1 at x = 1
π π
1 = sin 2t sin = sin 2t , t =
2 4
π
so, x = sin 2t 0≤t<
4
π
=1 t≥
4

                                                                                
Head office  Branch office 
 
fiziks, H.No. 40 D, G.F, Jia Sarai,  Anand Institute of Mathematics, 
 
Near IIT, Hauz Khas, New Delhi‐16  28‐B/6, Jia Sarai, Near IIT 
 
Phone: 011‐26865455/+91‐9871145498 Hauz Khas, New Delhi‐16 
                                                   
                                             Website: www.physicsbyfiziks.com                                                                                          
                                                            Email: fiziks.physics@gmail.com                                                                   34 
fiziks
Institute for NET/JRF, GATE, IIT‐JAM, JEST, TIFR and GRE in PHYSICAL SCIENCES 
 
Q68. The Hermite polynomial H n ( x ) satisfies the differential equation

d 2Hn dH n
2
− 2x + 2nH n ( x ) = 0
dx dx
The corresponding generating function
1
G ( t , x ) = ∑ n =0 H n ( x ) t n satisfies the equation

n!
∂ 2G ∂G ∂G ∂ 2G ∂G ∂G
(a) − 2x + 2t =0 (b) − 2x − 2t 2 =0
∂x 2
∂x ∂t ∂x 2
∂x ∂t
∂ 2G ∂G ∂G ∂ 2G ∂G ∂ 2G
(c) − 2x +2 =0 (d) − 2x +2 =0
∂x 2
∂x ∂t ∂x 2 ∂x ∂x∂t
Ans.: (a)
1
Solution: G = Hn ( x) tn
n!
1
G′ = H n′ ( x ) t n
n!
1
G ′′ = H n′′ ( x ) t n
n!
∂G 1
= H n ( x ) n t n −1
∂t n!
lets check the options one by one
∂G ∂G ∂G
− 2x + 2t =0
∂x 2
∂x ∂t
1 1 1
H n′′ ( x ) t n − 2 x H n′ ( x ) t n + 2t H n ( x ) n
n! n! n!

H n′′ ( x ) − 2 xH n′ ( x ) + 2 xH n ( x ) = 0 , which is Hermite Differential Equation.

                                                                                
Head office  Branch office 
 
fiziks, H.No. 40 D, G.F, Jia Sarai,  Anand Institute of Mathematics, 
 
Near IIT, Hauz Khas, New Delhi‐16  28‐B/6, Jia Sarai, Near IIT 
 
Phone: 011‐26865455/+91‐9871145498 Hauz Khas, New Delhi‐16 
                                                   
                                             Website: www.physicsbyfiziks.com                                                                                          
                                                            Email: fiziks.physics@gmail.com                                                                   35 
fiziks
Institute for NET/JRF, GATE, IIT‐JAM, JEST, TIFR and GRE in PHYSICAL SCIENCES 
 
8 1 1
Q69. The value of the integral ∫ 2 dx , valuated using Simpson’s rule with h = 2 is
0 x +5 3
(a) 0.565 (b) 0.620 (c) 0.698 (d) 0.736
Ans.: (a)
2
⎡ y0 + 4 ( y1 + y2 ) + 2 y2 + y4 ⎤⎦
x 1
Solution: I = y=
3⎣ x +5
2

2 ⎡1 ⎛1 1⎞ 1 1⎤ 1
= ⎢ + 4⎜ + ⎟ + 2× + ⎥ 0
3 ⎣5 ⎝9 4⎠ 21 69 ⎦ 5
2 1
2 ⎡1 ⎤
= ⎢ + 0.5734 + 0.09523 + 0.0145⎥ 9
3 ⎣5 ⎦ 4 1
2 21
= [ 0.2 + 0.5734 + 0.09523 + 0.0145] 1
3 6
31
2
= × 0.8831 = 0.5887 8 1
3 69
Q70. Consider a random walker on a square lattice. At each step the walker moves to a nearest
neighbour site with equal probability for each of the four sites. The walker starts at the
origin and takes 3 steps. The probability that during this walk no site is visited more than
one is
(a) 12 / 27 (b) 27 / 64 (c) 3 / 8 (d) 9 /16
Ans.: (d)
Solution: Total number of ways = 4 × 4 × 4
Number of preferred outcome = 4 × 3 × 3
(∵ Any four option in step-1 and only 3 option in step 2 &3 because he can not go to
previous position)
4 × 3× 3 9
probability = =
4 × 4 × 4 16

                                                                                
Head office  Branch office 
 
fiziks, H.No. 40 D, G.F, Jia Sarai,  Anand Institute of Mathematics, 
 
Near IIT, Hauz Khas, New Delhi‐16  28‐B/6, Jia Sarai, Near IIT 
 
Phone: 011‐26865455/+91‐9871145498 Hauz Khas, New Delhi‐16 
                                                   
                                             Website: www.physicsbyfiziks.com                                                                                          
                                                            Email: fiziks.physics@gmail.com                                                                   36 
fiziks
Institute for NET/JRF, GATE, IIT‐JAM, JEST, TIFR and GRE in PHYSICAL SCIENCES 
 
NET/JRF (JUNE-2016)
1
Q71. The radius of convergence of the Taylor series expansion of the function
cosh ( x )

around x = 0 , is
π
(a) ∞ (b) π (c) (d) 1
2
Ans: (c)
1 e4 z − 1
Q72. The value of the contour integral
2π i ∫
C
cosh ( z ) − 2sinh ( z )
dz around the unit circle C

traversed in the anti-clockwise direction, is


−8 ⎛1⎞
(a) 0 (b) 2 (c) (d) − tanh ⎜ ⎟
3 ⎝2⎠
Ans: (c)
e4 z − 1 e4 z − 1 e4 z − 1
Solution: f ( z ) = = z −z =
cosh z − 2sinh z e + e
2
− e z − e− z
ez 3
− + e− z
2 2
( )

⇒ f (z) =
(
2e z e 4 z − 1 ) = 2 (e 5z
− ez )
3−e 2z
3−e 2z

ln 3
For pole at z = z0 ,3 − e 2 z0 =0 ⇒ e 2 z0 = 3 ⇒ z0 =
2
It has simple pole at z0

a−1 = lim ( z − z0 ) f ( z ) = lim ( z − z0 )


(
2 e5 z − e z )
z → z0 z → z0 3−e 2z

( z − z0 ) × 2 ( 5e5 z − e z ) + 2 ( e5 z − e z ) ×1 ⎛ e5 z0 − e z0 ⎞
⇒ a−1 = lim = −⎜ ⎟
z → z0 −2e 2 z ⎝ e
2 z0


( 3) − 3 ⎞⎟
5
⎛9 3− 3⎞
⇒ a−1 = − ⎜⎜
8
⎟ = − ⎜
⎜ ⎟
⎟ = −
⎜ 3 ⎟ ⎝ 3 ⎠ 3
⎝ ⎠

                                                                                
Head office  Branch office 
 
fiziks, H.No. 40 D, G.F, Jia Sarai,  Anand Institute of Mathematics, 
 
Near IIT, Hauz Khas, New Delhi‐16  28‐B/6, Jia Sarai, Near IIT 
 
Phone: 011‐26865455/+91‐9871145498 Hauz Khas, New Delhi‐16 
                                                   
                                             Website: www.physicsbyfiziks.com                                                                                          
                                                            Email: fiziks.physics@gmail.com                                                                   37 
fiziks
Institute for NET/JRF, GATE, IIT‐JAM, JEST, TIFR and GRE in PHYSICAL SCIENCES 
 
1 1 8
∫ f ( z ) dz = × 2π i ∑ Residue = −
2π i 2π i 3

Q73. The Gauss hypergeometric function F ( a, b, c, z ) , defined by the Taylor series expansion

around z = 0 as F ( a, b, c, z ) =
∞ a ( a + 1) ... ( a + n − 1) b ( b + 1) ... ( b + n − 1)
∑n =0 c ( c + 1) ... ( c + n − 1) n !
zn ,

satisfies the recursion relation


d c
(a) F ( a, b, c; z ) = F ( a − 1, b − 1, c − 1; z )
dz ab
d c
(b) F ( a, b, c; z ) = F ( a + 1, b + 1, c + 1; z )
dz ab
d ab
(c) F ( a, b, c; z ) = F ( a − 1, b − 1, c − 1; z )
dz c
d ab
(d) F ( a, b, c; z ) = F ( a + 1, b + 1, c + 1; z )
dz c
Ans: (d)
dF ∞ a ( a + 1) ... ( a + n − 1) b ( b + 1) ... ( b + n − 1) n −1
Solution: =∑ nz
dz n =0 c ( c + 1) ... ( c + n − 1) n !
∞ a ( a + 1) ... ( a + n − 1) b ( b + 1) ... ( b + n − 1) z n −1
=∑
n =0 c ( c + 1) ... ( c + n − 1) n −1

ab ( a + 1) ... ( a + n − 1)( b + 1) ... ( b + n − 1) z n −1


=
c
∑ ( c + 1) ... ( c + n − 1) n −1

ab ∞ ( a + 1) ... ⎡⎣ a + ( n − 1) − 1⎤⎦ ( b + 1) ... ⎡⎣b + ( n − 1) − 1⎤⎦ z n −1


= ∑
c n =0 ( c + 1) ... ⎡⎣c + ( n − 1) − 1⎤⎦ n −1

dF ab
= F ( a + 1, b + 1, c + 1, z )
dz c

                                                                                
Head office  Branch office 
 
fiziks, H.No. 40 D, G.F, Jia Sarai,  Anand Institute of Mathematics, 
 
Near IIT, Hauz Khas, New Delhi‐16  28‐B/6, Jia Sarai, Near IIT 
 
Phone: 011‐26865455/+91‐9871145498 Hauz Khas, New Delhi‐16 
                                                   
                                             Website: www.physicsbyfiziks.com                                                                                          
                                                            Email: fiziks.physics@gmail.com                                                                   38 
fiziks
Institute for NET/JRF, GATE, IIT‐JAM, JEST, TIFR and GRE in PHYSICAL SCIENCES 
 
Q74. Let X and Y be two independent random variables, each of which follow a normal
distribution with the same standard deviation σ , but with means + μ and − μ ,
respectively. Then the sum X + Y follows a
(a) distribution with two peaks at ± μ and mean 0 and standard deviation σ 2
(b) normal distribution with mean 0 and standard deviation 2σ
(c) distribution with two peaks at ± μ and mean 0 and standard deviation 2σ

(d) normal distribution with mean 0 and standard deviation σ 2


Ans: (d)
Solution: μ ′ = μ x + μ y = μ − μ = 0

σ 12 = σ x2 + σ y2 = σ 2 + σ 2

σ ′ = 2σ
Q75. Using dimensional analysis, Planck defined a characteristic temperature TP from powers

of the gravitational constant G , Planck’s constant h , Boltzmann constant k B and the

speed of light c in vacuum. The expression for TP is proportional to

hc 5 hc3 G hk B2
(a) (b) (c) (d)
k B2G k B2G hc 4 k B2 Gc 3

Ans: (a)
E ML2T −2
Solution: E = hν ⇒ h = = = ML2T −1
ν T −1

E ML2T −2
E = k BT ⇒ k B = = = ML2T −2TP−1
T TP

m1m2 MLT −2 × L2
F =G 2
⇒ G = 2
⇒ G = M −1 L3T −2
r M

( )
5
hc 5 ML2T −1 × LT −1 ML7T −6
= = = TP2 = TP
k B2G M 2 L4T −4TP−2 × M −1 L3T −2 7 −6 −2
ML T TP

                                                                                
Head office  Branch office 
 
fiziks, H.No. 40 D, G.F, Jia Sarai,  Anand Institute of Mathematics, 
 
Near IIT, Hauz Khas, New Delhi‐16  28‐B/6, Jia Sarai, Near IIT 
 
Phone: 011‐26865455/+91‐9871145498 Hauz Khas, New Delhi‐16 
                                                   
                                             Website: www.physicsbyfiziks.com                                                                                          
                                                            Email: fiziks.physics@gmail.com                                                                   39 
fiziks
Institute for NET/JRF, GATE, IIT‐JAM, JEST, TIFR and GRE in PHYSICAL SCIENCES 
 
Q76. What is the Fourier transform ∫ dxe f ( x ) of
ikx


dn
f ( x) = δ ( x) + ∑ n
δ ( x)
n =1 dx

where δ ( x ) is the Dirac delta-function?

1 1 1 1
(a) (b) (c) (d)
1− i k 1+ i k k +i k −i
Ans: (b)
∞ ∞ ∞
dn dn
Solution: f ( x ) = δ ( x ) + ∑ n
δ ( x ) = ∑ n
δ ( x ) = ∑ δ (n) ( x )
n =1 dx n =0 dx n=0

∵ F ⎡⎣δ ( x ) ⎤⎦ = 1 ⇒ F ⎡⎣δ ( n ) ( x ) ⎤⎦ = ( −ik ) F ⎡⎣δ ( x ) ⎤⎦ = ( −ik )


n n


∵ f ( x ) = ∑ δ ( n) ( x )
n =0


1 1
⇒ F ⎡⎣ f ( x ) ⎤⎦ = ∑ ( −ik ) = 1 − ik + ( ik ) − ( ik ) + .... =
n 2 3
=
n =0 1 − ( −ik ) 1 + ik

Q77. In finding the roots of the polynomial f ( x ) = 3 x 3 − 4 x − 5 using the iterative Newton-

Raphson method, the initial guess is taken to be x = 2 . In the next iteration its value is
nearest to
(a) 1.671 (b) 1.656 (c) 1.559 (d) 1.551
Ans: (b)
Solution: f ( x ) = 3 x 3 − 4 x − 5 ; f ′ ( x ) = 9 x 2 − 4

3xn3 − 4 xn − 5 3 x03 − 4 x0 − 5
xn +1 = xn − ⇒ x1 = x0 −
9 xn2 − 4 9 x02 − 4
3× 8 − 4 × 2 − 5 11
Let x0 = 2 ⇒ x1 = 2 − = 2− ⇒ x1 = 1.656
9× 4 − 4 32

                                                                                
Head office  Branch office 
 
fiziks, H.No. 40 D, G.F, Jia Sarai,  Anand Institute of Mathematics, 
 
Near IIT, Hauz Khas, New Delhi‐16  28‐B/6, Jia Sarai, Near IIT 
 
Phone: 011‐26865455/+91‐9871145498 Hauz Khas, New Delhi‐16 
                                                   
                                             Website: www.physicsbyfiziks.com                                                                                          
                                                            Email: fiziks.physics@gmail.com                                                                   40 
fiziks
Institute for NET/JRF, GATE, IIT‐JAM, JEST, TIFR and GRE in PHYSICAL SCIENCES 
 
CLASSICAL MECHANICS SOLUTIONS

NET/JRF (JUNE-2011)
b
Q1. A particle of unit mass moves in a potential V ( x ) = ax 2 + , where a and b are positive
x2
constants. The angular frequency of small oscillations about the minimum of the potential
is
(a) 8b (b) 8a (c) 8a / b (d) 8b / a
Ans: (b)
1
b ∂V 2b ⎛ b ⎞4
Solution: V ( x ) = ax + 2 ⇒ = 0 ⇒ 2ax − 3 = 0 ⇒ ax 4 − b = 0 ⇒ ⇒ x0 = ⎜ ⎟ .
2

x ∂x x ⎝a⎠

k ∂2V
Since ω = , m = 1 and k = 2 where xo is stable equilibrium point.
m ∂x x = x
0

1
∂V 6b 2
6b ⎛ b ⎞4
Hence k = 2 = 2a + 4 = 2a + = 8a at x = x0 = ⎜ ⎟ .
∂x x0 b ⎝a⎠
a
Thus ω = 8a .

Q2. The acceleration due to gravity (g) on the surface of Earth is approximately 2.6 times that
on the surface of Mars. Given that the radius of Mars is about one half the radius of
Earth, the ratio of the escape velocity on Earth to that on Mars is approximately
(a) 1.1 (b) 1.3 (c) 2.3 (d) 5.2
Ans: (c)
Solution: Escape velocity = 2 gR

Escape velocity of Earth ge R e R g


= = 2.3 where e = 2 and e = 2.6.
Escape velocity of Mass gm R m Rm gm

                                                                                
Head office  Branch office 
 
fiziks, H.No. 40 D, G.F, Jia Sarai,  Anand Institute of Mathematics, 
 
Near IIT, Hauz Khas, New Delhi‐16  28‐B/6, Jia Sarai, Near IIT 
 
Phone: 011‐26865455/+91‐9871145498 Hauz Khas, New Delhi‐16 
                                                   
                                             Website: www.physicsbyfiziks.com                                                                                          
                                                            Email: fiziks.physics@gmail.com                                                                     41 
fiziks
Institute for NET/JRF, GATE, IIT‐JAM, JEST, TIFR and GRE in PHYSICAL SCIENCES 
 
Q3. The Hamiltonian of a system with n degrees of freedom is given
by H (q1 ,.....q n ; p1 ,....... p n ; t ) , with an explicit dependence on the time t. Which of the
following is correct?
(a) Different phase trajectories cannot intersect each other.
(b) H always represents the total energy of the system and is a constant of the motion.
(c) The equations qi = ∂H / ∂pi , pi = −∂H / ∂qi are not valid since H has explicit time
dependence.
(d) Any initial volume element in phase space remains unchanged in magnitude under
time evolution.
Ans: (a)
Q4. The Lagrangian of a particle of charge e and mass m in applied electric and magnetic
1
fields is given by L = mv 2 + eA ⋅ v − eφ , where A and φ are the vector and scalar
2
potentials corresponding to the magnetic and electric fields, respectively. Which of the
following statements is correct?
(a) The canonically conjugate momentum of the particle is given by p = mv

p2 e
(b) The Hamiltonian of the particle is given by H = + A ⋅ p + eφ
2m m
(c) L remains unchanged under a gauge transformation of the potentials
(d) Under a gauge transformation of the potentials, L changes by the total time derivative
Ans: (d)
∂L ρ
Solution: 2
V+ =−
∂t ε0

Q5. Consider the decay process τ − → π − + ν τ in the rest frame of the τ-. The masses of the

τ − , π − and ν τ are Mτ, Mπ and zero respectively.

A. The energy of π − is

(a)
(M 2
τ − M π2 )c 2
(b)
(M 2
τ + M π2 )c 2
(c) (M τ − M π )c 2 (d) Mτ M π c2
2M τ 2M τ

                                                                                
Head office  Branch office 
 
fiziks, H.No. 40 D, G.F, Jia Sarai,  Anand Institute of Mathematics, 
 
Near IIT, Hauz Khas, New Delhi‐16  28‐B/6, Jia Sarai, Near IIT 
 
Phone: 011‐26865455/+91‐9871145498 Hauz Khas, New Delhi‐16 
                                                   
                                             Website: www.physicsbyfiziks.com                                                                                          
                                                            Email: fiziks.physics@gmail.com                                                                     42 
fiziks
Institute for NET/JRF, GATE, IIT‐JAM, JEST, TIFR and GRE in PHYSICAL SCIENCES 
 
Ans: (b)

Solution: τ − → π − +ν τ

From conservation of energy M τ c2 = E π + E ν .

E 2π = p 2 c2 + M π2 c4 and E ν2 = p 2c2 since momentum of π − and ν τ is same.

M 2π c4
M τ c2 = E π + E ν , M π 2 c4 = E π2 − E ν2 ⇒ E π − E ν =
M τc2

Eπ − Eν =
M π c2
and E π + E ν = M τ c ⇒ E π =
2
M 2τ + Mπ2 c2
.
( )
Mτ 2Mτ

B. The velocity of π − is

(a)
(M τ− M π2 )c
2
(b)
(M 2
τ+ M π2 )c
(c)
Mπ c
(d)
Mτ c
M τ2 + M π2 M τ2 − M π2 Mτ Mπ
Ans: (a)
Solution: Velocity of π −

Eπ =
(M 2
τ + M π2 ) c 2
=
M πc 2 ⎛ v2 ⎞
⇒ ⎜1 − 2 ⎟ =
4 M π2 M τ2
⎝ c ⎠ ( M τ2 + M π2 )
2
2M τ v2
1−
c2
v2 4M π2 M τ2 v 2 M τ4 + M π4 + 2 M τ2 M π2 − 4 M π2 M τ2 ⎛ M τ2 − M π2 ⎞
⇒ = 1 − ⇒ = ⇒ v = ⎜ 2 2 ⎟
c.
( M τ2 + M π2 ) c (M 2 + M 2 ) ⎝ Mτ + Mπ ⎠
2
c2 2 2
τ π

Q6. The Hamiltonian of a particle of unit mass moving in the xy-plane is given to be:
1 2 1 2
H = xp x − yp y − x + y in suitable units. The initial values are given to be
2 2

and ( p x (0 ), p y (0 )) = ⎜ ,− ⎟ . During the motion, the curves traced out


⎛1 1⎞
(x(0), y(0)) = (1,1)
⎝2 2⎠
by the particles in the xy-plane and the p x p y – plane are

(a) both straight lines


(b) a straight line and a hyperbola respectively
(c) a hyperbola and an ellipse, respectively
(d) both hyperbolas
                                                                                
Head office  Branch office 
 
fiziks, H.No. 40 D, G.F, Jia Sarai,  Anand Institute of Mathematics, 
 
Near IIT, Hauz Khas, New Delhi‐16  28‐B/6, Jia Sarai, Near IIT 
 
Phone: 011‐26865455/+91‐9871145498 Hauz Khas, New Delhi‐16 
                                                   
                                             Website: www.physicsbyfiziks.com                                                                                          
                                                            Email: fiziks.physics@gmail.com                                                                     43 
fiziks
Institute for NET/JRF, GATE, IIT‐JAM, JEST, TIFR and GRE in PHYSICAL SCIENCES 
 
Ans: (d)
1 2 1 2
Solution: H = xp x − yp y − x + y
2 2 .

Solving Hamiltonion equation of motion


∂H ∂H
= − p x ⇒ p x − x = −p x and = −p y ⇒ −p y + y = −p y .
∂x ∂y
∂H ∂H
= x ⇒ x = x and = y ⇒ −y = y .
∂p x ∂p y

After solving these four differential equation and eliminating time t and using boundary
1 1 1
condition one will get ⇒ x = and p x =
y 2 py

NET/JRF (DEC-2011)
Q7. A double pendulum consists of two point masses m attached by strings of length l as
shown in the figure: The kinetic energy of the pendulum is

(a)
1 2 2
2
[
ml θ 1 + θ 22 ]
l
(b)
1 2
2
[
ml 2θ 12 + θ 22 + 2θ 1θ 2 cos(θ 1 − θ 2 ) ] θ1

(c)
1 2 2
2
[
ml θ1 + 2θ 22 + 2θ 1θ 2 cos(θ1 − θ 2 ) ] m

l
(d)
1 2
2
[
ml 2θ 12 + θ 22 + 2θ 1θ 2 cos(θ 1 + θ 2 ) ] θ2

Ans: (b) m

Solution: Let co-ordinate ( x1 , y1 ) and ( x 2 , y 2 ) . K .E. =


1
2
( 1
)
m x12 + y12 + m x 22 + y 22
2
( )
x1 = l sin θ 1 , y1 = l cos θ1 ⇒ x1 = l cos θ1θ1 , y1 = −l sin θ1θ1
x 2 = l sin θ 1 + l sin θ 2 , y 2 = l cos θ1 + l cos θ 2

(
⇒ x 2 = l cos θ1θ1 + l cos θ 2 θ2 , y 2 = l − sin θ1θ1 + l (− sin θ 2 )θ 2 )
                                                                                
Head office  Branch office 
 
fiziks, H.No. 40 D, G.F, Jia Sarai,  Anand Institute of Mathematics, 
 
Near IIT, Hauz Khas, New Delhi‐16  28‐B/6, Jia Sarai, Near IIT 
 
Phone: 011‐26865455/+91‐9871145498 Hauz Khas, New Delhi‐16 
                                                   
                                             Website: www.physicsbyfiziks.com                                                                                          
                                                            Email: fiziks.physics@gmail.com                                                                     44 
fiziks
Institute for NET/JRF, GATE, IIT‐JAM, JEST, TIFR and GRE in PHYSICAL SCIENCES 
 
Put the value of x1 , y1 , x 2 , y 2 in K.E equation, one will get
1
T= m ⎡⎣ 2θ12 + θ 22 + 2θ1θ 2 cos (θ1 − θ 2 ) ⎤⎦ .
2
Q8. A constant force F is applied to a relativistic particle of rest mass m. If the particle starts
from rest at t = 0, its speed after a time t is

(a) Ft / m
⎛ Ft ⎞
(b) c tanh⎜ ⎟ (
(c) c 1 − e − Ft / mc ) (d)
Fct
⎝ mc ⎠ F t + m2c 2
2 2

Ans: (d)
dp
Solution: = F ⇒ p = Ft + c .
dt
⎛F⎞
mu ⎜ ⎟t
At t=0, p=0 so c=0 ⇒ p = Ft ⇒ = Ft ⇒ u = ⎝m⎠ .
u2 ⎛ Ft ⎞
2
1− 2 1+ ⎜ ⎟
c ⎝ mc ⎠
Q9. The potential of a diatomic molecule as a function of the distance r between the atoms is
a b
given by V (r ) = − 6
+ 12 . The value of the potential at equilibrium separation between
r r
the atoms is:
(a) − 4 a 2 / b (b) − 2 a 2 / b (c) − a 2 / 2b (d) − a 2 / 4b
Ans: (d)
a b ∂V a 12b 1 ⎡ 12b ⎤
Solution: V (r ) = − + 12 , for equilibrium = 0 ⇒ − ( −6 ) 7 − 13 = 0 ⇒ 7 ⎢6a − 6 ⎥ = 0
r 6
r ∂r r r r ⎣ r ⎦
1 1
12b ⎛ 12b ⎞ 6 ⎛ 2b ⎞ 6
⇒ 6a − 6 = 0 ⇒ r = ⎜ ⎟ ⇒r =⎜ ⎟
r ⎝ 6a ⎠ ⎝ a ⎠
⎛ 1

⎜ ⎛ 2b ⎞ 6
⎟ a b a2 a2 a2
⇒V r =⎜ ⎟ =− + =− + =− .
⎜ ⎝ a ⎠ ⎟ ⎛ 2b ⎞ ⎛ 2b ⎞ 2 2b 4b 4b
⎝ ⎠ ⎜ ⎟ ⎜ ⎟
⎝ a ⎠ ⎝ a ⎠

                                                                                
Head office  Branch office 
 
fiziks, H.No. 40 D, G.F, Jia Sarai,  Anand Institute of Mathematics, 
 
Near IIT, Hauz Khas, New Delhi‐16  28‐B/6, Jia Sarai, Near IIT 
 
Phone: 011‐26865455/+91‐9871145498 Hauz Khas, New Delhi‐16 
                                                   
                                             Website: www.physicsbyfiziks.com                                                                                          
                                                            Email: fiziks.physics@gmail.com                                                                     45 
fiziks
Institute for NET/JRF, GATE, IIT‐JAM, JEST, TIFR and GRE in PHYSICAL SCIENCES 
 
Q10. Two particles of identical mass move in circular orbits under a central
1 2
potential V (r ) = kr . Let l1 and l2 be the angular momenta and r1, r2 be the radii of the
2
orbits respectively. If l1/l2 = 2, the value of r1 / r2 is:
(a) 2 (b) 1 / 2 (c) 2 (d) 1 / 2
Ans: (a)
J2 1
Solution: Veff = 2
+ kr 2 where J is angular momentum.
2mr 2
∂Veff J2
Condition for circular orbit = 0 ⇒ − 3 + kr = 0 ⇒ J 2 ∝ r 4 ⇒ J ∝ r 2 .
∂r mr
2
J ⎛r ⎞ r J r J
Thus 1 = ⎜ 1 ⎟ ⇒ 1 = 1 ⇒ 1 = 2 since 1 = 2 .
J 2 ⎝ r2 ⎠ r2 J2 r2 J2

Q11. A particle of mass m moves inside a bowl. If the surface of the bowl is given by the

equation z =
1
2
( )
a x 2 + y 2 , where a is a constant, the Lagrangian of the particle is

(a)
1
2
(
m r 2 + r 2φ 2 − gar 2 ) (b)
1
2
[( )
m 1 + a 2 r 2 r 2 + r 2φ 2 ]
(c)
1
2
(
m r 2 + r 2θ 2 + r 2 sin 2 θφ 2 − gar 2 ) (d)
1
2
[( )
m 1 + a 2 r 2 r 2 + r 2φ 2 − gar 2 ]
Ans: (d)

Solution: L =
1
2
(
m x 2 + y 2 + z 2 − mgz ) where z =
1
2
(
a x2 + y2 . )
1
It has cylindrical symmetry. Thus x = r cos φ , y = r sin φ , z = a (r2 ) .
2
x = r cos φ − r sin φφ , y = r sin φ + r cos φφ and z = a ( rr ) .

1
So L = m ⎡⎣(1 + a 2 r 2 ) r 2 + r 2 φ2 − gar 2 ⎤⎦ .
2

                                                                                
Head office  Branch office 
 
fiziks, H.No. 40 D, G.F, Jia Sarai,  Anand Institute of Mathematics, 
 
Near IIT, Hauz Khas, New Delhi‐16  28‐B/6, Jia Sarai, Near IIT 
 
Phone: 011‐26865455/+91‐9871145498 Hauz Khas, New Delhi‐16 
                                                   
                                             Website: www.physicsbyfiziks.com                                                                                          
                                                            Email: fiziks.physics@gmail.com                                                                     46 
fiziks
Institute for NET/JRF, GATE, IIT‐JAM, JEST, TIFR and GRE in PHYSICAL SCIENCES 
 
Q12. A planet of mass m moves in the inverse square central force field of the Sun of mass
M . If the semi-major and semi-minor axes of the orbit are a and b , respectively, the
total energy of the planet is:
GMm ⎛1 1⎞
(a) − (b) − GMm⎜ + ⎟
a+b ⎝a b⎠
GMm ⎛ 1 1 ⎞ ⎛ a−b ⎞
(c) − ⎜ − ⎟ (d) − GMm⎜⎜ ⎟

⎝ (a + b )
2
a ⎝b a⎠ ⎠
Ans: (a)
Solution: Assume Sun is at the centre of elliptical orbit.
1 2 GMm 1 2 GMm
Conservation of energy mv1 − = mv2 −
2 a 2 b v2

Conservation of momentum L = mv1a = mv2b b


v1
s a
⎛ a⎞
v2 = v1 ⎜ ⎟
⎝ b⎠
1 2 1 2 GMm GMm 1 ⎛ a2 ⎞ ⎛b−a⎞
mv1 − mv2 = − ⇒ m ⎜ v12 − v12 2 ⎟ = GMm ⎜ ⎟
2 2 a b 2 ⎝ b ⎠ ⎝ ab ⎠

1 2 ⎛ b2 − a 2 ⎞ ⎛ b − a⎞ 1 ⎛b⎞ 1
mv1 ⎜ ⎟ = GMm ⎜ ⎟ ⇒ mv12 = GMm ⎜ ⎟ ⋅
⎝ b ⎠ ⎝ ab ⎠ ⎝ a ⎠ (b + a )
2
2 2

1 2 GMm b 1 GMm
E= mv1 − = GMm −
2 a a (b + a) a

GMm ⎛ b ⎞ GMm ⎛ b − b − a ⎞ GMm


= ⎜⎜ − 1⎟ = ⎜ ⎟ =−
a ⎝ ( b + a ) ⎟⎠ a ⎜⎝ ( b + a ) ⎟⎠ (b + a )
Q13. An annulus of mass M made of a material of uniform density has inner and outer radii a
and b respectively. Its principle moment of inertia along the axis of symmetry
perpendicular to the plane of the annulus is:

(a)
1
M 2
(
b4 + a4 ) (b)
1
(
Mπ b 2 − a 2 )
2 (
b − a2 ) 2

M (b 2 − a 2 ) M (b 2 + a 2 )
1 1
(c) (d)
2 2
                                                                                
Head office  Branch office 
 
fiziks, H.No. 40 D, G.F, Jia Sarai,  Anand Institute of Mathematics, 
 
Near IIT, Hauz Khas, New Delhi‐16  28‐B/6, Jia Sarai, Near IIT 
 
Phone: 011‐26865455/+91‐9871145498 Hauz Khas, New Delhi‐16 
                                                   
                                             Website: www.physicsbyfiziks.com                                                                                          
                                                            Email: fiziks.physics@gmail.com                                                                     47 
fiziks
Institute for NET/JRF, GATE, IIT‐JAM, JEST, TIFR and GRE in PHYSICAL SCIENCES 
 
Ans: (d)
Q14. The trajectory on the zpz-plane (phase-space trajectory) of a ball bouncing perfectly
elastically off a hard surface at z = 0 is given by approximately by (neglect friction):
(a) (b)
PZ PZ

z z

PZ
PZ
(c) d)

z
z

Ans: (a)
Pz2 P2
Solution: H = + mgz and E = z + mgz .
2m 2m

                                                                                
Head office  Branch office 
 
fiziks, H.No. 40 D, G.F, Jia Sarai,  Anand Institute of Mathematics, 
 
Near IIT, Hauz Khas, New Delhi‐16  28‐B/6, Jia Sarai, Near IIT 
 
Phone: 011‐26865455/+91‐9871145498 Hauz Khas, New Delhi‐16 
                                                   
                                             Website: www.physicsbyfiziks.com                                                                                          
                                                            Email: fiziks.physics@gmail.com                                                                     48 
fiziks
Institute for NET/JRF, GATE, IIT‐JAM, JEST, TIFR and GRE in PHYSICAL SCIENCES 
 
NET/JRF (JUNE-2012)

Q15. The bob of a simple pendulum, which undergoes small oscillations, is immersed in water.
Which of the following figures best represents the phase space diagram for the
pendulum?
(a) (b)
p p

x x

(c) (d) p
p

x x

.
Ans: (d)
Solution: When simple pendulum oscillates in water it is damped oscillation so amplitude
continuously decrease and finally it stops.
Q16. Two events separated by a (spatial) distance 9 × 109m, are simultaneous in one inertial
frame.The time interval between these two events in a frame moving with a constant
speed 0.8 c (where the speed of light c = 3 × 108 m/s) is
(a) 60 s (b) 40 s (c) 20 s (d) 0 s
Ans: (b)
Solution: x 2' − x1' = 9 × 10 9 m and t 2' − t1' = 0 . Then

                                                                                
Head office  Branch office 
 
fiziks, H.No. 40 D, G.F, Jia Sarai,  Anand Institute of Mathematics, 
 
Near IIT, Hauz Khas, New Delhi‐16  28‐B/6, Jia Sarai, Near IIT 
 
Phone: 011‐26865455/+91‐9871145498 Hauz Khas, New Delhi‐16 
                                                   
                                             Website: www.physicsbyfiziks.com                                                                                          
                                                            Email: fiziks.physics@gmail.com                                                                     49 
fiziks
Institute for NET/JRF, GATE, IIT‐JAM, JEST, TIFR and GRE in PHYSICAL SCIENCES 
 
⎛ ⎞ ⎛ ⎞
⎜ t ' + v x' ⎟ ⎜ t1 + v x' ⎟
⎜ 2 c2 2 ⎟ ⎜ 1 c2 1 ⎟
t 2 − t1 = ⎜ − ⇒ t − t =
t 2' − t1'
+
v x 2' − x1'
=
v x 2' − x1'
.
( ) ( )
2 ⎟ ⎜ 2 ⎟
2 1 2 2
⎜ 1− v ⎟ ⎜ 1− v ⎟ v 2 c v 2 c v 2

⎜ 2 ⎟ ⎜ 2 ⎟
1− 2 1− 2 1− 2
⎝ c ⎠ ⎝ c ⎠ c c c
Put v = 0.8c ⇒ t 2 − t1 ≅ 40 sec

x2
Q17. If the Lagrangian of a particle moving in one dimensions is given by L = − V (x ) the
2x
Hamiltonian is
1 x2 1 p2
(a) xp 2 + V ( x ) (b) + V (x ) (c) x 2 + V (x ) (d) + V (x )
2 2x 2 2x
Ans: (a)
∂L x
Solution: Since H = p x x − L and = px ⇒ = px ⇒ x = px x .
∂x x

( px x )
2
x2 p x2 x
H = px x − + V ( x ) ⇒ H = px × px x − + V(x) ⇒ H = + V (x ) .
2x 2x 2
Q18. A horizontal circular platform mutes with a constant angular velocity Ω directed
vertically upwards. A person seated at the centre shoots a bullet of mass m horizontally
with speed v. The acceleration of the bullet, in the reference frame of the shooter, is
(a) 2v Ω to his right (b) 2v Ω to his left
(c) v Ω to his right (d) v Ω to his left
Ans: (a)

Solution: Velocity of bullet = vˆj , Angular velocity= Ωk̂ . There will be coriolis

force F = 2m v × Ω . ( )
F = 2mΩviˆ ⇒ a = 2vΩiˆ .

Q19. { }
The Poisson bracket r , p has the value

(a) r p (b) rˆ ⋅ pˆ (c) 3 (d) 1

Ans: (b)

                                                                                
Head office  Branch office 
 
fiziks, H.No. 40 D, G.F, Jia Sarai,  Anand Institute of Mathematics, 
 
Near IIT, Hauz Khas, New Delhi‐16  28‐B/6, Jia Sarai, Near IIT 
 
Phone: 011‐26865455/+91‐9871145498 Hauz Khas, New Delhi‐16 
                                                   
                                             Website: www.physicsbyfiziks.com                                                                                          
                                                            Email: fiziks.physics@gmail.com                                                                     50 
fiziks
Institute for NET/JRF, GATE, IIT‐JAM, JEST, TIFR and GRE in PHYSICAL SCIENCES 
 
Solution: r = xiˆ + yjˆ + zkˆ , r = ( x 2 + y 2 + z 2 ) , p = p xiˆ + p y ˆj + p z kˆ ,
1/ 2

p = ( px2 + p y2 + pz2 )
1/ 2

⎛∂ r ∂ p ∂ r ∂ p ⎞ ⎛∂ r ∂ p ∂ r ∂ p ⎞ ⎛∂ r ∂ p ∂ r ∂ p ⎞
{r , p}= ⎜ ⋅ − ⋅ ⎟ + ⎜⎜ ⋅ − ⋅ ⎟⎟ +⎜ ⋅ − ⋅ ⎟
⎝ ∂x ∂px ∂px ∂x ⎠ ⎝ ∂y ∂p y ∂p y ∂y ⎠ ⎝ ∂z ∂pz ∂pz ∂y ⎠

x px y p y z pz r⋅p
+ + ⇒ = ( rˆ ⋅ pˆ )
r p r p r p r p

Q20. Consider the motion of a classical particle in a one dimensional double-well

potential V ( x ) =
1 2
4
( 2
)
x − 2 . If the particle is displaced infinitesimally from the minimum

on the x-axis (and friction is neglected), then


(a) the particle will execute simple harmonic motion in the right well with an angular

frequency ω = 2

(b) the particle will execute simple harmonic motion in the right well with an angular

frequency ω =2

(c) the particle will switch between the right and left wells

(d) the particle will approach the bottom of the right well and settle there

Ans: (b)

Solution: V ( x ) = (x − 2)2 ⇒ ∂V = 2 x 2 − 2 × 2 x = 0 ⇒ x = 0 , x = ± 2 .
1 2
( )
4 ∂x 4
∂ 2V ∂ 2V
= 3 x 2
− 2 . At x = 0 , < 0 so V is maximum. Thus it is unstable point
∂x 2 ∂x 2

∂ 2V
∂ 2V ∂x 2 x = x0
= 4 and it is stable equilibrium point with ω = =2 ∵ μ = 1.
∂x 2 x=± 2
μ

                                                                                
Head office  Branch office 
 
fiziks, H.No. 40 D, G.F, Jia Sarai,  Anand Institute of Mathematics, 
 
Near IIT, Hauz Khas, New Delhi‐16  28‐B/6, Jia Sarai, Near IIT 
 
Phone: 011‐26865455/+91‐9871145498 Hauz Khas, New Delhi‐16 
                                                   
                                             Website: www.physicsbyfiziks.com                                                                                          
                                                            Email: fiziks.physics@gmail.com                                                                     51 
fiziks
Institute for NET/JRF, GATE, IIT‐JAM, JEST, TIFR and GRE in PHYSICAL SCIENCES 
 
Q21. What is proper time interval between the occurrence of two events if in one inertial frame
events are separated by 7.5 ×108m and occur 6.5 s a part?
(a) 6.50 s (b) 6.00 s (c) 5.75 s (d) 5.00 s
Ans: There is not possible to calculate proper time interval
Solution: x 2 − x1 = 7.5 × 108 m and t 2 − t1 = 6.5 s .

⎛ v ⎞ ⎛ v ⎞ v
2 ( 2
⎜ t2 − 2 x2 ⎟ ⎜ t1 − 2 x1 ⎟ t − t x − x1 )
t2' − t1' = ⎜ c ⎟−⎜ c ( )
⎟= 2 1 − c .
⎜ v2 ⎟ ⎜ v2 ⎟ v2 v2
⎜ 1− 2 ⎟ ⎜ 1− 2 ⎟ 1− 2 1− 2
⎝ c ⎠ ⎝ c ⎠ c c
In this problem v is not given so its not possible to calculate proper time interval.

NET/JRF (DEC-2012)

Q22. A solid cylinder of height H, radius R and density ρ, floats vertically on the surface of a
liquid of density ρ 0 . The cylinder will be set into oscillatory motion when a small
instantaneous downward force is applied. The frequency of oscillation is

ρg ρ g ρg ρ0 g
(a) (b) (c) (d)
ρ0 H ρ0 H ρ0H ρH

Ans: (d)
Solution: From Newton’s law of motion ma = mg − ρ0 Agh where A is area of cross section,

m = ρ AH .

ρ0 gh ρ0 g
⇒ ρ AHa = ρ AHg − ρ0 Agh ⇒ a = 1 − ⇒ω=
ρH ρH
Q23. Three particles of equal mass (m) are connected by two identical massless springs of
stiffness constant (K) as shown in the figure
K K
m m m
If x1, x2 and x3 denote the horizontal displacement of the masses from their respective
equilibrium positions the potential energy of the system is
                                                                                
Head office  Branch office 
 
fiziks, H.No. 40 D, G.F, Jia Sarai,  Anand Institute of Mathematics, 
 
Near IIT, Hauz Khas, New Delhi‐16  28‐B/6, Jia Sarai, Near IIT 
 
Phone: 011‐26865455/+91‐9871145498 Hauz Khas, New Delhi‐16 
                                                   
                                             Website: www.physicsbyfiziks.com                                                                                          
                                                            Email: fiziks.physics@gmail.com                                                                     52 
fiziks
Institute for NET/JRF, GATE, IIT‐JAM, JEST, TIFR and GRE in PHYSICAL SCIENCES 
 
1
[
(a) K x12 + x 22 + x32
2
] 1
(b) K x12 + x 22 + x32 − x 2 ( x1 + x3 )
2
[ ]
(c)
1
2
[
K x12 + 2 x 22 + x32 − 2 x 2 ( x1 + x3 ) ] (d)
1
2
[
K x12 + 2 x 22 − 2 x 2 ( x1 + x3 ) ]
Ans: (c)
1 1
K ( x2 − x1 ) + K ( x3 − x2 ) ,
2 2
Solution: V =
2 2
1 1 1
V=
2
( ) ( )
K x22 + x12 − 2 x2 x1 + K x32 + x22 − 2 x3 x2 ⇒ V = K ⎡⎣ x12 + 2 x22 + x32 − 2 x2 ( x1 + x3 ) ⎤⎦
2 2
Q24. A planet of mass m moves in the gravitational field of the Sun (mass M). If the semi-
major and semi-minor axes of the orbit are a and b respectively, the angular momentum
of the planet is

2GMm 2 ab 2GMm 2 ab
(a) 2GMm 2 (a + b ) (b) 2GMm 2 (a − b ) (c) (d)
a−b a+b
Ans: (d)
Solution: Assume Sun is at the centre of elliptical orbit.
1 2 GMm 1 2 GMm
Conservation of energy mv1 − = mv2 − v2
2 a 2 b
Conservation of momentum L = mv1a = mv2b b
v1
s a
⎛ a⎞
v2 = v1 ⎜ ⎟
⎝ b⎠

1 2 1 2 GMm GMm 1 ⎛ a2 ⎞ ⎛ b − a⎞
mv1 − mv2 = − ⇒ m ⎜ v12 − v12 2 ⎟ = GMm ⎜
2 2 a b 2 ⎝ b ⎠ ⎝ ab ⎟⎠

1 2 ⎛ b2 − a 2 ⎞ ⎛ b − a⎞ 1 ⎛ b⎞ 1
mv1 ⎜ ⎟ = GMm ⎜ ⎟ ⇒ mv12 = GMm ⎜ ⎟ ⋅
2 ⎝ b 2
⎠ ⎝ ab ⎠ 2 ⎝ a ⎠ (b + a)

⎛ b⎞ 1
v1 = 2GM ⎜ ⎟ ⋅
⎝ a ⎠ (b + a)

⎛ b⎞ ⎛ 1 ⎞ 2GMab 2GMm 2 ab
L = mv1 a = m 2GM ⎜ ⎟ ⋅ ⎜ ⎟ ⋅a = m ⇒L=
⎝ a⎠ ⎝ b + a⎠ (b + a) a+b
                                                                                
Head office  Branch office 
 
fiziks, H.No. 40 D, G.F, Jia Sarai,  Anand Institute of Mathematics, 
 
Near IIT, Hauz Khas, New Delhi‐16  28‐B/6, Jia Sarai, Near IIT 
 
Phone: 011‐26865455/+91‐9871145498 Hauz Khas, New Delhi‐16 
                                                   
                                             Website: www.physicsbyfiziks.com                                                                                          
                                                            Email: fiziks.physics@gmail.com                                                                     53 
fiziks
Institute for NET/JRF, GATE, IIT‐JAM, JEST, TIFR and GRE in PHYSICAL SCIENCES 
 
Q25. The Hamiltonian of a simple pendulum consisting of a mass m attached to a massless
pθ2
string of length l is H = + mgl (1 − cos θ ) . If L denotes the Lagrangian, the value of
2ml 2
dL
is:
dt
2g g
(a) − pθ sin θ (b) − pθ sin 2θ
l l
g
(c) pθ cos θ (d) lpθ2 cos θ
l
Ans: (a)
dL ∂L p2
Solution: = [L, H ] + where H = θ 2 + mgl (1 − cos θ ) .
dt ∂t 2ml
∂H p ml 2θ 2
L = ∑ pi qi − H = pθ θ − H , θ = = θ2 , ⇒ L = − mgl (1 − cos θ ) .
i ∂pθ ml 2

Hence we have to calculate [L, H ] which is only defined into phase space i.e. pθ and θ .

pθ2
Then ⇒ L = − mgl (1 − cos θ )
2ml 2

[L, H ] = ∂L × ∂H −
∂L ∂H
× =−
2g
pθ sin θ and
∂L
=0⇒
dL
=−
2g
pθ sin θ
∂θ ∂pθ ∂pθ ∂θ l ∂t dt l
Q26. Two bodies of equal mass m are connected by a massless rigid rod of length l lying in the
xy-plane with the centre of the rod at the origin. If this system is rotating about the z-axis
with a frequency ω, its angular momentum is
(a) ml 2ω / 4 (b) ml 2ω / 2 (c) ml 2ω (d) 2ml 2ω
Ans: (b)
Solution: Since rod is massless i.e. M = 0.
l
Moment of inertia of the system Ι = m1 r12 + m2 r22 , m1 = m2 = m and r1 = r2 =
2
ml 2 ml 2 ml 2 ml 2ω
Ι= + ⇒Ι= . Angular momentum, J = Iω and J = .
4 4 2 2

                                                                                
Head office  Branch office 
 
fiziks, H.No. 40 D, G.F, Jia Sarai,  Anand Institute of Mathematics, 
 
Near IIT, Hauz Khas, New Delhi‐16  28‐B/6, Jia Sarai, Near IIT 
 
Phone: 011‐26865455/+91‐9871145498 Hauz Khas, New Delhi‐16 
                                                   
                                             Website: www.physicsbyfiziks.com                                                                                          
                                                            Email: fiziks.physics@gmail.com                                                                     54 
fiziks
Institute for NET/JRF, GATE, IIT‐JAM, JEST, TIFR and GRE in PHYSICAL SCIENCES 
 
Q27. Which of the following set of phase-space trajectories is not possible for a particle
obeying Hamilton’s equations of motion?
(a) (b)
P P

x x
(c) (d)
P P

x x
Ans: (b)
Solution: Phase curve does not cut each other
Q28. The muon has mass 105 MeV/c2 and mean life time 2.2 μs in its rest frame. The mean
distance traversed by a muon of energy 315 MeV before decaying is approximately,
(a) 3 × 105 km (b) 2.2 cm (c) 6.6 μm (d) 1.98 km
Ans: (d)
MeV
Solution: Since E = 315MeV and m0 = 105 .
c2
m0 c 2 m0 c 2 105
E = mc 2 ⇒ E = ⇒ 315 = ⇒ 315 = ⇒ v = 0.94c .
v 2
v 2
v2
1− 2 1− 2 1− 2
c c c
t0 2.2 × 10 −6
Now, t = , t 0 = 2.2 μs ⇒ t = ⇒ t = 6.6 μs
v2 8
1− 2 1−
c 9

Now the distance traversed by muon is vt = 0.94c × 6.6 × 10 −6 = = 1.86 km .

                                                                                
Head office  Branch office 
 
fiziks, H.No. 40 D, G.F, Jia Sarai,  Anand Institute of Mathematics, 
 
Near IIT, Hauz Khas, New Delhi‐16  28‐B/6, Jia Sarai, Near IIT 
 
Phone: 011‐26865455/+91‐9871145498 Hauz Khas, New Delhi‐16 
                                                   
                                             Website: www.physicsbyfiziks.com                                                                                          
                                                            Email: fiziks.physics@gmail.com                                                                     55 
fiziks
Institute for NET/JRF, GATE, IIT‐JAM, JEST, TIFR and GRE in PHYSICAL SCIENCES 
 
NET/JRF (JUNE-2013)

Q29. The area of a disc in its rest frame S is equal to 1 (in some units). The disc will appear
distorted to an observer O moving with a speed u with respect to S along the plane of the
disc. The area of the disc measured in the rest frame of the observer O is ( c is the speed
of light in vacuum)
1/ 2 −1 / 2 −1
⎛ u2 ⎞ ⎛ u2 ⎞ ⎛ u2 ⎞ ⎛ u2 ⎞
(a) ⎜⎜1 − 2 ⎟⎟ (b) ⎜⎜1 − 2 ⎟⎟ (c) ⎜⎜1 − 2 ⎟⎟ (d) ⎜⎜1 − 2 ⎟⎟
⎝ c ⎠ ⎝ c ⎠ ⎝ c ⎠ ⎝ c ⎠
Ans: (a)
Solution: Area of disc from S frame is 1 i.e. π a 2 = 1 or π a ⋅ a = 1
u2 u2 u2
Area of disc from S ′ frame is π a ⋅ b = π a ⋅ a 1 − = 1 ⋅ 1 − = 1 −
c2 c2 c2
u2
where b = a 1 − .
c2
Q30. A planet of mass m and an angular momentum L moves in a circular orbit in a potential,
V (r ) = −k / r , where k is a constant. If it is slightly perturbed radially, the angular
frequency of radial oscillations is

(a) mk 2 / 2 L3 (b) mk 2 / L3 (c) 2mk 2 / L3 (d) 3mk 2 / L3


Ans: (b)
Veff
L2 k ∂Veff L2 k
Solution: Veff = − . For circular orbit =− 3 + 2 =0
2mr 2
r ∂r mr r

L2 k L2 k
⇒ = . Thus r = r0 = ⇒ ω= ,
mr 3 r 2 mk m
r
2
d Veff 3L 2
2k 3L 2
2k 3m k 3
2m k4
m k4 3 4 3
k= 2
=+ − 3 = − = − =
dr mr 4
r r = r0 ⎛ L2 ⎞
4
⎛ L2 ⎞
3
L6 L6 L6
r = r0
m⎜⎜ ⎟⎟ ⎜⎜ ⎟⎟
⎝ mk ⎠ ⎝ mk ⎠

d 2V
dr 2 r = r0 mk 2
ω= ⇒ ω= .
m L3
                                                                                
Head office  Branch office 
 
fiziks, H.No. 40 D, G.F, Jia Sarai,  Anand Institute of Mathematics, 
 
Near IIT, Hauz Khas, New Delhi‐16  28‐B/6, Jia Sarai, Near IIT 
 
Phone: 011‐26865455/+91‐9871145498 Hauz Khas, New Delhi‐16 
                                                   
                                             Website: www.physicsbyfiziks.com                                                                                          
                                                            Email: fiziks.physics@gmail.com                                                                     56 
fiziks
Institute for NET/JRF, GATE, IIT‐JAM, JEST, TIFR and GRE in PHYSICAL SCIENCES 
 
Q31. The number of degrees of freedom of a rigid body in d space-dimensions is
(a) 2d (b) 6 (c) d (d + 1) / 2 (d) d!
Ans: (c)
Q32. A system is governed by the Hamiltonian
1
( p x − ay )2 + 1 ( p x − bx )2
H =
2 2
where a and b are constants and p x , p y are momenta conjugate to x and y respectively.

For what values of a and b will the quantities ( p x − 3 y ) and ( p y + 2 x ) be conserved?

(a) a = −3, b = 2 (b) a = 3, b = −2


(c) a = 2, b = −3 (d) a = −2, b = 3
Ans: (d)
Solution: Poisson bracket [ px − 3 y, H ] = 0 and
⎣⎡ p y + 2 y , H ⎦⎤ = 0
p y (b − 3) + x(3b − b 2 ) = 0 and p x ( a + 2) − y (2 a + a 2 ) = 0

⇒ a = −2, b = 3
Q33. The Lagrangian of a particle of mass m moving in one dimension is given by
1 2
L= mx − bx
2
where b is a positive constant. The coordinate of the particle x(t ) at time t is given by: (in
following c1 and c 2 are constants)
b 2
(a) − t + c1t + c 2 (b) c1t + c 2
2m
⎛ bt ⎞ ⎛ bt ⎞ ⎛ bt ⎞ ⎛ bt ⎞
(c) c1 cos⎜ ⎟ + c 2 sin ⎜ ⎟ (d) c1 cosh⎜ ⎟ + c 2 sinh ⎜ ⎟
⎝m⎠ ⎝m⎠ ⎝m⎠ ⎝m⎠
Ans: (a)
d ⎛ ∂L ⎞ ∂L d
Solution: Equation of motion ⎜ ⎟− = 0 ⇒ (mx ) + b = 0 ⇒ mx + b = 0 ⇒
dt ⎝ ∂x ⎠ ∂x dt
mx = −b
d 2x b dx b b t2
=− ⇒ = − t + c1 ⇒ x = − + c1t + c2
dt 2 m dt m m 2
                                                                                
Head office  Branch office 
 
fiziks, H.No. 40 D, G.F, Jia Sarai,  Anand Institute of Mathematics, 
 
Near IIT, Hauz Khas, New Delhi‐16  28‐B/6, Jia Sarai, Near IIT 
 
Phone: 011‐26865455/+91‐9871145498 Hauz Khas, New Delhi‐16 
                                                   
                                             Website: www.physicsbyfiziks.com                                                                                          
                                                            Email: fiziks.physics@gmail.com                                                                     57 
fiziks
Institute for NET/JRF, GATE, IIT‐JAM, JEST, TIFR and GRE in PHYSICAL SCIENCES 
 
NET/JRF (DEC-2013)
Q34. Let A, B and C be functions of phase space variables (coordinates and momenta of a
mechanical system). If {, } represents the Poisson bracket, the value of
{A, {B, C}} − {{A, B}, C} is given by
(a) 0 (b) {B, {C , A}} (c) {A, {C , B}} (d) {{C , A}, B}
Ans: (d)
Solution: We know that Jacobi identity equation
{A, {B, C}} + {B, {C , A}} + {C , {A, B}} = 0
Now {A, {B, C}} − {{A, B}, C} = −{B, {C , A}} = {{C , A}, B}

z2
Q35. A particle moves in a potential V = x + y + 2
. Which component(s) of the angular 2

2
momentum is/are constant(s) of motion?
(a) None (b) Lx , L y and L z (c) only L x and Ly (d) only L z

Ans: (d)
z2
Solution: A particle moves in a potential V = x 2 + y 2 +
2
r2
V (r ,θ , φ ) = r sin θ cos φ + r sin θ sin φ + cos 2 θ
2 2 2 2 2 2

2
2
r
V (r ,θ , φ ) = r 2 sin 2 θ + cos 2 θ
2
Now φ is cyclic-co-ordinate ( pφ ) i.e Lz is constant of motion.

Q36. The Hamiltonian of a relativistic particle of rest mass m and momentum p is given

by H = p 2 + m 2 + V (x ) , in units in which the speed of light c = 1 . The corresponding


Lagrangian is

(a) L = m 1 + x 2 − V ( x ) (b) L = −m 1 − x 2 − V ( x )
1 2
(c) L = 1 + mx 2 − V ( x ) (d) L = mx − V ( x )
2
                                                                                
Head office  Branch office 
 
fiziks, H.No. 40 D, G.F, Jia Sarai,  Anand Institute of Mathematics, 
 
Near IIT, Hauz Khas, New Delhi‐16  28‐B/6, Jia Sarai, Near IIT 
 
Phone: 011‐26865455/+91‐9871145498 Hauz Khas, New Delhi‐16 
                                                   
                                             Website: www.physicsbyfiziks.com                                                                                          
                                                            Email: fiziks.physics@gmail.com                                                                     58 
fiziks
Institute for NET/JRF, GATE, IIT‐JAM, JEST, TIFR and GRE in PHYSICAL SCIENCES 
 
Ans: (b)
∂H
Solution: H = p 2 + m 2 + V (x ) ⇒
∂p
=x=
1
2
2p
1
(
⇒ x p2 + m2 )
1/ 2
=p
(p 2
+m )
2 2

xm
⇒ p=
1 − x2

Now L = ∑ xp − H = xp − H = xp − p 2 + m 2 − V ( x )

xm
Put value p = ⇒ L = −m 1 − x 2 − V (x )
1− x 2

Q37. A pendulum consists of a ring of mass M and radius R suspended by a massless rigid
rod of length l attached to its rim. When the pendulum oscillates in the plane of the ring,
the time period of oscillation is

l+R 2π
(a) 2π
g
(b) (l 2
+ R2 )1/ 4

2 R 2 + 2 Rl + l 2 2π
(c) 2π
g (R + l )
(d) (2R 2
+ 2 Rl + l 2 ) 1/ 4

g
Ans: (c)
Solution: The moment of inertia about pivotal point is given by y
I = I c.m + Md 2 = MR 2 + M (l + R) 2

If ring is displaced by angle θ then potential energy is


o
− Mg (l + R) cos θ
The Lagrangian is given by
1 2 1 o x
L= Iθ − V (θ ) = ( MR 2 + M (l + R) 2 )θ 2 + Mg (l + R) cos θ
2 2
d ⎛ ∂L ⎞ ⎛ ∂L ⎞
⎟−⎜ ⎟ = 0 ⇒ ( MR + M (l + R) )θ + Mg (l + R ) sin θ = 0
2 2

dt ⎝ ∂θ ⎠ ⎝ ∂θ ⎠

For small oscillation sin θ = θ ⇒ ( MR 2 + M (l + R) 2 )θ + Mg (l + R)θ = 0

2 R 2 + 2 Rl + l 2
Time period is given by 2π .
g (R + l )

                                                                                
Head office  Branch office 
 
fiziks, H.No. 40 D, G.F, Jia Sarai,  Anand Institute of Mathematics, 
 
Near IIT, Hauz Khas, New Delhi‐16  28‐B/6, Jia Sarai, Near IIT 
 
Phone: 011‐26865455/+91‐9871145498 Hauz Khas, New Delhi‐16 
                                                   
                                             Website: www.physicsbyfiziks.com                                                                                          
                                                            Email: fiziks.physics@gmail.com                                                                     59 
fiziks
Institute for NET/JRF, GATE, IIT‐JAM, JEST, TIFR and GRE in PHYSICAL SCIENCES 
 
Q38. Consider a particle of mass m attached to two identical springs each of length l and
spring constant k (see the figure). The equilibrium configuration is the one where the
springs are unstretched. There are no other external forces on the system. If the particle is
given a small displacement along the x -axis, which of the following describes the
equation of motion for small oscillations?
kx 3 kx 2
(a) mx + =0 (b) mx + kx = 0 (c) mx + 2kx = 0 (d) mx + =0
l2 l
Ans: (a)
Solution: The lagrangian of system is given by y
1 2 l
L= mx − V ( x)
2 x
o
The potential energy is given by
2 2 l
k⎡ 1
⎤ k⎡ 1

V ( x) = ⎢ x 2 + l 2
2⎣
( ) 2
(
− l ⎥ + ⎢ x2 + l 2 ) 2 − l⎥
⎦ 2⎣ ⎦ o x
2
⎡ 1

(
V ( x) = k ⎢ x 2 + l 2 ) 2 − l⎥
⎣ ⎦
For small oscillation one can approximate potential by Taylor expansion
2
⎡ 1
⎤ 2
2 ⎢
⎛ x ⎞
2 2
⎥ ⎡⎛ 1 x 2 1 x 4 ⎞ ⎤
V ( x) = kl ⎜ 1 + 2 ⎟ − 1 ⇒ V ( x) = kl ⎢⎜1 + 2 − 4 ⎟ − 1⎥
2
⎢ l ⎠ ⎥
⎢⎣⎝ ⎥⎦ ⎣⎢⎝ 2 l 8 l ⎠ ⎦⎥

2
⎛ x2⎞ ⎛ x 4⎞
V ( x) = kl ⎜ 2 ⎟2
⇒ V ( x) = k ⎜ 2 ⎟ .
⎝l ⎠ ⎝ 4l ⎠

1 2 ⎛ x 4⎞
So Lagrangian of system is given by L = mx − k ⎜ 2 ⎟
2 ⎝ 4l ⎠

d ⎛ ∂L ⎞ ⎛ ∂L ⎞ kx 3
The Lagranges equation of motion −
⎜ ⎟ ⎜ ⎟ = 0 ⇒ mx + = 0.
dt ⎝ ∂x ⎠ ⎝ ∂x ⎠ l2

                                                                                
Head office  Branch office 
 
fiziks, H.No. 40 D, G.F, Jia Sarai,  Anand Institute of Mathematics, 
 
Near IIT, Hauz Khas, New Delhi‐16  28‐B/6, Jia Sarai, Near IIT 
 
Phone: 011‐26865455/+91‐9871145498 Hauz Khas, New Delhi‐16 
                                                   
                                             Website: www.physicsbyfiziks.com                                                                                          
                                                            Email: fiziks.physics@gmail.com                                                                     60 
fiziks
Institute for NET/JRF, GATE, IIT‐JAM, JEST, TIFR and GRE in PHYSICAL SCIENCES 
 
NET/JRF (JUNE-2014)

Q39. The time period of a simple pendulum under the influence of the acceleration due to
gravity g is T . The bob is subjected to an additional acceleration of magnitude 3 g in
the horizontal direction. Assuming small oscillations, the mean position and time period
of oscillation, respectively, of the bob will be
(a) 0 o to the vertical and 3T (b) 30 o to the vertical and T / 2

(c) 60 o to the vertical and T / 2 (d) 0 o to the vertical and T / 3


Ans: (c)
l
Solution: T = 2π θ
g T

g′ = (3g )2 + g 2 = 4g 2 = 2 g 3 g
θ
l l 1 T
T ′ = 2π ⇒ T ′ = 2π ⋅ ⇒ T′ = g
2g g 2 2 g ′

T cosθ = mg , T sin θ = 3 mg ⇒ tan θ = 3 ⇒ θ = 60o


1 λ
Q40. A particle of mass m and coordinate q has the Lagrangian L = m q 2 − qq 2
2 2
where λ is a constant. The Hamiltonian for the system is given by
p 2 λqp 2 p2
(a) + (b)
2m 2m 2 2(m − λq )

p2 λqp 2 pq
(c) + (d)
2m 2(m − λq )2 2

Ans: (b)
1 1
Solution: H = ∑ qp − L where L = mq 2 − qq 2
2 2
∂L p
= p = mq − λ qq ⇒ p = q ( m − λ q ) ⇒ q =
∂q m − λq

                                                                                
Head office  Branch office 
 
fiziks, H.No. 40 D, G.F, Jia Sarai,  Anand Institute of Mathematics, 
 
Near IIT, Hauz Khas, New Delhi‐16  28‐B/6, Jia Sarai, Near IIT 
 
Phone: 011‐26865455/+91‐9871145498 Hauz Khas, New Delhi‐16 
                                                   
                                             Website: www.physicsbyfiziks.com                                                                                          
                                                            Email: fiziks.physics@gmail.com                                                                     61 
fiziks
Institute for NET/JRF, GATE, IIT‐JAM, JEST, TIFR and GRE in PHYSICAL SCIENCES 
 

⇒ H = qp − L =
p2 1
− m
( p ) + λ q ⋅ p2
2

( m − λ q ) 2 ( m − λ q )2 2 ( m − λ q )2
p2 p2
⇒ H = qp − L = − ( m − λq )
( m − λ q ) 2 ( m − λ q )2
p2 p2 p2
⇒ H = qp − L = − ⇒H =
( m − λq ) 2 ( m − λq ) 2 (m − λq )

Q41. The coordinates and momenta xi , pi (i = 1, 2, 3) of a particle satisfy the canonical Poisson

bracket relations {xi , p j } = δ ij . If C1 = x2 p3 + x3 p2 and C 2 = x1 p 2 + x 2 p1 are constants of

motion, and if C3 = {C1 , C 2 } = x1 p3 + x3 p1 , then

(a) {C 2 , C3 } = C1 and {C3 , C1 } = C 2

(b) {C 2 , C3 } = −C1 and {C3 , C1 } = −C 2

(c) {C 2 , C3 } = −C1 and {C3 , C1 } = C 2

(d) {C 2 , C3 } = C1 and {C3 , C1 } = −C 2


Ans: (d)

Solution:
C1 = x2 p3 + x3 p2 , C2 = x1 p2 − x2 p1 , C3 = x1 p3 + x3 p1

⎛ ∂C2 ∂C3 ∂C2 ∂C3 ⎞ ⎛ ∂C2 ∂C3 ∂C2 ∂C3 ⎞ ⎛ ∂C2 ∂C3 ∂C2 ∂C3 ⎞
{C2 , C3 } = ⎜ − ⎟+⎜ − ⎟+⎜ − ⎟
⎝ ∂x1 ∂p1 ∂p1 ∂x1 ⎠ ⎝ ∂x2 ∂p2 ∂p2 ∂x2 ⎠ ⎝ ∂x3 ∂p3 ∂p3 ∂x3 ⎠

{C2 , C3 } = ( p2 x3 − ( − x2 ) p3 ) + ( 0 − x1 ⋅ 0 ) + ( 0 ⋅ x1 − 0 ⋅ x1 ) = ( p2 x3 + x2 p3 ) = C1
⎛ ∂C3 ∂C1 ∂C3 ∂C1 ⎞ ⎛ ∂C3 ∂C1 ∂C3 ∂C1 ⎞ ⎛ ∂C3 ∂C1 ∂C3 ∂C1 ⎞
{C3 , C1} = ⎜ − ⎟+⎜ − ⎟+⎜ − ⎟
⎝ ∂x1 ∂p1 ∂p1 ∂x1 ⎠ ⎝ ∂x2 ∂p2 ∂p2 ∂x2 ⎠ ⎝ ∂x3 ∂p3 ∂p3 ∂x3 ⎠
{C3 , C1} = ( p3 ⋅ 0 − x3 ⋅ 0 ) + ( 0 ⋅ x3 − 0 ⋅ p3 ) + ( p1 x2 − x1 p2 ) = − ( x1 p2 − x2 p1 ) = −C2

                                                                                
Head office  Branch office 
 
fiziks, H.No. 40 D, G.F, Jia Sarai,  Anand Institute of Mathematics, 
 
Near IIT, Hauz Khas, New Delhi‐16  28‐B/6, Jia Sarai, Near IIT 
 
Phone: 011‐26865455/+91‐9871145498 Hauz Khas, New Delhi‐16 
                                                   
                                             Website: www.physicsbyfiziks.com                                                                                          
                                                            Email: fiziks.physics@gmail.com                                                                     62 
fiziks
Institute for NET/JRF, GATE, IIT‐JAM, JEST, TIFR and GRE in PHYSICAL SCIENCES 
 
Q42. The recently-discovered Higgs boson at the LHC experiment has a decay mode into a
photon and a Z boson. If the rest masses of the Higgs and Z boson are 125 GeV/c2 and

90 GeV/c2 respectively, and the decaying Higgs particle is at rest, the energy of the
photon will approximately be
(a) 35 3 GeV (b) 35 GeV (c) 30 GeV (d) 15 GeV
Ans: (c)

Solution:
H B → PH + Z B

From conservation of momentum 0 = P1 + P2 ⇒ P1 = − P2 ⇒ P1 = P2

Now EH B = EPH + EZ B ⇒ EPH + EZ B = M H B c 2

E P2H = P12 c 2 + 0 and EZ2B = P22 c 2 + M Z2B c 4

(
⇒ EZ B − EPH )( E ZB )
+ EPH = M Z2B c 4 ∵ P1 = P2

M Z2B c 4 M Z2B c 2
⇒ EZ B − EPH = 2
= ∵ EZ B + EPH = M H B c 2
M HB c M HB

⇒ 2 EPH = M H B c − 2
M z2B c 2
⇒ EPH =
(M 2
HB )
− M z2B c 2
M HB M HB

⎛ 125 ×125 − 90 × 90 ⎞ c
4
⇒ EPH = ⎜ ⎟ 4 = 30.1GeV
×
⎝ 2 ×125 ⎠ c
Q43. A canonical transformation relates the old coordinates (q, p ) to the new ones (Q, P ) by

the relations Q = q 2 and P = p / 2q . The corresponding time independent generating


function is
(a) P / q 2 (b) q 2 P (c) q 2 / P (d) qP 2
Ans: (b)

Solution: Q = q ; P = p / 2q
2

∂F2 ∂F
= p ⇒ 2 = P ⋅ 2q ⇒ F2 = q 2 P + f ( P)
∂q ∂q

                                                                                
Head office  Branch office 
 
fiziks, H.No. 40 D, G.F, Jia Sarai,  Anand Institute of Mathematics, 
 
Near IIT, Hauz Khas, New Delhi‐16  28‐B/6, Jia Sarai, Near IIT 
 
Phone: 011‐26865455/+91‐9871145498 Hauz Khas, New Delhi‐16 
                                                   
                                             Website: www.physicsbyfiziks.com                                                                                          
                                                            Email: fiziks.physics@gmail.com                                                                     63 
fiziks
Institute for NET/JRF, GATE, IIT‐JAM, JEST, TIFR and GRE in PHYSICAL SCIENCES 
 
∂F2
= Q = q 2 ⇒ F2 = q 2 P + f (q)
∂P
comparing both side f (q ) = f ( P) = 0 ⇒ F2 = q 2 P

NET/JRF (DEC-2014)
Q44. The equation of motion of a system described by the time-dependent Lagrangian
⎡1 ⎤
L = e γt ⎢ mx 2 − V (x )⎥ is
⎣2 ⎦
dV dV
(a) mx + γmx + =0 (b) mx + γmx − =0
dx dx
dV dV
(c) mx − γmx + =0 (d) mx + =0
dx dx
Ans: (a)
⎡1 ⎤ ∂L ∂L ∂V γ t
Solution: ∵ L = eγ t ⎢ mx 2 − V ( x ) ⎥ ⇒ = eγ t mx and =− e
⎣2 ⎦ ∂x ∂x ∂x
d ⎛ ∂L ⎞ ∂L d ∂V γ t ∂V γ t
∵ ⎜ ⎟− = 0 ⇒ ( eγ t mx ) + e = mxeγ t + mxγ eγ t + e =0
dt ⎝ ∂x ⎠ ∂x dt ∂x ∂x

⎛ ∂V ⎞ γ t ∂V
⎜⎝ mx + mγ x + ⎟⎠ e = 0 ⇒ mx + γ mx + =0
∂x ∂x
1 1
Q45. A particle of mass m is moving in the potential V ( x ) = − ax 2 + bx 4 where a, b are
2 4
positive constants. The frequency of small oscillations about a point of stable equilibrium
is
(a) a/m (b) 2a / m (c) 3a / m (d) 6a / m
Ans: (b)
1 1
Solution: ∵V ( x ) = − ax 2 + bx 4
2 4
1
∂V ⎛ a ⎞2
= 0 ⇒ −ax + bx3 = 0 ⇒ x ⎡⎣ − a + bx 2 ⎤⎦ = 0 ⇒ x = ± ⎜ ⎟ , 0
∂x ⎝b⎠

                                                                                
Head office  Branch office 
 
fiziks, H.No. 40 D, G.F, Jia Sarai,  Anand Institute of Mathematics, 
 
Near IIT, Hauz Khas, New Delhi‐16  28‐B/6, Jia Sarai, Near IIT 
 
Phone: 011‐26865455/+91‐9871145498 Hauz Khas, New Delhi‐16 
                                                   
                                             Website: www.physicsbyfiziks.com                                                                                          
                                                            Email: fiziks.physics@gmail.com                                                                     64 
fiziks
Institute for NET/JRF, GATE, IIT‐JAM, JEST, TIFR and GRE in PHYSICAL SCIENCES 
 
∂V
2
∂V2
∵ 2 = −a + 3bx 2 ⇒ At x = 0, 2 = − a (Negative so it is unstable point)
∂x ∂x
∂ 2V a
= − a + 3b = 2a (Positive so it is stable point)
∂x 2
1
⎛a⎞ 2
x =± ⎜ ⎟
b
⎝ b⎠

∂ 2V
∂x 2 = 2a
⇒ω =
m m
Q46. The radius of Earth is approximately 6400 km . The height h at which the acceleration
due to Earth’s gravity differs from g at the Earth’s surface by approximately 1 % is
(a) 64 km (b) 48 km (c) 32 km (d) 16 km
Ans: (c)
g 2h g 2h Δg 2h
Solution: = 1+ ⇒ −1 = ⇒ = ⇒ h = 32 k .m.
g′ R g′ R g′ R
Q47. According to the special theory of relativity, the speed v of a free particle of mass m and
total energy E is:

mc 2 2 E ⎛ mc 2 ⎞
(a) v = c 1 − (b) v = ⎜1 + ⎟⎟
E m ⎜⎝ E ⎠
2
⎛ mc 2 ⎞ ⎛ mc 2 ⎞
(c) v = c 1 − ⎜⎜ ⎟⎟ (d) v = c⎜⎜1 + ⎟
⎝ E ⎠ ⎝ E ⎟⎠
Ans: (c)
2 2
v 2 ⎛ mc 2 ⎞
mc 2 v2 m2c 4 ⎛ mc 2 ⎞
Solution: E = ⇒ 1− 2 = ⎜ ⎟ ⇒ 2 = 1− 2 ⇒ v = c 1− ⎜ ⎟
v2 c ⎝ E ⎠ c E ⎝ E ⎠
1− 2
c
p2
Q48. The Hamiltonian of a classical particle moving in one dimension is H = + αq 4 where
2m
α is a positive constant and p and q are its momentum and position respectively. Given
that its total energy E ≤ E 0 the available volume of phase space depends on E 0 as

(a) E 03 / 4 (b) E 0

(c) E0 (d) is independent of E 0


                                                                                
Head office  Branch office 
 
fiziks, H.No. 40 D, G.F, Jia Sarai,  Anand Institute of Mathematics, 
 
Near IIT, Hauz Khas, New Delhi‐16  28‐B/6, Jia Sarai, Near IIT 
 
Phone: 011‐26865455/+91‐9871145498 Hauz Khas, New Delhi‐16 
                                                   
                                             Website: www.physicsbyfiziks.com                                                                                          
                                                            Email: fiziks.physics@gmail.com                                                                     65 
fiziks
Institute for NET/JRF, GATE, IIT‐JAM, JEST, TIFR and GRE in PHYSICAL SCIENCES 
 
Ans: (a)
V( q)
P2
Solution: H = + α q4
2m
E0
Phase area = ∫ p ⋅ dq p q
1
⎛ E ⎞4 2mE0
A= ∫ p ⋅ dq = π 2mE × ⎜ ⎟
⎝α ⎠ ( E0 /α )1/4
− ( E0 /α )1/4

A∝E 1/2
0 ⋅E1/4
0 ⇒ A∝E 3/4
0 2mE0

p2 1
Q49. A mechanical system is described by the Hamiltonian H (q, p ) = + mω 2 q 2 . As a
2m 2
Q
result of the canonical transformation generated by F (q, Q ) = − , the Hamiltonian in
q
the new coordinate Q and momentum P becomes

1 2 2 mω 2 2 1 2 2 mω 2 2
(a) Q P + Q (b) Q P + P
2m 2 2m 2
1 2 mω 2 2 1 2 4 mω 2 − 2
(c) P + Q (d) Q P + P
2m 2 2m 2
Ans: (d)
p2 1 Q
+ mω 2 q 2 , F = F1 ( q, Q ) = −
Solution: H =
2m 2 q
∂F Q
⇒ 1=p ⇒ 2 =p ……….(a)
∂q q
∂F1 1 1
⇒ = −P ⇒ − = −P ⇒ q = ……….(b)
∂Q q P
1
From equation (a) and (b) ⇒ p = QP 2 ∵q =
P
p2 1 Q2 P4 1 ⎛ 1 ⎞ 1 2 4 1
H= + mω 2 q 2 = + mω 2 ⎜ 2 ⎟ = Q P + mω 2 P −2
2m 2 2m 2 ⎝ P ⎠ 2 m 2

                                                                                
Head office  Branch office 
 
fiziks, H.No. 40 D, G.F, Jia Sarai,  Anand Institute of Mathematics, 
 
Near IIT, Hauz Khas, New Delhi‐16  28‐B/6, Jia Sarai, Near IIT 
 
Phone: 011‐26865455/+91‐9871145498 Hauz Khas, New Delhi‐16 
                                                   
                                             Website: www.physicsbyfiziks.com                                                                                          
                                                            Email: fiziks.physics@gmail.com                                                                     66 
fiziks
Institute for NET/JRF, GATE, IIT‐JAM, JEST, TIFR and GRE in PHYSICAL SCIENCES 
 
Q50. The probe Mangalyaan was sent recently to explore the planet Mars. The inter-planetary
part of the trajectory is approximately a half-ellipse with the Earth (at the time of launch),
Sun and Mars (at the time the probe reaches the
destination) forming the major axis. Assuming that the
orbits of Earth and Mars are approximately circular with Sun
Earth Mars
radii RE and RM , respectively, the velocity (with respect RE
RM
to the Sun) of the probe during its voyage when it is at a
distance r (RE << r << RM ) from the Sun, neglecting the
effect of Earth and Mars, is
(R E + RM ) (R E + RM − r )
(a) 2GM (b) 2GM
r (R E + R M − r ) r (R E + RM )

RE 2GM
(c) 2GM (d)
rRM r

Ans: (b)
Solution: Total energy E = − K / 2a where 2a major axis and 2a = RE + RM .

1 2 GMm GMm ( R + RM − r )
⇒ v = 2GM E
mv − =−
2 r ( RE + RM ) r ( RE + RM )

                                                                                
Head office  Branch office 
 
fiziks, H.No. 40 D, G.F, Jia Sarai,  Anand Institute of Mathematics, 
 
Near IIT, Hauz Khas, New Delhi‐16  28‐B/6, Jia Sarai, Near IIT 
 
Phone: 011‐26865455/+91‐9871145498 Hauz Khas, New Delhi‐16 
                                                   
                                             Website: www.physicsbyfiziks.com                                                                                          
                                                            Email: fiziks.physics@gmail.com                                                                     67 
fiziks
Institute for NET/JRF, GATE, IIT‐JAM, JEST, TIFR and GRE in PHYSICAL SCIENCES 
 
NET/JRF (JUNE-2015)
Q51. A particle moves in two dimensions on the ellipse x 2 + 4 y 2 = 8 . At a particular instant it

is at the point ( x, y ) = ( 2,1) and the x -component of its velocity is 6 (in suitable units).

Then the y-component of its velocity is


(a) −3 (b) −2 (c) 1 (d) 4
Ans. (a)
dx dy
Solution: ∵ x 2 + 4 y 2 = 8 ⇒ 2 x + 8y =0
dt dt
⇒ 2 xvx + 8 y v y = 0 ⇒ 2 × 2 × 6 + 8 ×1× v y = 0 ⇒ v y = −3

Q52. Consider three inertial frames of reference A, B and C . the frame B moves with a
c c
velocity with respect to A , and C moves with a velocity with respect to B in the
2 10
same direction. The velocity of C as measured in A is

3c 4c c 3c
(a) (b) (c) (d)
7 7 7 7
Ans. (b)
c c A S B S′ C u x′
Solution: v = , u x′ =
2 10
C/2 C /10
u′ + v 4c
ux = x =
u ′v 7
1 + 2x
c
1 2
Q53. If the Lagrangian of a dynamical system in two dimensions is L = mx + mxy , then its
2
Hamiltonian is
1 1 2 1 1 2
(a) H = px p y + py (b) H = px p y + px
m 2m m 2m
1 1 2 1 1 2
(c) H = px p y − py (d) H = px p y − px
m 2m m 2m
Ans. (c)

                                                                                
Head office  Branch office 
 
fiziks, H.No. 40 D, G.F, Jia Sarai,  Anand Institute of Mathematics, 
 
Near IIT, Hauz Khas, New Delhi‐16  28‐B/6, Jia Sarai, Near IIT 
 
Phone: 011‐26865455/+91‐9871145498 Hauz Khas, New Delhi‐16 
                                                   
                                             Website: www.physicsbyfiziks.com                                                                                          
                                                            Email: fiziks.physics@gmail.com                                                                     68 
fiziks
Institute for NET/JRF, GATE, IIT‐JAM, JEST, TIFR and GRE in PHYSICAL SCIENCES 
 
1 ∂L
Solution: L = mx 2 + mxy ⇒ = mx + my = px (i)
2 ∂x
∂L py
⇒ = mx = p y or x = (ii)
∂y m
py px − p y
put x = in equation (i) ⇒ p y + my = px ⇒ y =
m m
1
H = px x + p y y − L = px x + p y y − mx 2 − mxy
2
px p y p y2
put value of x and y ⇒ H = −
m 2m
α β
Q54. A particle of mass m moves in the one dimensional potential V ( x ) = x3 + x 4 where
3 4
α , β > 0 . One of the equilibrium points is x = 0 . The angular frequency of small
oscillations about the other equilibrium point is
2α α α α
(a) (b) (c) (d)
3mβ mβ 12mβ 24mβ
Ans. (b)
α β ∂V α
Solution: V ( x ) = x3 + x4 ⇒ = α x 2 + β x3 = 0 ⇒ x0 = −
3 4 ∂x β

∂ 2V α2
Spring constant k = = ( +ve )
∂x 2 x = x0
β

k α
ω= =
m βm

Q55. A particle of unit mass moves in the xy -plane in such a way that x ( t ) = y ( t ) and

y ( t ) = − x ( t ) . We can conclude that it is in a conservative force-field which can be

derived from the potential


1 2 1 2
(a)
2
(
x + y2 ) (b)
2
(
x − y2 ) (c) x + y (d) x − y

                                                                                
Head office  Branch office 
 
fiziks, H.No. 40 D, G.F, Jia Sarai,  Anand Institute of Mathematics, 
 
Near IIT, Hauz Khas, New Delhi‐16  28‐B/6, Jia Sarai, Near IIT 
 
Phone: 011‐26865455/+91‐9871145498 Hauz Khas, New Delhi‐16 
                                                   
                                             Website: www.physicsbyfiziks.com                                                                                          
                                                            Email: fiziks.physics@gmail.com                                                                     69 
fiziks
Institute for NET/JRF, GATE, IIT‐JAM, JEST, TIFR and GRE in PHYSICAL SCIENCES 
 
Ans. (a)
Solution: ∵ x = y and y = − x
⇒ x = y = −x and y = − x = − y
⇒ x+x =0 and y + y = 0
1 2 1 2 1 2
that is possible for L =
2
mx + my − x + y 2
2 2
( )
1 2
⇒ V=
2
(
x + y2 )
1
Q56. A particle moves in one dimension in the potential V = k ( t ) x 2 , where k ( t ) is a time
2
d
dependent parameter. Then V , the rate of change of the expectation value V of the
dt
potential energy is
1 dk 2 k 1 dk 2 1
(a) x + ( xp + px ) (b) x + p2
2 dt 2m 2 dt 2m
k 1 dk 2
(c) xp + px (d) x
2m 2 dt
Ans. (a)
p2 1
Solution: H = + k (t ) x2
2m 2
d ∂V ⎡1 p2 1 ⎤ x 2 ∂k
V = [V , H ] + + = ⎢ k (t ) x2 , + k (t ) x2 ⎥ +
dt ∂t ⎣2 2m 2 ⎦ 2 ∂t

1 xp + px x 2 ∂k x 2 ∂k 1
= k ( H ) .2 + = + k ( t ) xp + px
2 2m 2 ∂t 2 ∂t 2m

                                                                                
Head office  Branch office 
 
fiziks, H.No. 40 D, G.F, Jia Sarai,  Anand Institute of Mathematics, 
 
Near IIT, Hauz Khas, New Delhi‐16  28‐B/6, Jia Sarai, Near IIT 
 
Phone: 011‐26865455/+91‐9871145498 Hauz Khas, New Delhi‐16 
                                                   
                                             Website: www.physicsbyfiziks.com                                                                                          
                                                            Email: fiziks.physics@gmail.com                                                                     70 
fiziks
Institute for NET/JRF, GATE, IIT‐JAM, JEST, TIFR and GRE in PHYSICAL SCIENCES 
 
Q57. Let q and p be the canonical coordinate and momentum of a dynamical system. Which
of the following transformations is canonical?
1 2 1 2
1. Q1 = q and P1 = p
2 2
1 1
2. Q2 = ( p + q ) and P2 = ( p − q )
2 2
(a) neither 1 nor 2 (b) both 1 and 2
(c) only 1 (d) only 2
Ans. (d)
q2 p2
Solution: For A : Q1 = , p1 =
2 2
∂Q ∂p ∂Q ∂p
[Q1 , p1 ] = 1 . 1 − 1 . 1 ≠ 1 (Not canonical)
∂q ∂p ∂p ∂q
1 1
For B : Q2 = ( p + q ) , p2 = ( p − q )
2 2

[Q2 , p2 ] = 1 (canonical)

Q58. Which of the following figures is a schematic representation of the phase space
trajectories (i.e., contours of constant energy) of a particle moving in a one-dimensional
1 2 1 4
potential V ( x ) = x + x p
2 4
p

(a) (b)
x
x

p p

(c) (d)
x x

                                                                                
Head office  Branch office 
 
fiziks, H.No. 40 D, G.F, Jia Sarai,  Anand Institute of Mathematics, 
 
Near IIT, Hauz Khas, New Delhi‐16  28‐B/6, Jia Sarai, Near IIT 
 
Phone: 011‐26865455/+91‐9871145498 Hauz Khas, New Delhi‐16 
                                                   
                                             Website: www.physicsbyfiziks.com                                                                                          
                                                            Email: fiziks.physics@gmail.com                                                                     71 
fiziks
Institute for NET/JRF, GATE, IIT‐JAM, JEST, TIFR and GRE in PHYSICAL SCIENCES 
 
Ans. (a)
− x2 x4 V ( x)
Solution: V ( x ) = +
2 4
∂V x
= 0 ⇒ x = 0, x = ±1
∂x
∂ 2V
= −ve for x = 0 (unstable point)
∂x 2 E>0
= + ve for x = ±1 (stable point)
E<0 E=0

NET/JRF (DEC-2015)
Q59. Two masses m each, are placed at the points ( x, y ) = ( a , a ) and ( − a, − a ) and two

masses, 2m each, are placed at the points ( a, − a ) and ( − a, a ) . The principal moments of

inertia of the system are


(a) 2m 2 , 4ma 2 (b) 4ma 2 ,8ma 2

(c) 4ma 2 , 4ma 2 (d) 8ma 2 ,8ma 2


Ans.: (b)

(
Solution: I xx = ∑ mi yi2 + zi2 = ∑ mi yi2 ) ∵ zi = 0
i

⇒ I xx = ma 2 + ma 2 + 2ma 2 + 2ma 2

I xx = 6ma 2

Similarly, I yy = 6ma 2 and I zz = 12ma 2

I xz = I zx = 0, I yz = I zy = 0

I xy = I yx = − mi ∑ xi yi = − ma 2 − ma 2 + 2ma 2 + 2ma 2 ⇒ I xy = I yx = 2ma 2


i

moment of inertia tensor


⎛ 6ma 2 2ma 2 0 ⎞
⎜ ⎟
I = ⎜ 2ma 2 6ma 2 0 ⎟
⎜ 0 0 12ma 2 ⎟⎠

                                                                                
Head office  Branch office 
 
fiziks, H.No. 40 D, G.F, Jia Sarai,  Anand Institute of Mathematics, 
 
Near IIT, Hauz Khas, New Delhi‐16  28‐B/6, Jia Sarai, Near IIT 
 
Phone: 011‐26865455/+91‐9871145498 Hauz Khas, New Delhi‐16 
                                                   
                                             Website: www.physicsbyfiziks.com                                                                                          
                                                            Email: fiziks.physics@gmail.com                                                                     72 
fiziks
Institute for NET/JRF, GATE, IIT‐JAM, JEST, TIFR and GRE in PHYSICAL SCIENCES 
 
eigen value of matrices is principal moment of inertia which is given by
λ1 = 4ma 2 = I x

λ2 = 8ma 2 = I y

λ3 = 12ma 2 = I z

so I x = 4ma 2 and I y = 8ma 2

Q60. The Lagrangian of a system is given by


1 2 ⎛5 ⎞
L= mq1 + 2mq22 − k ⎜ q12 + 2q22 − 2q1q2 ⎟
2 ⎝4 ⎠
where m and k are positive constants. The frequencies of its normal modes are

(a)
k
,
2m m
3k
(b)
k
2m
(
13 ± 73 )
5k k k 6k
(c) , (d) ,
2m m 2m m
Ans.: (a)
1 2 ⎡5 ⎤
Solution: L = mq1 + 2mq22 − k ⎢ q12 + 2q22 − 2q1 q2 ⎥
2 ⎣4 ⎦
1 2 4 2 k ⎡10 2 ⎤
L= mq1 + mq2 − ⎢ q1 + 4q22 − 2q1 q2 − 2q2 q1 ⎥
2 2 2⎣ 4 ⎦

⎛ 10 ⎞
⎛m 0 ⎞ k −2 k ⎟
T =⎜ ⎟, V =⎜ 4
⎝ 0 4m ⎠ ⎜⎜ ⎟
⎝ −2k 4k ⎟⎠

The secular equation V − ω 2 m = 0

⎛ 10 ⎞
⎜ 4 k −ω m −2 k
2
⎛ 10 ⎞
⎜⎜
⎟=0
⎟ ⎝ 4 ⎠
(
⎜ k − ω m ⎟ 4k − 4ω m − 4k = 0
2 2 2
)
⎝ −2 k 4k − ω 2 4m ⎟⎠

⇒ 10k 2 − 10ω 2 km − 4ω 2 km + 4ω 4 m 2 − 4k 2 = 0
⇒ 3k 2 − 7ω 2 km + 2ω 4 m 2 = 0

                                                                                
Head office  Branch office 
 
fiziks, H.No. 40 D, G.F, Jia Sarai,  Anand Institute of Mathematics, 
 
Near IIT, Hauz Khas, New Delhi‐16  28‐B/6, Jia Sarai, Near IIT 
 
Phone: 011‐26865455/+91‐9871145498 Hauz Khas, New Delhi‐16 
                                                   
                                             Website: www.physicsbyfiziks.com                                                                                          
                                                            Email: fiziks.physics@gmail.com                                                                     73 
fiziks
Institute for NET/JRF, GATE, IIT‐JAM, JEST, TIFR and GRE in PHYSICAL SCIENCES 
 
⇒ 3k − 6ω km − ω km + 2ω m = 0
2 2 2 4 2

k 3k
( )(
⇒ k − 2ω 2 m 3k − ω 2 m = 0 ⇒ ω = ) 2m
, ω=
m
Q61. Consider a particle of mass m moving with a speed v . If TR denotes the relativistic

kinetic energy and TN its non-relativistic approximation, then the value of


(TR − TN ) for
TR
v = 0.01 c , is

(a) 1.25 × 10−5 (b) 5.0 × 10−5


(c) 7.5 × 10−5 (d) 1.0 × 10−4
Ans.: (c)
1
Solution: TN = m0 v 2
2
m0 c 2
TR = mc 2 − m0 c 2 = − m0 c 2
2
v
1−
c2
v = 0.01 c

( 0.01)
2
1 v2
(TR − TN ) m0 v 2
TN 2 2 2
Now, = 1− = 1− = 1− = 1−
TR TR m c 2
c 2
1
0
− m0 c 2 − c2 −1
1 − ( 0.01)
2 2 2
v v
1− 2 1− 2
c c
TR − TN
= 7.5 × 10−5
TR

                                                                                
Head office  Branch office 
 
fiziks, H.No. 40 D, G.F, Jia Sarai,  Anand Institute of Mathematics, 
 
Near IIT, Hauz Khas, New Delhi‐16  28‐B/6, Jia Sarai, Near IIT 
 
Phone: 011‐26865455/+91‐9871145498 Hauz Khas, New Delhi‐16 
                                                   
                                             Website: www.physicsbyfiziks.com                                                                                          
                                                            Email: fiziks.physics@gmail.com                                                                     74 
fiziks
Institute for NET/JRF, GATE, IIT‐JAM, JEST, TIFR and GRE in PHYSICAL SCIENCES 
 
Q62. A canonical transformation ( p, q ) → ( P, Q ) is performed on the Hamiltonian

1 1
H= 2
+ mω 2 q 2 via the generating function
2mp 2
1
F= mω q 2 cot Q . If Q ( 0 ) = 0 , which of the following graphs shows schematically the
2
dependence of Q ( t ) on t ?

(a) (b)
Q (t )
Q (t )

(c) (d)
Q (t ) Q (t )

Ans.: (d)
p2 1 1
Solution: H = + mω 2 q 2 , F1 = mω q 2 cot Q
2m 2 2
∂F1 ∂F1 ∂F1
= p, = − P, k=H+
∂q ∂Q ∂t
∂F1 θ (t )
= p ⇒ mω q cot Q = p …..(i)
∂q
∂F1 t
= −P
∂Q
1
⇒ − mω 2 q 2 cosec 2 Q = − P
2
1 P
⇒ mω 2 q 2 = …..(ii)
2 cosec 2Q
From (i) and (ii)
p = P 2m cos Q
                                                                                
Head office  Branch office 
 
fiziks, H.No. 40 D, G.F, Jia Sarai,  Anand Institute of Mathematics, 
 
Near IIT, Hauz Khas, New Delhi‐16  28‐B/6, Jia Sarai, Near IIT 
 
Phone: 011‐26865455/+91‐9871145498 Hauz Khas, New Delhi‐16 
                                                   
                                             Website: www.physicsbyfiziks.com                                                                                          
                                                            Email: fiziks.physics@gmail.com                                                                     75 
fiziks
Institute for NET/JRF, GATE, IIT‐JAM, JEST, TIFR and GRE in PHYSICAL SCIENCES 
 
∂F ∂F
⇒k =H+ 1; ∵ 1 =0
∂t ∂t
p2 1
⇒k=H = + mω 2 q 2 put the value of p and q
2m 2
⇒ k = P 2 using equation of motion Q and P
∂k
= − P = 0 ⇒ P = 0 ⇒ P = constant
∂Q
∂k ∂k
=θ ⇒ = 2 P = Q ⇒ Q = constant (∵ P = constant )
∂P ∂P
Q = c ⇒ Q = ct + α ( from boundary condition α = 0 )
⇒ Q = ct
Therefore, option (d) is correct.
Q63. The Lagrangian of a particle moving in a plane s given in Cartesian coordinates as
L = xy − x 2 − y 2
In polar coordinates the expression for the canonical momentum pr (conjugate to the
radial coordinate r ) is
(a) r sin θ + rθ cos θ (b) r cos θ + rθ sin θ
(c) 2r cos θ − rθ sin 2θ (d) r sin 2θ + rθ cos 2θ
Ans.: (d)
Solution: L = xy − x 2 − y 2

(
L = xy − x 2 + y 2 )
x = r cos θ , y = r sin θ ⇒ x = r cos θ − r sin θθ , y = r sin θ + r cos θ θ

L = r 2 sin θ cos θ − r 2 sin θ cos θ θ 2 + r r cos 2 θθ − rr sin 2 θθ


∂L
Pr =
∂r
⇒ 2r sin θ cos θ + rθ cos 2 θ − sin 2 θ( )
⇒ Pr = r sin 2θ + rθ cos 2θ

                                                                                
Head office  Branch office 
 
fiziks, H.No. 40 D, G.F, Jia Sarai,  Anand Institute of Mathematics, 
 
Near IIT, Hauz Khas, New Delhi‐16  28‐B/6, Jia Sarai, Near IIT 
 
Phone: 011‐26865455/+91‐9871145498 Hauz Khas, New Delhi‐16 
                                                   
                                             Website: www.physicsbyfiziks.com                                                                                          
                                                            Email: fiziks.physics@gmail.com                                                                     76 
fiziks
Institute for NET/JRF, GATE, IIT‐JAM, JEST, TIFR and GRE in PHYSICAL SCIENCES 
 
NET/JRF (JUNE-2016)
Q64. Let ( x, t ) and ( x′, t ′) be the coordinate systems used by the observers O and O′ ,

respectively. Observer O′ moves with a velocity v = β c along their common positive x -


axis. If x+ = x + ct and x− = x − ct are the linear combinations of the coordinates, the

Lorentz transformation relating O and O′ takes the form

x− − β x+ x+ − β x− 1+ β 1− β
(a) x+′ = and x−′ = (b) x+′ = x+ and x−′ = x−
1− β 2
1− β 2 1− β 1+ β

x+ − β x− x− − β x+ 1− β 1+ β
(c) x+′ = and x−′ = (d) x+′ = x+ and x−′ = x−
1− β 2 1− β 2 1+ β 1− β

Ans: (d)
Solution: x+′ = x′ + ct ′

⎛ vx ⎞ ⎛ v⎞ ⎛ v⎞ v v v
c⎜t − 2 ⎟ x ⎜1 − ⎟ ct ⎜1 − ⎟ 1− 1− 1−
x − vt c ⎠ c c
= + ⎝ = ⎝ ⎠+ ⎝ ⎠ =x c + ct c = c ( x + ct )
v2 v2 v 2
v 2 v v v
1− 2 1− 2 1− 2 1− 2 1+ 1+ 1+
c c c c c c c

1− β
x+′ = x+
1+ β
⎛ vx ⎞ ⎛ v⎞ ⎛ v⎞
c⎜t − 2 ⎟ x ⎜1 + ⎟ ct ⎜ 1 + ⎟
x − vt c ⎠ c⎠ c⎠
x−′ = x′ − ct ′ ⇒ − ⎝ ⇒ ⎝ − ⎝
v2 v2 v2 v2
1− 2 1− 2 1− 2 1− 2
c c c c

v v
1+ 1+
x−′ = x c − ct c ⇒ x′ = 1 + β ( x − ct ) ⇒ x′ = 1 + β x
− − −
1−
v
1−
v 1− β 1− β
c c
Q65. A ball of mass m , initially at rest, is dropped from a height of 5 meters. If the coefficient
of restitution is 0.9 , the speed of the ball just before it hits the floor the second time is
approximately (take g = 9.8 m / s 2 )
(a) 9.80 m / s (b) 9.10 m / s (c) 8.91 m / s (d) 7.02 m / s
                                                                                
Head office  Branch office 
 
fiziks, H.No. 40 D, G.F, Jia Sarai,  Anand Institute of Mathematics, 
 
Near IIT, Hauz Khas, New Delhi‐16  28‐B/6, Jia Sarai, Near IIT 
 
Phone: 011‐26865455/+91‐9871145498 Hauz Khas, New Delhi‐16 
                                                   
                                             Website: www.physicsbyfiziks.com                                                                                          
                                                            Email: fiziks.physics@gmail.com                                                                     77 
fiziks
Institute for NET/JRF, GATE, IIT‐JAM, JEST, TIFR and GRE in PHYSICAL SCIENCES 
 
Ans: (c)
Solution: velocity just before hitting first time is
v1 = 2 gh = 2 × 9.8 × 5 = 9.89 m / s

After hitting velocity will be = ev1 = 0.9 × 9.89

v2 = 8.9 m / s

velocity hitting before second time will be same as v2

Q66. The Hamiltonian of a system with generalized coordinate and momentum ( q. p ) is

H = p 2 q 2 . A solution of the Hamiltonian equation of motion is (in the following A and


B are constants)
A 2 At A −2 At
(a) p = Be −2 At , q= e (b) p = Ae −2 At , q= e
B B
A − At 2 A A2t
(c) p = Ae At , q= e (d) p = 2 Ae − A t , q= e
B B
Ans: (a)
Solution: H = p 2 q 2
From Hamilton’s equation
∂H dp
= −p ⇒ = −2 p 2 q (i)
∂q dt
∂H dq
=q⇒ = 2 pq 2 (ii)
∂p dt
from equations (i) and (ii)
dp dq
=−
p q
Integrating both sides, ln p = − ln q + ln A
pq = A (iii)
from equation (i)
dp
= −2 p 2 q = −2 pA
dt
                                                                                
Head office  Branch office 
 
fiziks, H.No. 40 D, G.F, Jia Sarai,  Anand Institute of Mathematics, 
 
Near IIT, Hauz Khas, New Delhi‐16  28‐B/6, Jia Sarai, Near IIT 
 
Phone: 011‐26865455/+91‐9871145498 Hauz Khas, New Delhi‐16 
                                                   
                                             Website: www.physicsbyfiziks.com                                                                                          
                                                            Email: fiziks.physics@gmail.com                                                                     78 
fiziks
Institute for NET/JRF, GATE, IIT‐JAM, JEST, TIFR and GRE in PHYSICAL SCIENCES 
 
dp p
∫ p = − ∫ 2 Adt + ln B ⇒ ln B = −2 At ⇒ p = Be
−2 At

A 2 At
Putting this value of p in equation (iii) gives q = e
B
Hence, the correct option is (a)
Q67. A canonical transformation ( q, p ) → ( Q, P ) is made through the generating function

F ( q, p ) = q 2 p on the Hamiltonian

p2 β
H ( q, p ) = + q4
2α q 2
4

where α and β are constants. The equations of motion for ( Q, p ) are

P 4P −β Q
(a) Q = and P = − β Q (b) Q = and P =
α α 2
P 2P 2 2P
(c) Q = and P = − − βQ (d) Q = and P = − β Q
α Q α
Ans: (b)
Solution: F ( q, P ) = q 2 P

This is F2 type generating function so

∂F2 ∂F
= p & 2 =Q
∂q ∂P
1 1
p = 2qP & Q = q 2 ⇒ q = ( Q ) 2 & p = 2 ( Q ) 2 P

4QP 2 β 2 2P2 β Q2
H ( Q, P ) = + Q ⇒ +
2α Q 4 α 4
∂H 4P
⇒ =Q ⇒ Q=
∂P α
∂H βQ
and = −P ⇒ P = −
∂Q 2

                                                                                
Head office  Branch office 
 
fiziks, H.No. 40 D, G.F, Jia Sarai,  Anand Institute of Mathematics, 
 
Near IIT, Hauz Khas, New Delhi‐16  28‐B/6, Jia Sarai, Near IIT 
 
Phone: 011‐26865455/+91‐9871145498 Hauz Khas, New Delhi‐16 
                                                   
                                             Website: www.physicsbyfiziks.com                                                                                          
                                                            Email: fiziks.physics@gmail.com                                                                     79 
fiziks
Institute for NET/JRF, GATE, IIT‐JAM, JEST, TIFR and GRE in PHYSICAL SCIENCES 
 
Q68. The Lagrangian of a system moving in three dimensions is
1 2 1 1
mx1 + m ( x22 + x32 ) − kx12 − k ( x2 + x3 )
2
L=
2 2 2
The independent constants of motion is/are
(a) energy alone
(b) only energy, one component of the linear momentum and one component of the
angular momentum
(c) only energy, one component of the linear momentum
(d) only energy, one component of the angular momentum
Ans: (a)
Solution: The motion is in 3D . So don’t get confine with x1 , x2 x3 they are actually x, y, z
Langrangian is then
1 2 1 1
mx + m ( y 2 + z 2 ) − kx 2 − k ( y + z )
2
L=
2 2 2
∂L ∂L ∂L
when ≠ 0, ≠ 0, ≠0
∂x ∂y ∂z
so not any component at Linear momentum is conserve.
Now transform the Lagrangian to Hamiltonian
2
Px2 Py Pz2 1 2 1
+ kx + k ( y + z )
2
H= + +
2m 4m 4m 2 2
∂H
= 0 so energy is conserved
∂t
now Let us assume Lx = yPz − zPy

dLx ∂L
= [ Lx , H ] + x
dt ∂t

[ Lx , H ] = ⎡⎣ yPz − zPy , H ⎤⎦ ⇒ [ y, H ] Pz + y [ Pz , H ] − [ z, H ] Py − z ⎡⎣ Py , H ⎤⎦
⎡ Py2 ⎤ ⎡ 1 2⎤ ⎡ Pz2 ⎤ ⎡ 1 2⎤
⇒ ⎢ y, ⎥ Pz + y ⎢ Pz , k ( y + z ) ⎥ − ⎢ z , ⎥ Py − ⎢ Py , k ( y + z ) ⎥ z
⎣⎢ 4m ⎦⎥ ⎣ 2 ⎦ ⎣ 4m ⎦ ⎣ 2 ⎦

                                                                                
Head office  Branch office 
 
fiziks, H.No. 40 D, G.F, Jia Sarai,  Anand Institute of Mathematics, 
 
Near IIT, Hauz Khas, New Delhi‐16  28‐B/6, Jia Sarai, Near IIT 
 
Phone: 011‐26865455/+91‐9871145498 Hauz Khas, New Delhi‐16 
                                                   
                                             Website: www.physicsbyfiziks.com                                                                                          
                                                            Email: fiziks.physics@gmail.com                                                                     80 
fiziks
Institute for NET/JRF, GATE, IIT‐JAM, JEST, TIFR and GRE in PHYSICAL SCIENCES 
 
P ⎡ 1 ⎤ ⎡ P ⎤ ⎡ 1 ⎤
⇒ 2 Py z + y ⎢0 − k .2 ( y + z ) ⎥ − ⎢ 2 Py z ⎥ − ⎢0 − k .2 ( y + z ) ⎥
4m ⎣ 2 ⎦ ⎣ 4m ⎦ ⎣ 2 ⎦

⇒ ( ) ( )
− k y 2 + yz + k z 2 + yz ⇒ − k ⎡⎣ y 2 − z 2 ⎤⎦ ⇒ k ⎡⎣ z 2 − y 2 ⎤⎦

dLx dLy dL
⇒ ≠ 0 . Similarly ≠ 0 and ⇒ z ≠ 0
dt dt dt
Q69. For a particle of energy E and momentum p (in a frame F ), the rapidity y is defined

1 ⎛ E + p3c ⎞
as y = ln ⎜ ⎟ . In a frame F ′ moving with velocity v = ( 0, 0, β c ) with respect to
2 ⎝ E − p3c ⎠

F , the rapidity y′ will be

1 1 ⎛ 1+ β ⎞
(a) y′ = y + ln (1 − β 2 ) (b) y′ = y − ln ⎜ ⎟
2 2 ⎝ 1− β ⎠

⎛ 1+ β ⎞ ⎛ 1+ β ⎞
(c) y′ = y + ln ⎜ ⎟ (d) y′ = y + 2 ln ⎜ ⎟
⎝ 1− β ⎠ ⎝ 1− β ⎠
Ans: (b)

1 ⎛ E + p3c ⎞
Solution: y = ln ⎜ ⎟
2 ⎝ E − p3c ⎠

1 ⎛ E ′ + p3′c ⎞
Then y′ = ln ⎜ ⎟
2 ⎝ E ′ − p3′c ⎠

⎛ ⎛ E ⎞⎞
Where p3′ = γ ⎜ p3 − v ⎜ 2 ⎟ ⎟ E ′ = γ ( E − vp3 )
⎝ ⎝ c ⎠⎠

⎛ v ⎞
1 ⎜ ( E + p3c ) − ( E + p3c ) ⎟
Put the value of p3′ and E ′ one will get y′ = ln ⎜ c

2 ⎜ E− p c +v E− p c ⎟
⎜ ( 3 ) ( 3 ) ⎟
⎝ c ⎠

1 ⎛ ( E + p3 c ) (1 − β ) ⎞ 1 ⎛ ( E + p3 c ) ⎞ 1 ⎛ 1 − β ⎞
ln ⎜
⎜ ⎟⎟ ⇒ ln ⎜⎜ ⎟ + ln ⎜ ⎟
2 ⎝ ( E − p3 c ) (1 + β ) ⎠ 2 ⎝ ( E − p3 c ) ⎟⎠ 2 ⎝ 1 + β ⎠

1 ⎛ 1− β ⎞ 1 ⎛ 1+ β ⎞
y + ln ⎜ ⎟ ⇒ y − ln ⎜ ⎟
2 ⎝ 1+ β ⎠ 2 ⎝ 1− β ⎠

                                                                                
Head office  Branch office 
 
fiziks, H.No. 40 D, G.F, Jia Sarai,  Anand Institute of Mathematics, 
 
Near IIT, Hauz Khas, New Delhi‐16  28‐B/6, Jia Sarai, Near IIT 
 
Phone: 011‐26865455/+91‐9871145498 Hauz Khas, New Delhi‐16 
                                                   
                                             Website: www.physicsbyfiziks.com                                                                                          
                                                            Email: fiziks.physics@gmail.com                                                                     81 
fiziks
Institute for NET/JRF, GATE, IIT‐JAM, JEST, TIFR and GRE in PHYSICAL SCIENCES 
 
ELECTROMAGNETIC THEORY
JRF/NET–(JUNE-2011)
Q1. The electrostatic potential V(x, y) in free space in a region where the charge density ρ is
zero is given by V ( x, y ) = 4e 2 x + f ( x ) − 3 y 2 . Given that the x-component of the electric

field Ex, and V are zero at the origin, f ( x ) is

(a) 3x 2 − 4e 2 x + 8 x (b) 3x 2 − 4e 2 x + 16 x
(c) 4e 2 x − 8 (d) 3x 2 − 4e 2 x
Ans: (d)
Solution: V = 4e 2 x + f ( x ) − 3 y 2 . Since ρ = 0 ⇒ ∇ 2V = 0 ⇒ 16e 2 x + f ′′( x ) − 6 = 0 .

Since E x = 0 at origin ⇒ E = −∇V ⇒ E x = − 8e 2 x + f ′( x ) [ ]


E x (0, 0) = −[8 + f ′(0)] = 0 ⇒ f ′(0 ) = −8 .

Since V (0, 0 ) = 0 ⇒ 4 + f (0 ) = 0 ⇒ f (0 ) = −4

Solve equation 16e 2 x + f ′′( x ) − 6 = 0 ⇒ f ′′( x ) = 6 − 16e 2 x ⇒ f ′(x ) = 6 x − 8e 2 x + c1 , since


f ′(0 ) = −8 ⇒ c1 = 0 .

Again Integrate f ′( x ) = 6 x − 8e 2 x ⇒ f ( x ) = 3 x 2 − 4e 2 x + c 2

since f (0 ) = −4 ⇒ c 2 = 0 . Thus f ( x ) = 3x 2 − 4e 2 x

Q2. For constant uniform electric and magnetic field E = E 0 and B = B0 , it is possible to

choose a gauge such that the scalar potential φ and vector potential A are given by

(a) φ = 0 and A =
1
2
(
B0 × r ) (b) φ = − E 0 ⋅ r and A =
1
2
(
B0 × r )
(c) φ = − E 0 ⋅ r and A = 0 (d) φ = 0 and A = − E 0 t
Ans: (a)
Solution: Let E = E 0 ( xˆ + yˆ + zˆ ) and B = B0 ( xˆ + yˆ + zˆ ) since they are constant vector.

∂V
Lorentz Gauge condition is ∇ ⋅ A = − μ 0 ε 0
∂t

                                                                                
Head office  Branch office 
 
fiziks, H.No. 23, G.F, Jia Sarai,  Anand Institute of Mathematics, 
 
Near IIT, Hauz Khas, New Delhi‐16  28‐B/6, Jia Sarai, Near IIT 
 
Phone: 011‐26865455/+91‐9871145498 Hauz Khas, New Delhi‐16 
                                                   
                                             Website: www.physicsbyfiziks.com                                                                                          
                                                            Email: fiziks.physics@gmail.com                                                                     82 
fiziks
Institute for NET/JRF, GATE, IIT‐JAM, JEST, TIFR and GRE in PHYSICAL SCIENCES 
 
{since B × r = B 0 ( z − y )xˆ − B 0 ( z − x )ŷ + B 0 ( y − x )zˆ }
∂φ ∂φ
(a) = 0 and ∇ ⋅ A = 0 (b) ≠ 0, and ∇ ⋅ A = 0
∂t ∂t
∂φ ∂φ
(c) ≠ 0 and ∇ ⋅ A = 0 (d) ≠ 0 and ∇ ⋅ A ≠ 0
∂t ∂t
Q3. A plane electromagnetic wave is propagating in a lossless dielectric. The electric field is
given by
[ {
E ( x, y, z , t ) = E 0 ( xˆ + Azˆ ) exp ik 0 − ct + x + 3 z , ( )}]
where c is the speed of light in vacuum, E0, A and k0 are constant and x̂ and ẑ are unit
vectors along the x- and z-axes. The relative dielectric constant of the medium εr and the
constant A are
1 1
(a) ε r = 4 and A = − (b) ε r = 4 and A = +
3 3

(c) ε r = 4 and A = 3 (d) ε r = 4 and A = − 3


Ans: (a)
[ {
Solution: E ( x, y, z , t ) = E 0 ( xˆ + Azˆ ) exp ik 0 − ct + x + 3 z . ( )}]
(
Comparing with term e i (k ⋅r −ωt ) ⇒ k = k 0 xˆ + 3 zˆ and ω = k 0 c . )
ω k0c c
Since v = = = ⇒ Refractive index n = ε r = 2 ⇒ ε r = 4.
k k 02 + 3k 02 2

( )
Since k ⋅ nˆ = 0 ⇒ k 0 xˆ + 3 zˆ ⋅ ( xˆ + Azˆ ) = 0 ⇒ k 0 1 + A 3 = 0 ⇒ A = − ( ) 1
3
A − Kr
Q4. A static, spherically symmetric charge distribution is given by ρ (r ) = e where A and
r
K are positive constants. The electrostatic potential corresponding to this charge
distribution varies with r as

(a) re− Kr (b)


1 − Kr
r
re (c)
1 − Kr
r2
re (d)
1
r
(1 − e−Kr )

                                                                                
Head office  Branch office 
 
fiziks, H.No. 23, G.F, Jia Sarai,  Anand Institute of Mathematics, 
 
Near IIT, Hauz Khas, New Delhi‐16  28‐B/6, Jia Sarai, Near IIT 
 
Phone: 011‐26865455/+91‐9871145498 Hauz Khas, New Delhi‐16 
                                                   
                                             Website: www.physicsbyfiziks.com                                                                                          
                                                            Email: fiziks.physics@gmail.com                                                                     83 
fiziks
Institute for NET/JRF, GATE, IIT‐JAM, JEST, TIFR and GRE in PHYSICAL SCIENCES 
 
Ans: (b)
Solution: since ∇ 2V = − ρ / ε 0

A − kr 1 ∂ ⎛ 2 ∂V ⎞
∇ 2V must be proportional to e where ∇ 2V = 2 ⎜r ⎟.
r r ∂r ⎝ ∂r ⎠
Q5. The magnetic field of the TE11 mode of a rectangular waveguide of dimensions a × b as
shown in the figure is given by H z = H 1 cos(0.3π x ) cos(0.4π y ) , where x and y are in cm.
x

z
b

y
A. The dimensions of the waveguide are
(a) a = 3.33 cm, b = 2.50 cm (b) a = 0.40 cm, b = 0.30 cm
(c) a = 0.80 cm, b = 0.60 cm (d) a = 1.66 cm, b = 1.25 cm
Ans: (a)
Solution: Since H z = H 0 cos(0.3πx ) cos(0.4πy )

mπ nπ
⇒ = 0.3π where m = 1 and = 0.4π where n = 1
a b
⇒ a = 3.33cm, b = 2.50cm
B. The entire range of frequencies f for which the TE11 mode will propagate is
(a) 6.0 GHz < f < 7.5 GHz (b) 7.5 GHz < f < 9.0 GHz
(c) 7.5 GHz < f < 12.0 GHz (d) 7.5 GHz < f
Ans: (d)
2 2
c ⎛m⎞ ⎛n⎞ c 1 1
Solution: f m , n = ⎜ ⎟ + ⎜ ⎟ ⇒ f1,1 = + 2 = 7.5 GH z .
2 ⎝ a ⎠ ⎝b⎠ 2 a 2
b
For propagation, frequency of incident wave must be greater than cutoff frequency.

                                                                                
Head office  Branch office 
 
fiziks, H.No. 23, G.F, Jia Sarai,  Anand Institute of Mathematics, 
 
Near IIT, Hauz Khas, New Delhi‐16  28‐B/6, Jia Sarai, Near IIT 
 
Phone: 011‐26865455/+91‐9871145498 Hauz Khas, New Delhi‐16 
                                                   
                                             Website: www.physicsbyfiziks.com                                                                                          
                                                            Email: fiziks.physics@gmail.com                                                                     84 
fiziks
Institute for NET/JRF, GATE, IIT‐JAM, JEST, TIFR and GRE in PHYSICAL SCIENCES 
 
JRF/NET-(DEC-2011)
Q6. Consider three polarizer’s P1, P2 and P3 placed along an axis as shown in the figure.
P1 P2 P3
(unpolarized) θ
I0

The pass axis of P1 and P3 are at right angles to each other while the pass axis of P2
makes an angle θ with that of P1. A beam of unpolarized light of intensity I0 is incident
on P1 as shown. The intensity of light emerging from P3 is
I0 I0 I0
(a) 0 (b) (c) sin 2 2θ (d) sin 2 2θ
2 8 4
Ans: (c)
Solution: I = I 0 cos 2 θ (Malus Law)

I0 I0 I0 I
⇒ I1 = , I2 = cos 2 θ , I3 = cos 2 θ × cos 2 (90 − θ ) = 0 sin 2 2θ .
2 2 2 8
Q7. Four equal point charges are kept fixed at the four vertices of a square. How many neutral
points (i.e. points where the electric field vanishes) will be found inside the square?
(a) 1 (b) 4 (c) 5 (d) 7
Ans: (a)
Solution: Inside the square, there is only one point where field vanishes.

Q8. A static charge distribution gives rise to an electric field of the form E = α 1 − e − r / R ( ) rˆ ,
r2
where α and R are positive constants. The charge contained within a sphere of radius R,
centred at the origin is
e e2 R R2
(a) παε 0 (b) παε 0 (c) 4παε 0 (d) παε 0
R2 R2 e e
Ans: None of the options given are correct
π 2π
Solution: Qenc = ε 0 ∫ E ⋅ da = αε 0 ∫ 1 − e − r / R ( ) rˆ ⋅ (r 2
)
sin θdθdφrˆ = αε 0 × ∫ ∫ (1 − e
−r / R
)sin θdθdφ
r 2
0 0

                                                                                
Head office  Branch office 
 
fiziks, H.No. 23, G.F, Jia Sarai,  Anand Institute of Mathematics, 
 
Near IIT, Hauz Khas, New Delhi‐16  28‐B/6, Jia Sarai, Near IIT 
 
Phone: 011‐26865455/+91‐9871145498 Hauz Khas, New Delhi‐16 
                                                   
                                             Website: www.physicsbyfiziks.com                                                                                          
                                                            Email: fiziks.physics@gmail.com                                                                     85 
fiziks
Institute for NET/JRF, GATE, IIT‐JAM, JEST, TIFR and GRE in PHYSICAL SCIENCES 
 
⎛ 1⎞
at r = R , Qenc = 4παε 0 ⎜1 − ⎟ . So none of the options given are correct.
⎝ e⎠
Q9. In a Young’s double slit interference experiment, the slits are at a distance 2L from each
other and the screen is at a distance D from the slits. If a glass slab of refractive index μ
and thickness d is placed in the path of one of the beams, the minimum value of d for the
central fringe to be dark is
λD λD λ λ
(a) (b) (c) (d)
(μ − 1) D +L
2 2 (μ − 1)L (μ − 1) 2(μ − 1)

Ans: (d)
nλ λ
Solution: For central fringe to be dark, (μ − 1)d = ⇒d =
2 2(μ − 1)
Q10. Consider a solenoid of radius R with n turns per unit length, in which a time dependent
current I = I0 sinωt (where ωR/c << 1) flows. The magnitude of the electric field at a
perpendicular distance r < R from the axis of symmetry of the solenoid, is
1 1 1
(a) 0 (b) ωμ 0 nI 0 R 2 cos ωt (c) ωμ 0 nI 0 r sin ωt (d) ωμ 0 nI 0 r cos ωt
2r 2 2
Ans: (d)

Solution: ∫ E ⋅ dl = −∫
∂B
∂t
⋅ da ; (B = μ nI (t )zˆ ).
0

2πr 2
r
dI
⇒ E × 2πr = − μ 0 n
dt ∫ 2πr ′dr ′ = − μ 0 n × I 0ω cos ωt ×
r ′=0
2

1
⇒ E = − × ωμ 0 nI 0 r cos ωt
2
Q11. A constant electric current I in an infinitely long straight wire is suddenly switched on at t
= 0. The vector potential at a perpendicular distance r from the wire is given

by A =
kˆμ 0 I ⎡ 1

(
ln ⎢ ct + c 2 t 2 − r 2
⎣r
)⎤⎥⎦ . The electric field at a distance r (< ct) is
μ0 I 1 ˆ ˆ cμ 0 I cμ 0 I
(a) 0 (b)
2π t 2
(i − j ) (c)
1 ˆ ˆ
(
i+ j ) (d) − kˆ
2π c 2 t 2 − r 2 2 2π c 2 t 2 − r 2

                                                                                
Head office  Branch office 
 
fiziks, H.No. 23, G.F, Jia Sarai,  Anand Institute of Mathematics, 
 
Near IIT, Hauz Khas, New Delhi‐16  28‐B/6, Jia Sarai, Near IIT 
 
Phone: 011‐26865455/+91‐9871145498 Hauz Khas, New Delhi‐16 
                                                   
                                             Website: www.physicsbyfiziks.com                                                                                          
                                                            Email: fiziks.physics@gmail.com                                                                     86 
fiziks
Institute for NET/JRF, GATE, IIT‐JAM, JEST, TIFR and GRE in PHYSICAL SCIENCES 
 
Ans: (d)

∂A ∂A μ I r ⎡1 ⎛ 2c 2 t ⎞⎤
⎢ ⎜⎜ c + ⎟⎥ .
Solution: E = −∇φ −
∂t
=−
∂t
⇒E=− 0
2π ct + c 2 t 2 − r 2 ( ) ⎢⎣ r ⎝ 2 c 2t 2 − r 2

⎠⎥⎦
− cμ 0 I
⇒E= kˆ
2π c t − r2 2 2

JRF/NET-(JUNE-2012)
1 10
Q12. The magnetic field corresponding to the vector potential A = F × r + 3 r where F is
2 r
a constant vector, is
30 30
(a) F (b) − F (c) F + r (d) F − r
r4 r4
Ans: (a)

Solution: B = ∇ × A =
1
[ ( ⎛
)]
r ⎞
∇ × F × r + 10⎜ ∇ × 3 ⎟ . Since F is a constant vector, let
2 ⎝ r ⎠

xˆ yˆ zˆ
F = F0 ( xˆ + yˆ + zˆ ) , F × r = F0 F0 F0 = xˆ ( z − y )F0 − yˆ ( z − x )F0 + zˆ ( y − x )F0
x y z

xˆ yˆ zˆ
(
∇× F ×r =

∂x
) ∂
∂y

∂z
= xˆ[F0 + F0 ] − yˆ [− F0 − F0 ] + zˆ[F0 + F0 ] = 2 F0 ( xˆ + yˆ + zˆ )
(z − y )F0 (x − z )F0 ( y − x )F0

1
2
[ ( )] r
r
∇ × F × r = F0 ( xˆ + yˆ + zˆ ) = F , ∇ × 3 = 0 . Thus B = F

Q13. An electromagnetic wave is incident on a water-air interface. The phase of the


perpendicular component of the electric field, E ⊥ , of the reflected wave into the water is
found to remain the same for all angles of incidence. The phase of the magnetic field H
(a) does not change (b) changes by 3π/2
(c) changes by π/2 (d) changes by π
Ans: (d)

                                                                                
Head office  Branch office 
 
fiziks, H.No. 23, G.F, Jia Sarai,  Anand Institute of Mathematics, 
 
Near IIT, Hauz Khas, New Delhi‐16  28‐B/6, Jia Sarai, Near IIT 
 
Phone: 011‐26865455/+91‐9871145498 Hauz Khas, New Delhi‐16 
                                                   
                                             Website: www.physicsbyfiziks.com                                                                                          
                                                            Email: fiziks.physics@gmail.com                                                                     87 
fiziks
Institute for NET/JRF, GATE, IIT‐JAM, JEST, TIFR and GRE in PHYSICAL SCIENCES 
 
Q14. The magnetic field at a distance R from a long straight wire carrying a steady current I is
proportional to
(a) IR (b) I /R2 (c) I2/R2 (d) I / R
Ans: (d)
Q15. Which of the following questions is Lorentz invariant?
2 2 2 2 2 2 2
(a) E × B (b) E − B (c) E + B (d) E B

Ans: (b)
Q16. Charges Q, Q and -2Q are placed on the vertices of an equilateral triangle ABC of sides
of length a, as shown in the figure. The dipole moment of this configuration of charges,
irrespective of the choice of origin, is - 2Q
ĵ C
(a) + 2aQ iˆ
a a
(b) + 3aQ ˆj
A B
a Q
(c) − 3aQ ˆj Q

(d) 0 iˆ
Ans: (c)
Solution: Let coordinates of A is (l, m), then
⎡⎛ a ⎞ ⎛ 3a ⎞ ˆ ⎤
[ ] [ ]
p = qi ri ′ = Q liˆ + mˆj + Q (l + a )iˆ + mˆj − 2Q ⎢⎜ l + ⎟iˆ + ⎜⎜ m + ⎟ j⎥
2 ⎟⎠ ⎥⎦
⎢⎣⎝ 2 ⎠ ⎝

[ ] [ ] [
p = Q liˆ + mˆj + Q (l + a )iˆ + mˆj − Q (2l + a )iˆ + 2m + 3a ˆj ⇒ p = − 3aQˆj ( )]
m×r
Q17. The vector potential A due to a magnetic moment m at a point r is given by A = .
r3

If m is directed along the positive z-axis, the x-component of the magnetic field, at the

point r , is
3myz 3mxy 3mxz 3m(z 2 − xy )
(a) (b) − (c) (d)
r5 r5 r5 r5
Ans: (c)

                                                                                
Head office  Branch office 
 
fiziks, H.No. 23, G.F, Jia Sarai,  Anand Institute of Mathematics, 
 
Near IIT, Hauz Khas, New Delhi‐16  28‐B/6, Jia Sarai, Near IIT 
 
Phone: 011‐26865455/+91‐9871145498 Hauz Khas, New Delhi‐16 
                                                   
                                             Website: www.physicsbyfiziks.com                                                                                          
                                                            Email: fiziks.physics@gmail.com                                                                     88 
fiziks
Institute for NET/JRF, GATE, IIT‐JAM, JEST, TIFR and GRE in PHYSICAL SCIENCES 
 
Solution: m = mzˆ and
r
m
r
1
(
B = ∇ × A = 3 2 cos θrˆ + sin θθˆ = 3 [3(m ⋅ rˆ )rˆ − m] )
1 ⎡ ⎛ xxˆ + yyˆ + zzˆ ⎞ r ⎤ 3mxz
B= ⎢3mzˆ ⋅ ⎜ ⎟ − mzˆ ⎥ ⇒ Bx =
r3 ⎣ ⎝ r ⎠r ⎦ r5
JRF/NET-(DEC-2012)
Q18. Three charges are located on the circumference of a circle of radius R as shown in the
figure below. The two charges Q subtend an angle 90° at the centre
Q Q
of the circle. The charge q is symmetrically placed with respect to
the charges Q. If the electric field at the centre of the circle is zero,
what is the magnitude of Q?
(a) q / 2 (b) 2q (c) 2q (d) 4q q
Ans: (a)
1 Q 1 q
Solution: E1 = E 2 = and E3 =
4πε 0 R 2
4πε 0 R 2
q
Resultant of E1 and E 2 is E = E12 + E 22 = 2E1 , Thus E3 = E ⇒ Q =
2
Q19. Consider a hollow charged shell of inner radius a and outer radius b. The volume charge
k
density is ρ (r ) = (k is constant) in the region a < r < b. The magnitude of the electric
r2
field produced at distance r > a is
k (b − a )
(a) for all r > a ,
ε 0r 2
k (b − a ) kb
(b) for a < r < b and for r > b
ε 0r 2
ε 0r 2
k (r − a ) k (b − a )
(c) for a < r < b and for r > b
ε 0r 2
ε 0r 2
k (r − a ) k (b − a )
(d) for a < r < b and for r > b
ε 0a 2
ε 0r 2
Ans: (c)

                                                                                
Head office  Branch office 
 
fiziks, H.No. 23, G.F, Jia Sarai,  Anand Institute of Mathematics, 
 
Near IIT, Hauz Khas, New Delhi‐16  28‐B/6, Jia Sarai, Near IIT 
 
Phone: 011‐26865455/+91‐9871145498 Hauz Khas, New Delhi‐16 
                                                   
                                             Website: www.physicsbyfiziks.com                                                                                          
                                                            Email: fiziks.physics@gmail.com                                                                     89 
fiziks
Institute for NET/JRF, GATE, IIT‐JAM, JEST, TIFR and GRE in PHYSICAL SCIENCES 
 
Solution: For r < a
2 1 1 1 k 2
∫ E.da = E(4πr ) = Qenc = ∫ ρdV = ∫ r sin θdrdθdφ
ε0 ε0 ε0 r 2
4πk r 4πk k ⎛ r−a ⎞
E(4πr 2 ) = ∫a dr = (r − a) ⇒ E = ⎜ rˆ
ε0 ε0 ε0 ⎝ r 2 ⎟⎠
For r > a
4πk b 4πk k ⎛ b −a ⎞
E4πr 2 = ∫a dr = (b − a) ⇒ E = ⎜ rˆ
ε0 ε0 ε0 ⎝ r 2 ⎟⎠
Q20. Consider the interference of two coherent electromagnetic waves whose electric field
vectors are given by E1 = iˆE 0 cos ω t and E 2 = ˆjE 0 cos(ω t + ϕ ) where ϕ is the phase

ε0
difference. The intensity of the resulting wave is given by E 2 , where E 2 is the
2
time average of E2. The total intensity is
(a) 0 (b) ε 0 E 02 (c) ε 0 E 02 sin 2 ϕ (d) ε 0 E 02 cos 2 ϕ
Ans: (a)
Solution: Since waves are polarized in perpendicular direction hence there will be no
interference.
Q21. Four charges (two + q and two –q) are kept fixed at the four vertices of a square of side a
as shown. At the point P which is at a distance R q −q
from the centre (R >> a), the potential is
proportional to
(a) 1/R (b) 1/R2 a R P

(c) 1/R3 (d) 1/R4


Ans: (c)
−q q
Solution: Given configuration is quadrupole.
Q22. A point charges q of mass m is kept at a distance d below a grounded infinite conducting
sheet which lies in the xy - plane. For what value of d will the charge remains stationary?
(a) q / 4 mgπε 0 (b) q / mgπε 0

(c) There is no finite value of d (d) mgπε 0 / q


                                                                                
Head office  Branch office 
 
fiziks, H.No. 23, G.F, Jia Sarai,  Anand Institute of Mathematics, 
 
Near IIT, Hauz Khas, New Delhi‐16  28‐B/6, Jia Sarai, Near IIT 
 
Phone: 011‐26865455/+91‐9871145498 Hauz Khas, New Delhi‐16 
                                                   
                                             Website: www.physicsbyfiziks.com                                                                                          
                                                            Email: fiziks.physics@gmail.com                                                                     90 
fiziks
Institute for NET/JRF, GATE, IIT‐JAM, JEST, TIFR and GRE in PHYSICAL SCIENCES 
 
Ans: (a)
Solution: There is attractive force between point charge q and grounded conducting sheet that
q2 q 1
can be calculate from method of images i.e. = mg ⇒ d =
4πε 0 (2d ) 2
4 mgπε 0

Q23. An infinite solenoid with its axis of symmetry along the z-direction carries a steady
current I.

The vector potential A at a distance R from the axis
(a) is constant inside and varies as R outside the solenoid
R
(b) varies as R inside and is constant outside the solenoid
1
(c) varies as inside and as R outside the solenoid
R
1
(d) varies as R inside and as outside the solenoid
R
Ans: (d)
Q24. Consider an infinite conducting sheet in the xy-plane with a time dependent current
density Kt iˆ , where K is a constant. The vector potential at ( x, y , z ) is given
μ0 K
by A = (ct − z )2 iˆ . The magnetic field B is
4c
μ 0 Kt ˆ μ 0 Kz ˆ μ0 K μ0 K
(a) j (b) − j (c) − (ct − z )iˆ (d) − (ct − z ) ˆj
2 2c 2c 2c
Ans: (d)
∂Ax μ K
B = ∇× A = yˆ = − 0 (ct − z ) ˆj
∂z 2c
Q25. When a charged particle emits electromagnetic radiation, the electric field E and the
1 1
Poynting vector S = E × B at a larger distance r from emitter vary as n and
μ0 r
1
respectively. Which of the following choices for n and m are correct?
rm
(a) n = 1 and m = 1 (b) n = 2 and m = 2
(c) n = 1 and m = 2 (d) n = 2 and m = 4
Ans: (c)
                                                                                
Head office  Branch office 
 
fiziks, H.No. 23, G.F, Jia Sarai,  Anand Institute of Mathematics, 
 
Near IIT, Hauz Khas, New Delhi‐16  28‐B/6, Jia Sarai, Near IIT 
 
Phone: 011‐26865455/+91‐9871145498 Hauz Khas, New Delhi‐16 
                                                   
                                             Website: www.physicsbyfiziks.com                                                                                          
                                                            Email: fiziks.physics@gmail.com                                                                     91 
fiziks
Institute for NET/JRF, GATE, IIT‐JAM, JEST, TIFR and GRE in PHYSICAL SCIENCES 
 
JRF/NET-(JUNE-2013)
Q26. A particle of charge e and mass m is located at the midpoint of the line joining two fixed
collinear dipoles with unit charges as shown in the figure. (The particle is constrained to
move only along the line joining the dipoles). Assuming that the length of the dipoles is
much shorter than their separation, the natural frequency of oscillation of the particle is
R R

+ − e, m − +

2d 2d

6eR 2 6eR 6ed 2 6ed


(a) (b) (c) (d)
πε 0 md 5 πε 0 md 4 πε 0 mR 5 πε 0 mR 4

Ans: (d) R R

x
+ − e, m A − +

2d 2d

Let us displace the charge particle by small amount x at A. Then the resultant electric
field at point A is given by

2p ⎡ 1 1 ⎤ 6d
E= ⎢ − 3 ⎥
=− x where p = 1.2d = 2d .
4πε 0 ⎣ (R + x ) 3
(R − x ) ⎦ πε 0 R 4
6ed
F = eE = − x
πε 0 R 4
k 6ed
Then ω = =
m πε 0 mR 4

                                                                                
Head office  Branch office 
 
fiziks, H.No. 23, G.F, Jia Sarai,  Anand Institute of Mathematics, 
 
Near IIT, Hauz Khas, New Delhi‐16  28‐B/6, Jia Sarai, Near IIT 
 
Phone: 011‐26865455/+91‐9871145498 Hauz Khas, New Delhi‐16 
                                                   
                                             Website: www.physicsbyfiziks.com                                                                                          
                                                            Email: fiziks.physics@gmail.com                                                                     92 
fiziks
Institute for NET/JRF, GATE, IIT‐JAM, JEST, TIFR and GRE in PHYSICAL SCIENCES 
 
Q27. A current I is created by a narrow beam of protons moving in vacuum with constant
velocity u . The direction and magnitude, respectively, of the Poynting vector S outside
the beam at a radial distance r (much larger than the width of the beam) from the axis,
are
I2 I2
(a) S ⊥ u and S = (b) S || (− u ) and S =
4π 2 ε 0 u r 2 4π 2 ε 0 u r 4

I2 I2
(c) S || u and S = (d) S || u and S =
4π 2 ε 0 u r 2 4π 2 ε 0 u r 4

Ans: (c)
Solution: Let charge per unit length be λ , hence I = λ u in z-direction.
μ0 I ˆ
The magnetic field at a distance r is B = φ.
2πr
λ I
The electric field at a distance r is E = rˆ = rˆ .
2πε 0 r 2πε 0 ur
E×B I2
Hence Poynting vector S = = zˆ
μ0 4π 2 ε 0 ur 2
Q28. If the electric and magnetic fields are unchanged when the potential A changes (in
suitable units) according to A → A + r̂ , where r = r (t )r̂ , then the scalar potential Φ must
simultaneously change to
(a) Φ − r (b) Φ + r (c) Φ − ∂ r / ∂ t (d) Φ + ∂ r / ∂ t
Ans: (c)

Solution: A′ = A + ∇λ = A + rˆ ⇒ ∂ λ / ∂ r = 1 ⇒ λ = r + C
V ′ =V − ∂λ / ∂t =V − ∂ r / ∂t
Q29. Consider an axially symmetric static charge distribution of the form,
2
⎛ r0 ⎞ − r / r0
ρ = ρ0 ⎜ ⎟ e cos 2 ϕ
⎝r⎠
The radial component of the dipole moment due to this charge distribution is
(a) 2πρ 0 r04 (b) πρ 0 r04 (c) ρ 0 r04 (d) πρ 0 r04 / 2

                                                                                
Head office  Branch office 
 
fiziks, H.No. 23, G.F, Jia Sarai,  Anand Institute of Mathematics, 
 
Near IIT, Hauz Khas, New Delhi‐16  28‐B/6, Jia Sarai, Near IIT 
 
Phone: 011‐26865455/+91‐9871145498 Hauz Khas, New Delhi‐16 
                                                   
                                             Website: www.physicsbyfiziks.com                                                                                          
                                                            Email: fiziks.physics@gmail.com                                                                     93 
fiziks
Institute for NET/JRF, GATE, IIT‐JAM, JEST, TIFR and GRE in PHYSICAL SCIENCES 
 
Ans: (a)
2
⎛r ⎞
p = ∫ r ′ρ (r ′)dτ ′ = ∫ ∫ ∫ r ′ × ρ 0 ⎜ 0 ⎟ e − r ′ / r0 cos 2 ϕ × r ′ 2 sin θdr ′dθdϕ
V ⎝ r′ ⎠
r0 π 2π

∫ r ′e 0 dr ′∫ sin θdθ ∫ cos ϕdϕ = 2πρ 0 r0


−r′ / r
p = ρ 0 r02 2 4

r ′=0 0 0

Q30. The components of a vector potential Aμ ≡ ( A0 , A1 , A2 , A3 ) are given by

Aμ = k (− xyz, yzt , zxt , xyt )

where k is a constant. The three components of the electric field are


(a) k ( yz, zx, xy ) (b) k ( x, y, z ) (c) (0, 0, 0 ) (d) k ( xt , yt , zt )
Ans: (c)
Solution: Aμ = (φ , Ax , Ay , Az ) ⇒ φ = −kxyz, Ax = kyzt , Ay = kzxt , Az = kxyt

∂A
Since ∇φ = k (− yzxˆ − xzyˆ − xyzˆ ) and = k ( yzxˆ + xzyˆ + xyzˆ )
∂t
∂A
E = −∇φ − = k ( yzxˆ + xzyˆ + xyzˆ ) − k ( yzxˆ + xzyˆ + xyzˆ ) = 0 ⇒ E = (0,0,0 )
∂t
Q31. An oscillating current I (t ) = I 0 exp(− iωt ) flows in the direction of the y-axis through a

thin metal sheet of area 1.0 cm2 kept in the xy -plane. The rate of total energy radiated per
unit area from the surfaces of the metal sheet at a distance of 100 m is
(
(a) I 0ω / 12πε 0 c 3 ) (
(b) I 02ω 2 / 12πε 0 c 3 )
(c) I 02ω 3 / (12πε 0 c 3 ) (d) I 02ω 4 / (12πε c ) 0
3

Ans: (d)

                                                                                
Head office  Branch office 
 
fiziks, H.No. 23, G.F, Jia Sarai,  Anand Institute of Mathematics, 
 
Near IIT, Hauz Khas, New Delhi‐16  28‐B/6, Jia Sarai, Near IIT 
 
Phone: 011‐26865455/+91‐9871145498 Hauz Khas, New Delhi‐16 
                                                   
                                             Website: www.physicsbyfiziks.com                                                                                          
                                                            Email: fiziks.physics@gmail.com                                                                     94 
fiziks
Institute for NET/JRF, GATE, IIT‐JAM, JEST, TIFR and GRE in PHYSICAL SCIENCES 
 
JRF/NET-(DEC-2013)
Q32. A horizontal metal disc rotates about the vertical axis in a uniform magnetic field
pointing up as shown in the figure. A circuit is made by connecting one end A of a
resistor to the centre of the disc and the other end B to its edge through a sliding contact.
The circuit that flows through the resistor is
B
A B

(a) zero (b) DC from A to B


(c) DC from B to A (d) AC
Ans: (b)
Q33. The force between two long and parallel wires carrying currents I 1 and I 2 and separated
by a distance D is proportional to
(b) (I 1 + I 2 ) / D (c) (I 1 I 2 / D )
2
(a) I 1 I 2 / D (d) I 1 I 2 / D 2
Ans: (a)
Q34. The electric field of an electromagnetic wave is given by

E = E 0 cos[π (0.3x + 0.4 y − 1000t )]kˆ .

The associated magnetic field B is


(a) 10 −3 E 0 cos[π (0.3 x + 0.4 y − 1000t )]kˆ

)
(b) 10 −4 E 0 cos[π (0.3 x + 0.4 y − 1000t )] 4iˆ − 3 ˆj (
(c) E cos[π (0.3 x + 0.4 y − 1000t )](0.3iˆ + 0.4 ˆj )
0

(d) 10 E cos[π (0.3 x + 0.4 y − 1000t )](3iˆ + 4 ˆj )


2
0

Ans: (b)

Solution: k = π (0.3 xˆ + 0.4 yˆ ), ω = 1000t

                                                                                
Head office  Branch office 
 
fiziks, H.No. 23, G.F, Jia Sarai,  Anand Institute of Mathematics, 
 
Near IIT, Hauz Khas, New Delhi‐16  28‐B/6, Jia Sarai, Near IIT 
 
Phone: 011‐26865455/+91‐9871145498 Hauz Khas, New Delhi‐16 
                                                   
                                             Website: www.physicsbyfiziks.com                                                                                          
                                                            Email: fiziks.physics@gmail.com                                                                     95 
fiziks
Institute for NET/JRF, GATE, IIT‐JAM, JEST, TIFR and GRE in PHYSICAL SCIENCES 
 
k×E
π (0.3 xˆ + 0.4 yˆ ) × E 0 cos[π (0.3x + 0.4 y − 1000t )]kˆ
1
B= =
ω ω
⇒ B = 10 −4 E 0 cos[π (0.3x + 0.4 y − 1000t )] 4iˆ − 3 ˆj ( )
Q35. A point charge q is placed symmetrically at a distance d from two perpendicularly
placed grounded conducting infinite plates as shown in the figure. The net force on the
charge (in units of 1 / 4πε 0 ) is
q
(a)
q2
8d 2
( )
2 2 − 1 away from the corner
d

d
(b)
q2
8d 2
( )
2 2 − 1 towards the corner

q2
(c) towards the corner
2 2d 2
3q 2 F3
(d) away from the corner d d
8d 2 −q q
Ans: (b) F1
F2 d
q 2
q 2
Solution: F 1 = F 2 = k 2
and F 3 = k 2
4d 8d d
q2 +q −q
Resultant of F 1 , F 2 is F12 = F12 + F22 = 2 2k . 2d
8d 2

Net force F = k
q2
8d 2
( )
2 2 − 1 (towards the corner)

Q36. If the electrostatic potential V (r , θ , φ ) in a charge free region has the form
V (r ⋅ θ ⋅ φ ) = f (r ) cos θ , then the functional form of f (r ) (in the following a and b are
constants) is:
b b b ⎛r⎞
(a) ar 2 + (b) ar + (c) ar + (d) a ln⎜ ⎟
r r2 r ⎝b⎠
Ans: (b)

                                                                                
Head office  Branch office 
 
fiziks, H.No. 23, G.F, Jia Sarai,  Anand Institute of Mathematics, 
 
Near IIT, Hauz Khas, New Delhi‐16  28‐B/6, Jia Sarai, Near IIT 
 
Phone: 011‐26865455/+91‐9871145498 Hauz Khas, New Delhi‐16 
                                                   
                                             Website: www.physicsbyfiziks.com                                                                                          
                                                            Email: fiziks.physics@gmail.com                                                                     96 
fiziks
Institute for NET/JRF, GATE, IIT‐JAM, JEST, TIFR and GRE in PHYSICAL SCIENCES 
 
1 ∂ ⎛ ∂V ⎞ 1 ∂ ⎛ ∂V ⎞ 1 ⎛ ∂ 2V ⎞
Solution: ∇ 2V = 2 ⎜ r 2 ⎟ + ⎜ sin θ ⎟ + ⎜ ⎟=0
r ∂r ⎝ ∂r ⎠ r 2 sin θ ∂θ ⎝ ∂θ ⎠ r 2 sin 2 θ ⎝ ∂φ 2 ⎠

1 ∂ ⎛ 2 ∂f ⎞ 1 ∂
⇒ ⎜r cos θ ⎟ + 2 ( sin θ f × − sin θ ) = 0
r ∂r ⎝ ∂r ⎠ r sin θ ∂θ
2

cos θ ⎛ 2 ∂2 f ∂f ⎞ f
⇒ ⎜ r 2 + 2r ⎟ − 2 ( 2sin θ cos θ ) = 0
r2 ⎝ ∂r ∂r ⎠ r sin θ

∂2 f ∂f
⇒ r2 + 2r − 2 f ( r ) = 0
∂r
2
∂r
b
f (r ) = ar + satisfy the above equation.
r2
Q37. Let four point charges q, − q / 2, q and − q / 2 be placed at the vertices of a square of
side a . Let another point charge − q be placed at the cnetre of the square (see the figure).

−q/2 q

−q

q −q/2
Let V (r ) be the electrostatic potential at a point P at a distance r >> a from the centre
of the square. Then V (2r ) / V (r ) is
1 1 1
(a) 1 (b) (c) (d)
2 4 8
Ans: (d)
q q
Solution: According to multipole expansion Qmono = − +q− +q−q =0
2 2
q
p = q(axˆ + ayˆ ) − (axˆ + ayˆ ) − q(axˆ − ayˆ ) + q(− axˆ − ayˆ ) − q (− axˆ + ayˆ ) + 0 = 0
2 2
1 V (2r ) 1
Thus V ∝ ⇒ = .
r 3
V (r ) 8

                                                                                
Head office  Branch office 
 
fiziks, H.No. 23, G.F, Jia Sarai,  Anand Institute of Mathematics, 
 
Near IIT, Hauz Khas, New Delhi‐16  28‐B/6, Jia Sarai, Near IIT 
 
Phone: 011‐26865455/+91‐9871145498 Hauz Khas, New Delhi‐16 
                                                   
                                             Website: www.physicsbyfiziks.com                                                                                          
                                                            Email: fiziks.physics@gmail.com                                                                     97 
fiziks
Institute for NET/JRF, GATE, IIT‐JAM, JEST, TIFR and GRE in PHYSICAL SCIENCES 
 
Q38. ( ) ( )
Let V , A and V ′, A′ denote two sets of scalar and vector potentials, and ψ a scalar
function. Which of the following transformations leave the electric and magnetic fields
(and hence Maxwell’s equations) unchanged?
∂ψ ∂ψ
(a) A′ = A + ∇ψ and V ′ = V − (b) A′ = A − ∇ψ and V ′ = V + 2
∂t ∂t
∂ψ ∂ψ
(c) A′ = A + ∇ψ and V ′ = V + (d) A′ = A − ∇ψ and V ′ = V −
∂t ∂t
Ans: (a)

JRF/NET–(JUNE-2014)
Q39. A time-dependent current I (t ) = Ktzˆ (where K is a constant) is switched on at t = 0 in
an infinite current-carrying wire. The magnetic vector potential at a perpendicular
distance a from the wire is given (for time t > a / c ) by

μ 0 K c t −a μ K
2 2 2
ct
ct − a 2 + z 2 t
(a) zˆ
4π c − c ∫t − a
dz
2 2 (a 2 + z 2 )1 / 2
2
(b) zˆ 0
4π ∫ dz (a
− ct
2
+ z2 )1/ 2

c 2t 2 − a 2
μ K ct ct − a 2 + z 2 μ K t
(c) zˆ 0 ∫ dz (d) zˆ 0 ∫ dz
4π c −ct (a 2 + z 2 )1 / 2 4π − c 2t 2 − a 2 (a 2
+ z2 )
1/ 2

Ans: (a)
μ0 ∞
I ( tr ) μ0 ∞
K (t − R / c ) I
Solution: A = zˆ
4π ∫
−∞
R
dz = zˆ
4π ∫
−∞
R
dz dz
z R
c 2t 2 − a 2
μ0 K ct − a + z 2 2
⇒ A = zˆ ∫ dz •P
4π c − (a ) a
1/2
c t −a
2 2 2
2
+ z2

                                                                                
Head office  Branch office 
 
fiziks, H.No. 23, G.F, Jia Sarai,  Anand Institute of Mathematics, 
 
Near IIT, Hauz Khas, New Delhi‐16  28‐B/6, Jia Sarai, Near IIT 
 
Phone: 011‐26865455/+91‐9871145498 Hauz Khas, New Delhi‐16 
                                                   
                                             Website: www.physicsbyfiziks.com                                                                                          
                                                            Email: fiziks.physics@gmail.com                                                                     98 
fiziks
Institute for NET/JRF, GATE, IIT‐JAM, JEST, TIFR and GRE in PHYSICAL SCIENCES 
 
Q40. A current i p flows through the primary coil of a transformer. The graph of i p (t ) as a

function of time t is shown in the figure below.


i p (t )

1 2 3 t
Which of the following graphs represents the current i S in the secondary coil?
(a) (b)
is ( t ) is ( t )

1 2 3 t

1 2 3 t

is ( t )
(c) (d)
is ( t )

1 2 3t

1 2 3t
Ans: (c)
di p
Solution: is ∝ −
dt
Q41. If the electrostatic potential in spherical polar coordinates is
ϕ (r ) = ϕ 0 e − r / r0

where ϕ 0 and r0 are constants, then the charge density at a distance r = r0 will be

ε 0ϕ 0 eε 0ϕ 0 ε 0ϕ 0 2eε 0ϕ 0
(a) (b) (c) − (d) −
er 0
2
2r02 er 0
2
r02

                                                                                
Head office  Branch office 
 
fiziks, H.No. 23, G.F, Jia Sarai,  Anand Institute of Mathematics, 
 
Near IIT, Hauz Khas, New Delhi‐16  28‐B/6, Jia Sarai, Near IIT 
 
Phone: 011‐26865455/+91‐9871145498 Hauz Khas, New Delhi‐16 
                                                   
                                             Website: www.physicsbyfiziks.com                                                                                          
                                                            Email: fiziks.physics@gmail.com                                                                     99 
fiziks
Institute for NET/JRF, GATE, IIT‐JAM, JEST, TIFR and GRE in PHYSICAL SCIENCES 
 
Ans: (a)
ρ
Solution: ∵ ∇ 2φ = − ⇒ ρ = −ε 0 ( ∇ 2φ )
ε0

1 ∂ ⎛ 2 ∂φ ⎞ 1 ∂ ⎛ 2 φ0 − r / r0 ⎞ 1 φ0 ∂ 2 − r / r0 1 φ ⎡ ⎤
∇ 2φ = 2 ⎜r ⎟= 2
r ∂r ⎝ ∂r ⎠ r ∂r ⎝
⎜r ×− e
r0
⎟=− 2
r r0 ∂r
r ×e
r r0 ⎣
1
r0
( )
= − 2 0 ⎢ r 2 × − e − r / r0 + 2re − r / r0 ⎥
⎠ ⎦
φ ⎡ 1 2 ⎤
⇒ ∇ 2φ = − 0 ⎢ − e − r / r0 + e − r / r0 ⎥
r0 ⎣ r0 r ⎦

φ0 ⎡ 1 2 −1 ⎤ φ0 ⎛ φ0 ⎞ φ0ε 0
At a distance r = r0 , ∇ 2φ = − ⎢ e + e ⎥ = − 2 ⇒ ρ = −ε 0 ⎜ − 2 ⎟ = 2
−1

r0 ⎣ r0 r0 ⎦ r0 e ⎝ r0 e ⎠ r0 e

Q42. If A = yziˆ + zxˆj + xykˆ and C is the circle of unit radius in the plane defined by z = 1 ,

with the centre on the z - axis, then the value of the integral ∫ C
A ⋅ d is

π π
(a) (b) π (c) (d) 0
2 4
Ans: (d)

iˆ ˆj kˆ
∂ ∂ ∂
Solution: ∇ × A = = iˆ ( x − x ) − ˆj ( y − y ) + kˆ ( z − z ) = 0
∂x ∂y ∂z
yz zx xy

Since ∫ A ⋅ d = ∫ ∇ × A ⋅ d a = 0
C
( )
S

Q43. Consider an electromagnetic wave at the interface between two homogenous dielectric
media of dielectric constants ε 1 and ε 2 . Assuming ε 2 > ε 1 and no charges on the

surface, the electric field vector E and the displacement vector D in the two media
satisfy the following inequalities

(a) E 2 > E1 and D2 > D1 (b) E 2 < E1 and D2 < D1

(c) E 2 < E1 and D2 > D1 (d) E 2 > E1 and D2 < D1

                                                                                
Head office  Branch office 
 
fiziks, H.No. 23, G.F, Jia Sarai,  Anand Institute of Mathematics, 
 
Near IIT, Hauz Khas, New Delhi‐16  28‐B/6, Jia Sarai, Near IIT 
 
Phone: 011‐26865455/+91‐9871145498 Hauz Khas, New Delhi‐16 
                                                   
                                             Website: www.physicsbyfiziks.com                                                                                          
                                                            Email: fiziks.physics@gmail.com                                                                     100 
fiziks
Institute for NET/JRF, GATE, IIT‐JAM, JEST, TIFR and GRE in PHYSICAL SCIENCES 
 
Q44. A charge (− e ) is placed in vacuum at the point (d ,0,0 ) , where d > 0 . The region x ≤ 0

⎛d ⎞
is filled uniformly with a metal. The electric field at the point ⎜ ,0,0 ⎟ is
⎝2 ⎠
10e 10e
(a) − (1, 0, 0) (b) (1, 0, 0)
9πε 0 d 2 9πε 0 d 2
e e
(c) (1, 0, 0) (d) − (1, 0, 0)
πε 0 d 2 πε 0 d 2
Ans: (b) E−
d
E+
2 P
x
+e d 0 d −e

1 e 1 4e 1 e 1 4e
E+ = = and E− = =
4πε 0 ( 3d / 2 ) 2
4πε 0 9d 2
4πε 0 ( d / 2 ) 2
4πε 0 d 2

Thus resultant electric field at point P is


1 4e 1 4e 1 40e 1 10e 1 10e
E = E+ + E− = + = = ⇒E= xˆ
4πε 0 9d 2
4πε 0 d 2
4πε 0 9d 2
9πε 0 d 2
9πε 0 d 2
Q45. A beam of light of frequency ω is reflected from a dielectric-metal interface at normal
incidence. The refractive index of the dielectric medium is n and that of the metal is
n 2 = n(1 + iρ ) . If the beam is polarised parallel to the interface, then the phase change
experienced by the light upon reflection is
(a) tan (2 / ρ ) (b) tan −1 (1 / ρ ) (c) tan −1 (2 / ρ ) (d) tan −1 (2 ρ )
Ans: (c)
⎛ 1− β ⎞ v1 c/n
Solution: Since E0 R = ⎜ ⎟ E0 I where β = = = 1 + iρ
⎝ 1+ β ⎠ v2 c / n (1 + i ρ )

⎛ −i ρ ⎞ ⎛ ρ e−iπ /2 ⎞ ⎛ ρ ⎞ −i(π /2+θ ) ρ


⇒ E0 R = ⎜ ⎟ 0 I ⎜⎜
E = ⎟ E0 I = ⎜ ⎟e E0 I where tan θ = .
+ i ρ 2 iθ ⎟ ⎜ 4 + ρ2 ⎟
⎝ 2 ⎠ ⎝ 4+ ρ e ⎠ ⎝ ⎠ 2

2 ⎛2⎞
Thus phase change φ = − (π / 2 + θ ) ⇒ tan φ = cot θ = ⇒ φ = tan −1 ⎜ ⎟
ρ ⎝ρ⎠
                                                                                
Head office  Branch office 
 
fiziks, H.No. 23, G.F, Jia Sarai,  Anand Institute of Mathematics, 
 
Near IIT, Hauz Khas, New Delhi‐16  28‐B/6, Jia Sarai, Near IIT 
 
Phone: 011‐26865455/+91‐9871145498 Hauz Khas, New Delhi‐16 
                                                   
                                             Website: www.physicsbyfiziks.com                                                                                          
                                                            Email: fiziks.physics@gmail.com                                                                     101 
fiziks
Institute for NET/JRF, GATE, IIT‐JAM, JEST, TIFR and GRE in PHYSICAL SCIENCES 
 
Q46. A thin, infinitely long solenoid placed along the z - axis contains a magnetic flux φ .
Which of the following vector potentials corresponds to the magnetic field at an arbitrary
point (x, y, z ) ?

⎛ φ φ ⎞
(a) (Ax , Ay , Az ) = ⎜⎜ −
y x
, ,0 ⎟⎟
⎝ 2π x + y 2π x + y
2 2 2 2

⎛ φ φ ⎞
(b) (Ax , Ay , Az ) = ⎜⎜ −
y x
, ,0 ⎟⎟
⎝ 2π x + y + z 2π x + y + z
2 2 2 2 2 2

⎛ φ x+ y φ x+ y ⎞
(c) (Ax , Ay , Az ) = ⎜⎜ − , ,0 ⎟⎟
⎝ 2π x + y 2π x + y
2 2 2 2

⎛ φ φ ⎞
(d) (Ax , Ay , Az ) = ⎜⎜ −
x y
, ,0 ⎟⎟
⎝ 2π x + y 2π x + y
2 2 2 2

Ans: (a) B = ∇ × A = 0
Q47. An electromagnetically-shielded room is designed so that at a frequency ω = 10 7 rad/s
the intensity of the external radiation that penetrates the room is 1% of the incident

× 10 6 (Ωm ) is the conductivity of the shielding material, its


1 −1
radiation. If σ =

minimum thickness should be (given that ln 10 = 2.3 )
(a) 4.60 mm (b) 2.30 mm (c) 0.23 mm (d) 0.46 mm
Ans: (b)
Solution:
1 ⎛ I0 ⎞
I = I 0 e −2κ z ⇒ z =
2κ ⎜⎝ I ⎟⎠
ln

I0 σμω 1 1
where = 100, κ = = × × 106 × 4π × 10−7 × 107 = 103
I 2 2 2π
1
⇒z= ln (100 ) = 2.30 mm
2 × 103

                                                                                
Head office  Branch office 
 
fiziks, H.No. 23, G.F, Jia Sarai,  Anand Institute of Mathematics, 
 
Near IIT, Hauz Khas, New Delhi‐16  28‐B/6, Jia Sarai, Near IIT 
 
Phone: 011‐26865455/+91‐9871145498 Hauz Khas, New Delhi‐16 
                                                   
                                             Website: www.physicsbyfiziks.com                                                                                          
                                                            Email: fiziks.physics@gmail.com                                                                     102 
fiziks
Institute for NET/JRF, GATE, IIT‐JAM, JEST, TIFR and GRE in PHYSICAL SCIENCES 
 
Q48. A charged particle is at a distance d from an infinite conducting plane maintained at zero
potential. When released from rest, the particle reaches a speed u at a distance d / 2 from
the plane. At what distance from the plane will the particle reach the speed 2u ?
(a) d / 6 (b) d / 3 (c) d / 4 (d) d / 5
Ans: (b) x
d x2
1 q d x A2
q2 2
Solution: F = ma = m =− ⇒ 2 = − 2 where A = . P +q
dt 2
4πε 0 4d 2
dt x 16π mε 0
d
dv A dv dv A dx 1 d 2 d ⎛ A⎞

dt
=− 2 v ⇒v
x dt dt
=− 2
x dt

2 dt
v = ⎜ ⎟
dt ⎝ x ⎠
( ) 0
v2 A A ⎛1 1⎞ d
⇒ = + C at ⇒ x = d , v = 0 ⇒ C = − ⇒ v = 2 A ⎜ − ⎟ .
2 x d ⎝x d⎠
−q
⎛ 1 1⎞ 2A ⎛1 1⎞ d
Thus u = 2 A ⎜ − ⎟= then 2u = 2 A ⎜ − ⎟ ⇒ x =
⎝d /2 d ⎠ d ⎝x d⎠ 5

JRF/NET–(DEC-2014)
Q49. A charged particle moves in a helical path under the influence of a constant
magnetic field. The initial velocity is such that the component along the
magnetic field is twice the component in the plane normal to the magnetic l
field.
The ratio / R of the pitch to the radius R of the helical path is 2R
(a) π / 2 (b) 4π (c) 2π (d) π
Ans: (b)
Solution: v = 2v⊥

2π R 2π R l
Pitch of the helix l = v T = v = 2v⊥ = 4π R ⇒ = 4π
v⊥ v⊥ R
Q50. A parallel beam of light of wavelength λ is incident normally on a thin polymer film
with air on both sides. If the film has a refractive index n > 1 , then second-order bright
fringes can be observed in reflection when the thickness of the film is
(a) λ / 4n (b) λ / 2n (c) 3λ / 4n (d) λ / n

                                                                                
Head office  Branch office 
 
fiziks, H.No. 23, G.F, Jia Sarai,  Anand Institute of Mathematics, 
 
Near IIT, Hauz Khas, New Delhi‐16  28‐B/6, Jia Sarai, Near IIT 
 
Phone: 011‐26865455/+91‐9871145498 Hauz Khas, New Delhi‐16 
                                                   
                                             Website: www.physicsbyfiziks.com                                                                                          
                                                            Email: fiziks.physics@gmail.com                                                                     103 
fiziks
Institute for NET/JRF, GATE, IIT‐JAM, JEST, TIFR and GRE in PHYSICAL SCIENCES 
 
Ans: (c)
λ
Solution: For constructive interference: 2nd cos θ = ( 2m + 1)
2
For normal incidence (θ = 0 ) and second order ( m = 1)

λ 3λ
⇒ 2nd cos 0 = ( 2 ×1 + 1) ⇒d =
2 4n
Q51. A solid sphere of radius R has a charge density, given by
⎛ ar ⎞
ρ (r ) = ρ 0 ⎜1 − ⎟
⎝ R⎠
where r is the radial coordinate and ρ 0 , a and R are positive constants. If the

magnitude of the electric field at r = R / 2 is 1.25 times that at r = R , then the value of a
is
(a) 2 (b) 1 (c) 1 / 2 (d) 1 / 4
Ans: (b)
r
→ → 1 → 1 ⎛ ar ⎞
∫ E .d a = ε Qenc ⇒ E × 4π r = ∫ρ ⎜1 − ⎟ 4π r dr
2 2
Solution:
S 0 ε0 0
0
⎝ R⎠

4πρ0 ⎛ 2 ar 3 ⎞ 2 4πρ0 ⎛ r 3 ar 4 ⎞ ρ0 ⎛ r ar 2 ⎞
→ r →

ε 0 ∫0 ⎝
⇒ E × 4π r 2 = ⎜ r − ⎟ r dr = ⎜ − ⎟ ⇒ E = ⎜ − ⎟
R ⎠ ε 0 ⎝ 3 4R ⎠ ε 0 ⎝ 3 4R ⎠

ρ0 ⎛ R / 2 aR 2 / 4 ⎞ ρ0 ⎛ R aR 2 ⎞
∵ Er = R / 2 = 1.25 Er = R ⇒ ⎜ − ⎟ = 1.25 ⎜ − ⎟
ε0 ⎝ 3 4R ⎠ ε 0 ⎝ 3 4R ⎠

⎛1 a ⎞ 5 ⎛ 1 a ⎞ ⎛ 1 a ⎞ ⎛ 5 5a ⎞ 5a a 5 1
⇒⎜ − ⎟= ⎜ − ⎟⇒⎜ − ⎟=⎜ − ⎟⇒ − = −
⎝ 6 16 ⎠ 4 ⎝ 3 4 ⎠ ⎝ 6 16 ⎠ ⎝ 12 16 ⎠ 16 16 12 6
4a 5 − 2 a 3
⇒ = ⇒ = ⇒ a =1
16 12 4 12

                                                                                
Head office  Branch office 
 
fiziks, H.No. 23, G.F, Jia Sarai,  Anand Institute of Mathematics, 
 
Near IIT, Hauz Khas, New Delhi‐16  28‐B/6, Jia Sarai, Near IIT 
 
Phone: 011‐26865455/+91‐9871145498 Hauz Khas, New Delhi‐16 
                                                   
                                             Website: www.physicsbyfiziks.com                                                                                          
                                                            Email: fiziks.physics@gmail.com                                                                     104 
fiziks
Institute for NET/JRF, GATE, IIT‐JAM, JEST, TIFR and GRE in PHYSICAL SCIENCES 
 
Q52. The electrostatic lines of force due to a system of four point charges is sketched below.

At large distance r , the leading asymptotic behaviour of the electrostatic potential is


proportional to
(a) r (b) r −1 (c) r −2 (d) r −3
Ans: (d)
1
Solution: The given electrostatic line of force is due to a quadrupole. So V ∝ .
r3
Q53. A plane electromagnetic wave incident normally on the surface of a material is partially
reflected. Measurements on the standing wave in the region in front of the interface show
that the ratio of the electric field amplitude at the maxima and the minima is 5. The ratio
of the reflected intensity to the incident intensity is
(a) 4 / 9 (b) 2 / 3 (c) 2 / 5 (d) 1 / 5
Ans: (a)
E0 I + E0 R E 2
Solution: = 5 ⇒ E0 I + E0 R = 5 ( E0 I − E0 R ) ⇒ 6 E0 R = 4 E0 I ⇒ 0 R =
E0 I − E0 R E0 I 3
2
I ⎛E ⎞ 4
⇒ R = ⎜ 0R ⎟ =
I I ⎝ E0 I ⎠ 9
Q54. A non-relativistic particle of mass m and charge e , moving with a velocity v and
acceleration a , emits radiation of intensity I . What is the intensity of the radiation
emitted by a particle of mass m / 2 , charge 2e , velocity v / 2 and acceleration 2a ?
(a) 16 I (b) 8 I (c) 4 I (d) 2 I
Ans: (a)
q 2 a 2 sin 2 θ I 2 q22 a22 I 2 4e 2 × 4a 2
Solution: ∵ I ∝ ⇒ = ⇒ = = 16 ⇒ I 2 = 16 I
r2 I1 q12 a12 I e2 a 2
                                                                                
Head office  Branch office 
 
fiziks, H.No. 23, G.F, Jia Sarai,  Anand Institute of Mathematics, 
 
Near IIT, Hauz Khas, New Delhi‐16  28‐B/6, Jia Sarai, Near IIT 
 
Phone: 011‐26865455/+91‐9871145498 Hauz Khas, New Delhi‐16 
                                                   
                                             Website: www.physicsbyfiziks.com                                                                                          
                                                            Email: fiziks.physics@gmail.com                                                                     105 
fiziks
Institute for NET/JRF, GATE, IIT‐JAM, JEST, TIFR and GRE in PHYSICAL SCIENCES 
 
JRF/NET–(JUNE-2015)
Q55. A Plan electromagnetic wave is travelling along the positive z -direction. The maximum
electric field along the x - direction is 10 V / m . The approximate maximum values of the
power per unit area and the magnetic induction B , respectively, are
(a) 3.3 × 10−7 watts / m 2 and 10 tesla
(b) 3.3 × 10−7 watts / m 2 and 3.3 × 10−8 tesla
(c) 0.265 watts / m 2 and 10 tesla

(d) 0.265 watts / m 2 and 3.3 ×10−8 tesla


Ans. (d)
P 1 1
= c ∈0 E02 = × 3 × 108 × 8.86 × 10−12 × (10 ) = 0.132 W / m 2
2
Solution: E0 = 10V / m, I =
A 2 2
E0 10
B0 = = = 3.3 × 10−8 Tesla
c 3 ×10 8

Q56. ( ) (
Which of the following transformations V , A → V ', A ' of the electrostatic potential )
V and the vector potential A is a gauge transformation?

(
(a) V ′ = V + ax, A′ = A + at kˆ ) (
(b) V ′ = V + ax, A′ = A − at kˆ )
(
(c) V ′ = V + ax, A′ = A + at iˆ ) (
(d) V ′ = V + ax, A′ = A − at iˆ )
Ans. (d)
∂λ ∂λ
Solution: V ′ = V − ⇒ = − ax ⇒ λ = − axt + c
∂t ∂t
⇒ Δλ − atxˆ

Thus A = A − atxˆ

                                                                                
Head office  Branch office 
 
fiziks, H.No. 23, G.F, Jia Sarai,  Anand Institute of Mathematics, 
 
Near IIT, Hauz Khas, New Delhi‐16  28‐B/6, Jia Sarai, Near IIT 
 
Phone: 011‐26865455/+91‐9871145498 Hauz Khas, New Delhi‐16 
                                                   
                                             Website: www.physicsbyfiziks.com                                                                                          
                                                            Email: fiziks.physics@gmail.com                                                                     106 
fiziks
Institute for NET/JRF, GATE, IIT‐JAM, JEST, TIFR and GRE in PHYSICAL SCIENCES 
 
Q57. Suppose the yz -plane forms a chargeless boundary between two media of permittivities
∈left and ∈right where ∈left :∈right = 1: 2 if the uniform electric field on the left is

( )
Eleft = c iˆ + ˆj + kˆ (where c is a constant), than the electric field on the right Eright is

(
(a) c 2iˆ + ˆj + kˆ ) (
(b) c iˆ + 2 ˆj + 2kˆ )
⎛1 ⎞ ⎛ 1 1 ⎞
(c) c ⎜ iˆ + ˆj + kˆ ⎟ (d) c ⎜ iˆ + ˆj + kˆ ⎟
⎝2 ⎠ ⎝ 2 2 ⎠
Ans. (c)

(
Solution: E1′′ = c ˆj + kˆ = E2′′ ) 1
y
2
∈1 ⊥
D1⊥ = D2⊥ ⇒∈1 E1⊥ =∈2 E2⊥ ⇒ E21 = E1
∈2
1 ˆ x
⇒ E2⊥ = ci
2
⎛1 ⎞ z
⇒ E2 = c ⎜ iˆ + ˆj + kˆ ⎟
⎝2 ⎠
Q58. A proton moves with a speed of 300 m / s in a circular orbit in the xy -plan in a magnetic
field 1 tesla along the positive z - direction. When an electric field of 1 V / m is applied
along the positive y -direction, the center of the circular orbit
(a) remains stationary
(b) moves at 1 m / s along the negative x − direction
(c) moves at 1 m / s along the positive z − direction
(d) moves at 1 m / s along the positive x − direction
z
Ans. (d)
Solution: change particle will deflect in + x -direction with B
E 1 y
v= = =1 m/ s .
B 1 E
x

                                                                                
Head office  Branch office 
 
fiziks, H.No. 23, G.F, Jia Sarai,  Anand Institute of Mathematics, 
 
Near IIT, Hauz Khas, New Delhi‐16  28‐B/6, Jia Sarai, Near IIT 
 
Phone: 011‐26865455/+91‐9871145498 Hauz Khas, New Delhi‐16 
                                                   
                                             Website: www.physicsbyfiziks.com                                                                                          
                                                            Email: fiziks.physics@gmail.com                                                                     107 
fiziks
Institute for NET/JRF, GATE, IIT‐JAM, JEST, TIFR and GRE in PHYSICAL SCIENCES 
 
Q59. Consider a rectangular wave guide with transverse dimensions 2 m ×1 m driven with an

angular frequency ω = 109 rad / s . Which transverse electric (TE ) modes will propagate

in this wave guide?


(a) TE10 , TE01 and TE20 (b) TE01 , TE11 and TE20

(c) TE01 , TE10 and TE11 (d) TE01 , TE10 and TE22
Ans. (a)

m2 n2
Solution: ωmn = Cπ +
a 2 b2
cπ 3 ×108 × 3.14
ω10 = = = 4.71× 108 rod / sec
a 2
cπ 3 ×108 × 3.14
ω01 = = = 9.42 × 108 rod / sec
b 1

1 1
ω11 = cπ + 2 = 10.53 × 108 rod / sec
a b
2

2cπ
ω20 = = 9.72 ×108 rod / sec
a

4 4
ω22 = cπ + 2 = 10.5 × 108 rod / sec
a b
2

Since ω > ω10 , ω01 , ω22

Q60. The electric and magnetic fields in the charge free region z > 0 are given by
E ( r , t ) = E0 e − k1z cos ( k2 x − ωt ) ˆj

E0
B (r,t ) = e − k1z ⎡ k1 sin ( k2 x − ωt ) iˆ + k2 cos ( k2 x − ωt ) kˆ ⎤
ω ⎣ ⎦

where ω , k1 and k2 are positive constants. The average energy flow in the x -direction is

E02 k2 −2 k1z E02 k2 E02 k1 −2 k1z 1


(a) e (b) e −2 k1z (c) e (d) c ∈0 E02 e −2 k1 z
2μ0ω μ0ω 2μ0ω 2
Ans. (a)

                                                                                
Head office  Branch office 
 
fiziks, H.No. 23, G.F, Jia Sarai,  Anand Institute of Mathematics, 
 
Near IIT, Hauz Khas, New Delhi‐16  28‐B/6, Jia Sarai, Near IIT 
 
Phone: 011‐26865455/+91‐9871145498 Hauz Khas, New Delhi‐16 
                                                   
                                             Website: www.physicsbyfiziks.com                                                                                          
                                                            Email: fiziks.physics@gmail.com                                                                     108 
fiziks
Institute for NET/JRF, GATE, IIT‐JAM, JEST, TIFR and GRE in PHYSICAL SCIENCES 
 
2 −2 k1 z
E e
Solution: S =
1
μ
E×B = 0
μω
( )
⎡⎣ k1 cos θ sin θ ( − zˆ ) + k2 cos 2 θ xˆ ⎤⎦ ,θ = k2 x − ωt
0 0
2 −2 k1 z
k2 E e E02 k2 −2 k1 z
⇒ S = 0
= e
2 μ0ω 2μ0ω
Q61. A uniform magnetic field in the positive z -direction passes through a circular wire loop
of radius 1 cm and resistance 1 Ω lying in the xy -plane. The field strength is reduced
from 10 tesla to 9 tesla in 1s . The charge transferred across any point in the wire is
approximately
(a) 3.1× 10−4 coulomb (b) 3.4 × 10−4 coulomb
(c) 4.2 ×10−4 coulomb (d) 5.2 ×10−4 coulomb
Ans. (a)
dφ dq ε 1 dφ
Solution: ε = − ⇒I= = =−
dt dt R R dt
A −π r 2
⇒ dq = − dB = dB
R R

( )
2
−3.14 × 10−2
⇒ dq = × 1 = 3.14 × 10−4 coulomb
1
Q62. A rod of length L carries a total charge Q distributed uniformly. If this is observed in a
frame moving with a speed v along the rod, the charge per unit length (as measured by
the moving observer) is

Q ⎛ v2 ⎞ Q v2
(a) ⎜1 − 2 ⎟ (b) 1− 2
L⎝ c ⎠ L c

Q Q
(c) (d)
v2 ⎛ v2 ⎞
L 1− 2 L ⎜1 − 2 ⎟
c ⎝ c ⎠
Ans. (c)
λ0 Q
Solution: λ = =
v2 v2
1− L 1−
c2 c2
                                                                                
Head office  Branch office 
 
fiziks, H.No. 23, G.F, Jia Sarai,  Anand Institute of Mathematics, 
 
Near IIT, Hauz Khas, New Delhi‐16  28‐B/6, Jia Sarai, Near IIT 
 
Phone: 011‐26865455/+91‐9871145498 Hauz Khas, New Delhi‐16 
                                                   
                                             Website: www.physicsbyfiziks.com                                                                                          
                                                            Email: fiziks.physics@gmail.com                                                                     109 
fiziks
Institute for NET/JRF, GATE, IIT‐JAM, JEST, TIFR and GRE in PHYSICAL SCIENCES 
 
JRF/NET–(DEC-2015)
Q63. A hollow metallic sphere of radius a , which is kept at a potential V0 has a charge Q at its
centre. The potential at a point outside the sphere, at a distance r from the centre, is
Q Va Q V a2 V0 a
(a) V0 (b) + 0 (c) + 02 (d)
4π ∈0 r r 4π ∈0 r r r
Ans.: (d)
Q0
Solution: Let charge on conductor is Q0 , then V0 =
4π ∈0 a
Q0 Va
Now V= ⇒V = 0
4π ∈0 r r
Q64. Consider a charge Q at the origin of 3 - dimensional coordinate system. The
flux of the electric field through the curved surface of a cone that has a height
h
h and a circular base of radius R (as shown in the figure) is
R Q
Q Q hQ QR
(a) (b) (c) (d)
∈0 2 ∈0 R ∈0 2h ∈0
Ans.: (b)
Q65. Given a uniform magnetic field B = B0 kˆ (where B0 is a constant), a possible choice for
the magnetic vector potential A is

(a) B0 yiˆ (b) − B0 yiˆ (


(c) B0 xjˆ + yiˆ ) (
(d) B0 xiˆ + yjˆ )
Ans.: (b)
Solution: (a) ∇ × A = − B0 kˆ

(b) ∇ × A = B0 kˆ

(c) ∇ × A = 0
(d) ∇ × A = 2 B0 zˆ

Q66. A beam of unpolarized light in a medium with dielectric constant ∈1 is reflected from a

plane interface formed with another medium of dielectric constant ∈2 = 3 ∈1 . The two

                                                                                
Head office  Branch office 
 
fiziks, H.No. 23, G.F, Jia Sarai,  Anand Institute of Mathematics, 
 
Near IIT, Hauz Khas, New Delhi‐16  28‐B/6, Jia Sarai, Near IIT 
 
Phone: 011‐26865455/+91‐9871145498 Hauz Khas, New Delhi‐16 
                                                   
                                             Website: www.physicsbyfiziks.com                                                                                          
                                                            Email: fiziks.physics@gmail.com                                                                     110 
fiziks
Institute for NET/JRF, GATE, IIT‐JAM, JEST, TIFR and GRE in PHYSICAL SCIENCES 
 
media have identical magnetic permeability. If the angle of incidence is 600 , then the
reflected light
(a) is plane polarized perpendicular to the plane of incidence
(b) is plane polarized parallel to the plane of incidence
(c) is circularly polarized
(d) has the same polarization as the incident light
Ans.: (a)
θI
⎛n ⎞ ∈1
Solution: θ B = tan ⎜ 2 ⎟ −1

⎝ n1 ⎠ ∈2

⎛ ∈x ⎞
θ B = tan −1 ⎜
⎜ ∈
⎟ = tan −1
⎟ ( 3)
⎝ 1 ⎠
⇒ θ B = 600 (hence reflected light is plane polarized perpendicular to plane of incidence))

Q67. A small magnetic needle is kept at ( 0, 0 ) with its moment along the x -axis. Another

small magnetic needle is at the point (1,1) and is free to rotate in the xy - plane. In

equilibrium the angle θ between their magnetic moments is such that


1
(a) tan θ = (b) tan θ = 0 (c) tan θ = 3 (d) tan θ = 1
3
Ans.: (c)
μ0 μ mm
Solution: U = ⎡ m ⋅ m2 − 3 ( m1 ⋅ rˆ )( m2 ⋅ rˆ ) ⎤⎦ ⇒ U = 0 1 3 2 ⎡cos θ − 3cos 450 cos (θ − 450 ) ⎤
3 ⎣ 1
4π r 4π r ⎣ ⎦

For stable position energy is minimum i.e. r̂ y


m2
∂U μ mm ⎡ ⎤ r
∂θ
=0⇒ 0 132
4π r ⎢sin θ +
3
(
sin θ − 450 ⎥ = 0 ) θ
⎣ 2 ⎦ 450 x
m1
3 ⎛ sin θ cos θ ⎞
⇒ sin θ = ⎜ − ⎟ ⇒ tan θ = 3
2⎝ 2 2 ⎠
so, option (c) is correct .

                                                                                
Head office  Branch office 
 
fiziks, H.No. 23, G.F, Jia Sarai,  Anand Institute of Mathematics, 
 
Near IIT, Hauz Khas, New Delhi‐16  28‐B/6, Jia Sarai, Near IIT 
 
Phone: 011‐26865455/+91‐9871145498 Hauz Khas, New Delhi‐16 
                                                   
                                             Website: www.physicsbyfiziks.com                                                                                          
                                                            Email: fiziks.physics@gmail.com                                                                     111 
fiziks
Institute for NET/JRF, GATE, IIT‐JAM, JEST, TIFR and GRE in PHYSICAL SCIENCES 
 
Q68. A dipole of moment p , oscillating at frequency ω , radiates spherical waves. The vector
potential at large distance is
μ0 eikr
A(r ) = iω p
4π r
⎛1⎞
To order ⎜ ⎟ the magnetic field B at a point r = rnˆ is
⎝r⎠
μ0 ω 2 eikr μ0 ω 2 eikr
(a) − ( )
n
ˆ ⋅ p n
ˆ (b) − ( n
ˆ × p )
4π C r 4π C r
μ0 2 eikr π 0 ω 2 eikr
(c) − ω k ( nˆ ⋅ p ) p (d) − p
4π r 4π C r
Ans.: (b)
Solution: Let p = pzˆ , then B must be in φˆ direction.

Check nˆ × p = rˆ × zˆ = φˆ . So, correct option is (b).


Q69. The frequency dependent dielectric constant of a material is given by
A
ε (ω ) = 1 +
ω − ω 2 − iωγ
2
0

where A is a positive constant, ω0 the resonant frequency and γ the damping

coefficient. For an electromagnetic wave of angular frequency ω << ω0 which of the

γ
following is true? (Assume that << 1 ).
ω0
(a) There is negligible absorption of the wave
(b) The wave propagation is highly dispersive
(c) There is strong absorption of the electromagnetic wave
(d) The group velocity and the phase velocity will have opposite sign
Ans.: (a)
Solution: When ω << ω0 , there is negligible absorption of the wave.

                                                                                
Head office  Branch office 
 
fiziks, H.No. 23, G.F, Jia Sarai,  Anand Institute of Mathematics, 
 
Near IIT, Hauz Khas, New Delhi‐16  28‐B/6, Jia Sarai, Near IIT 
 
Phone: 011‐26865455/+91‐9871145498 Hauz Khas, New Delhi‐16 
                                                   
                                             Website: www.physicsbyfiziks.com                                                                                          
                                                            Email: fiziks.physics@gmail.com                                                                     112 
fiziks
Institute for NET/JRF, GATE, IIT‐JAM, JEST, TIFR and GRE in PHYSICAL SCIENCES 
 
JRF/NET–(JUNE-2016)
Q70. Four equal charges of +Q each are kept at the vertices of a square of side R . A particle of

mass m and charge +Q is placed in the plane of the square at a short distance a ( R)

from the centre. If the motion of the particle is confined to the plane, it will undergo
small oscillations with an angular frequency

Q2 Q2
(a) (b)
2πε 0 R 3 m πε 0 R 3m

2Q 2 Q2
(c) (d)
πε 0 R 3 m 4πε 0 R 3m

Ans: (c)
kQ +Q +Q
Solution: E1 = E2 =
⎡⎛ R ⎞ R2 ⎤
2
E2 E1 y
⎢⎜ a + ⎟ + ⎥
⎣⎢⎝ 2⎠ 4 ⎥⎦
θa x
Resultant field E12, y = 2 E1 cos θ
R /2
2kQ ⎛ R⎞ 2kQ ⎛ R⎞
E12, y = ⎜a + ⎟ ≈ ⎜a + ⎟ +Q R /2 +Q
⎝ 2⎠ ⎝ 2⎠
33
⎡⎛ R⎞ R ⎤
2 2 ⎡R ⎤
2 2
2

⎢⎜ a + ⎟ + ⎥ ⎢ 2 ⎥
⎢⎣⎝ 2⎠ 4 ⎥⎦ ⎣ ⎦

4 2kQ ⎛ R⎞
E12, y = ⎜a + ⎟
R 3
⎝ 2⎠
+Q R /2 +Q
kQ R
Similarly; E3 = E4 = φ 2 −a
⎡⎛ R ⎞ R ⎤
2 2

⎢⎜ − a ⎟ + ⎥
⎢⎣⎝ 2 ⎠ 4 ⎥⎦ E3 a E4

2kQ ⎛R ⎞
Resultant E34, y = 2 E3 cos φ = ⎜ − a⎟
⎝2 ⎠
3
⎡⎛ R ⎞ R ⎤
2 2 2

⎢⎜ − a ⎟ + ⎥ +Q +Q
⎢⎣⎝ 2 ⎠ 4 ⎥⎦

                                                                                
Head office  Branch office 
 
fiziks, H.No. 23, G.F, Jia Sarai,  Anand Institute of Mathematics, 
 
Near IIT, Hauz Khas, New Delhi‐16  28‐B/6, Jia Sarai, Near IIT 
 
Phone: 011‐26865455/+91‐9871145498 Hauz Khas, New Delhi‐16 
                                                   
                                             Website: www.physicsbyfiziks.com                                                                                          
                                                            Email: fiziks.physics@gmail.com                                                                     113 
fiziks
Institute for NET/JRF, GATE, IIT‐JAM, JEST, TIFR and GRE in PHYSICAL SCIENCES 
 
4 2kQ ⎛ R ⎞
⇒ E34, y = ⎜ + a⎟
R 3
⎝2 ⎠

4 2kQ ⎡⎛ R ⎞ ⎛R ⎞⎤ 8 2kQ
Resultant E = ⎢ ⎜ − a ⎟ − ⎜ + a ⎟⎥ = − a
R 3
⎣⎝ 2 ⎠ ⎝2 ⎠⎦ R3

−8 2 1 2 2Q
⇒ E= × Qa ⇒ E = − a
R 3
4πε 0 πε 0 R 3

2 2Q 2 2 2Q 2
⇒ F = QE = − a⇒ω=
πε 0 R 3 πε 0 mR 3
Q71. Two parallel plate capacitors, separated by distances x and 1.1x respectively, have a
dielectric material of dielectric constant 3.0 inserted between the plates and are
connected to a battery of voltage V . The difference in charge on the second capacitor
compared to the first is
(a) +66% (b) +20% (c) −3.3% (d) −10%
Ans: (d)
3ε 0 A 3ε A
Solution: Q1 = C1V1 = V , Q2 = C2V2 = 0 V
x 1.1x
⎛ 1 ⎞ 3ε A
⎜ − 1⎟ × 0 V
Q2 − Q1 x
× 100% = ⎝
1.1 ⎠
× 100 = −10%
Q1 3 ε A
0
V
x
Q72. The half space region x > 0 and x < 0 are filled with dielectric
x<0 x>0
media of dielectric constants ε1 and ε 2 respectively. There is a ε2 ε1
uniform electric field in each part. In the right half, the electric field θ1 E1

makes an angle θ1 to the interface. The corresponding angle θ 2 in


the left half satisfies θ2
(a) ε1 sin θ 2 = ε 2 sin θ1 (b) ε1 tan θ 2 = ε 2 tan θ1
E2
(c) ε1 tan θ1 = ε 2 tan θ 2 (d) ε1 sin θ1 = ε 2 sin θ 2
Ans: (c)

                                                                                
Head office  Branch office 
 
fiziks, H.No. 23, G.F, Jia Sarai,  Anand Institute of Mathematics, 
 
Near IIT, Hauz Khas, New Delhi‐16  28‐B/6, Jia Sarai, Near IIT 
 
Phone: 011‐26865455/+91‐9871145498 Hauz Khas, New Delhi‐16 
                                                   
                                             Website: www.physicsbyfiziks.com                                                                                          
                                                            Email: fiziks.physics@gmail.com                                                                     114 
fiziks
Institute for NET/JRF, GATE, IIT‐JAM, JEST, TIFR and GRE in PHYSICAL SCIENCES 
 

E1
tan θ1 E E⊥
Solution: = 1⊥ = 1⊥
tan θ 2 E2 E2
∵ E1 = E2 ( )
E2
E1⊥ ε 2
D1⊥ = D2⊥ ⇒ ε1 E1⊥ = ε 2 E2⊥ ⇒ =
E2⊥ ε1
tan θ1 ε 2
⇒ = ⇒ ε1 tan θ1 = ε 2 tan θ 2
tan θ 2 ε1

Q73. The x - and z -components of a static magnetic field in a region are Bx = B0 x 2 − y 2 ( )


and Bz = 0 , respectively. Which of the following solutions for its y -component is
consistent with the Maxwell equations?
(a) By = B0 xy (b) By = −2 B0 xy
⎛1 ⎞
(
(c) By = − B0 x 2 − y 2 ) (d) By = B0 ⎜ x3 − xy 2 ⎟
⎝3 ⎠
Ans: (b)

(
Solution: Bx = B0 x 2 − y 2 , Bz = 0 )
∂Bx ∂By ∂Bz ∂By ∂B
∵∇ ⋅ B = 0 ⇒ + + =0⇒ = − x = −2 B0 x ⇒ By = −2 B0 xy
∂x ∂y ∂z ∂y ∂x
Q74. A magnetic field B is Bzˆ in the region x > 0 and zero elsewhere. A rectangular loop, in
the xy -plane, of sides l (along the x -direction) and h (along the y - direction) is
inserted into the x > 0 region from the x < 0 region at constant velocity v = vxˆ . Which of
the following values of l and h will generate the largest EMF?
(a) l = 8, h = 3 (b) l = 4, h = 6 (c) l = 6, h = 4 (d) l = 12, h = 2
Ans: (b) z
Solution: φm ∝ Bhx h
l
−dφm v
ε∝ ∝ Bvh ∝ h
dt y

x
                                                                                
Head office  Branch office 
 
fiziks, H.No. 23, G.F, Jia Sarai,  Anand Institute of Mathematics, 
 
Near IIT, Hauz Khas, New Delhi‐16  28‐B/6, Jia Sarai, Near IIT 
 
Phone: 011‐26865455/+91‐9871145498 Hauz Khas, New Delhi‐16 
                                                   
                                             Website: www.physicsbyfiziks.com                                                                                          
                                                            Email: fiziks.physics@gmail.com                                                                     115 
fiziks
Institute for NET/JRF, GATE, IIT‐JAM, JEST, TIFR and GRE in PHYSICAL SCIENCES 
 
Q75. Consider a sphere S1 of radius R which carries a uniform charge
S2
R S
of density ρ . A smaller sphere S2 of radius a < is cut out and 1 P
2
b
removed from it. The centres of the two spheres are separated by r
nR
ˆ
the vector b = , as shown in the figure. The electric field at a
2
point P inside S2 is

ρR ρR ρR ρa
(a) nˆ (b) ( r − na
ˆ ) (c) nˆ (d) r
3ε 0 3ε 0 a 6ε 0 3ε 0 R
Ans: (c)
ρ
Solution: Electric field at P due to S1 is E1 = r+
3ε 0 S2
−ρ S1 r−
Electric field at P due to S2 (assume − ρ ) is E2 = r−
3ε 0 b P
r+
ρ
Thus E = E1 + E2 = ( r+ − r− ) ; ∵ b + r− = r+ ⇒ r+ − r− = b
3ε 0

ρ ρR ⎛ R ⎞
E= b= nˆ ⎜∵ b = nˆ ⎟
3ε 0 6ε 0 ⎝ 2 ⎠
Q76. The value of the electric and magnetic fields in a particular reference frame (in Gaussian
units) are E = 3xˆ + 4 yˆ and B = 3 zˆ respectively. An inertial observer moving with respect

to this frame measures the magnitude of the electric field to be E ′ = 4 . The magnitude of

the magnetic field B′ measured by him is

(a) 5 (b) 9 (c) 0 (d) 1


Ans: (c)
Solution:
∵ E 2 − B 2 = E ′2 − B′2 = constant ⇒ ( 9 + 16 ) − 9 = 16 − B′2 ⇒ B′ = 0

                                                                                
Head office  Branch office 
 
fiziks, H.No. 23, G.F, Jia Sarai,  Anand Institute of Mathematics, 
 
Near IIT, Hauz Khas, New Delhi‐16  28‐B/6, Jia Sarai, Near IIT 
 
Phone: 011‐26865455/+91‐9871145498 Hauz Khas, New Delhi‐16 
                                                   
                                             Website: www.physicsbyfiziks.com                                                                                          
                                                            Email: fiziks.physics@gmail.com                                                                     116 
fiziks
Institute for NET/JRF, GATE, IIT‐JAM, JEST, TIFR and GRE in PHYSICAL SCIENCES 
 
Q77. A loop of radius a , carrying a current I , is placed in a uniform magnetic field B . If the
normal to the loop is denoted by n̂ , the force F and the torque T on the loop are
μ0
(a) F = 0 and T = π a 2 I n̂× B (b) F = I×B

μ0 1
(c) F = I × B and T = I nˆ × B (d) F = 0 and T = IB
4π μ 0ε 0
Ans: (a)
Solution: In uniform field F = 0
Torque T = m × B = π a 2 Inˆ × B
Q78. A waveguide has a square cross-section of side 2a . For the TM modes of wave vector k ,
the transverse electromagnetic modes are obtained in terms of a function ψ ( x, y ) which

obeys the equation


⎡ ∂2 ∂2 ⎛ ω 2 ⎞⎤
⎢ 2 + + ⎜ 2 − k 2 ⎟ ⎥ψ ( x, y ) = 0
⎣ ∂x ∂y ⎝ c
2
⎠⎦

with the boundary condition ψ ( ± a, y ) = ψ ( x, ± a ) = 0 . The frequency ω of the lowest

mode is given by
⎛ 4π 2 ⎞ ⎛ π2 ⎞
(a) ω 2 = c 2 ⎜ k 2 + 2 ⎟ (b) ω 2 = c 2 ⎜ k 2 + 2 ⎟
⎝ a ⎠ ⎝ a ⎠

⎛ 2 π2 ⎞ ⎛ 2 π2 ⎞
(c) ω = c ⎜ k + 2 ⎟
2 2
(d) ω = c ⎜ k + 2 ⎟
2 2

⎝ 2a ⎠ ⎝ 4a ⎠

Ans: (c)
Solution: c 2 k 2 = ω 2 − ωmn
2
⇒ ω 2 = c 2 k 2 + ωmn
2

⎛ m2 n2 ⎞ ⎡ 1 1 ⎤
⇒ ωmn
2
= c 2π 2 ⎜ 2 + 2 ⎟ ⇒ ω112 = c 2π 2 ⎢ + ⎥
⎝a b ⎠ ⎢⎣ ( 2a ) ( 2a ) ⎥⎦
2 2

1 c 2π 2 2⎛ 2 π2 ⎞
⇒ ω112 = c 2π 2 × = ⇒ ω 2
= c ⎜ k + ⎟
2a 2 2a 2 ⎝ 2a 2 ⎠

                                                                                
Head office  Branch office 
 
fiziks, H.No. 23, G.F, Jia Sarai,  Anand Institute of Mathematics, 
 
Near IIT, Hauz Khas, New Delhi‐16  28‐B/6, Jia Sarai, Near IIT 
 
Phone: 011‐26865455/+91‐9871145498 Hauz Khas, New Delhi‐16 
                                                   
                                             Website: www.physicsbyfiziks.com                                                                                          
                                                            Email: fiziks.physics@gmail.com                                                                     117 
fiziks
Institute for NET/JRF, GATE, IIT‐JAM, JEST, TIFR and GRE in PHYSICAL SCIENCES 
 
QUANTUM MECHANICS SOLUTIONS
NET/JRF (JUNE-2011)
⎛ 1 ⎞
Q1. The wavefunction of a particle is given by ψ = ⎜ φ 0 + iφ1 ⎟ , where φ0 and φ1 are the
⎝ 2 ⎠
normalized eigenfunctions with energies E0 and E1 corresponding to the ground state
and first excited state, respectively. The expectation value of the Hamiltonian in the state
ψ is
E E E − 2 E1 E + 2 E1
(a) 0 + E1 (b) 0 − E1 (c) 0 (d) 0
2 2 3 3
Ans: (d)
1 ψ Hψ E 0 + 2 E1
Solution: ψ = φ0 + iφ1 and H = =
2 ψψ 3
Q2. The energy levels of the non-relativistic electron in a hydrogen atom (i.e. in a Coulomb
potential V (r ) ∝ −1 / r ) are given by E nlm ∝ −1 / n 2 , where n is the principal quantum

number, and the corresponding wave functions are given by ψ nlm , where l is the orbital

angular momentum quantum number and m is the magnetic quantum number. The spin of
the electron is not considered. Which of the following is a correct statement?
(a) There are exactly (2l + 1) different wave functionsψ nlm , for each Enlm.

(b) There are l(l + 1) different wave functions ψ nlm , for each Enlm.

(c) Enlm does not depend on l and m for the Coulomb potential.
(d) There is a unique wave function ψ nlm for each Enlm.
Ans: (c)
Q3. The Hamiltonian of an electron in a constant magnetic field B is given by H = μσ ⋅ B .
where μ is a positive constant and σ = (σ 1 , σ 2 , σ 3 ) denotes the Pauli matrices. Let

ω = μB / and I be the 2 × 2 unit matrix. Then the operator e i H t / simplifies to

ωt iσ ⋅ B ωt iσ ⋅ B
(a) I cos + sin (b) I cos ω t + sin ω t
2 B 2 B
iσ ⋅ B iσ ⋅ B
(c) I sin ω t + cos ω t (d) I sin 2ω t + cos 2ω t
B B
                                                                                
Head office  Branch office 
 
fiziks, H.No. 40‐D, G.F, Jia Sarai,  Anand Institute of Mathematics, 
 
Near IIT, Hauz Khas, New Delhi‐16  28‐B/6, Jia Sarai, Near IIT 
 
Phone: 011‐26865455/+91‐9871145498 Hauz Khas, New Delhi‐16 
                                                   
                                             Website: www.physicsbyfiziks.com                                                                                          
                                                            Email: fiziks.physics@gmail.com                                                                     118 
fiziks
Institute for NET/JRF, GATE, IIT‐JAM, JEST, TIFR and GRE in PHYSICAL SCIENCES 
 
Ans: (b)
Solution: H = μσ B where σ = (σ 1 , σ 2 , σ 3 ) are pauli spin matrices and B are constant magnetic

( ) ( )
field. σ = σ 1iˆ, σ 2 ˆj , σ 3 kˆ , B = Bx iˆ + By ˆj + Bz kˆ and Hamiltonion H = μσ ⋅ B in matrices

⎛ Bz Bx − iBy ⎞
form is given by μ = ⎜ . Eigenvalue of given matrices are given by
⎝ Bx + iBy − Bz ⎟⎠

+ μ B and − μ B . H matrices are not diagonals so eiH t / is equivalent to

⎛ 1 1 ⎞
⎛ iμ Bt ⎞
e 0 ⎟ ⎜ 2 2 ⎟
S −1 ⎜ S where S is unitary matrices and S −1 = S = ⎜ ⎟.
⎜ i μ Bt ⎟
⎜ 1 1 ⎟
⎜⎝ 0 e ⎟⎠ −
⎜⎝ ⎟
2 2⎠

⎛ 1 1 ⎞ iμ Bt ⎛ 1 1 ⎞
⎛ iμ Bt ⎞ ⎛ ⎞
e 0 ⎟ ⎜ 2 2 ⎟ ⎜e 0 ⎟ ⎜ 2 2 ⎟
S −1 ⎜ S=⎜ ⎟ ⎜ ⎟ where ω = μB / .
⎜ i μ Bt ⎟
⎜ 1 1 ⎟ ⎜⎜ i μ Bt ⎟
1 1 ⎟
⎜⎝ 0 e ⎟⎠ ⎜⎝ − ⎟⎝ 0 e ⎟⎠ ⎜⎜ − ⎟
2 2⎠ ⎝ 2 2⎠

⎛ cos ω t i sin ω t ⎞
eiHt / = ⎜ which is equivalent to I cos ω t + iσ x sin ω t can be written
⎝ i sin ω t cos ω t ⎟⎠

iσ ⋅ B iσ ⋅ B
as I cos ω t + sin ω t where σ x =
B B
Q4. If the perturbation H' = ax, where a is a constant, is added to the infinite square well
potential

V (x ) = ⎧⎨0 for 0≤ x ≤π
⎩∞ otherwise.
The correction to the ground state energy, to first order in a, is
aπ aπ aπ
(a) (b) aπ (c) (d)
2 4 2
Ans: (a)
π π
a⋅2 πx aπ 2 πx
Solution: E01 = ∫ψ 0* H 'ψ 0 dx = ∫ x sin
2
dx = ∵ψ 0 = sin .
0
π 0
π 2 π π

                                                                                
Head office  Branch office 
 
fiziks, H.No. 40‐D, G.F, Jia Sarai,  Anand Institute of Mathematics, 
 
Near IIT, Hauz Khas, New Delhi‐16  28‐B/6, Jia Sarai, Near IIT 
 
Phone: 011‐26865455/+91‐9871145498 Hauz Khas, New Delhi‐16 
                                                   
                                             Website: www.physicsbyfiziks.com                                                                                          
                                                            Email: fiziks.physics@gmail.com                                                                     119 
fiziks
Institute for NET/JRF, GATE, IIT‐JAM, JEST, TIFR and GRE in PHYSICAL SCIENCES 
 
Q5. A particle in one dimension moves under the influence of a potential V ( x ) = ax 6 , where a
is a real constant. For large n the quantized energy level En depends on n as:
(a) En ~ n3 (b) En ~ n4/3 (c) En ~ n6/5 (d) En ~ n3/2
Ans: (d)
p x2 p x2
+ ax 6 and p x = 2m(E − ax 6 ) 2 . [ ]
1
Solution: V ( x ) = ax 6 , H = + ax 6 , E =
2m 2m
According to W.K.B approximation pdx ≅ nh

∫ ( 2m ( E − ax ))
1/ 2
6
dx ∝ n

We can find this integration without solving the integration


Px
1 2mE
2 6 1/ 6 1/ 6
p 2
x p x ⎛E⎞ 6
E= + ax 6 ⇒ + =1 ⇒ x = ⎜ ⎟
x
at p x = 0 . − (E / a ) (E / a )
2m 2mE E / a ⎝a⎠ x
Area of Ellipse = semi major axis × semiminor axis
− 2mE
1
3
⎛E⎞ 6
= π 2mE × ⎜ ⎟ ∝ n ⇒ E ∝ n 2 .
⎝a⎠
1
Q6. (A) In a system consisting of two spin particles labeled 1 and 2, let S (1) = σ (1) and
2 2

S (2 ) = σ (2 ) denote the corresponding spin operators. Here σ ≡ (σ x , σ y , σ z ) and


2
σ x , σ y , σ z are the three Pauli matrices.

In the standard basis the matrices for the operators S x(1)S y(2 ) and S y(1)S x(2 ) are respectively,
2 2 2 2
(a) ⎛1 0 ⎞, ⎛ − 1 0 ⎞ ⎛ i 0 ⎞, ⎛ − i 0 ⎞
⎜ ⎟ ⎜ ⎟ (b) ⎜ ⎟ ⎜ ⎟
4 ⎝ 0 − 1⎠ 4 ⎝ 0 1 ⎠ 4 ⎝0 − i⎠ 4 ⎝ 0 i ⎠
⎛0 1 0 0 ⎞ ⎛0 −i 0 0⎞
⎛0 0 0 − i⎞ 2 ⎛0 0 0 − i⎞ ⎜ ⎟ 2⎜ ⎟
2
⎜0 0 i 0 ⎟, ⎜ 0 0 −i 0⎟
2
⎜1 0 0 0 ⎟ ⎜i 0 0 0⎟
(c) ⎜ (d) , ⎜
4 ⎜0 − i 0 0 ⎟ 4 ⎜0 i 0 0⎟ 4 ⎜0 0 0 ⎟
−i 4 0 0 0 1⎟
⎝i 0 0 0 ⎟⎠ ⎜i
⎝ 0 0 0 ⎟⎠ ⎜
⎜0
⎟ ⎜ ⎟
⎝ 0 i 0 ⎟⎠ ⎜0
⎝ 0 1 0 ⎟⎠
Ans: (c)

                                                                                
Head office  Branch office 
 
fiziks, H.No. 40‐D, G.F, Jia Sarai,  Anand Institute of Mathematics, 
 
Near IIT, Hauz Khas, New Delhi‐16  28‐B/6, Jia Sarai, Near IIT 
 
Phone: 011‐26865455/+91‐9871145498 Hauz Khas, New Delhi‐16 
                                                   
                                             Website: www.physicsbyfiziks.com                                                                                          
                                                            Email: fiziks.physics@gmail.com                                                                     120 
fiziks
Institute for NET/JRF, GATE, IIT‐JAM, JEST, TIFR and GRE in PHYSICAL SCIENCES 
 
⎛0 0 0 −i ⎞
2 ⎜ ⎟
(1) ( 2 )
2
⎛0 1 ⎞ ⎛0 −i ⎞ ⎜ 0 0 i 0 ⎟
Solution: Sx Sy = ⎜ ⎟⊗⎜ ⎟⇒ ⎜
4 ⎝1 0⎠ ⎝i 0 ⎠ 4 0 −i 0 0 ⎟
⎜⎜ i 0 0

0 ⎟⎠

⎛0 0 0 −i ⎞
−i ⎞ ⎛ 0 ⎜ ⎟
2
⎛0 1⎞ ⎜0
2
0 −i 0 ⎟
S y(1) S x( 2) = ⊗ ⇒
4 ⎜⎝ i 0 ⎟⎠ ⎜⎝1 ⎟
0⎠ 4 ⎜0 i 0 0 ⎟
⎜⎜ ⎟
⎝i 0 0 0 ⎟⎠
(B) These two operators satisfy the relation
{ }
(a) S x(1)S y(2 ) , S y(1)S x(2 ) = S z(1)S z(2 ) { }
(b) S x(1)S y(2 ) , S y(1)S x(2 ) = 0

(c) [S ( )S ( ) , S ( )S ( ) ] = iS ( )S ( )
x
1
y
2
y
1
x
2
z
1
z
2
(d) [S ( )S ( ) , S ( )S ( ) ] = 0
x
1
y
2
y
1
x
2

Ans: (d)
Solution: We have matrix S(x1)S(y2) and S(y1)S(x2) from question 6(A) so commutation is given by

[S ( ) S ( ) , S ( ) S ( ) ] = 0 .
x
1
y
2
y
1
x
2

NET/JRF (DEC-2011)

Q7. The energy of the first excited quantum state of a particle in the two-dimensional

potential V ( x, y ) =
1
2
(
mω 2 x 2 + 4 y 2 is)
3 5
(a) 2 ω (b) 3 ω (c) ω (d) ω
2 2
Ans: (d)
1 1 1 ⎛ 1⎞ ⎛ 1⎞
Solution: V ( x, y ) = mω 2 (x 2 + 4 y 2 ) = mω2 x 2 + m4ω2 y 2 , E = ⎜ n x + ⎟ ω + ⎜ n y + ⎟ 2 ω
2 2 2 ⎝ 2⎠ ⎝ 2⎠
ω 1 3 ω
For ground state energy n x = 0, n y = 0 ⇒ E = + 2 ω=
2 2 2
3 ω 5 ω
First exited state energy n x = 1, n y = 0 ⇒ + ω=
2 2

                                                                                
Head office  Branch office 
 
fiziks, H.No. 40‐D, G.F, Jia Sarai,  Anand Institute of Mathematics, 
 
Near IIT, Hauz Khas, New Delhi‐16  28‐B/6, Jia Sarai, Near IIT 
 
Phone: 011‐26865455/+91‐9871145498 Hauz Khas, New Delhi‐16 
                                                   
                                             Website: www.physicsbyfiziks.com                                                                                          
                                                            Email: fiziks.physics@gmail.com                                                                     121 
fiziks
Institute for NET/JRF, GATE, IIT‐JAM, JEST, TIFR and GRE in PHYSICAL SCIENCES 
 
Q8. Consider a particle in a one dimensional potential that satisfies V ( x ) = V (− x ) . Let ψ 0

and ψ1 denote the ground and the first excited states, respectively, and let

ψ = α 0 ψ 0 + α 1 ψ 1 be a normalized state with α 0 and α 1 being real constants. The

expectation value x of the position operator x in the state ψ is given by

(a) α 02 ψ 0 x ψ 0 + α 12 ψ 1 x ψ 1 (b) α 0α 1 [ ψ 0 x ψ 1 + ψ 1 x ψ 0 ]
(c) α 02 + α 12 (d) 2α 0α 1
Ans: (b)
Solution: Since V ( x ) = V (− x ) so potential is symmetric.

ψ 0 xψ 0 = 0 , ψ 1 xψ 1 = 0

ψ x ψ = (α 0 ψ 0 + α 1 ψ 1 ) × (α 0 ψ 0 + α 1 ψ 1 ) = α 0 α1 ⎡⎣ ψ 0 x ψ1 + ψ1 x ψ 0 ⎤⎦

Q9. The perturbation H ' = bx 4 , where b is a constant, is added to the one dimensional
1
harmonic oscillator potential V ( x ) = mω 2 x 2 . Which of the following denotes the
2
correction to the ground state energy to first order in b?
[Hint: The normalized ground state wave function of the one dimensional harmonic
1/ 4
⎛ mω ⎞ 2
oscillator potential isψ 0 = ⎜ ⎟ e − mω x /2
. You may use the following
⎝ π ⎠
∞ 1
−n− ⎛ 1⎞
integral ∫ x e 2n − ax 2
dx = a 2
Γ⎜ n + ⎟ ].
−∞ ⎝ 2⎠

3b 2 3b 2 3b 2 15b 2
(a) (b) (c) (d)
4m 2 ω 2 2m 2 ω 2 2π m 2ω 2 4m 2 ω 2
Ans: (a)
1
Solution: H ' = bx 4 , V ( x ) = mω 2 x 2 .
2
1/ 4 mω x 2
⎛ mω ⎞ −
Correction in ground state is given by E = ψ 0 H 'ψ 0 1
0 whereψ 0 = ⎜ ⎟ e 2
.
⎝ π ⎠

                                                                                
Head office  Branch office 
 
fiziks, H.No. 40‐D, G.F, Jia Sarai,  Anand Institute of Mathematics, 
 
Near IIT, Hauz Khas, New Delhi‐16  28‐B/6, Jia Sarai, Near IIT 
 
Phone: 011‐26865455/+91‐9871145498 Hauz Khas, New Delhi‐16 
                                                   
                                             Website: www.physicsbyfiziks.com                                                                                          
                                                            Email: fiziks.physics@gmail.com                                                                     122 
fiziks
Institute for NET/JRF, GATE, IIT‐JAM, JEST, TIFR and GRE in PHYSICAL SCIENCES 
 
1 1
mωx 2 ∞
⎛ mω ⎞ 2

⎛ mω ⎞ 2 2 2 − mωx
( )
− 2
E = ∫ψ bx ψ 0 dx = ⎜ ⎟ ⋅ b∫ x e dx = ⎜ ⎟ ⋅b ∫ x e
1 * 4 4
dx
⎝ π ⎠ ⎝ π ⎠
0 0
−∞ −∞


− n −1/ 2 ⎛ 1⎞
∫−∞ e dx = α
2 n −αn 2
It is given in the equation x ⎜⎝ n + ⎟⎠
2

Thus n = 2 and α =

1 1 1
∞ −2 −
⎛ mω ⎞ 2 2 2 − mωx ⎛ mω ⎞ 2 ⎛ mω ⎞
( )
2 2 ⎛ 1⎞
⇒ E 01 = ⎜ ⎟ ⋅b ∫ x e dx = b ⎜ ⎜⎝ 2 + ⎟⎠
⎝ π ⎠ −∞
⎝ π ⎟⎠ ⎜⎝ ⎟⎠
2
1 −5
⎛ mω ⎞ 2 ⎛ mω ⎞ 2 5 3 b 2
⇒ E = b⎜1
= .
⎝ π ⎟⎠ ⎜⎝
0 ⎟⎠
2 4 m 2ω 2

Q10. Let 0 and 1 denote the normalized eigenstates corresponding to the ground and first

excited states of a one dimensional harmonic oscillator. The uncertainty ∆p in the


1
state (0 + 1 ) , is
2

(a) Δp = mω / 2 (b) Δp = mω / 2

(c) Δp = mω (d) Δp = 2 mω
Ans: (c)

Solution: ψ =
1
2
( 0〉 + 1〉 ) , p = i 2 (a †
)
−a →

a† ψ =
1
2
( 1 1 + 2 2 ) and a ψ =
1
2
(0 + 10 )

p =i

2
(ψa †
)
− a ψ = 0 , p2 = −

2
(a †2
+ a 2 − ( 2 N + 1) )
− mω ⎡ †2 mω mω ⎛ 1 ⎞
p2 = a + a2 − 2N + 1 ⎤ = 2N + 1 = ⎜ 2 ⋅ + 1⎟ = mω
2 ⎣ ⎦ 2 2 ⎝ 2 ⎠
2
Δp = p2 − p = mω .

                                                                                
Head office  Branch office 
 
fiziks, H.No. 40‐D, G.F, Jia Sarai,  Anand Institute of Mathematics, 
 
Near IIT, Hauz Khas, New Delhi‐16  28‐B/6, Jia Sarai, Near IIT 
 
Phone: 011‐26865455/+91‐9871145498 Hauz Khas, New Delhi‐16 
                                                   
                                             Website: www.physicsbyfiziks.com                                                                                          
                                                            Email: fiziks.physics@gmail.com                                                                     123 
fiziks
Institute for NET/JRF, GATE, IIT‐JAM, JEST, TIFR and GRE in PHYSICAL SCIENCES 
 
1
Q11. The wave function of a particle at time t = 0 is given by ψ (0) = ( u1 + u 2 ) , where
2
u1 and u2 are the normalized eigenstates with eigenvalues E1 and E2

respectively, (E 2 > E1 ) . The shortest time after which ψ (t ) will become orthogonal to

ψ (0) is

− π π 2 π 2 π
(a) (b) (c) (d)
2(E 2 − E1 ) E 2 − E1 E 2 − E1 E 2 − E1
Ans: (b)

1 ⎛ ⎞
− iE1t − iE2t
1
Solution: ψ ( 0) =
2
( u1 + u2 ) ⇒ ψ ( 0) =
2
u
⎜⎝ 1 e + u 2 e ⎟⎠

− iE1t − iE2t
1 1
ψ ( t ) is orthogonal to ψ ( 0) ⇒ ψ ( 0) ψ ( t ) = 0 ⇒ e + e =0
2 2
− iE1t − iE2t − iE1t − iE2t ( E2 − E1 )
i
⇒e +e =0⇒e = −e ⇒e = −1

⇒ cos
( E2 − E1 ) t = cos π ⇒ t = π
E2 − E1
Q12. A constant perturbation as shown in the figure below acts on a particle of mass m
confined in an infinite potential well between 0 and L.

V0 V0
2
0 L
L/2

the first-order correction to the ground state energy of the particle is


V0 3V0 V0 3V0
(a) (b) (c) (d)
2 4 4 2
Ans: (b)

                                                                                
Head office  Branch office 
 
fiziks, H.No. 40‐D, G.F, Jia Sarai,  Anand Institute of Mathematics, 
 
Near IIT, Hauz Khas, New Delhi‐16  28‐B/6, Jia Sarai, Near IIT 
 
Phone: 011‐26865455/+91‐9871145498 Hauz Khas, New Delhi‐16 
                                                   
                                             Website: www.physicsbyfiziks.com                                                                                          
                                                            Email: fiziks.physics@gmail.com                                                                     124 
fiziks
Institute for NET/JRF, GATE, IIT‐JAM, JEST, TIFR and GRE in PHYSICAL SCIENCES 
 
L
L
2
V 2 πx 2 πx
Solution: E = ψ 1 V p ψ 1
1
1 = ∫ 0 sin 2 dx + ∫ V0 sin 2 dx
0
2 L L L L L
2

L
L
V 2 1⎛ 2πx ⎞ 2V 1 ⎛ 2πx ⎞
E11 = 0 ∫ ⎜1 − cos ⎟ dx + 0 ∫ ⎜1 − cos ⎟ dx
L 0 2⎝ L ⎠ L L 2⎝ L ⎠
2

V0 ⎛ L ⎞ 2V0 ⎛ L ⎞ V0 2V0 3V0


⇒ E11 = ⎜ ⎟+ ⎜L− ⎟ = + =
2L ⎝ 2 ⎠ 2L ⎝ 2⎠ 4 4 4

NET/JRF (JUNE-2012)
Q13. The component along an arbitrary direction n̂ , with direction cosines (n x , n y , n z ) , of the

1
spin of a spin − particle is measured. The result is
2

(a) 0 (b) ± nz (c) ± (n x + n y + nz ) (d) ±


2 2 2
Ans: (d)
⎛0 1⎞ ⎛0 −i ⎞ ⎛1 0 ⎞
Solution: S x = ⎜ ⎟⎟ , S y = ⎜⎜ ⎟⎟ , S z = ⎜⎜ ⎟
2 ⎜⎝1 0⎠ 2 ⎝i 0 ⎠ 2 ⎝0 −1⎟⎠

n = n x ˆi + n y ˆj + n z kˆ and n x2 + n y2 + n z2 = 1 , S = S x iˆ + S y ˆj + S z kˆ

⎛ ⎞ ⎛ i ⎞ ⎛ ⎞
⎜0 ⎟ ⎜0 − ⎟ ⎜ 0 ⎟
n ⋅ S = nx ⎜ 2⎟ + n ⎜ 2 ⎟ + nz ⎜ 2 ⎟
⎜ ⎟ y
⎜i ⎟ ⎜ − ⎟
⎜ ⎟ ⎜ 0 ⎟ ⎜0 ⎟
⎝2 ⎠ ⎝2 ⎠ ⎝ 2 ⎠


⎜ nz (n − in y )⎟
x

n⋅S = ⎜ 2 2 ⎟
⎜ ⎟
⎜ (n x + in y ) − nz ⎟
⎝2 2 ⎠

Let λ is eigen value of n ⋅ S

                                                                                
Head office  Branch office 
 
fiziks, H.No. 40‐D, G.F, Jia Sarai,  Anand Institute of Mathematics, 
 
Near IIT, Hauz Khas, New Delhi‐16  28‐B/6, Jia Sarai, Near IIT 
 
Phone: 011‐26865455/+91‐9871145498 Hauz Khas, New Delhi‐16 
                                                   
                                             Website: www.physicsbyfiziks.com                                                                                          
                                                            Email: fiziks.physics@gmail.com                                                                     125 
fiziks
Institute for NET/JRF, GATE, IIT‐JAM, JEST, TIFR and GRE in PHYSICAL SCIENCES 
 
nz −λ (n x − in y )
2 2
=0
(n x + in y ) −n z −λ
2 2

⎛ n z2 2

( )
2 2
⎛ nz ⎞ ⎛ nz ⎞
⇒ −⎜ − λ⎟⎜ + λ ⎟ − ( n x + n y ) = 0 ⇒ −⎜⎜
2 2

−λ ⎟−
2
n x2 + n y2 = 0 .
⎝ 2 ⎠⎝ 2 ⎠ 4 ⎝ 4 ⎠ 4

(n )
2
⇒− 2
x + n y2 + n z2 + λ2 = 0 ⇒ λ = ± .
4 2
Q14. A particle of mass m is in a cubic box of size a. The potential inside the box
(0 ≤ x < a,0 ≤ y < a,0 ≤ z < a ) is zero and infinite outside. If the particle is in an

14π 2
eigenstate of energy E = , its wavefunction is
2ma 2
3/ 2 3/ 2
⎛2⎞ 3πx 5πy 6πz ⎛2⎞ 7πx 4πy 3πz
(a) ψ = ⎜ ⎟ sin sin sin (b) ψ = ⎜ ⎟ sin sin sin
⎝a⎠ a a a ⎝a⎠ a a a
3/ 2 3/ 2
⎛2⎞ 4πx 8πy 2πz ⎛2⎞ πx 2πy 3πz
(c) ψ = ⎜ ⎟ sin sin sin (d) ψ = ⎜ ⎟ sin sin sin
⎝a⎠ a a a ⎝a⎠ a a a
Ans: (d)

( )π 14π 2 2
2 2
Solution: E nx ,n y ,nz = n x2 + n y2 + n z2 =
2ma 2 2ma 2
⇒ n x2 + n y2 + n z2 = 14 ⇒ n x = 1, n y = 2, n z = 3 .

Q15. Let ψ nlml denote the eigenfunctims of a Hamiltonian for a spherically symmetric

potential V (r ) . The wavefunction ψ =


1
4
[ ]
ψ 210 + 5ψ 21−1 + 10ψ 211 is an eigenfunction

only of
(a) H, L2 and Lz (b) H and Lz (c) H and L2 (d) L2 and Lz
Ans: (c)
Solution: Hψ = Enψ

L2ψ = l (l + 1) 2ψ and Lzψ ≠ m ψ .

                                                                                
Head office  Branch office 
 
fiziks, H.No. 40‐D, G.F, Jia Sarai,  Anand Institute of Mathematics, 
 
Near IIT, Hauz Khas, New Delhi‐16  28‐B/6, Jia Sarai, Near IIT 
 
Phone: 011‐26865455/+91‐9871145498 Hauz Khas, New Delhi‐16 
                                                   
                                             Website: www.physicsbyfiziks.com                                                                                          
                                                            Email: fiziks.physics@gmail.com                                                                     126 
fiziks
Institute for NET/JRF, GATE, IIT‐JAM, JEST, TIFR and GRE in PHYSICAL SCIENCES 
 
Q16. 2
The commentator x , p is2
[ ]
(a) 2i xp (b) 2i ( xp + px ) (c) 2i px (d) 2i ( xp − px )
Ans: (b)
Solution: ⎡⎣ x 2 , p 2 ⎤⎦ = x ⎡⎣ x, p 2 ⎤⎦ + ⎡⎣ x, p 2 ⎤⎦ x = xp [ x, p ] + x [ x, p ] p + p [ x, p ] x + [ x, p ] px

[x 2
]
, p 2 = xp(i ) + x(i ) p + p(i )x + (i ) px = 2i (xp + px ) .
Q17. A free particle described by a plane wave and moving in the positive z-direction
undergoes scattering by a potential
⎧V if r ≤ R
V (r ) = ⎨ 0
⎩0 if r > R

If V0 is changed to 2V0, keeping R fixed, then the differential scattering cross-section, in


the Born approximation.
(a) increases to four times the original value
(b) increases to twice the original value
(c) decreases to half the original value
(d) decreases to one fourth the original value
Ans: (a)
V (r ) = V0 , r≤R
Solution:
= 0, r>R
m 4
Low energy scattering amplitude f (θ , φ ) = − V0 πR 3 and differential scattering is
2π 2
3
2
dσ 2 ⎛ 2mV0 R 3 ⎞
given by 1 = f =⎜ ⎟⎠
dΩ ⎝ 3
2

d σ 2 ⎛ 2m ( 2V0 ) R ⎞
3 2
⎛ 2mV0 ⎞
Now V (r ) = 2V0 for r < R ⇒ =⎜ ⎟ = 4⎜ ⎟⎠
dΩ ⎝ 3 2
⎠ ⎝

d σ 2 ⎛ 2m ( 2V0 ) R ⎞
3 2
⎛ 2mV0 R 3 ⎞ dσ
⇒ =⎜ ⎟ = 4⎜⎜ ⎟=4 1

dΩ ⎝ 3 2
⎠ ⎝ 3
2
⎠ dΩ

                                                                                
Head office  Branch office 
 
fiziks, H.No. 40‐D, G.F, Jia Sarai,  Anand Institute of Mathematics, 
 
Near IIT, Hauz Khas, New Delhi‐16  28‐B/6, Jia Sarai, Near IIT 
 
Phone: 011‐26865455/+91‐9871145498 Hauz Khas, New Delhi‐16 
                                                   
                                             Website: www.physicsbyfiziks.com                                                                                          
                                                            Email: fiziks.physics@gmail.com                                                                     127 
fiziks
Institute for NET/JRF, GATE, IIT‐JAM, JEST, TIFR and GRE in PHYSICAL SCIENCES 
 
Q18. A variational calculation is done with the normalized trial wavefunction

ψ (x ) =
15 2
5/ 2
( )
a − x 2 for the one-dimensional potential well
4a
⎧⎪0 if x ≤ a
V (x ) = ⎨
⎪⎩∞ if x > a

The ground state energy is estimated to be


5 2 3 2 3 2 5 2
(a) (b) (c) (d)
3ma 2 2ma 2 5ma 2 4ma 2
Ans: (d)

15
Solution: ψ ( x ) = 5
(a 2
− x2 ), V (x ) = 0 , x ≤ a and V ( x ) = ∞ , x > a
4a 2

− 2 ∂ 2ψ
a
E = ∫ψHψdx where H =
−a
2m ∂x 2

⎡ 15 2
a
⎤ ⎡ − 2 d 2 ⎧⎪ 15 2 ⎫⎤
2 ⎪ 15 − 2
a

∫− a ⎢ 4a5/ 2
E = a − x 2
⎥⎢ ( 2 ⎨ )5/ 2
a − x ⎬⎥ dx = (5 ∫ )
(a 2 − x 2 )(− 2)dx
⎣ ⎦ ⎢⎣ 2m dx ⎩⎪ 4a ⎭⎪ ⎥⎦ 16a 2m − a
a
15 2 2 15 2 4a 3 5 2
⇒ E =
16a 5 2m −∫a
( a 2
− x 2
) dx =
16a 5 m 3
=
4ma 2

Q19. A particle in one-dimension is in the potential


⎧∞ if x < 0

V ( x ) = ⎨ − V0 if 0 ≤ x ≤ l
⎪0 if x > l

If there is at least one bound state, the minimum depth of potential is
2
π2 2
π2 2 2π 2 2
π2
(a) (b) (c) (d)
8ml 2 2ml 2 ml 2 ml 2
Ans: (a)

                                                                                
Head office  Branch office 
 
fiziks, H.No. 40‐D, G.F, Jia Sarai,  Anand Institute of Mathematics, 
 
Near IIT, Hauz Khas, New Delhi‐16  28‐B/6, Jia Sarai, Near IIT 
 
Phone: 011‐26865455/+91‐9871145498 Hauz Khas, New Delhi‐16 
                                                   
                                             Website: www.physicsbyfiziks.com                                                                                          
                                                            Email: fiziks.physics@gmail.com                                                                     128 
fiziks
Institute for NET/JRF, GATE, IIT‐JAM, JEST, TIFR and GRE in PHYSICAL SCIENCES 
 
Solution: For bound state − V0 < E < 0
Wave function in region I, ψ I = 0 , ψ II = A sin kx + B cos kx , ψ III = ce−γx
Ι ΙΙ ΙΙΙ
2m(V0 + E ) 2m ( − E ) − V0 o
Where k = 2
, γ= 2
. l

Use Boundary condition at x = 0 and x = l (wave function is continuous and differential


at x = 0 and x = l ) one will get
k cot kl = −γ ⇒ kl cot kl = −γl ⇒ η = −ξ cot ξ where γl = η , kl = ξ .

2mV0l 2 η
⇒ η +ξ =
2 2
2

1/ 2 o π 3π
⎛ 2mV0l 2 ⎞ π π2 2
For one bound state ⎜ ⎟ = ⇒ V0 = . 2
ξ 2

2
⎠ 2 8ml 2
Q20. Which of the following is a self-adjoint operator in the spherical polar coordinate
system (r , θ , φ ) ?
i ∂ ∂ i ∂ ∂
(a) − (b) − i (c) − (d) − i sin θ
sin θ ∂θ
2
∂θ sin θ ∂θ ∂θ
Ans: (c)
−i ∂
Solution: is Hermitian.
sin θ ∂θ

                                                                                
Head office  Branch office 
 
fiziks, H.No. 40‐D, G.F, Jia Sarai,  Anand Institute of Mathematics, 
 
Near IIT, Hauz Khas, New Delhi‐16  28‐B/6, Jia Sarai, Near IIT 
 
Phone: 011‐26865455/+91‐9871145498 Hauz Khas, New Delhi‐16 
                                                   
                                             Website: www.physicsbyfiziks.com                                                                                          
                                                            Email: fiziks.physics@gmail.com                                                                     129 
fiziks
Institute for NET/JRF, GATE, IIT‐JAM, JEST, TIFR and GRE in PHYSICAL SCIENCES 
 
NET/JRF (DEC-2012)
Q21. Let v, p and E denote the speed, the magnitude of the momentum, and the energy of a
free particle of rest mass m. Then

(a) dE = constant (b) p = mv


dp

(c) v = cp (d) E = mc2


p +m c
2 2 2

Ans: (c)
m0 v m02 v 2 p 2v 2
Solution: p = mv = ⇒ p2 = ⇒ m 2 2
0 v = p 2

v2 v2 c2
1− 1− 2
c2 c

⎛ 2 p2 ⎞ p2 pc
⎜ 2

⇒ v ⎜ m0 + 2 ⎟ = p ⇒ v = 2 2
2 2
⇒v=
⎝ c ⎠ m0 c + p 2 p + m02 c 2
2

c2
Q22. The wave function of a state of the Hydrogen atom is given by,
ψ = ψ 200 + 2ψ 211 + 3ψ 210 + 2ψ 21−1
where ψ nlm is the normalized eigen function of the state with quantum numbers n, l, m in

the usual notation. The expectation value of Lz in the state ψ is


15 11 3
(a) (b) (c) (d)
6 6 8 8
Ans: (d)
1 2 3 2
Solution: Firstly normalizeψ , ψ = ψ 200 + ψ 211 + ψ 210 + ψ 21−1
16 16 16 16
1 9 10
P(0 )= + = .
16 16 16
4 2
Probability of getting (iħ) i.e. P(i )= and P(− i )= .
16 16
ψ Lz ψ 10 4 2 4 2 2
Now, Lz = = 0 × + 1 × + (− 1 ) × = − = =
ψψ 16 16 16 16 16 16 8

                                                                                
Head office  Branch office 
 
fiziks, H.No. 40‐D, G.F, Jia Sarai,  Anand Institute of Mathematics, 
 
Near IIT, Hauz Khas, New Delhi‐16  28‐B/6, Jia Sarai, Near IIT 
 
Phone: 011‐26865455/+91‐9871145498 Hauz Khas, New Delhi‐16 
                                                   
                                             Website: www.physicsbyfiziks.com                                                                                          
                                                            Email: fiziks.physics@gmail.com                                                                     130 
fiziks
Institute for NET/JRF, GATE, IIT‐JAM, JEST, TIFR and GRE in PHYSICAL SCIENCES 
 
1
Q23. The energy eigenvalues of a particle in the potential V ( x ) = mω 2 x 2 − ax are
2
⎛ 1⎞ a2 ⎛ 1⎞ a2
(a) En = ⎜ n + ⎟ ω − (b) En = ⎜ n + ⎟ ω +
⎝ 2⎠ 2mω 2 ⎝ 2⎠ 2mω 2

⎛ 1⎞ a2 ⎛ 1⎞
(c) En = ⎜ n + ⎟ ω − (d) En = ⎜ n + ⎟ ω
⎝ 2⎠ mω 2 ⎝ 2⎠
Ans: (a)
p x2 1
Solution: Hamiltonian ( H ) of Harmonic oscillator, H = + mω 2 x 2
2m 2
⎛ 1⎞
Eigenvalue of this, E n = ⎜ n + ⎟ ω
⎝ 2⎠

p x2 1 p2 1 ⎡ 2ax a2 ⎤ a2
But here, H = + mω 2 x 2 − ax ⇒ H = x + mω 2 ⎢ x 2 − + ⎥ −
2m 2 2m 2 ⎣ mω 2 m 2ω 4 ⎦ 2mω 2
2
p x2 1 2⎡ a ⎤ a2
H= + mω ⎢ x − −
2m 2 ⎣ mω 2 ⎥⎦ 2mω 2

⎛ 1⎞ a2
Energy eigenvalue, E n = ⎜ n + ⎟ ω −
⎝ 2⎠ 2mω 2

Q24. If a particle is represented by the normalized wave function


⎧ 15 (a 2 − x 2 )
⎪ for − a < x < a
ψ ( x ) = ⎨ 4a 5 / 2
⎪0 otherwise

the uncertainty Δp in its momentum is

(a) 2 / 5a (b) 5 / 2a (c) 10 / a (d) 5 / 2a


Ans: (d)
2
Solution: Δp = p2 − p and

                                                                                
Head office  Branch office 
 
fiziks, H.No. 40‐D, G.F, Jia Sarai,  Anand Institute of Mathematics, 
 
Near IIT, Hauz Khas, New Delhi‐16  28‐B/6, Jia Sarai, Near IIT 
 
Phone: 011‐26865455/+91‐9871145498 Hauz Khas, New Delhi‐16 
                                                   
                                             Website: www.physicsbyfiziks.com                                                                                          
                                                            Email: fiziks.physics@gmail.com                                                                     131 
fiziks
Institute for NET/JRF, GATE, IIT‐JAM, JEST, TIFR and GRE in PHYSICAL SCIENCES 
 

ψ −i ψ
15 (a 2 − x 2 )
a
∂ 2
p =
ψψ
∂x =∫ (− i ) 15 (a − x 2 )dx
−a 4a 5/2
4a ∂x
5/ 2

a a
=∫
15
(− i ) a − x (− 2 x )dx = +ih 2 × 155
( 2 2
) ∫ (a
2
x − x 3 dx )
− a 16a
5
16 × a −a ↓
odd func n

=0
a
∂2 2
∫( ) ( )
15
p 2
=− 2
× a −x
2
a − x 2 dx
2

16a 5 −a ∂x 2

a
a
⎧ 2 x3 ⎫
=− 2
×
15
16a 5
(
× (− 2 ) ∫ a 2 − x 2 dx = ) 2
×
15
16a 5
× 2 ⎨ a ⋅ x − ⎬
3 ⎭ −a
−a ⎩

15 ⎡ 3 2a 3 ⎤ 15 3⎡ 1 ⎤ 15 2 2
= × × 2 ⎢ 2a − ⎥= × × 2 × 2a ⎢1 − ⎥ = ×
2 2
2
16a 5 ⎣ 3 ⎦ 16 ⎣ 3 ⎦ 4a 3

5 2
p2 =
2a 2

2 5 2 5
Now, Δp = p2 − p = −0 =
2a 2 2a
Q25. Given the usual canonical commutation relations, the commutator [A, B] of
A = i (xp y − yp x ) and B = ( yp z + zp y ) is

(a) (xp z − p x z ) (b) − (xp z − p x z )


(c) (xp z + p x z ) (d) − (xp z + p x z )
Ans: (c)
[
Solution: [A, B ] = (ixp y − iyp x ), ( yp z + zp y ) ]
[A, B] = i[xp y , yp z ] − i[ yp x , yp z ] + i[xp y , zp y ] − i[yp x , zp y ]
[A, B] = i[xp y , yp z ] − 0 + 0 − i[yp x , zp y ] = i[xp y , yp z ] − i[yp x , zp y ]
[A, B] = ix[ p y , yp z ] + i[x, yp z ] p y − iy[ p x , zp y ] − i[y, zp y ]p x
[A, B] = ix[ p y , yp z ] + 0 − 0 − i[y, zp y ]p x = ix[ p y , yp z ] − i[y, zp y ]p x
                                                                                
Head office  Branch office 
 
fiziks, H.No. 40‐D, G.F, Jia Sarai,  Anand Institute of Mathematics, 
 
Near IIT, Hauz Khas, New Delhi‐16  28‐B/6, Jia Sarai, Near IIT 
 
Phone: 011‐26865455/+91‐9871145498 Hauz Khas, New Delhi‐16 
                                                   
                                             Website: www.physicsbyfiziks.com                                                                                          
                                                            Email: fiziks.physics@gmail.com                                                                     132 
fiziks
Institute for NET/JRF, GATE, IIT‐JAM, JEST, TIFR and GRE in PHYSICAL SCIENCES 
 
[A, B] = ix × (− i ) p z − izi × p x
[A, B] = (xp z + p x z )
Q26. Consider a system of three spins S1, S2 and S3 each of which can take values +1 and -1.
The energy of the system is given by E = − J [S1 S 2 + S 2 S 3 + S 3 S1 ] where J is a positive
constant. The minimum energy and the corresponding number of spin configuration are,
respectively,
(a) J and 1 (b) −3 J and 1 (c) −3 J and 2 (d) −6 J and 2
Ans: (c)
Solution: If we take S1 = S2 = S3 = +1 i.e. ↑ ↑ ↑
S1 S2 S3

Then energy, E = − J [1 × 1 + 1 × 1 + 1 × 1] = −3J

Again S1 = S2 = S3 = −1 , then ↓ ↓ ↓

Energy ( E ) = −3J

So, minimum energy is ( −3J ) and there are two spin configuration.

If we take ↑ ↓ ↑
S1 S2 S3

Then we get Maximum energy E = J .


1
Q27. The energies in the ground state and first excited state of a particle of mass m = in a
2
potential V ( x ) are −4 and −1 , respectively, (in units in which = 1 ). If the

corresponding wavefunctions are related by ψ 1 ( x ) = ψ 0 ( x )sinh x, then the ground state


eigenfunction is
(a) ψ 0 ( x ) = sec hx (b) ψ 0 ( x ) = sec hx

(c) ψ 0 ( x ) = sec h 2 x (d) ψ 0 ( x ) = sec h 3 x


Ans: (c)
Solution: Given that ground state energy E0 = −4 , first excited state energy E1 = −1 and ψ 0 , ψ 1
are corresponding wave functions.

                                                                                
Head office  Branch office 
 
fiziks, H.No. 40‐D, G.F, Jia Sarai,  Anand Institute of Mathematics, 
 
Near IIT, Hauz Khas, New Delhi‐16  28‐B/6, Jia Sarai, Near IIT 
 
Phone: 011‐26865455/+91‐9871145498 Hauz Khas, New Delhi‐16 
                                                   
                                             Website: www.physicsbyfiziks.com                                                                                          
                                                            Email: fiziks.physics@gmail.com                                                                     133 
fiziks
Institute for NET/JRF, GATE, IIT‐JAM, JEST, TIFR and GRE in PHYSICAL SCIENCES 
 
1
Solving Schrödinger equation (use m = and = 1 )
2
− 2 ∂ 2ψ 0 ∂ 2ψ 0
+ Vψ 0 = E0ψ 0 ⇒ − 2 + Vψ 0 = −4ψ 0 …..(1)
2m ∂x 2 ∂x
− 2 ∂ 2ψ 1 ∂ 2ψ 1
+ Vψ 1 = E1ψ 1 ⇒ − 2 + Vψ 1 = −1ψ 1 ……..(2)
2m ∂x 2 ∂x
Put ψ 1 = ψ 0 sinh x in equation (2) one will get

⎡ ∂ 2ψ ∂ψ ⎤
− ⎢ 20 .sinh x + 2 0 cosh x + ψ 0 sinh x ⎥ + Vψ 0 sinh x = −ψ 0 sinh x
⎣ ∂x ∂x ⎦

⎡ ∂ 2ψ ∂ψ ⎤
− ⎢ 20 + 2 0 coth x +ψ 0 ⎥ + Vψ 0 = −ψ 0
⎣ ∂x ∂x ⎦
⎡ ∂ 2ψ 0 ⎤ ∂ψ 0 ∂ 2ψ 0
⎢− + Vψ 0 ⎥ − 2 coth x − ψ 0 = −ψ 0 using relation − 2 + Vψ 0 = −4ψ 0
⎣ ∂x ∂x ∂x
2

∂ψ 0 dψ 0
−4ψ 0 − 2 coth x −ψ 0 = −ψ 0 ⇒ = −2 tanh xdx ⇒ ψ 0 = sec h 2 x .
∂x ψ0

NET/JRF (JUNE-2013)
Q28. In a basis in which the z - component S z of the spin is diagonal, an electron is in a spin

⎛ (1 + i ) / 6 ⎞
state ψ = ⎜ ⎟ . The probabilities that a measurement of S z will yield the values
⎝ 2/3 ⎠

/ 2 and − / 2 are, respectively,


(a) 1/ 2 and 1/ 2 (b) 2 / 3 and 1/ 3 (c) 1/ 4 and 3 / 4 (d) 1/ 3 and 2 / 3
Ans: (d)
⎛1⎞ ⎛0⎞
Solution: Eigen state of S z is φ1 = ⎜ ⎟ and φ2 = ⎜ ⎟ corresponds to Eigen value and −
⎝0⎠ ⎝1⎠ 2 2
respectively.
2 2
⎛ ⎞ φ1 ψ 1+ i
2
2 1 ⎛ ⎞ φ2 ψ 2
P⎜ ⎟ = = = = , P⎜− ⎟ = =
⎝ 2⎠ ψ ψ 6 6 3 ⎝ 2⎠ ψ ψ 3
                                                                                
Head office  Branch office 
 
fiziks, H.No. 40‐D, G.F, Jia Sarai,  Anand Institute of Mathematics, 
 
Near IIT, Hauz Khas, New Delhi‐16  28‐B/6, Jia Sarai, Near IIT 
 
Phone: 011‐26865455/+91‐9871145498 Hauz Khas, New Delhi‐16 
                                                   
                                             Website: www.physicsbyfiziks.com                                                                                          
                                                            Email: fiziks.physics@gmail.com                                                                     134 
fiziks
Institute for NET/JRF, GATE, IIT‐JAM, JEST, TIFR and GRE in PHYSICAL SCIENCES 
 
Q29. Consider the normalized state ψ of a particle in a one-dimensional harmonic oscillator:

ψ = b1 0 + b2 1

where 0 and 1 denote the ground and first excited states respectively, and b1 and b2

are real constants. The expectation value of the displacement x in the state ψ will be a
minimum when
1 1
(a) b2 = 0, b1 = 1 (b) b2 = b1 (c) b2 = b1 (d) b2 = b1
2 2
Ans: (d)
Solution: x = b12 0 x 0 + b22 1 x 1 + 2b1b2 0 x 1

Since 0 x 0 = 0 and 1 x 1 = 0 ⇒ x = 2b1b2 0 x 1 .

Min of x means min 2b1b2 . We know that b12 + b22 = 1.

x min ⎣
2
( ⎦ ) 2
[
= ⎡( b1 + b2 ) − b12 + b22 ⎤ 0 x 1 = (b1 + b2 ) − 1 0 x 1 ⇒ 1 − (b1 − b2 ) 0 x 1
2
] [ ]
for min value of ⎡1 − ( b1 − b2 ) ⎤ there must be maximum of ( b1 − b2 ) so
2 2
⇒ b1 = −b2
⎣ ⎦
none of answer is matched but if we consider about magnitude then option (d) is correct .
Q30. The un-normalized wavefunction of a particle in a spherically symmetric potential is
given by
ψ ( r ) = zf ( r )

where f (r ) is a function of the radial variable r . The eigenvalue of the operator

L2 (namely the square of the orbital angular momentum) is


2 2 2 2
(a) /4 (b) /2 (c) (d) 2
Ans: (d)
Solution: ψ (r ) = zf (r ) = r cosθ f (r )

ψ ( r = Y10 (θ , φ ) ) , L2ψ ( r ) = L2Y10 (θ , φ ) where l = 1

L2 = l (l + 1) 2
= 1(1 + 1) 2
=2 2

                                                                                
Head office  Branch office 
 
fiziks, H.No. 40‐D, G.F, Jia Sarai,  Anand Institute of Mathematics, 
 
Near IIT, Hauz Khas, New Delhi‐16  28‐B/6, Jia Sarai, Near IIT 
 
Phone: 011‐26865455/+91‐9871145498 Hauz Khas, New Delhi‐16 
                                                   
                                             Website: www.physicsbyfiziks.com                                                                                          
                                                            Email: fiziks.physics@gmail.com                                                                     135 
fiziks
Institute for NET/JRF, GATE, IIT‐JAM, JEST, TIFR and GRE in PHYSICAL SCIENCES 
 
Q31. If ψ nlm denotes the eigenfunciton of the Hamiltonian with a potential V = V (r ) then the

expectation value of the operator L2x + L2y in the state

υ/ =
1
5
[
3υ/ 211 + υ/ 210 − 15υ/ 21−1 ]
is
2 2 2 2
(a) 39 / 25 (b) 13 / 25 (c) 2 (d) 26 / 25
Ans: (d)
Solution: L2x + L2y = L2 − L2z ⇒ L2x + L2y = L2 − L2z = L2 − L2z

⎛ 9 1 15 ⎞
L2 − L2z = 2 2
− ⎜ ×1 2
+ ×0 2
+ ×1 2 ⎟
⎝ 25 25 25 ⎠
24 50 − 24 26
L2 − L2z = 2 2
− 2
= 2
= 2

25 25 25
Q32. Consider a two-dimensional infinite square well
⎧0 0 < x < a, 0< y<a
V ( x, y ) = ⎨
⎩∞ otherwise

2 ⎛ n xπx ⎞ ⎛ n y πy ⎞
Its normalized Eigenfunctions are ψ nx ,n y ( x, y ) = sin ⎜ ⎟ sin⎜ ⎟
a ⎝ a ⎠ ⎜⎝ a ⎟⎠

⎧⎪ a a
V 0< x< , 0< y<
where n x , n y = 1,2,3, …..If a perturbation H ' = ⎨ 0 2 2
⎪⎩ 0 otherwise

is applied, then the correction to the energy of the first excited state to order V0 is

V0 V0 ⎡ 64 ⎤
(a)
4
(b)
4 ⎢⎣1 ± 9π 2 ⎥⎦

V0 ⎡ 16 ⎤ V0 ⎡ 32 ⎤
(c)
4 ⎢⎣1 ± 9π 2 ⎥⎦ (d)
4 ⎢⎣1 ± 9π 2 ⎥⎦

Ans: (b)
Solution: For first excited state, which is doubly degree

                                                                                
Head office  Branch office 
 
fiziks, H.No. 40‐D, G.F, Jia Sarai,  Anand Institute of Mathematics, 
 
Near IIT, Hauz Khas, New Delhi‐16  28‐B/6, Jia Sarai, Near IIT 
 
Phone: 011‐26865455/+91‐9871145498 Hauz Khas, New Delhi‐16 
                                                   
                                             Website: www.physicsbyfiziks.com                                                                                          
                                                            Email: fiziks.physics@gmail.com                                                                     136 
fiziks
Institute for NET/JRF, GATE, IIT‐JAM, JEST, TIFR and GRE in PHYSICAL SCIENCES 
 
2 πx 2π y 2 2π x πy
φ1 = sin sin , φ2 = sin sin
a a a a a a
2 a / 2 2 π x 2 a / 2 2 2π y 1 1 V
H11 = φ1 H φ1 = V0 ∫
a 0
sin
a
dx ∫ sin
a 0 a
dy = V0 ⋅ ⋅ = 0
2 2 4
2 a/2 πx 2π x 2 a / 2 2π y πy
H12 = φ1 H φ2 = V0
a ∫0
sin
a
sin
a
dx ∫ sin
a 0 a
sin
a
dy

⎛ 4 ⎞⎛ 4 ⎞ 16 16 V
H 12 = V0 ⎜ ⎟⎜ ⎟ = V0 , H 21 = φ2 H ′ φ1 = V0 2 and H 22 = φ2 H ′ φ2 = 0 .
⎝ 3π ⎠⎝ 3π ⎠ 9π 9π
2
4

⎛ V0 16V0 ⎞
⎜ −λ ⎟ 2 2

Thus ⎜ 4 9π 2 ⎟ = 0 ⇒ ⎛⎜ V0 − λ ⎞⎟ − ⎛⎜ 16V0 ⎞⎟ = 0
⎜ 16V0 V0
− λ ⎟⎟ ⎝ 4 ⎠ ⎝ 9π ⎠
2

⎝ 9π 2 4 ⎠
⎛V ⎞ 16V0 V ⎛ 64 ⎞
⇒ ⎜ 0 −λ⎟ = ± ⇒λ = 0 ⎜1 ± 2 ⎟
⎝ 4 ⎠ 9π 2
4 ⎝ 9π ⎠
Q33. The bound on the ground state energy of the Hamiltonian with an attractive delta-
function potential, namely
2
d2
− aδ ( x )
H =−
2m dx 2
using the variational principle with the trial wavefunction ψ ( x ) = A exp − bx 2 is ( )
⎡ ∞

⎢ Note : ∫0 e −t a
t dt = Γ (a + 1)⎥
⎣ ⎦

(a) − ma 2 / 4π 2
(b) − ma 2 / 2π 2

(c) − ma 2 / π 2
(d) − ma 2 / 5π 2

Ans: (c)
2 2
b 2b b 2b
Solution: For given wavefunction T = and V = −a ⇒ E = −a
2m π 2m π
d E d E 2
2 1 − 12 π 4
For variation of parameter =0⇒ = −a × b =0⇒b= .
db db 2m π 2 8m 2
ma 2
⇒ E =− .
min
π 2

                                                                                
Head office  Branch office 
 
fiziks, H.No. 40‐D, G.F, Jia Sarai,  Anand Institute of Mathematics, 
 
Near IIT, Hauz Khas, New Delhi‐16  28‐B/6, Jia Sarai, Near IIT 
 
Phone: 011‐26865455/+91‐9871145498 Hauz Khas, New Delhi‐16 
                                                   
                                             Website: www.physicsbyfiziks.com                                                                                          
                                                            Email: fiziks.physics@gmail.com                                                                     137 
fiziks
Institute for NET/JRF, GATE, IIT‐JAM, JEST, TIFR and GRE in PHYSICAL SCIENCES 
 
Q34. If the operators A and B satisfy the commutation relation [ A, B] = I , where I is the
identity operator, then
[ ]
(a) e A , B = e A [ ] [
(b) e A , B = e B , A ]
(c) [e , B] = [e
A −B
,A ] (d) [e , B] = I
A

Ans: (a)
⎡ A A2 ⎤
Solution: [A, B] = I and e = ⎢1 + + + .......⎥
A

⎣ 1 2 ⎦
⎡⎣ A2 , B ⎤⎦ ⎡⎣ A3 , B ⎤⎦
[ ]
⎡ A A2
e , B = ⎢1 + +
A ⎤
+ ......., B ⎥ = [1, B ] + [ A, B ] + + .........
⎣ 1 2 ⎦ 2 3

[e , B] = 0 + I + A[A, B]2+![A, B]A + A[A , B]3+![A , B]A + ...........


2 2
A

[e , B] = 1 + A + A2! + ........ = e [ ] [ ]
2
A A
where [A, B] = I , A 2 , B = 2 A and A3 , B = 3 A 2 .

Q35. Two identical bosons of mass m are placed in a one-dimensional potential


1
V (x ) = mω 2 x 2 . The bosons interact via a weak potential,
2
[
V12 = V0 exp − mΩ( x1 − x 2 ) / 4
2
]
where x1 and x 2 denote coordinates of the particles. Given that the ground state
1
mωx 2
⎛ mω ⎞ 4 −
wavefunction of the harmonic oscillator isυ/ 0 ( x ) = ⎜ ⎟ e 2
. The ground state
⎝π ⎠
energy of the two-boson system, to the first order in V0 , is

V0 Ω
(a) ω + 2V0 (b) ω +
ω
1

⎛ Ω ⎞ 2 ⎛ ω⎞
(c) ω + V0 ⎜1 + ⎟ (d) ω + V0 ⎜1 + ⎟
⎝ 2ω ⎠ ⎝ Ω⎠
Ans: (c)

                                                                                
Head office  Branch office 
 
fiziks, H.No. 40‐D, G.F, Jia Sarai,  Anand Institute of Mathematics, 
 
Near IIT, Hauz Khas, New Delhi‐16  28‐B/6, Jia Sarai, Near IIT 
 
Phone: 011‐26865455/+91‐9871145498 Hauz Khas, New Delhi‐16 
                                                   
                                             Website: www.physicsbyfiziks.com                                                                                          
                                                            Email: fiziks.physics@gmail.com                                                                     138 
fiziks
Institute for NET/JRF, GATE, IIT‐JAM, JEST, TIFR and GRE in PHYSICAL SCIENCES 
 
Solution: There is two bosons trapped in harmonic oscillator so energy for ground state without
ω
perturbation is = 2. = ω.
2
If perturbation is introduced we have to calculate 〈V1,2 〉 where

[
V12 = V0 exp − mΩ( x1 − x 2 ) / 4 .
2
]
⎛ 1
mω x12 mω x22 ⎞
⎜ ⎛ mω ⎞ 2 − −
⎟ is very tedious task.
But calculating 〈V1,2 〉 on state υ/ 0 ( x ) = ⎜ e 2
e 2
⎜ ⎝ π ⎟⎠ ⎟
⎝ ⎠
So lets use a trick i.e perturbation is nothing but approximation used in Taylor series. So
[
just expand V12 = V0 exp − mΩ( x1 − x 2 ) / 4
2
] and take average value of first term
Ω ( x1 − x2 )
2

[
V12 = V0 exp − mΩ( x1 − x 2 ) / 4 = V0 (1 −
2
] 4
+ ...)

V12 = V0 (1 −
(
mΩ x12 + x22 − 2 x1.x2 ) + ...)
4

〈V1,2 〉 = V0 (1 −
(
mΩ 〈 x12 〉 + 〈 x22 〉 − 2〈 x1 〉.〈 x2 〉 ) + ...) = V (1 − mΩ( 2mω + 2mω − 0) )...
o
4 4
−1
Ω ⎛ Ω ⎞2
⇒ 〈V12 〉 = Vo (1 − ) ≈ V0 ⎜1 + ⎟ .
4ω ⎝ 2ω ⎠
−1
⎛ Ω ⎞2
So E = ω + V0 ⎜1 + ⎟ .
⎝ 2ω ⎠

                                                                                
Head office  Branch office 
 
fiziks, H.No. 40‐D, G.F, Jia Sarai,  Anand Institute of Mathematics, 
 
Near IIT, Hauz Khas, New Delhi‐16  28‐B/6, Jia Sarai, Near IIT 
 
Phone: 011‐26865455/+91‐9871145498 Hauz Khas, New Delhi‐16 
                                                   
                                             Website: www.physicsbyfiziks.com                                                                                          
                                                            Email: fiziks.physics@gmail.com                                                                     139 
fiziks
Institute for NET/JRF, GATE, IIT‐JAM, JEST, TIFR and GRE in PHYSICAL SCIENCES 
 
NET/JRF (DEC-2013)
1 1 ⎛1 + i ⎞
Q36. A spin - particle is in the state χ = ⎜⎜ ⎟⎟ in the eigenbasis of S 2 and S z . If we
2 11 ⎝ 3 ⎠
h h
measure S z , the probabilities of getting + and − , respectively are
2 2
1 1 2 9 1 3
(a) and (b) and (c) 0 and 1 (d) and
2 2 11 11 11 11
Ans: (b)
2
⎛ ⎞ 1 ⎛1 + i⎞ 1 2
Solution: P ⎜ ⎟ = (10) ⎜ ⎟ = × 2 = ∵ ψ ψ =1
⎝ 2⎠ 11 ⎝ 3 ⎠ 2 11
2
⎛ ⎞ 1 ⎛1 + i⎞ 9
P⎜− ⎟ =
⎝ 2⎠
( 01) ⎜ ⎟ =
11 ⎝ 3 ⎠ 11

⎛ ⎞ ⎛ ⎞
i.e. probability of S z getting ⎜ ⎟ and ⎜ − ⎟
⎝ 2⎠ ⎝ 2⎠

Q37. The motion of a particle of mass m in one dimension is described by the


p2 1
Hamiltonian H = + mω 2 x 2 + λx . What is the difference between the (quantized)
2m 2
energies of the first two levels? (In the following, x is the expectation value of x in the
ground state.)
λ2
(a) ω − λ x (b) ω + λ x (c) ω + (d) ω
2mω 2
Ans: (d)
p2 1 1
Solution: H = + mω 2 x 2 + λ x ⇒ V ( x ) = mω 2 x 2 + λ x
2m 2 2
1 ⎡ 2 ⎤ 1 2⎡ 2 λ λ2 λ2 ⎤
V ( x) = mω 2 ⎢ x 2 + λ x = mω ⎢ x + 2 ⋅ x ⋅ + − ⎥
2 ⎣ mω 2 ⎥⎦ 2 ⎣ mω 2 m 2ω 4 m 2ω 4 ⎦
2
1 ⎛ λ ⎞ λ2
V ( x ) = mω 2 ⎜ x + ⎟ −
2 ⎝ mω 2 ⎠ 2mω 2

⎛ 1⎞ λ2 3 1
∴ En = ⎜ n + ⎟ ω − ⇒ E1 − E0 = ω− ω = ω
⎝ 2⎠ 2mω 2
2 2
                                                                                
Head office  Branch office 
 
fiziks, H.No. 40‐D, G.F, Jia Sarai,  Anand Institute of Mathematics, 
 
Near IIT, Hauz Khas, New Delhi‐16  28‐B/6, Jia Sarai, Near IIT 
 
Phone: 011‐26865455/+91‐9871145498 Hauz Khas, New Delhi‐16 
                                                   
                                             Website: www.physicsbyfiziks.com                                                                                          
                                                            Email: fiziks.physics@gmail.com                                                                     140 
fiziks
Institute for NET/JRF, GATE, IIT‐JAM, JEST, TIFR and GRE in PHYSICAL SCIENCES 
 
Q38. Let ψ nlm denote the eigenfunctions of a Hamiltonian for a spherically symmetric

potential V (r ) . The expectation value of Lz in the state

ψ =
1
6
[ψ 200 + 5ψ 210 + 10ψ 21−1 + 20ψ 211 is ]
5 5 5
(a) − (b) (c) (d)
18 6 18
Ans: (d)
1 5 10 20 10 5
Solution: Lz = ψ L z ψ = × 0 + × 0 + × (−1 ) + (1 ) = = ∵ψ ψ =1
36 36 36 36 36 18
Q39. If ψ ( x ) = A exp(− x 4 ) is the eigenfunction of a one dimensional Hamiltonian with
eiggenvalue E = 0 , the potential V ( x ) (in units where = 2m = 1 ) is

(a) 12 x 2 (b) 16 x 6 (c) 16 x 6 + 12 x 2 (d) 16 x 6 − 12 x 2


Ans: (d)
Solution: Schrodinger equation
− ∇ 2ψ + Vψ = 0 (where = 2m = 1 and E = 0 )

∂2
∂x
( 4
) ∂
− 2 Ae − x + VAe − x = 0 ⇒ − ⎡e − x × −4 x3 ⎤ + Ve− x = 0
4

∂x ⎣
4


4

[{ 4
(
4 3x 2 e − x + x 3 − 4 x 3 e − x
4
)}]+ Ve − x4 4 4
= 0 ⇒ 12 x 2 e − x − 16 x 6 e − x + Ve − x = 0
4

⇒ V = 16 x 6 − 12 x 2
Q40. A particle is in the ground state of an infinite square well potential given by,
⎧0 for − a ≤ x ≤ a
V (x ) = ⎨
⎩∞ otherwise

a a
The probability to find the particle in the interval between − and is
2 2
1 1 1 1 1 1
(a) (b) + (c) − (d)
2 2 π 2 π π
Ans: (b)

                                                                                
Head office  Branch office 
 
fiziks, H.No. 40‐D, G.F, Jia Sarai,  Anand Institute of Mathematics, 
 
Near IIT, Hauz Khas, New Delhi‐16  28‐B/6, Jia Sarai, Near IIT 
 
Phone: 011‐26865455/+91‐9871145498 Hauz Khas, New Delhi‐16 
                                                   
                                             Website: www.physicsbyfiziks.com                                                                                          
                                                            Email: fiziks.physics@gmail.com                                                                     141 
fiziks
Institute for NET/JRF, GATE, IIT‐JAM, JEST, TIFR and GRE in PHYSICAL SCIENCES 
 
a a
Solution: The probability to find the particle in the interval between − and is
2 2

πx πx πx 1 1⎡ ⎛ 2πx ⎞ ⎤
a/2 a/2 a/2
2 2 1
= ∫ ⋅ cos ⋅ cos dx = ∫ cos 2 dx = × ⎢ ∫ ⎜1 + cos ⎟dx ⎥
−a / 2
2a 2a 2a 2a −a / 2
a 2a a 2 ⎣−a / 2 ⎝ 2a ⎠ ⎦
a/2
1 ⎡ a πx ⎤ 1 ⎡a a a ⎤ 1 ⎡ 2a ⎤ ⎛ 1 1 ⎞
= ⎢ x + sin ⎥ = ⎢ + + (1 + 1)⎥ = ⎢ a+ ⎥ =⎜ + ⎟
2a ⎣ π a ⎦ − a / 2 2a ⎣ 2 2 π ⎦ 2a ⎣ π ⎦ ⎝2 π ⎠
Q41. The expectation value of the x - component of the orbital angular momentum L x in the

state ψ =
1
5
[
3ψ 2,1, −1 + 5ψ 2,1,0 − 11ψ 2,1, +1 ]
(where ψ nlm are the eigenfunctions in usual notation), is

(a) −
10
25
( 11 − 3) (b) 0 (c)
10
25
( 11 + 3) (d) 2

Ans: (a)

Solution: L− l , m = l ( l + 1) − m ( m − 1) l , m − 1 and L+ l , m = l ( l + 1) − m ( m + 1) l , m + 1

L+ + L− L + L−
Lx = ⇒ Lx = +
2 2
1
L+ψ = ⎡⎣3 2 ψ 210 + 5 2 ψ 211 ⎤⎦
5
1 1 1
ψ L+ ψ = .3 10 − 110 = 10(3 − 11)
25 25 25
1
L−ψ = ⎡⎣ 2 5ψ 21−1 − 2 11ψ 210 ⎤⎦
5
1 1
ψ L− ψ = .3 10 − 10 11
25 25
L+ + L− 1
Lx = = 10(3 − 11)
2 25

ψ Lx ψ =
1
25
.3 10 −
1
25
10 11 = −
10
25
( 11 − 3)
                                                                                
Head office  Branch office 
 
fiziks, H.No. 40‐D, G.F, Jia Sarai,  Anand Institute of Mathematics, 
 
Near IIT, Hauz Khas, New Delhi‐16  28‐B/6, Jia Sarai, Near IIT 
 
Phone: 011‐26865455/+91‐9871145498 Hauz Khas, New Delhi‐16 
                                                   
                                             Website: www.physicsbyfiziks.com                                                                                          
                                                            Email: fiziks.physics@gmail.com                                                                     142 
fiziks
Institute for NET/JRF, GATE, IIT‐JAM, JEST, TIFR and GRE in PHYSICAL SCIENCES 
 
Q42. A particle is prepared in a simultaneous eigenstate of L2 and Lz . If l ( + 1) 2 and m are

respectively the eigenvalues of L2 and Lz , then the expectation value L2x of the particle

in this state satisfies


(a) L2x = 0 (b) 0 ≤ L2x ≤ 2 2

( + 1) 2 2
( + 1) 2

(c) 0 ≤ L2x ≤ (d) ≤ L2x ≤


2 2 2
Ans: (d)
1
Solution: L2x =
2
(
l ( l + 1) 2
− m2 2
)
For max value m = 0 and for min m = l
l 2
l (l + 1) 2
≤ L2x ≤
2 2
A, B, C are Non zero Hermitian operator.
[A, B] = C ⇒ AB − BA ⇒ AB − Ab = 0 = C
but C≠0
if AB = BA ie [A, B ] = C false (2)

                                                                                
Head office  Branch office 
 
fiziks, H.No. 40‐D, G.F, Jia Sarai,  Anand Institute of Mathematics, 
 
Near IIT, Hauz Khas, New Delhi‐16  28‐B/6, Jia Sarai, Near IIT 
 
Phone: 011‐26865455/+91‐9871145498 Hauz Khas, New Delhi‐16 
                                                   
                                             Website: www.physicsbyfiziks.com                                                                                          
                                                            Email: fiziks.physics@gmail.com                                                                     143 
fiziks
Institute for NET/JRF, GATE, IIT‐JAM, JEST, TIFR and GRE in PHYSICAL SCIENCES 
 
NET/JRF (JUNE-2014)

Q43. Consider a system of two non-interacting identical fermions, each of mass m in an


infinite square well potential of width a . (Take the potential inside the well to be zero
and ignore spin). The composite wavefunction for the system with total energy
5π 2 2
E= is
2ma 2
2 ⎡ ⎛ πx1 ⎞ ⎛ 2πx 2 ⎞ ⎛ 2πx1 ⎞ ⎛ πx 2 ⎞⎤
(a) ⎢sin⎜ ⎟ sin⎜ ⎟ − sin⎜ ⎟ sin⎜ ⎟⎥
a⎣ ⎝ a ⎠ ⎝ a ⎠ ⎝ a ⎠ ⎝ a ⎠⎦

2 ⎡ ⎛ πx1 ⎞ ⎛ 2πx 2 ⎞ ⎛ 2πx1 ⎞ ⎛ πx 2 ⎞⎤


(b) ⎢sin⎜ ⎟ sin⎜ ⎟ + sin⎜ ⎟ sin⎜ ⎟⎥
a⎣ ⎝ a ⎠ ⎝ a ⎠ ⎝ a ⎠ ⎝ a ⎠⎦

2 ⎡ ⎛ πx1 ⎞ ⎛ 3πx 2 ⎞ ⎛ 3πx1 ⎞ ⎛ πx 2 ⎞⎤


(c) ⎢sin⎜ ⎟ sin⎜ ⎟ − sin⎜ ⎟ sin⎜ ⎟⎥
a ⎣ ⎝ a ⎠ ⎝ 2a ⎠ ⎝ 2a ⎠ ⎝ a ⎠ ⎦

2 ⎡ ⎛ πx1 ⎞ ⎛ πx 2 ⎞ ⎛ πx ⎞ ⎛ πx ⎞⎤
(d) ⎢sin⎜ ⎟ cos⎜ ⎟ − sin⎜ 2 ⎟ cos⎜ 2 ⎟⎥
a⎣ ⎝ a ⎠ ⎝ a ⎠ ⎝ a ⎠ ⎝ a ⎠⎦
Ans: (a)
Solution: Fermions have antisymmetric wave function
2 ⎡ ⎛ π x1 ⎞ ⎛ 2π x2 ⎞ ⎛ 2π x1 ⎞ ⎛ π x2 ⎞ ⎤
ψ ( x1 x2 ) = ⎢ sin ⎜ ⎟ sin ⎜ ⎟ − sin ⎜ ⎟ ⋅ sin ⎜ ⎟⎥
a⎣ ⎝ a ⎠ ⎝ a ⎠ ⎝ a ⎠ ⎝ a ⎠⎦

5π 2 2
∵ En = ⇒ nx1 = 1, nx2 = 2
2ma 2

Q44. A particle of mass m in the potential V ( x, y ) =


1
2
( )
mω 2 4 x 2 + y 2 , is in an eigenstate of

5
energy E = ω . The corresponding un-normalized eigen function is
2
⎡ mω ⎡ mω
(a) y exp ⎢− (2 x 2 + y 2 )⎥

(b) x exp ⎢− (2 x 2 + y 2 )⎥

⎣ 2 ⎦ ⎣ 2 ⎦
⎡ mω 2 ⎡ mω 2
(c) y exp ⎢− (x + y 2 )⎥

(d) xy exp ⎢− (x + y 2 )⎥

⎣ 2 ⎦ ⎣ 2 ⎦
                                                                                
Head office  Branch office 
 
fiziks, H.No. 40‐D, G.F, Jia Sarai,  Anand Institute of Mathematics, 
 
Near IIT, Hauz Khas, New Delhi‐16  28‐B/6, Jia Sarai, Near IIT 
 
Phone: 011‐26865455/+91‐9871145498 Hauz Khas, New Delhi‐16 
                                                   
                                             Website: www.physicsbyfiziks.com                                                                                          
                                                            Email: fiziks.physics@gmail.com                                                                     144 
fiziks
Institute for NET/JRF, GATE, IIT‐JAM, JEST, TIFR and GRE in PHYSICAL SCIENCES 
 
Ans: (a)

mω 2 (4 x 2 + y 2 ) , E =
1 5
Solution: V ( x, y ) = ω
2 2
1 1
⇒ V ( x, y ) = m ( 2ω ) x 2 + mω 2 y 2
2

2 2
⎛ 1⎞ ⎛ 1⎞ ⎛ 1⎞ ⎛ 1⎞
Now, E n = ⎜ n x + ⎟ ω x + ⎜ n y + ⎟ ω y = ⎜ n x + ⎟2 ω + ⎜ n y + ⎟ ω
⎝ 2⎠ ⎝ 2⎠ ⎝ 2⎠ ⎝ 2⎠

⎛ 3⎞
⇒ En = ⎜ 2 n x + n y + ⎟ ω
⎝ 2⎠
5
∵ En = ω when n x = 0 and n y = 1 .
2
Q45. A particle of mass m in three dimensions is in the potential
⎧0 r<a
V (r ) = ⎨
⎩∞ r>a
Its ground state energy is
π2 2
π2 2
3π 2 2 9π 2 2
(a) (b) (c) (d)
2ma 2 ma 2 2ma 2 2ma 2

Ans: (a)
⎞ d u ( r ) l ( l + 1)
2
⎛ 2
Solution: ⎜ − ⎟ 2
+ + V ( r ) u ( r ) = Eu ( r )
⎝ 2m ⎠ dr 2mr 2

d 2u ( r ) 2mE
2
= − Ku ( r ) ∵ K = 2
, l = 0, V ( r ) = 0
dr
u ( r ) = A sin Kr + B cos Kr

Using boundary condition, B = 0,

π2 2
u ( r ) = A sin Kr , r = a, u ( r ) = 0 ⇒ sin Ka = 0 ⇒ Ka = nπ ⇒ E = ∵n =1
2ma 2

                                                                                
Head office  Branch office 
 
fiziks, H.No. 40‐D, G.F, Jia Sarai,  Anand Institute of Mathematics, 
 
Near IIT, Hauz Khas, New Delhi‐16  28‐B/6, Jia Sarai, Near IIT 
 
Phone: 011‐26865455/+91‐9871145498 Hauz Khas, New Delhi‐16 
                                                   
                                             Website: www.physicsbyfiziks.com                                                                                          
                                                            Email: fiziks.physics@gmail.com                                                                     145 
fiziks
Institute for NET/JRF, GATE, IIT‐JAM, JEST, TIFR and GRE in PHYSICAL SCIENCES 
 
⎛ ∂ 1⎞
Q46. Given that pˆ r = −i ⎜ + ⎟ , the uncertainty Δp r in the ground state.
⎝ ∂r r ⎠
1
υ/ 0 (r ) = e − r / a0
πa 3
0

of the hydrogen atom is


2 2
(a) (b) (c) (d)
a0 a0 2a 0 a0

Ans: (a)
⎛ ∂ 1⎞ 1
Solution: pˆ r = −i ⎜ + ⎟ , ψ 0 (r ) = e − r / a0
⎝ ∂r r ⎠ πa0
3

2
ΔPr = Pr2 − Pr


1 ⎧⎪⎡ −r / a
⎛ ∂ 1 ⎞⎤ e 0 ⎫⎪
Now Pr = ∫ e − r / a0 ⎨ ⎢ − i ⎜ + ⎟ ⎥ ⎬4π r dr
2

0 πa 03 ⎪⎩⎣ ⎝ ∂r r ⎠⎦ πa 03 ⎪⎭

4π i ⎡∞ −r / a ⎛ −r / a ⎛ 1 ⎞ 1 − r / a0 ⎞ 2 ⎤
=− ⎢ ∫ e 0 ⎜⎜ e 0 ⎜⎜ − ⎟⎟ + e ⎟ r dr ⎥

πa 03 ⎢⎣ 0 ⎝ ⎝ a0 ⎠ r ⎠ ⎥⎦

4π i ⎡ 1 ⎤
∞ ∞

∫ ∫
− 2 r / a0 2
=− 3 ⎢
− e r dr + re − 2 r / a0 dr ⎥
πa 0 ⎣ a 0 0 0 ⎦

4π i ⎡ 1⎛ 2 ! 1 ! ⎞⎤
=− ⎢− ⎜ + ⎟⎥
π a03 ⎢⎣ a0 ⎜⎝ ( 2 / a0 ) ( 2 / a0 ) ⎟⎠ ⎥⎦
3 2

4π i ⎡ a02 a02 ⎤
=− − + =0 ⇒ Pr = 0
π a03 ⎢⎣ 4 4 ⎥⎦

1

⎧ ⎛ ∂ 2 2 ∂ ⎞ − r / a0 ⎫
3 ∫ ⎬4π r dr
Pr2 = e − r / a0 ⎨− 2
⎜⎜ 2 + ⎟⎟e 2

πa 0 0 ⎩ ⎝ ∂r r ∂r ⎠ ⎭

4π 2 ⎡∞ −r / a ⎛ −r / a ⎛ 1 ⎞ 2 ⎛ 1 ⎞ −r / a ⎞ ⎤ 2
=− ⎢ ∫ e 0 ⎜ e 0 ⎜ 2 ⎟ + ⋅ ⎜ − ⎟ e 0 ⎟ ⎥ r dr
π a03 ⎢⎣ 0 ⎝ ⎝ a0 ⎠ r ⎝ a0 ⎠ ⎠ ⎥⎦

                                                                                
Head office  Branch office 
 
fiziks, H.No. 40‐D, G.F, Jia Sarai,  Anand Institute of Mathematics, 
 
Near IIT, Hauz Khas, New Delhi‐16  28‐B/6, Jia Sarai, Near IIT 
 
Phone: 011‐26865455/+91‐9871145498 Hauz Khas, New Delhi‐16 
                                                   
                                             Website: www.physicsbyfiziks.com                                                                                          
                                                            Email: fiziks.physics@gmail.com                                                                     146 
fiziks
Institute for NET/JRF, GATE, IIT‐JAM, JEST, TIFR and GRE in PHYSICAL SCIENCES 
 
4 2⎡ 1 ⎤ 4 2⎡1 2 ! 2 1 ! ⎤
∞ ∞
2
= − 3 ⎢ ∫ 2 r 2 e −2 r / a0 dr − ∫ re −2 r / a0 dr ⎥ = − 3 ⎢ 2 − ⎥
a0 ⎣ 0 a0 a0 0 ⎦ a0 ⎢ a0 ( 2 / a0 )3 a0 ( 2 / a0 ) 2 ⎥
⎣ ⎦

4 2 ⎡ 2 ! a03 2 a02 ⎤ 4 2 ⎡ a0 a0 ⎤ 4 2 ⎛ a0 ⎞ 2
=− ⎢ × − × ⎥ = − ⎢4 − = − 3 ×⎜− ⎟ = 2
a03 ⎣ a02 8 a0 4⎦ a03 ⎣ 2 ⎥⎦ a0 ⎝ 4 ⎠ a0
2
2
∴ ΔP = Pr2 − Pr = −0 =
a02 a0

Q47. The ground state eigenfunction for the potential V ( x ) = −δ ( x ) where δ ( x ) is the delta

function, is given by υ/ ( x ) = Ae
−α x
, where A and α > 0 are constants. If a perturbation

H ′ = bx 2 is applied, the first order correction to the energy of the ground state will be
b b 2b b
(a) (b) (c) (d)
2α 2
α 2
α 2
2α 2
Ans: (d)

Solution: V ( x ) = −δ ( x ) , ψ (x ) = Ae
−α x

ψ ψ = 1 ⇒ ψ ( x ) = α e −α x

E = φ1 H ′ φ1 = ∫ α e −α x bx 2 α e −α x dx
1
1
−∞

∞ ∞
⎡0 ∞
⎤ ⎡ ∞

∫ αe bx dx = b ∫ α e x dx = bα ⎢ ∫ x e dx + ∫ x e dx ⎥ = bα ⎢2 × ∫ x e dx ⎥
− 2α x 2 −2α x 2 2 2α x 2 −2α x 2 − 2αx

−∞ −∞ ⎣ −∞ 0 ⎦ ⎣ 0 ⎦

⎡ 2! ⎤ 2! b
∫−∞
− 2α x
α e bx 2
dx = 2bα ⎢ 3⎥
= 2 × bα 3 =
⎣ (2α ) ⎦ 8α 2α 2

Q48. An electron is in the ground state of a hydrogen atom. The probability that it is within the
Bohr radius is approximately equal to
(a) 0.60 (b) 0.90 (c) 0.16 (d) 0.32
Ans: (d)

                                                                                
Head office  Branch office 
 
fiziks, H.No. 40‐D, G.F, Jia Sarai,  Anand Institute of Mathematics, 
 
Near IIT, Hauz Khas, New Delhi‐16  28‐B/6, Jia Sarai, Near IIT 
 
Phone: 011‐26865455/+91‐9871145498 Hauz Khas, New Delhi‐16 
                                                   
                                             Website: www.physicsbyfiziks.com                                                                                          
                                                            Email: fiziks.physics@gmail.com                                                                     147 
fiziks
Institute for NET/JRF, GATE, IIT‐JAM, JEST, TIFR and GRE in PHYSICAL SCIENCES 
 
2
a0 a0
1 4π
∫ 4π r dr = 2 ∫r e
− r / a0 2 2 −2 r / a0
Solution: Probability: e dr
0 πa 3
0
π ao 0

2
⎪⎧ ⎡ 2 −2 r / a0
a0 a0 a0
1 4 ⎛ a0 ⎞ ⎤ ⎡ −2 r / a0 ⎛ a0 ⎞ ⎛ a0 ⎞ ⎤
∫ e − r / a0
4π r dr = 3
2
⎨⎢r e ⎜ − 2 ⎟ ⎥ − ⎢ 2r e ( )
⎜ − 2 ⎟ ⎜ − 2 ⎟⎥
0 π a03 a0 ⎪⎩ ⎣ ⎝ ⎠⎦0 ⎣ ⎝ ⎠⎝ ⎠⎦0

⎛ a0 ⎞ ⎛ a0 ⎞ ⎛ a0 ⎞ ⎤ ⎪⎫
a0

+ ⎢ 2e −2 r / a0 ⎜ − 2 ⎟ ⎜ − 2 ⎟ ⎜ − 2 ⎟⎥ ⎬
⎣ ⎝ ⎠⎝ ⎠⎝ ⎠ ⎦ 0 ⎪⎭

4 ⎡ 2 − 2aa0 ⎛ a0 ⎞ ⎛ a02 ⎞ −2 a0 / a0 a03 −2 a0 / a0 −0 ⎛ a0


3
⎞⎤
= ⎢ a0 e 0 ⎜− 2 ⎟ − 2 a0 ⎜ ⎟ e − e + 2 e ⎜ ⎟⎥
a03 ⎢⎣ ⎝ ⎠ ⎝ 4 ⎠ 4 ⎝ 8 ⎠ ⎥⎦

4 ⎡ a03 1 a03 1 a03 a03 ⎤ ⎡ 5 1⎤ ⎡ 1 ⎤


= ⎢ − − − + ⎥ = 4 ⎢ − 2 + ⎥ = ⎢ −5 × 2 + 1⎥
a03 ⎣ 2 e
2
2 e 2
4e 2
4⎦ ⎣ 4e 4⎦ ⎣ e ⎦

= [ −5 × 0.137 + 1] = [ −0.685 + 1] = 0.32

Q49. A particle in the infinite square well


⎧0 0< x<a
V (x ) = ⎨
⎩∞ otherwise
is prepared in a state with the wavefunction
⎧ 3 ⎛ πx ⎞
⎪ A sin ⎜ ⎟ 0< x<a
υ/ (x ) = ⎨ ⎝a⎠
⎪0 otherwise

The expectation value of the energy of the particle is
5 2π 2 9 2π 2 9 2π 2 2
π2
(a) (b) (c) (d)
2ma 2 2ma 2 10ma 2 2ma 2

Ans: (c)
⎧ 3 ⎛ πx ⎞ ⎫
⎧0 0< x<a ⎪ A sin ⎜ ⎟ 0< x<a ⎪
Solution: V ( x ) = ⎨ ψ (x ) = ⎨ ⎝a⎠ ⎬
⎩∞ otherwise , ⎪0 otherwise⎪⎭

                                                                                
Head office  Branch office 
 
fiziks, H.No. 40‐D, G.F, Jia Sarai,  Anand Institute of Mathematics, 
 
Near IIT, Hauz Khas, New Delhi‐16  28‐B/6, Jia Sarai, Near IIT 
 
Phone: 011‐26865455/+91‐9871145498 Hauz Khas, New Delhi‐16 
                                                   
                                             Website: www.physicsbyfiziks.com                                                                                          
                                                            Email: fiziks.physics@gmail.com                                                                     148 
fiziks
Institute for NET/JRF, GATE, IIT‐JAM, JEST, TIFR and GRE in PHYSICAL SCIENCES 
 
⎛πx ⎞ 3 πx 1 3π x
ψ ( x ) = A sin 3 ⎜ ⎟ = A sin − A sin ∵sin 3 A = 3sin A − 4 sin 3 A
⎝ a ⎠ 4 a 4 a

3 πx 1 3π x A ⎡ a 2 πx a 2 3π x ⎤
= A sin − A sin = ⎢ × 3sin − sin ⎥
4 a 4 a 4⎣ 2 a a 2 a a ⎦

A⎡ a a ⎤
⇒ ψ ( x) = ⎢3 φ1 ( x ) − φ3 ( x ) ⎥
4⎣ 2 2 ⎦

a 2 a 2 10a 2 32
ψ ψ =1 ⇒9 A + A =1 ⇒ A =1 ⇒ A =
32 32 32 10a

1⎛ a 32 a 32 ⎞
ψ ( x ) = ⎜⎜ 3. φ1 ( x ) − φ3 ( x ) ⎟⎟
4⎝ 2 10a 2 10a ⎠
3 1
ψ (x ) = φ1 (x ) − φ3 ( x )
10 10

π2 2
9π 2 2
Now, E1 = , E2 = ⇒ E = an P ( an )
2ma 2 2ma 2
2 2
ϕ1 ψ 9 φ2 ψ 1
Probably P ( E1 ) = = , P ( E2 ) = =
ψψ 10 ψψ 10

9 π 2 2 1 9π 2 2 9π 2 2
E = × + × ⇒ E =
10 2ma 2 10 2ma 2 10ma 2

                                                                                
Head office  Branch office 
 
fiziks, H.No. 40‐D, G.F, Jia Sarai,  Anand Institute of Mathematics, 
 
Near IIT, Hauz Khas, New Delhi‐16  28‐B/6, Jia Sarai, Near IIT 
 
Phone: 011‐26865455/+91‐9871145498 Hauz Khas, New Delhi‐16 
                                                   
                                             Website: www.physicsbyfiziks.com                                                                                          
                                                            Email: fiziks.physics@gmail.com                                                                     149 
fiziks
Institute for NET/JRF, GATE, IIT‐JAM, JEST, TIFR and GRE in PHYSICAL SCIENCES 
 
NET/JRF (DEC-2014)
Q50. Suppose Hamiltonian of a conservative system in classical mechanics is H = ωxp , where
ω is a constant and x and p are the position and momentum respectively. The
corresponding Hamiltonian in quantum mechanics, in the coordinate representation, is
⎛ ∂ 1⎞ ⎛ ∂ 1⎞
(a) − i ω ⎜ x − ⎟ (b) − i ω ⎜ x + ⎟
⎝ ∂x 2 ⎠ ⎝ ∂x 2 ⎠
∂ i ω ∂
(c) − i ωx (d) − x
∂x 2 ∂x
Ans: (b)
Solution: Classically H = ω xp , quantum mechanically H must be Hermitian,
ω ω
So, H = ( xp + px ) and Hψ = ( xpψ + pxψ )
2 2
ω⎛ ∂ψ −i ∂ ( xψ ) ⎞ ω ∂ψ ∂ψ
⇒ Hψ = ⎜ x ( −i ) + ⎟ = ( −i ) ⎛⎜ x +x

+ψ ⎟
2⎝ ∂x ∂x ⎠ 2 ⎝ ∂x ∂x ⎠

−i ω ⎛ ∂ψ ⎞ −i ω ⎛ ∂ ⎞
⇒ Hψ = ⎜ 2x +ψ ⎟ = ⎜ 2 x + 1⎟ψ
2 ⎝ ∂x ⎠ 2 ⎝ ∂x ⎠

⎛ ∂ 1⎞
⇒ H ψ = −i ω ⎜ x + ⎟ ψ
⎝ ∂x 2 ⎠

Q51. Let ψ 1 and ψ 2 denote the normalized eigenstates of a particle with energy eigenvalues
E1 and E2 respectively, with E 2 > E1 . At time t = 0 the particle is prepared in a state
1
Ψ (t = 0) = (ψ 1 + ψ 2 )
2
The shortest time T at which Ψ (t = T ) will be orthogonal to Ψ (t = 0 ) is
2 π π π π
(a) (b) (c) (d)
(E 2 − E1 ) (E 2 − E1 ) 2(E 2 − E1 ) 4(E 2 − E1 )
Ans: (b)
1 1 − iE1T 1 − iE2T
Solution: ψ ( t = 0) = (ψ 1 + ψ 2 ) and ψ ( t = T ) = e ψ1 + e ψ2
2 2 2

                                                                                
Head office  Branch office 
 
fiziks, H.No. 40‐D, G.F, Jia Sarai,  Anand Institute of Mathematics, 
 
Near IIT, Hauz Khas, New Delhi‐16  28‐B/6, Jia Sarai, Near IIT 
 
Phone: 011‐26865455/+91‐9871145498 Hauz Khas, New Delhi‐16 
                                                   
                                             Website: www.physicsbyfiziks.com                                                                                          
                                                            Email: fiziks.physics@gmail.com                                                                     150 
fiziks
Institute for NET/JRF, GATE, IIT‐JAM, JEST, TIFR and GRE in PHYSICAL SCIENCES 
 
iE1 iE2 iE iE T
1 − T 1 − T − 1T − 2T i ( E2 − E1 )
∫ ψ *
( 0 )ψ ( T ) dx = 0 ⇒ e + e = 0 ⇒ e = − e ⇒ e = −1
2 2
⎛T ⎞ π
Equate real part ⇒ cos ⎜ ( E2 − E1 )⎟ = −1 ⇒ T = cos −1 ( −1) =
⎝ ⎠ ( E2 − E1 ) ( E2 − E1 )
Q52. Consider the normalized wavefunction
φ = a1ψ 11 + a 2ψ 10 + a3ψ 1−1
where ψ lm is a simultaneous normalized eigenfunction of the angular momentum

operators L2 and Lz , with eigenvalues l (l + 1) 2


and m respectively. If φ is an
eigenfunction of the operator L x with eigenvalue , then

1 1 1 1
(a) a1 = −a3 = , a2 = (b) a1 = a3 = , a2 =
2 2 2 2
1 1 1
(c) a1 = a3 = , a2 = − (d) a1 = a 2 = a3 =
2 2 3
Ans: (b)
L+ + L−
Solution: Lx φ = φ ⇒ ψ =λ ψ
2
For L+ , L+ [ a1ψ 11 + a2ψ 10 + a3ψ 1−1 ] = a1 0 ψ 12 + a2 2 ψ 11 + a3 2 ψ 10

= a2 2 ψ 11 + a3 2 ψ 10

For L− , L− [ a1ψ 11 + a2ψ 10 + a3ψ 1−1 ] = a1 2 ψ 10 + a2 2 ψ 1−1

L+ + L−
Given φ = φ
2
L+ + L− 1
⇒ φ = ⎡⎣ a2 2 ψ 11 + ( a1 + a3 ) 2 ψ 10 + a2 2 ψ 1−1 ⎤⎦
2 2
L+ + L−

2
φ = [a ψ
1 11 + a2ψ 10 + a3ψ 1−1 ] (Given)

a2
Thus = a1 ⇒ a2 = 2a1
2

                                                                                
Head office  Branch office 
 
fiziks, H.No. 40‐D, G.F, Jia Sarai,  Anand Institute of Mathematics, 
 
Near IIT, Hauz Khas, New Delhi‐16  28‐B/6, Jia Sarai, Near IIT 
 
Phone: 011‐26865455/+91‐9871145498 Hauz Khas, New Delhi‐16 
                                                   
                                             Website: www.physicsbyfiziks.com                                                                                          
                                                            Email: fiziks.physics@gmail.com                                                                     151 
fiziks
Institute for NET/JRF, GATE, IIT‐JAM, JEST, TIFR and GRE in PHYSICAL SCIENCES 
 
a1 + a3 a +a a a2
= a2 ⇒ 1 3 = 2a1 ⇒ 2 = a3 ∵ a12 + a22 + 2 = 1
2 2 2 2
1 1
a1 = a3 = , a2 =
2 2
Q53. Let x and p denote, respectively, the coordinate and momentum operators satisfying the
canonical commutation relation [x, p ] = i in natural units ( = 1) . Then the commutator

[x, pe ] is
−p

(a) i (1 − p )e − p (b) i (1 − p 2 )e − p (c) i (1 − e − p ) (d) ipe − p


Ans: (a)
Solution: ∵ [ x, p ] = i

⎡ p 2 p3 ⎤
⎡⎣ x, pe − p ⎤⎦ = [ x, p ] e − p + p ⎡⎣ x, e − p ⎤⎦ = ie− p + p ⎢ x,1 − p + − ....⎥
⎣ 2 3 ⎦
⎡ ⎡ p2 ⎤ ⎤ ⎡ 2ip 3ip 2 ⎤
= ie− p + p ⎢[ x,1] − [ x, p ] + ⎢ x, ⎥ ....⎥ = ie − p + p ⎢0 − i + − ......⎥
⎣ ⎣ 2⎦ ⎦ ⎣ 2 3 ⎦
⎡ p3 ⎤
⇒ ⎡⎣ x, pe− p ⎤⎦ = ie − p − i ⎢ p − p 2 + .....⎥ = ie − p − ipe − p = i (1 − p ) e− p
⎣ 2 ⎦

Q54. Let σ = (σ 1 , σ 2 , σ 3 ) , where σ 1 , σ 2 , σ 3 are the Pauli matrices. If a and b are two

arbitrary constant vectors in three dimensions, the commutator a ⋅ σ , b ⋅ σ is equal to (in [ ]


the following I is the identity matrix)
( )
(a) a ⋅ b (σ 1 + σ 2 + σ 3 ) (
(b) 2i a × b ⋅σ )
(c) (a ⋅ b ) I (d) a b I

Ans: (b)
Solution: a = a1iˆ + a2 ˆj + a3 kˆ , b = b1iˆ + b2 ˆj + b3 kˆ , σ = σ x iˆ + σ y ˆj + σ z kˆ

⎡ a ⋅ σ , b ⋅ σ ⎤ = ⎡ a1σ x + a2σ y + + a3σ z , b1σ x + b2σ y + b3σ z ⎤


⎣ ⎦ ⎣ ⎦

⎡ a ⋅ σ , b ⋅ σ ⎤ = a1b1 [σ x , σ x ] + a1b2 ⎡σ x , σ y ⎤ + a1b3 [σ x , σ z ] + a2b1 ⎡σ y , σ x ⎤ + a2b2 ⎡σ y , σ y ⎤


⎣ ⎦ ⎣ ⎦ ⎣ ⎦ ⎣ ⎦

+ a2b3 ⎡⎣σ y , σ z ⎤⎦ + a3b1 [σ z , σ x ] + a3b2 ⎡⎣σ z , σ y ⎤⎦ + a3b3 [σ z , σ z ]


                                                                                
Head office  Branch office 
 
fiziks, H.No. 40‐D, G.F, Jia Sarai,  Anand Institute of Mathematics, 
 
Near IIT, Hauz Khas, New Delhi‐16  28‐B/6, Jia Sarai, Near IIT 
 
Phone: 011‐26865455/+91‐9871145498 Hauz Khas, New Delhi‐16 
                                                   
                                             Website: www.physicsbyfiziks.com                                                                                          
                                                            Email: fiziks.physics@gmail.com                                                                     152 
fiziks
Institute for NET/JRF, GATE, IIT‐JAM, JEST, TIFR and GRE in PHYSICAL SCIENCES 
 
= a1b1 ⋅ 0 + a1b2 ⋅ 2iσ z − 2ia1b3σ y − a2b1 ⋅ 2iσ z + 0 + a2b3 ⋅ 2iσ x + a3b1 ⋅ 2iσ y − a3b2 ⋅ 2iσ x + 0

⇒ ⎡⎣ a ⋅ σ , b ⋅ σ ⎤⎦ = = 2i a × b ⋅ σ( )
Q55. The ground state energy of the attractive delta function potential
V ( x ) = −bδ ( x ) ,
where b > 0 , is calculated with the variational trial function
⎧ πx ⎫
⎪ A cos , for − a < x < a, ⎪
ψ (x ) = ⎨ 2a ⎬
⎪⎩ 0, otherwise, ⎪⎭

is
mb 2 2mb 2 mb 2 mb 2
(a) − (b) − (c) − (d) −
π2 2
π2 2
2π 2 2 4π 2 2
Ans: (b)
⎧ πx
Solution: V ( x ) = −bδ ( x ) ; b > 0 and ψ ( x ) = ⎨ A cos ; −a < x < a
⎩ 2a

2 πx
Normalized ψ = cos
2a 2a

− 2 ∂2
a π2 2
T =∫ ψ ψ dx =
*
−a 2 m ∂x 2 8ma 2
a 2 b
V = ∫ ψ * − bδ ( x )ψ dx = ( −b ) = −
−a 2a a
π2 2
b
E = 2

8ma a
∂ E −2π 2 2 b −π 2 2 −π 2 2
⇒ = + = 0 ⇒ + b = 0 ⇒ a =
∂a 8ma 3 a2 4ma 4ma

Put the value of a in equation: E =


b π
π2
− =
2 22
( 4mb ) b ( 4mb ) 2mb 2
− 2 2 =− 2 2
2

8ma 2 a 8m π 2 2
2
π π ( ) ( )

                                                                                
Head office  Branch office 
 
fiziks, H.No. 40‐D, G.F, Jia Sarai,  Anand Institute of Mathematics, 
 
Near IIT, Hauz Khas, New Delhi‐16  28‐B/6, Jia Sarai, Near IIT 
 
Phone: 011‐26865455/+91‐9871145498 Hauz Khas, New Delhi‐16 
                                                   
                                             Website: www.physicsbyfiziks.com                                                                                          
                                                            Email: fiziks.physics@gmail.com                                                                     153 
fiziks
Institute for NET/JRF, GATE, IIT‐JAM, JEST, TIFR and GRE in PHYSICAL SCIENCES 
 
Q56. Let ψ = c0 0 + c1 1 (where c 0 and c1 are constants with c02 + c12 = 1 ) be a linear

combination of the wavefunctions of the ground and first excited states of the one-
dimensional harmonic oscillator. For what value of c 0 is the expectation value x a

maximum?

1 1
(a) x = , c0 = (b) x = , c0 =
mω 2 2mω 2

1 1
(c) x = , c0 = (d) x = , c0 =
2mω 2 mω 2
Ans: (c)
Solution: ψ = c0 0 + c1 1

X = ψ Xψ

⎣ ( ⎦ ⎣)
⇒ X = 2c0 c1 0 X 1 = ⎡ c02 + c12 − ( c0 − c1 ) ⎤ 0 X 1 = ⎡1 − ( c0 − c1 ) ⎤ 0 X 1
2 2


1
For max X = c0 = c1 ∵ c02 + c12 = 1 ⇒ c0 =
2
1 1
⇒ X =2 0 X 1 = 0 X 1
2 2

2mω
( 0 a+a 1)⇒ +
X =
2mω
Q57. Consider a particle of mass m in the potential V ( x ) = a x , a > 0 . The energy eigen-

values E n (n = 0, 1, 2, ....) , in the WKB approximation, are


1/ 3 2/3
⎡ 3a π ⎛ 1 ⎞⎤ ⎡ 3a π ⎛ 1 ⎞⎤
(a) ⎢ ⎜ n + ⎟⎥ (b) ⎢ ⎜ n + ⎟⎥
⎣ 4 2m ⎝ 2 ⎠⎦ ⎣ 4 2m ⎝ 2 ⎠⎦
4/3
3a π ⎛ 1⎞ ⎡ 3a π ⎛ 1 ⎞⎤
(c) ⎜n + ⎟ (d) ⎢ ⎜ n + ⎟⎥
4 2m ⎝ 2⎠ ⎣ 4 2m ⎝ 2 ⎠⎦
Ans: (b)
Solution: V ( x ) = a x , x>0

                                                                                
Head office  Branch office 
 
fiziks, H.No. 40‐D, G.F, Jia Sarai,  Anand Institute of Mathematics, 
 
Near IIT, Hauz Khas, New Delhi‐16  28‐B/6, Jia Sarai, Near IIT 
 
Phone: 011‐26865455/+91‐9871145498 Hauz Khas, New Delhi‐16 
                                                   
                                             Website: www.physicsbyfiziks.com                                                                                          
                                                            Email: fiziks.physics@gmail.com                                                                     154 
fiziks
Institute for NET/JRF, GATE, IIT‐JAM, JEST, TIFR and GRE in PHYSICAL SCIENCES 
 
α2 ⎛ 1⎞
According to W.K.B., ∫ pdq = ⎜ n + ⎟ where α1 and α 2 are positive mid point
α1 ⎝ 2⎠

P2
E= + a x ⇒ P = 2m ( E − a x )
2m −E / α E /α

⎛ 1⎞
2m ( E − a x )dx = ⎜ n + ⎟
E/α
∫ − E /α
⎝ 2⎠
0 E /α ⎛ 1⎞
∫ 2m ( E + ax )dx + ∫ 2m ( E − ax )dx = ⎜ n + ⎟
− E /α 0
⎝ 2⎠
E /α ⎛ 1⎞
2∫ 2m ( E − ax )dx = ⎜ n + ⎟
0
⎝ 2⎠

2m ( E − ax ) = t At x = 0, t = 2mE; x = E / a, t = 0

⇒ −2madx = dt
2 mE
2 mE ⎛ 1⎞ 2 ⎛ 1⎞
⇒ 2ma ∫ t dt = ⎜ n + ⎟ ⇒ 2ma t
1/ 2
= ⎜n+ ⎟
0
⎝ 2⎠ 3 0 ⎝ 2⎠

4 ⎛ 1⎞ 4 ⎛ 1⎞
= ⎜ n + ⎟ ⇒ ma ( 2mE ) = ⎜ n + ⎟
2 mE 3/ 2
⇒ ma t 3/2
3 0
⎝ 2⎠ 3 ⎝ 2⎠
2/3
4 ⎛ 1⎞ ⎡ 3a π ⎛ 1 ⎞⎤
⇒ ⋅ 23/ 2 am5 / 2 E 3/ 2 = ⎜ n + ⎟ ⇒ E = ⎢ ⎜ n + ⎟⎥
3 ⎝ 2⎠ ⎣ 4 2m ⎝ 2 ⎠⎦

Q58. The Hamiltonian H 0 for a three-state quantum system is given by the matrix

⎛1 0 0⎞ ⎛ 0 1 0⎞
⎜ ⎟ ⎜ ⎟
H 0 = ⎜ 0 2 0 ⎟ . When perturbed by H ′ =∈ ⎜ 1 0 1 ⎟ where ∈<< 1 , the resulting shift
⎜0 0 2⎟ ⎜ 0 1 0⎟
⎝ ⎠ ⎝ ⎠
in the energy eigenvalue E 0 = 2 is

(a) ∈, − 2 ∈ (b) − ∈, 2 ∈ (c) ± ∈ (d) ± 2 ∈


Ans: None of the answer is correct.
⎛ 1 0 0⎞ ⎛ 0 1 0⎞
Solution: H 0 = ⎜ 0 2 0⎟ , H ′ =∈0 ⎜ 1 0 1⎟
⎜ ⎟ ⎜ ⎟
⎝ 0 0 2⎠ ⎝ 0 1 0⎠
                                                                                
Head office  Branch office 
 
fiziks, H.No. 40‐D, G.F, Jia Sarai,  Anand Institute of Mathematics, 
 
Near IIT, Hauz Khas, New Delhi‐16  28‐B/6, Jia Sarai, Near IIT 
 
Phone: 011‐26865455/+91‐9871145498 Hauz Khas, New Delhi‐16 
                                                   
                                             Website: www.physicsbyfiziks.com                                                                                          
                                                            Email: fiziks.physics@gmail.com                                                                     155 
fiziks
Institute for NET/JRF, GATE, IIT‐JAM, JEST, TIFR and GRE in PHYSICAL SCIENCES 
 
⎛ 2 0⎞ ⎛ 0 1⎞
⎜⎝ 0 2⎟⎠ in H 0 is not ∈0 ⎜⎝ 1 0⎟⎠ in H ′ because H ′ is not in block diagonal form. So we

must diagonalised whole H ′ . The Eigen value at H ′ = 0, + 2 ∈0 , − 2 ∈0 .

⎛0 0 0 ⎞
⎜ ⎟
After diagonalisation H ′ =∈0 ⎜ 0 2 0 ⎟ , λ = 0 is correction for Eigenvalue at H 0 .
⎜⎝ 0 0 − 2 ⎟⎠

So ± 2 ∈0 is the correction for eigenvalue of H 0 = 2


None of the answer is correct.

NET/JRF (JUNE-2015)
Q59. The ratio of the energy of the first excited state E1 , to that of the ground state E0 , to that

L
of a particle in a three-dimensional rectangular box of side L, L and , is
2
(a) 3 : 2 (b) 2 :1 (c) 4 :1 (d) 4 : 3
Ans. (a)
π2 2
Solution: E = ⎡⎣ nx2 + n y2 + 4nz2 ⎤⎦
2
2mL
For ground state nz = 1, n y = 1, nz = 1

6π 2 2
E0 =
2mL2
For first excited state. nx = 1, n y = 2nz = 1

π2 2
9π 2 2
2 (
E = E1 = 1 + 4 + 4) ⇒
2mL 2mL2
E1 9 3
∴ = ⇒
E0 6 2

                                                                                
Head office  Branch office 
 
fiziks, H.No. 40‐D, G.F, Jia Sarai,  Anand Institute of Mathematics, 
 
Near IIT, Hauz Khas, New Delhi‐16  28‐B/6, Jia Sarai, Near IIT 
 
Phone: 011‐26865455/+91‐9871145498 Hauz Khas, New Delhi‐16 
                                                   
                                             Website: www.physicsbyfiziks.com                                                                                          
                                                            Email: fiziks.physics@gmail.com                                                                     156 
fiziks
Institute for NET/JRF, GATE, IIT‐JAM, JEST, TIFR and GRE in PHYSICAL SCIENCES 
 
Q60. If Li are the components of the angular momentum operator L , then the operator

∑ i =1,2,3
⎡ L, Li ⎤ equals
⎣ ⎦

(a) L (b) 2 L (c) 3L (d) − L


Ans. (b)
Solution: Let L = Lxiˆ + Ly ˆj + Lz kˆ

x = 1, y = 2, z = 3

⎡ L, Lx ⎤ = ⎡⎣ Ly , Lx ⎤⎦ j + [ Lz , Lx ] kˆ = −i Lz ˆj + Ly kˆ i
⎣ ⎦

⎡ ⎡ L, Lx ⎤ , Lx ⎤ = i [ − Lz , Lx ] ˆj + ⎡ Ly , Lx ⎤ i − i .i Ly ˆj − ( i ) Lz ( i ) Lz ( i ) .kˆ = 2 ⎡ Ly ˆj + Lz kˆ ⎤
⎣⎣ ⎦ ⎦ ⎣ ⎦ ⎣ ⎦
similarly ⎡ ⎡⎣ L, Ly ⎤⎦ Ly ⎤ = 2 ⎡ Lx iˆ + Lz kˆ ⎤
⎣ ⎦ ⎣ ⎦

⎡ ⎡ L, Lz ⎤ Lz ⎤ = 2
⎡ Lx iˆ + Ly ˆj ⎤
⎣⎣ ⎦ ⎦ ⎣ ⎦

∑ ⎡⎣[ L, Li ] Li ⎤⎦ = 2
i =1,2,3
2 ⎡ Lx iˆ + Ly ˆj + Lz kˆ ⎤ = 2 L
⎣ ⎦
put =1

Q61. The wavefunction of a particle in one-dimension is denoted by ψ ( x ) in the coordinate


− ipx
representation and by φ ( p ) = ∫ψ ( x ) e dx in the momentum representation. If the

action of an operator Tˆ on ψ ( x ) is given by Tˆψ ( x ) = ψ ( x + a ) , where a is a constant

then Tˆφ ( p ) is given by


− iap
i
(a) − apφ ( p ) (b) e φ ( p)

+ iap
⎛ i ⎞
(c) e φ ( p) (d) ⎜1 + ap ⎟ φ ( p )
⎝ ⎠
Ans. (c)
− ipx
Solution: φ ( p ) = ∫ψ ( x )e dx

Tψ ( x ) = ψ ( x + a )

                                                                                
Head office  Branch office 
 
fiziks, H.No. 40‐D, G.F, Jia Sarai,  Anand Institute of Mathematics, 
 
Near IIT, Hauz Khas, New Delhi‐16  28‐B/6, Jia Sarai, Near IIT 
 
Phone: 011‐26865455/+91‐9871145498 Hauz Khas, New Delhi‐16 
                                                   
                                             Website: www.physicsbyfiziks.com                                                                                          
                                                            Email: fiziks.physics@gmail.com                                                                     157 
fiziks
Institute for NET/JRF, GATE, IIT‐JAM, JEST, TIFR and GRE in PHYSICAL SCIENCES 
 
− ipx − ipx ipa − ip ( x + a )

T φ ( p ) = ∫ Tψ ( x ) e dx = ∫ψ ( x + a ) e dx = e ∫ψ ( x + a )e dx
ipa
⇒ Tφ ( p ) = e φ ( p )

Q62. The differential cross-section for scattering by a target is given by



(θ , φ ) = a 2 + b2 cos 2 θ

If N is the flux of the incoming particles, the number of particles scattered per unit time
is
4π ⎛ 1 ⎞
(a)
3
(
N a2 + b2 ) (b) 4π N ⎜ a 2 + b 2 ⎟
⎝ 6 ⎠

⎛1 1 ⎞ ⎛ 1 ⎞
(c) 4π N ⎜ a 2 + b 2 ⎟ (d) 4π N ⎜ a 2 + b 2 ⎟
⎝2 3 ⎠ ⎝ 3 ⎠
Ans. (d)

Solution: = a 2 + b 2 cos 2 θ

π 2π π 2π
2 ⎡ b2 ⎤
σ = a 2 ∫ ∫ sin θ dθ dφ + b 2 ∫ cos 2 θ sin θ dθ ∫ dφ = a 2 .4π + b 2 .2π × = 4π ⎢ a 2 + ⎥
0 0 0 0
3 ⎣ 3⎦

⎛ b2 ⎞
number of particle per unit σ .N = 4π N ⎜ a 2 + ⎟
⎝ 3⎠

1
Q63. A particle of mass m is in a potential V = mω 2 x 2 , where ω is a constant. Let
2
mω ⎛ ipˆ ⎞ daˆ
aˆ = ⎜ xˆ + ⎟ . In the Heisenberg picture is given by
2 ⎝ mω ⎠ dt

(a) ω â (b) −iω aˆ (c) ω â † (d) iω aˆ †


Ans. (b)
1
Solution: V = mω 2 x 2
2

                                                                                
Head office  Branch office 
 
fiziks, H.No. 40‐D, G.F, Jia Sarai,  Anand Institute of Mathematics, 
 
Near IIT, Hauz Khas, New Delhi‐16  28‐B/6, Jia Sarai, Near IIT 
 
Phone: 011‐26865455/+91‐9871145498 Hauz Khas, New Delhi‐16 
                                                   
                                             Website: www.physicsbyfiziks.com                                                                                          
                                                            Email: fiziks.physics@gmail.com                                                                     158 
fiziks
Institute for NET/JRF, GATE, IIT‐JAM, JEST, TIFR and GRE in PHYSICAL SCIENCES 
 
mω ⎛ ipˆ ⎞
a= ⎜ xˆ + ⎟
2 ⎝ mω ⎠
da 1 ∂a ∂a
= [ a, H ] + =0
dt i ∂t ∂t

1 mω ⎡ ⎡ p 2 ⎤ imω 2 ⎤ 1 mω ⎛ 2 p iω ⎞
= ⎢ ⎢ x, ⎥+ ⎡⎣ px, x 2 ⎤⎦ ⎥ = ⎜i + ( −2 x ) i ⎟
i 2 ⎣ ⎣ 2m ⎦ 2mω ⎦ i 2 ⎝ 2m 2 ⎠

mω ⎛p ⎞ mω ⎛ ip ⎞
= ⎜ − iω x ⎟ = −iω ⎜x+ ⎟ = −iω aˆ
2 ⎝m ⎠ 2 ⎝ mω ⎠
Q64. Two different sets of orthogonal basis vectors
⎧⎛ 1 ⎞ ⎛ 0 ⎞ ⎫ ⎧ 1 ⎛1⎞ 1 ⎛ 1 ⎞ ⎫
⎨⎜ ⎟ , ⎜ ⎟ ⎬ and ⎨ ⎜ ⎟, ⎜ ⎟ ⎬ are given for a two dimensional real vector space.
⎩⎝ 0 ⎠ ⎝ 1 ⎠ ⎭ ⎩ 2 ⎝ 1 ⎠ 2 ⎝ −1 ⎠ ⎭
The matrix representation of a linear operator  in these bases are related by a unitary
transformation. The unitary matrix may be chosen to be
⎛ 0 −1 ⎞ ⎛0 1⎞
(a) ⎜ ⎟ (b) ⎜ ⎟
⎝1 0 ⎠ ⎝1 0⎠
1 ⎛1 1 ⎞ 1 ⎛1 0 ⎞
(c) ⎜ ⎟ (d) ⎜ ⎟
2 ⎝1 −1⎠ 2 ⎝1 1 ⎠
Ans. (c)
⎛1 0⎞
Solution: u1 = ⎜ ⎟
⎝0 1⎠
1 ⎛1 1 ⎞
u2 = ⎜ ⎟
2 ⎝ 1 −1 ⎠

1 ⎛1 1 ⎞
u = u1 ⊗ u2 = ⎜ ⎟
2 ⎝1 −1⎠

                                                                                
Head office  Branch office 
 
fiziks, H.No. 40‐D, G.F, Jia Sarai,  Anand Institute of Mathematics, 
 
Near IIT, Hauz Khas, New Delhi‐16  28‐B/6, Jia Sarai, Near IIT 
 
Phone: 011‐26865455/+91‐9871145498 Hauz Khas, New Delhi‐16 
                                                   
                                             Website: www.physicsbyfiziks.com                                                                                          
                                                            Email: fiziks.physics@gmail.com                                                                     159 
fiziks
Institute for NET/JRF, GATE, IIT‐JAM, JEST, TIFR and GRE in PHYSICAL SCIENCES 
 
Q65. The Dirac Hamiltonian H = cα . p + β mc for a free electron corresponds to the classical
2

relation E 2 = p 2 c 2 + m 2 c 4 . The classical energy-momentum relation of a piratical of


2
⎛ q ⎞

( )
charge q in a electromagnetic potential φ , A is ( E − qφ ) = c ⎜ p − A ⎟ + m 2 c 4 .
c ⎠
2 2

Therefore, the Dirac Hamiltonian for an electron in an electromagnetic field is


e ⎛ e ⎞
(a) cα . p + A. A + β mc 2 − eφ (b) cα . ⎜ p + A ⎟ + β mc 2 + eφ
c ⎝ c ⎠

⎛ e ⎞ ⎛ e ⎞
(c) c ⎜ α . p + eφ + A ⎟ + β mc 2 (d) cα . ⎜ p + A ⎟ + β mc 2 − eφ
⎝ c ⎠ ⎝ c ⎠
Ans. (d)
Solution: Electromagnetic interaction of Dirac particle
1
⎡⎛ qA ⎞ ⎤ 2
H = ⎢⎜ P − 2 ⎟ c 2 + m 2 c 4 ⎥ + qφ
⎣⎢⎝ c ⎠ ⎦⎥
Quantum mechanical Hamiltonian

∂ψ ⎡ ⎛ qA ⎞ ⎤
i = ⎢ cα ⎜ P − ⎟ + β mc + qφ ⎥ψ
2

∂t ⎢⎣ ⎝ c ⎠ ⎦⎥
put q = −e

⎡ ⎛ e ⎞ ⎤
H = ⎢ cα . ⎜ P + A ⎟ + β mc 2 − eφ ⎥
⎣ ⎝ c ⎠ ⎦
Q66. A particle of energy E scatters off a repulsive spherical potential
⎧V for r < a
V (r ) = ⎨ 0
⎩ 0 for r ≤ a
where V0 and a are positive constants. In the low energy limit, the total scattering cross-
2
⎛ 1 ⎞ 2m
section is σ = 4π a 2 ⎜ tanh ka − 1⎟ , where k 2 = 2 (V0 − E ) > 0 . In the limit V0 → ∞
⎝ ka ⎠ h
the ratio of σ to the classical scattering cross-section off a sphere a radius a is
1
(a) 4 (b) 3 (c) 1 (d)
2

                                                                                
Head office  Branch office 
 
fiziks, H.No. 40‐D, G.F, Jia Sarai,  Anand Institute of Mathematics, 
 
Near IIT, Hauz Khas, New Delhi‐16  28‐B/6, Jia Sarai, Near IIT 
 
Phone: 011‐26865455/+91‐9871145498 Hauz Khas, New Delhi‐16 
                                                   
                                             Website: www.physicsbyfiziks.com                                                                                          
                                                            Email: fiziks.physics@gmail.com                                                                     160 
fiziks
Institute for NET/JRF, GATE, IIT‐JAM, JEST, TIFR and GRE in PHYSICAL SCIENCES 
 
Ans. (a)
2
⎡1 ⎤
Solution: σ = 4π a ⎢ tanh ka − 1⎥ 2

⎣ ka ⎦
ka → ∞ tanh ka → 1
2
⎛ 1 ⎞
σ = 4π a ⎜ − 1⎟
2

⎝ ka ⎠
ka → ∞
σ H = 4π a 2
σH
classically σ c = π a 2
σc = 4

NET/JRF (DEC-2015)
Q67. A Hermitian operator O has two normalized eigenstates 1 and 2 with eigenvalues 1

and 2 , respectively. The two states u = cos θ 1 + sin θ 2 and v = cos φ 1 + sin φ 2

are such that v O ν = 7 / 4 and u v = 0 . Which of the following are possible values of

θ and φ ?
π π π π
(a) θ = − and φ = (b) θ = and φ =
6 3 6 3
π π π π
(c) θ = − and φ = (d) θ = and φ = −
4 4 3 6
Ans.: (a)
Solution: u = cos θ 1 + sin θ 2

v = cos φ 1 + sin φ 2

it is given Oˆ 1 = 1

Oˆ 2 = 2

                                                                                
Head office  Branch office 
 
fiziks, H.No. 40‐D, G.F, Jia Sarai,  Anand Institute of Mathematics, 
 
Near IIT, Hauz Khas, New Delhi‐16  28‐B/6, Jia Sarai, Near IIT 
 
Phone: 011‐26865455/+91‐9871145498 Hauz Khas, New Delhi‐16 
                                                   
                                             Website: www.physicsbyfiziks.com                                                                                          
                                                            Email: fiziks.physics@gmail.com                                                                     161 
fiziks
Institute for NET/JRF, GATE, IIT‐JAM, JEST, TIFR and GRE in PHYSICAL SCIENCES 
 
7
v Oˆ v =
4
7 7
cos 2 φ + 2sin 2 φ = ⇒ cos 2 φ + sin 2 φ = 1 ⇒ sin 2 φ = − 1
4 4
3 π
sin φ = ⇒φ =
2 3
u v =0

⇒ cos θ cos φ + sin θ sin φ = 0 ⇒ cos (θ − φ ) = 0

π π π π π π 5π π
⇒ θ −φ = or φ − θ = ⇒θ = + or θ = − ⇒θ = or θ = −
2 2 2 3 3 2 6 6
⎛x⎞
Q68. The ground state energy of a particle of mass m in the potential V ( x ) = V0 cosh ⎜ ⎟ ,
⎝L⎠
2
where L and V0 are constants (with V0 >> ) is approximately
2ml 2
2V0 V0 V0 V0
(a) V0 + (b) V0 + (c) V0 + (d) V0 +
L m L m 4L m 2L m
Ans.: (d)
⎛x⎞ V
Solution: (
V0 = cosh ⎜ ⎟ = 0 e x / L + e − x / L
⎝L⎠ 2
)
V ⎡ x 1 ⎛ x ⎞ 2 ⎤ V0 ⎡ x 1 ⎛ x ⎞2 ⎤
= 0 ⎢1 + + ⎜ ⎟ ....⎥ + ⎢1 − + ⎜ ⎟ + ....⎥
2 ⎢⎣ L 2! ⎝ L ⎠ ⎥⎦ 2 ⎢⎣ L 2! ⎝ L ⎠ ⎥⎦
2
V V V ⎛x⎞ 1⎛V ⎞ 2
= 0 + 0 + 0 ⎜ ⎟ = V0 + ⎜ 02 ⎟x
2 2 2 ⎝L⎠ 2⎝ L ⎠

V0 V0
K= , ω=
L2 mL2
So, ground state energy is

ω V0 V0
V0 + ⇒ V0 + 2
⇒ V0 +
2 2 mL 2L m

                                                                                
Head office  Branch office 
 
fiziks, H.No. 40‐D, G.F, Jia Sarai,  Anand Institute of Mathematics, 
 
Near IIT, Hauz Khas, New Delhi‐16  28‐B/6, Jia Sarai, Near IIT 
 
Phone: 011‐26865455/+91‐9871145498 Hauz Khas, New Delhi‐16 
                                                   
                                             Website: www.physicsbyfiziks.com                                                                                          
                                                            Email: fiziks.physics@gmail.com                                                                     162 
fiziks
Institute for NET/JRF, GATE, IIT‐JAM, JEST, TIFR and GRE in PHYSICAL SCIENCES 
 
Q69. Let ψ nlm denote the eigenstates of a hydrogen atom in the usual notation. The state

1⎡
2ψ 200 − 3ψ 211 + 7ψ 210 − 5ψ 21−1 ⎤⎦
5⎣
is an eigenstate of
(a) L2 , but not of the Hamiltonian or Lz (b) the Hamiltonian, but not of L2 or Lz

(c) the Hamiltonian, L2 and Lz (d) L2 and Lz , but not of the Hamiltonian
Ans.: (b)
1
Solution: ψ = ⎡⎣ 2ψ 200 − 3ψ 211 + 7ψ 2 10 − 5ψ 2 1 −1 ⎤⎦
5
13.6
H ψ =− ψ
4
So ψ is eigen state of H

But L2 ψ ≠ α ψ and Lz ψ ≠ β ψ

So ψ is not eigen state of L2 and Lz

1
Q70. The Hamiltonian for a spin- particle at rest is given by H = E0 (σ z + ασ x ) , where σ x
2
and σ z are Pauli spin matrices and E0 and α are constants. The eigenvalues of this
Hamiltonian are

(a) ± E0 1 + α 2 (b) ± E0 1 − α 2

⎛ 1 ⎞
(c) E0 (doubly degenerate) (d) E0 ⎜1 ± α 2 ⎟
⎝ 2 ⎠
Ans.: (a)
⎡⎛ 1 0 ⎞ ⎛ 0 1 ⎞⎤ ⎛1 α⎞
Solution: H = E0 (σ z + ασ x ) = E0 ⎢⎜ ⎟ +α ⎜ ⎟ ⎥ ⇒ H = E0 ⎜ ⎟
⎣ ⎝ 0 −1 ⎠ ⎝ 1 0 ⎠⎦ ⎝α −1 ⎠

if λ is eigen value, then


⎛ (1 − λ ) α ⎞
H − λ I = 0 ⇒ E0 ⎜ ⎟ λ = ± E0 1 + α 2
⎝ α − (1 + λ ) ⎠

                                                                                
Head office  Branch office 
 
fiziks, H.No. 40‐D, G.F, Jia Sarai,  Anand Institute of Mathematics, 
 
Near IIT, Hauz Khas, New Delhi‐16  28‐B/6, Jia Sarai, Near IIT 
 
Phone: 011‐26865455/+91‐9871145498 Hauz Khas, New Delhi‐16 
                                                   
                                             Website: www.physicsbyfiziks.com                                                                                          
                                                            Email: fiziks.physics@gmail.com                                                                     163 
fiziks
Institute for NET/JRF, GATE, IIT‐JAM, JEST, TIFR and GRE in PHYSICAL SCIENCES 
 
Q71. A hydrogen atom is subjected to the perturbation
2r
V pert ( r ) =∈ cos
a0

where a0 is the Bohr radius. The change in the ground state energy to first order in ∈

∈ ∈ −∈ −∈
(a) (b) (c) (d)
4 2 2 4
Ans.: (d)
Solution: For First order perturbation
−r
1 ⎛ 2r ⎞
E11 = φ100 V p φ100 , φ100 = e a , V p =∈ cos ⎜ ⎟
π a03 ⎝ a0 ⎠
∞ −2 r ∞ −2 r
1 ⎛ 2r ⎞ 4∈ ⎛ 2r ⎞
E = ∫ 3 e a0 ∈ cos ⎜ ⎟ 4π r 2 dr = 3 ∫ e a0 cos ⎜ ⎟ r 2 dr
1
1
0
π a0 ⎝ a0 ⎠ a0 0 ⎝ a0 ⎠

⎡ ia2 r −i 2 r

2 ∈ ⎡ a0 (1−i ) 2 ⎤
∞ −2 r ∞ −2 r ∞ −2 r (1+ i )
4∈ ⎢e + e 0
0 a
⎥ 2
= 3 ∫e
a03 ⎢⎣ ∫0 ∫0
a0
⎢ ⎥ r dr = ⎢ e r dr + e a0
r 2
dr ⎥
a0 0 2 ⎥⎦
⎢⎣ ⎥⎦

⎡ ⎤
⎢ ⎥
2∈⎢ 2! 2! ⎥ ∈⎡ 1 1 ⎤
⇒ + ⇒ ⎢ + ⎥
a03 ⎢ ⎡ 2 ⎤
3
⎡ 2 ⎤
3⎥
2 ⎢ (1 − i )3 (1 + i )3 ⎥
⎢ (1 − i )⎥ ⎢ (1 + i )⎥ ⎥⎥ ⎣ ⎦
⎢⎣ ⎢⎣ a0 ⎦ a
⎣ 0 ⎦ ⎦

⎡ ⎤
⎢ ⎥ ⎡ ⎤
∈⎢ 1 1 ⎥⇒ ∈ ⎢ 1 + 1 ⎥
⇒ +
2⎢ 3
3 ⎛ 1+ i ⎞ ⎥
3
4 2⎢ 4
− i 3π i 3π

( ) ⎛ 1− i ⎞
( )
3
⎢ 2 ⎜ ⎟ 2 ⎜ ⎟ ⎥ ⎣ e e 4

⎢⎣ ⎝ 2⎠ ⎝ 2 ⎠ ⎥⎦

∈ ⎡ i 34π − i 3π
⎤ ∈ ⎡ ⎛ 3π ⎞ ⎤
⇒ ⎢ e + e 4
⎥⇒ ⎢ 2 cos ⎜ 4 ⎟ ⎥
4 2 ⎣ ⎦ 4 2 ⎣ ⎝ ⎠⎦

∈ ⎡ ⎛ 1 ⎞⎤ −∈ −∈
⇒ ⎢2 ⎜ − ⎟⎥ ⇒ ⇒ E11 =
4 2 ⎣ ⎝ 2 ⎠⎦ 4 4

                                                                                
Head office  Branch office 
 
fiziks, H.No. 40‐D, G.F, Jia Sarai,  Anand Institute of Mathematics, 
 
Near IIT, Hauz Khas, New Delhi‐16  28‐B/6, Jia Sarai, Near IIT 
 
Phone: 011‐26865455/+91‐9871145498 Hauz Khas, New Delhi‐16 
                                                   
                                             Website: www.physicsbyfiziks.com                                                                                          
                                                            Email: fiziks.physics@gmail.com                                                                     164 
fiziks
Institute for NET/JRF, GATE, IIT‐JAM, JEST, TIFR and GRE in PHYSICAL SCIENCES 
 
Q72. The product of the uncertainties ( ΔLx ) ( ΔLy ) for a particle in the state a 1,1 + b 1, −1

where l , m denotes an eigenstate of L2 and Lz will be a minimum for

(a) a = ±ib (b) a = 0 and b = 1

3 1
(c) a = and b = (d) a = ±b
2 2
Ans.: (d)
Solution: ψ = a 1,1 + b 1, −1

L+ ψ = 2 b 1, 0

L2+ ψ = 2 2b 1,1

L− ψ = 2 a 1, 0

L2− ψ = 2 2 a 1, −1

ψ L2 ψ = a 2
2 2
+b 2
2 2
( 2
= a + b2 2 ) 2

ψ L2z ψ = a + b 2 ( 2
) 2

Lx = 0, Ly = 0

L2x =
1 ⎡ 2
4 ⎣
L+ + L2
− + 2 L2
− L2
2
⎤ = 1 ⎡ a*b + b*a 2
⎦ 4 ⎣⎢
( ) ( ) 2
+2 2 ( 2
− 2
)( a 2
+b
2
)⎤⎦⎥
2
L2x =
2 ⎣
(
⎡ a*b + b*a + a 2 + b 2 ⎤
⎦)

L =
2 ( )
2 L2 − L22 − L2+ − L2−
y
4
2
L2y = ⎡ a 2 + b 2 − a*b + b*a ⎤
2 ⎣ ⎦ ( )

(a ))
2
ΔLx ΔLy =
2
2 2
+ b − a*b + b*a ( 2
∵ a + b =1
2

                                                                                
Head office  Branch office 
 
fiziks, H.No. 40‐D, G.F, Jia Sarai,  Anand Institute of Mathematics, 
 
Near IIT, Hauz Khas, New Delhi‐16  28‐B/6, Jia Sarai, Near IIT 
 
Phone: 011‐26865455/+91‐9871145498 Hauz Khas, New Delhi‐16 
                                                   
                                             Website: www.physicsbyfiziks.com                                                                                          
                                                            Email: fiziks.physics@gmail.com                                                                     165 
fiziks
Institute for NET/JRF, GATE, IIT‐JAM, JEST, TIFR and GRE in PHYSICAL SCIENCES 
 
2

( )
2
ΔLx ΔLy = 1 − a*b + b*a (i)
2
1 −i 2
Now check option (a) a = ±ib ⇒ a = ,b = ⇒ ΔLx ΔLy =
2 2 2
2
Option (b) a = 0, b = 1 ⇒ ΔLx ΔLy =
2
2
3 1
Option (c) a = ,b = ⇒ ΔLx ΔLy =
2 2 4
1 1
Option (d) a = ±b ⇒ a = ,b = ⇒ ΔLx ΔLy = 0 option (d) is correct
2 2
Q73. The ground state energy of a particle in potential V ( x ) = g x , estimated using the trail

wavefunction
⎧ c 2

ψ ( x ) = ⎨ a5
a − x2 , ( ) x< a
⎪0, x≥ a

(where g and c are constants) is
1/ 3 1/ 3 1/ 3 1/ 3
15 ⎛ 2 g 2 ⎞ 5 ⎛ 2g2 ⎞ 3 ⎛ 2g2 ⎞ 7 ⎛ 2g2 ⎞
(a) ⎜ ⎟ (b) ⎜ ⎟ (c) ⎜ ⎟ (d) ⎜ ⎟
16 ⎝ m ⎠ 6⎝ m ⎠ 4⎝ m ⎠ 8⎝ m ⎠

Ans.: (a)
a
15
∫ ψ ψ dx = 1 ⇒ c = 16
*
Solution:
−a

a
− 2 ⎛ 15 ⎞ ∂2 2 10 2
T =
2m ⎝ 16a 2 ⎠ −∫a
⎜ ⎟ a 2
− x 2

∂x 2
(
a − x 2
dx ⇒ T)=
4ma 2
( )
a
15 × 2 g 5
V =
16a 05 ∫ (
x a 2 − x 2 dx ⇒ V = ga
16
)
E= T + V (i)

10 2 5 ga
E= +
4ma 2 16
                                                                                
Head office  Branch office 
 
fiziks, H.No. 40‐D, G.F, Jia Sarai,  Anand Institute of Mathematics, 
 
Near IIT, Hauz Khas, New Delhi‐16  28‐B/6, Jia Sarai, Near IIT 
 
Phone: 011‐26865455/+91‐9871145498 Hauz Khas, New Delhi‐16 
                                                   
                                             Website: www.physicsbyfiziks.com                                                                                          
                                                            Email: fiziks.physics@gmail.com                                                                     166 
fiziks
Institute for NET/JRF, GATE, IIT‐JAM, JEST, TIFR and GRE in PHYSICAL SCIENCES 
 
1
dE 8 ⎛ 2 ⎞3
= 0 ⇒ a3 = ⇒ a = 2⎜ ⎟
da mg ⎝ mg ⎠
put the value of a in equation (i)
1
15 ⎛ 2 g 2 ⎞ 3
E= ⎜ ⎟
16 ⎝ m ⎠

NET/JRF (JUNE-2016)
Q74. The state of a particle of mass m in a one dimensional rigid box in the interval 0 to L is

2 ⎛ 3 ⎛ 2π x ⎞ 4 ⎛ 4π x ⎞ ⎞
given by the normalized wavefunction ψ ( x ) = ⎜ sin ⎜ ⎟ + sin ⎜ ⎟ ⎟ . If its
L ⎝ 5 ⎝ L ⎠ 5 ⎝ L ⎠⎠
energy is measured the possible outcomes and the average value of energy are,
respectively
h 2 2h 2 73 h 2 h2 h2 19 h 2
(a) , and (b) , and
2mL2 mL2 50 mL2 8mL2 2mL2 40 mL2
h 2 2h 2 19 h 2 h 2 2h 2 73 h 2
(c) , and (d) , and
2mL2 mL2 10 mL2 8mL2 mL2 200 mL2
Ans: (a)

2 ⎛ 3 ⎛ 2π x ⎞ 4 ⎛ 4π x ⎞ ⎞
Solution: ψ ( x ) = ⎜ sin ⎜ ⎟ + sin ⎜ ⎟⎟
L ⎝ 5 ⎝ L ⎠ 5 ⎝ L ⎠⎠

n 2π 2 2
Measurement E =
2mL2
h2 2h 2
∵ n = 2 ⇒ E2 = and n = 4 ⇒ E4 =
2mL2 mL2
9 16
probability p ( E2 ) = and p ( E4 ) =
25 25
now average value at energy
9 h2 16 2h 2 73h 2
E = ∑ an p ( an ) = × + × =
25 2mL2 25 mL2 50mL2

                                                                                
Head office  Branch office 
 
fiziks, H.No. 40‐D, G.F, Jia Sarai,  Anand Institute of Mathematics, 
 
Near IIT, Hauz Khas, New Delhi‐16  28‐B/6, Jia Sarai, Near IIT 
 
Phone: 011‐26865455/+91‐9871145498 Hauz Khas, New Delhi‐16 
                                                   
                                             Website: www.physicsbyfiziks.com                                                                                          
                                                            Email: fiziks.physics@gmail.com                                                                     167 
fiziks
Institute for NET/JRF, GATE, IIT‐JAM, JEST, TIFR and GRE in PHYSICAL SCIENCES 
 
Q75. If Lˆ x , Lˆ y Lˆ z are the components of the angular momentum operator in three dimensions

the commutator ⎡⎣ Lˆ x , Lˆ x Lˆ y Lˆ z ⎤⎦ may be simplified to

( )
(a) i Lx Lˆ2z − Lˆ2y (b) i Lˆ z Lˆ y Lˆ x

(c) i L ( 2 Lˆ − Lˆ )
x
2
z
2
y (d) 0

Ans: (a)
Solution: ⎡⎣ Lx , Lx Ly Lz ⎤⎦ = Lx ⎡⎣ Lx , Ly Lz ⎤⎦ + [ Lx Lx ] Ly Lz

⇒ Lx ⎡⎣ Lx , Ly ⎤⎦ Lz + Lx Ly [ Lx , Lz ] + 0 ⇒ Lx [i Lz ] Lz + Lx Ly ( −i Ly )

⇒ i Lx L2z − i Lx L2y ⇒ i Lx L2z − L2y ( )


Q76. Suppose that the Coulomb potential of the hydrogen atom is changed by adding an
Ze 2 g
inverse-square term such that the total potential is V ( r ) = − + 2 , where g is a
r r
constant. The energy eigenvalues Enlm in the modified potential

(a) depend on n and l , but not on m


(b) depend on n but not on l and m
(c) depend on n and m , but not on l
(d) depend explicitly on all three quantum numbers n , l and m
Ans: (b)
ze 2 g
Solution: V ( r ) = − + 2 is central potential
r r
So angular momentum is conserve then eigen value En ,l ,m will depend end only on n

which is principal quantum number.

                                                                                
Head office  Branch office 
 
fiziks, H.No. 40‐D, G.F, Jia Sarai,  Anand Institute of Mathematics, 
 
Near IIT, Hauz Khas, New Delhi‐16  28‐B/6, Jia Sarai, Near IIT 
 
Phone: 011‐26865455/+91‐9871145498 Hauz Khas, New Delhi‐16 
                                                   
                                             Website: www.physicsbyfiziks.com                                                                                          
                                                            Email: fiziks.physics@gmail.com                                                                     168 
fiziks
Institute for NET/JRF, GATE, IIT‐JAM, JEST, TIFR and GRE in PHYSICAL SCIENCES 
 
Q77. The eigenstates corresponding to eigenvalues E1 and E2 of a time independent

Hamiltonian are 1 and 2 respectively. If at t = 0 , the system is in a state

ψ ( t = 0 ) = sin θ 1 + cos θ 2 the value of ψ ( t ) ψ ( t ) at time t will be

(a) 1 (b)
( E sin 1
2
θ + E2 cos 2 θ )
E12 + E22

(c) eiE1t / sin θ + eiE2t / cos θ (d) e − iE1t / sin 2 θ + e− iE2t / cos 2 θ

Ans: (a)
Solution: ψ ( t = 0 ) = sin θ 1 + cos θ 2
− iE1t − iE2t
ψ ( t ) = sin θ 1 e + cos θ 2 e
− i ( E1 − E2 )t

ψ ( t ) ψ ( t ) = sin 2 θ 1 1 + cos 2 θ 2 2 + 2 Re e sin θ ⋅ cos θ 1 2

sin 2 θ + cos 2 θ ∵ 1 2 =0

=1 2 2 =1

1
Q78. Consider a particle of mass m in a potential V ( x ) = mω 2 x 2 + g cos kx . The change in
2
1
the ground state energy, compared to the simple harmonic potential mω 2 x 2 , to first
2
order in g is

⎛ k 2h ⎞ ⎛ k 2h ⎞
(a) g exp ⎜ − ⎟ (b) g exp ⎜ ⎟
⎝ 2mω ⎠ ⎝ 2mω ⎠

⎛ 2k 2 h ⎞ ⎛ k 2h ⎞
(c) g exp ⎜ − ⎟ (d) g exp ⎜ − ⎟
⎝ mω ⎠ ⎝ 4mω ⎠
Ans: (d)
                                                                                
Head office  Branch office 
 
fiziks, H.No. 40‐D, G.F, Jia Sarai,  Anand Institute of Mathematics, 
 
Near IIT, Hauz Khas, New Delhi‐16  28‐B/6, Jia Sarai, Near IIT 
 
Phone: 011‐26865455/+91‐9871145498 Hauz Khas, New Delhi‐16 
                                                   
                                             Website: www.physicsbyfiziks.com                                                                                          
                                                            Email: fiziks.physics@gmail.com                                                                     169 
fiziks
Institute for NET/JRF, GATE, IIT‐JAM, JEST, TIFR and GRE in PHYSICAL SCIENCES 
 
Solution: Ground state wavefunction
1
mω x 2
⎛ mω ⎞ 4 −
ψ 0 ( x) = ⎜ ⎟ e
2

⎝π ⎠
The perturbation term is H p = g cos kx

First order correction E01 = ∫ ψ 0* ( x ) H Pψ 0 ( x ) dx
−∞

1 1
∞ ∞ mω x 2 ∞ mω x 2
⎛ mω ⎞ 2 − ⎛ eikx + e − ikx ⎞ ⎛ mω ⎞ 2 − ⎛ eikx + eikx ⎞
= ∫⎜ ⎟ ∫e ⎜ ⎟ dx ⇒ g ⎜ ⎟ ∫ e ⎜ ⎟ dx
−∞ ⎝
π ⎠ −∞ ⎝ 2 ⎠ ⎝π ⎠ ∞ ⎝ 2 ⎠
1
g ⎛ mω ⎞ 2 ⎡ − mω x ikx ⎤
2 2
∞ ∞ − mω x

⎟ ∫ ∫
− ikx
⇒ ⎜ ⎢ e .e dx + e . e dx ⎥
2 ⎝ π ⎠ ⎣⎢ −∞ −∞ ⎦⎥
1 2
1 2
mω x ∞ − mω x ∞
g ⎛ mω ⎞ 2 − + ikx g ⎛ mω ⎞ 2 −ikx
⇒ ⎜ ⎟ ∫e dx + ⎜ ⎟ ∫e dx
2 ⎝ π ⎠ −∞ 2 ⎝ π ⎠ −∞
mω ⎡ 2 2 ikx ⎛ ik ⎞ ⎛ ik ⎞ ⎤
1 2 2

⎛ mω ⎞ 2
− ⎢x + +⎜ ⎟ −⎜ ⎟ ⎥
ω ω ω
⎟ ∫
⎣⎢ 2 m ⎝ 2 m ⎠ ⎝ 2 m ⎠ ⎥⎦
=⎜ e dx
⎝ π ⎠ −∞
1 2
∞ mω ⎛ ik ⎞ k2
⎛ mω ⎞ 2 − ⎜ x+ ⎟ −
⎟ ∫e
⎝ 2 mω ⎠
⇒ ⎜ e 4 mω
dx
⎝ π ⎠ −∞
1 2
k2 ∞ mω ⎛ ik ⎞ k2
− ⎛ mω ⎞ 2 ⎜ x+ ⎟ −
⎟ ∫e
⎝ 2 mω ⎠
⇒ e 4 mω
⎜ dx = e 4 mω

⎝ π ⎠ −∞
1 2
mω x ∞
⎛ mω ⎞ 2 − −ikx
Similarly (ii) is ⎜ ⎟ ∫ e
⎝ π ⎠ −∞
1 2
k 2h ∞ mω ⎛ ik ⎞ k2
⎛ mω ⎞ 2 − ⎜ x− ⎟ −
⎟ ∫e
⎝ 2 mω ⎠
⇒ e 4 mω
⎜ dx = e 4 mω

⎝ π ⎠ −∞

g ⎡ −k ⎤
2 2 2
k k
− −
E = ⎢ e 4 mω + e 4 mω ⎥ = ge 4 mω
1
0
2 ⎢⎣ ⎥⎦

                                                                                
Head office  Branch office 
 
fiziks, H.No. 40‐D, G.F, Jia Sarai,  Anand Institute of Mathematics, 
 
Near IIT, Hauz Khas, New Delhi‐16  28‐B/6, Jia Sarai, Near IIT 
 
Phone: 011‐26865455/+91‐9871145498 Hauz Khas, New Delhi‐16 
                                                   
                                             Website: www.physicsbyfiziks.com                                                                                          
                                                            Email: fiziks.physics@gmail.com                                                                     170 
fiziks
Institute for NET/JRF, GATE, IIT‐JAM, JEST, TIFR and GRE in PHYSICAL SCIENCES 
 
Q79. The energy levels for a particle of mass m in the potential V ( x ) = α x , determined in

the WKB approximation


b
⎛ 1⎞
2m ∫ E − V ( x )dx = ⎜ n + ⎟ π
a ⎝ 2⎠

(where a, b are the turning points and n = 0,1, 2... ), are


2 2
⎡ hπα ⎛ 1 ⎞⎤ 3 ⎡ 3hπα ⎛ 1 ⎞⎤ 3
(a) En = ⎢ ⎜ n + ⎟ (b) En = ⎢ ⎜ n + ⎟
⎣4 m ⎝ 2 ⎠ ⎥⎦ ⎣ 4 2m ⎝ 2 ⎠ ⎥⎦
2 2
⎡ 3hπα ⎛ 1 ⎞⎤ 3 ⎡ hπα ⎛ 1 ⎞⎤ 3
(c) En = ⎢ ⎜ n + ⎟⎥ (d) En = ⎢ ⎜ n + ⎟⎥
⎣4 m ⎝ 2 ⎠⎦ ⎣ 4 2m ⎝ 2 ⎠⎦
Ans: (b)
Solution: V ( x ) = α x
V ( x)
⇒ V ( x ) = −α x x<0

=αx x>0
E E
b −
⎛ 1⎞ α α
2m ∫ E − V ( x )dx = ⎜ n + ⎟ π
a ⎝ 2⎠ x

⎛ E⎞ ⎛E⎞
from figure a = ⎜ − ⎟ , b = ⎜ ⎟
⎝ α⎠ ⎝α ⎠
E
α
⎛ 1⎞
2m ∫ E − V ( x )dx = ⎜ n + ⎟ π
E ⎝ 2⎠

α

E
0 α
⎛ 1⎞
2m ∫ E
E + α xdx + ∫ E − α xdx = ⎜ n + ⎟ π
0 ⎝ 2⎠

α

E
α
⎛ 1⎞
2 2m ∫ E − α x ( dx ) = ⎜ n + ⎟ π
0 ⎝ 2⎠

                                                                                
Head office  Branch office 
 
fiziks, H.No. 40‐D, G.F, Jia Sarai,  Anand Institute of Mathematics, 
 
Near IIT, Hauz Khas, New Delhi‐16  28‐B/6, Jia Sarai, Near IIT 
 
Phone: 011‐26865455/+91‐9871145498 Hauz Khas, New Delhi‐16 
                                                   
                                             Website: www.physicsbyfiziks.com                                                                                          
                                                            Email: fiziks.physics@gmail.com                                                                     171 
fiziks
Institute for NET/JRF, GATE, IIT‐JAM, JEST, TIFR and GRE in PHYSICAL SCIENCES 
 
dt
put E − α x = t dx = −
α
E
limit x → 0 ⇒ t → E x→ ⇒t →0
α
0
⎛ − dt ⎞ ⎛ 1⎞
2 2m ∫ t ⎜ ⎟ = ⎜ n + ⎟π
E ⎝ α ⎠ ⎝ 2⎠
0
2 2m ⎡ 2 32 ⎤ ⎛ 1⎞ 2 2m 2 32 ⎛ 1⎞
− ⎢ t ⎥ = ⎜ n + ⎟ π ⇒ .E = ⎜ n + ⎟ π kh
α ⎣ 3 ⎦E ⎝ 2⎠ α 3 ⎝ 2⎠
2
⎛ 1 ⎞ π α3
3
⎡ 3 πα ⎛ 1 ⎞⎤ 3
⇒ E = ⎜n+ ⎟2
⇒ En = ⎢ ⎜ n + ⎟⎥
⎝ 2 ⎠ 4 2m ⎣ 4 2m ⎝ 2 ⎠⎦
Q80. A particle of mass m moves in one dimension under the influence of the potential
V ( x ) = −αδ ( x ) , where α is a positive constant. The uncertainty in the product

( Δx )( Δp ) in its ground state is

(a) 2 (b) (c) (d) 2


2 2
Ans: (c)
Solution: V ( x ) = −α ( x )

For this potential wavefunction


ψ ( x)
ψ ( x ) = α eα x x<0

= α e −α x x>0
which evenfunction about x = 0
so x = 0, p = 0 x→

1
now x 2 = 2α ∫ x 2 e −2α x dx =
0
2α 2

2 1
Δx = x2 − x =

                                                                                
Head office  Branch office 
 
fiziks, H.No. 40‐D, G.F, Jia Sarai,  Anand Institute of Mathematics, 
 
Near IIT, Hauz Khas, New Delhi‐16  28‐B/6, Jia Sarai, Near IIT 
 
Phone: 011‐26865455/+91‐9871145498 Hauz Khas, New Delhi‐16 
                                                   
                                             Website: www.physicsbyfiziks.com                                                                                          
                                                            Email: fiziks.physics@gmail.com                                                                     172 
fiziks
Institute for NET/JRF, GATE, IIT‐JAM, JEST, TIFR and GRE in PHYSICAL SCIENCES 
 
∞ 2 0 2 ∞
d d d
p 2 = − 2 ∫ ψ * 2 ψ dx = − 2 ∫ α eα x 2 α eα x dx − 2 ∫ α e −α x 2 α e −α x dx
−∞
dx −∞
dx 0
dx

0
α3 α
2 3 2

∫e ∫e ⇒ − 2α 2 which is not possible


2α x −2α x
=− α 2 3
dx − α 2 2
dx = − −
−∞ 0
2α 2α



2
so we will use the formula p = 2
dx
−∞
dx

which is equivalent to α2
2

2
Δp = p2 − p = α

1
now Δx.Δp = . α=
2α 2
2
β
Q81. The ground state energy of a particle of mass m in the potential V ( x ) = x4 ,
6m
1 2
−α x
⎛ α ⎞4
estimated using the normalized trial wavefunction ψ ( x ) = ⎜ ⎟ e 2 , is
⎝π ⎠

α ∞ 1 α ∞ 3
∫ ∫
2 2
[use dxx 2 e −α x and dx x 4 e −α x = ]
π −∞ 2α π −∞ 4α 2
1 1 1 1
3 8 2 3
(a) 2
β3 (b) 2
β3 (c) 2
β3 (d) 2
β3
2m 3m 3m 8m
Ans: (d)
Solution: E = T + V
1
α x2
⎛ α ⎞4 −
For ψ ( x ) = ⎜ ⎟ e 2

⎝π ⎠
2
α
T =
4m
1 1
∞ ∞
⎛ α ⎞2 β 4 −α x 2 ⎛ α ⎞2 β β β
2 2 2 2
3
V =⎜ ⎟ ∫ x e dx = ⎜ ⎟ ∫xe
4 −α x 2
dx = . =
⎝ π ⎠ −∞ 6m ⎝ π ⎠ 6m 6m 4α 8mα 2
2
−∞

                                                                                
Head office  Branch office 
 
fiziks, H.No. 40‐D, G.F, Jia Sarai,  Anand Institute of Mathematics, 
 
Near IIT, Hauz Khas, New Delhi‐16  28‐B/6, Jia Sarai, Near IIT 
 
Phone: 011‐26865455/+91‐9871145498 Hauz Khas, New Delhi‐16 
                                                   
                                             Website: www.physicsbyfiziks.com                                                                                          
                                                            Email: fiziks.physics@gmail.com                                                                     173 
fiziks
Institute for NET/JRF, GATE, IIT‐JAM, JEST, TIFR and GRE in PHYSICAL SCIENCES 
 
2
α β 2
E = + (1)
4m 8mα 2
dE 2
2 2β 2
⎛ β ⎞ 1
= − = 0 = ⎜ 1 − ⎟ = 0 ⇒ α = ( β ) 3
dα 4m 8mα 3 4m ⎝ α 3 ⎠
putting value of α in equation (1)
⎡ 1

2 1 2
β 2

1
( β ) 3

E = (β ) 3 + = (β ) +
3
4m 2
4m ⎢ 2 ⎥
θ m ( β )3 ⎢⎣ ⎥⎦
1
3
E = 2
β 3
8m

                                                                                
Head office  Branch office 
 
fiziks, H.No. 40‐D, G.F, Jia Sarai,  Anand Institute of Mathematics, 
 
Near IIT, Hauz Khas, New Delhi‐16  28‐B/6, Jia Sarai, Near IIT 
 
Phone: 011‐26865455/+91‐9871145498 Hauz Khas, New Delhi‐16 
                                                   
                                             Website: www.physicsbyfiziks.com                                                                                          
                                                            Email: fiziks.physics@gmail.com                                                                     174 
fiziks
Institute for NET/JRF, GATE, IIT‐JAM, JEST, TIFR and GRE in PHYSICAL SCIENCES 
 
THERMODYNAMICS AND STATISTICAL PHYSICS
NET/JRF (JUNE-2011)
Q1. Consider the transition of liquid water to steam as water boils at a temperature of 100oC
under a pressure of 1 atmosphere. Which one of the following quantities does not change
discontinuously at the transition?
(a) The Gibbs free energy (b) The internal energy
(c) The entropy (d) The specific volume
Ans: (a)
Solution: In first order transition Gibbs free energy is continuous.
Q2. A particle is confined to the region x ≥ 0 by a potential which increases linearly as
u (x ) = u 0 x . The mean position of the particle at temperature T is

k BT k BT
(b) (k B T ) / u 0
2
(a) (c) (d) u 0 k B T
u0 u0

Ans: (a)
p2 u x
1 − 2 mkBT − 0
Solution: Partition function Z = ∫ e dp ∫ e kBT dx and x = ∫ xp ( x )dxdpx
h
∞ μ0 x 2 ∞
p2 u0 x − ⎛ k BT ⎞
∫ xe ∫ te
k BT −t
− −
dx ⎜⎝ u ⎟⎠ dt
∫∫ xe
2 mk BT k BT
dp e dx k BT
⇒ x = = 0
μ x = 0 0

=
⎛ k BT ⎞
2


p

u0x − 0 u0
∫e ∫e
−t
∫∫ e dt
k BT
2 mk BT
dp e k BT
dx dx ⎜⎝ u ⎟⎠
0 0 0

Q3. A cavity contains blackbody radiation in equilibrium at temperature T. The specific heat
per unit volume of the photon gas in the cavity is of the form CV = γT 3 , where γ is a

constant. The cavity is expanded to twice its original volume and then allowed to
equilibrate at the same temperature T. The new internal energy per unit volume is
γT 4
(a) 4γT 4 (b) 2γT 4 (c) γT 4 (d)
4
Ans: (d)
γT 4
Solution: du = C v dT = ∫ γT dT ⇒ u = 3

4
                                                                                
Head office  Branch office 
 
fiziks, H.No. 23, G.F, Jia Sarai,  Anand Institute of Mathematics, 
 
Near IIT, Hauz Khas, New Delhi‐16  28‐B/6, Jia Sarai, Near IIT 
 
Phone: 011‐26865455/+91‐9871145498 Hauz Khas, New Delhi‐16 
                                                   
                                             Website: www.physicsbyfiziks.com                                                                                          
                                                            Email: fiziks.physics@gmail.com                                                                     175 
fiziks
Institute for NET/JRF, GATE, IIT‐JAM, JEST, TIFR and GRE in PHYSICAL SCIENCES 
 
Q4. Consider a system of N non-interacting spins, each of which has classical magnetic
moment of magnitude μ. The Hamiltonian of this system in an external magnetic field H
N
is ∑ μ i .H , where μ i the magnetic moment of the ith spin. The magnetization per spin at
i =1

temperature T is

μ 2H ⎡ ⎛ μH ⎞ k B T ⎤
(a) (b) μ ⎢coth ⎜⎜ ⎟⎟ − ⎥
k BT ⎣ ⎝ k B T ⎠ μH ⎦
⎛ μH ⎞ ⎛ μH ⎞
(c) μ sinh⎜⎜ ⎟⎟ (d) μ tanh⎜⎜ ⎟⎟
⎝ k BT ⎠ ⎝ k BT ⎠
Ans: (b)
2π π
μH cos θ
∫ ∫ μ cos θ exp kT
sin θ d θ d φ
Solution: For classical limit M = 0 0
βμH cos θ
∫∫ exp kBT sin θ d θd φ
⎡ ⎛ μH ⎞ k B T ⎤
M = μ ⎢coth ⎜ ⎟− ⎥
⎣ ⎝ k B T ⎠ μH ⎦
Q5. Consider an ideal Bose gas in three dimensions with the energy-momentum relation
ε ∝ p s with s > 0 . The range of s for which this system may undergo a Bose-Einstein
condensation at a non-zero temperature is
(a) 1 < s < 3 (b) 0 < s < 2 (c) 0 < s < 3 (d) 0 < s < ∞
Ans: (a)
NET/JRF (DEC-2011)
bS 3
Q6. The internal energy E of a system is given by E = , where b is a constant and other
VN
symbols have their usual meaning. The temperature of this system is equal to
2
bS 2 3bS 2 bS 3 ⎛S⎞
(a) (b) (c) 2 (d) ⎜ ⎟
VN VN V N ⎝N⎠
Ans: (b)
⎛ ∂E ⎞ 3bS 2
Solution: TdS = dE + PdV ⇒ dE = TdS − PdV ⇒ ⎜ ⎟ = T ⇒ T =
⎝ ∂S ⎠V VN
                                                                                
Head office  Branch office 
 
fiziks, H.No. 23, G.F, Jia Sarai,  Anand Institute of Mathematics, 
 
Near IIT, Hauz Khas, New Delhi‐16  28‐B/6, Jia Sarai, Near IIT 
 
Phone: 011‐26865455/+91‐9871145498 Hauz Khas, New Delhi‐16 
                                                   
                                             Website: www.physicsbyfiziks.com                                                                                          
                                                            Email: fiziks.physics@gmail.com                                                                     176 
fiziks
Institute for NET/JRF, GATE, IIT‐JAM, JEST, TIFR and GRE in PHYSICAL SCIENCES 
 
Q7. Consider a Maxwellian distribution of the velocity of the molecules of an ideal gas. Let
Vmp and Vrms denote the most probable velocity and the root mean square velocity,
respectively. The magnitude of the ratio Vmρ / Vrms is
(a) 1 (b) 2/3 (c) 2/3 (d) 3 / 2
Ans: (c)

2kT 3kT V 2
Solution: For Maxwellian distribution Vmp = , Vrms = ⇒ mb =
m m Vrms 3
Q8. If the number density of a free electron gas in three dimensions is increased eight times,
its Fermi temperature will
(a) increase by a factor of 4 (b) decrease by a factor of 4
(c) increase by a factor of 8 (d) decrease by a factor of 8
Ans: (a)
2
⎛ 3N ⎞ 3 2 N
Solution: Fermi energy E F = ⎜⎜ ⎟⎟ where is number density and g is degeneracy
⎝ 4πVg ⎠ 2m V
2 2

⎛ n ⎞3 2 TF ⎛ n ⎞ 3 n1
E F = TF K ⇒ TF ∝ ⎜ ⎟ ⇒ TF ∝ (n ) 3 ⇒ 1 = ⎜⎜ 1 ⎟⎟ = 4 since = 8.
⎝V ⎠ TF2 ⎝ n2 ⎠ n2
1
Q9. A system of N non-interacting spin - particles is placed in an external magnetic field H.
2
The behavior of the entropy of the system as a function of energy is given by
(a) S (b) S

E →
− μBH μBH − μB H E → μB H
S
(c) S (d)

− μB H E→ μBH − μB H E→

                                                                                
Head office  Branch office 
 
fiziks, H.No. 23, G.F, Jia Sarai,  Anand Institute of Mathematics, 
 
Near IIT, Hauz Khas, New Delhi‐16  28‐B/6, Jia Sarai, Near IIT 
 
Phone: 011‐26865455/+91‐9871145498 Hauz Khas, New Delhi‐16 
                                                   
                                             Website: www.physicsbyfiziks.com                                                                                          
                                                            Email: fiziks.physics@gmail.com                                                                     177 
fiziks
Institute for NET/JRF, GATE, IIT‐JAM, JEST, TIFR and GRE in PHYSICAL SCIENCES 
 
Ans: (a)
S − Nε + U ⎛ Nε + U ⎞ Nε − U ⎛ Nε − U ⎞
Solution: = ln ⎜ ⎟− ln ⎜ ⎟ where ε = μH . S is symmetrical
Nk 2Nε ⎝ 2ε ⎠ 2 Nε ⎝ 2Nε ⎠
about E.
Q10. A gas of N non-interacting particles is in thermal equilibrium at temperature T. Each
particle can be in any of the possible non-degenerate states of energy 0, 2ε and 4ε. The
average energy per particle of the gas, when βε << 1 , is
(a) 2ε (b) 3ε (c) 2ε / 3 (d) ε
Ans: (a)
Solution: E1 = 0, E 2 = 2ε , E 3 = 4ε , Z = e −0 β + e −2ε β + e −4ε β

0 ⋅ e − oβ + 2εe −2ε β + 4 ∈ e −4ε β


⇒ E =
e − 0 β + e − 2ε β + e − 4ε β

2εe −2ε β + 4εe −4ε β 2ε (1 − 2εβ....) + 4ε (1 − ( 4εβ ) ....) 2ε + 4ε 6ε


⇒ E = = = = = 2ε
1 + e − 2ε β + e − 4ε β 1 + (1 − 2εβ....) + (1 − 4εβ.....) 1+1+1 3

where βε << 1 .
Q11. A one-dimensional chain consists of a set of N rods each of length a. When stretched by a
load, each rod can align either parallel or perpendicular to the length of the chain. The
energy of a rod is –ε when perpendicular to it. When the chain is in thermal equilibrium
at temperature T, its average length is
(a) Na / 2 (b) Na
(
(c) Na / 1 + e −2ε / k BT ) (
(d) Na 1 + e −2ε / k BT )
Ans: (c)
Solution: Let n1 no. of rods are parallel and n2 no. of rods are perpendicular.
Energy of rod when it is perpendicular = −ε
Energy of rod when it is parallel is ε .
e − β ( −ε ) eβε e − βε
P(− ε ) = = and P (ε ) =
e − β (−ε ) + e − βε eβε + e−βε e βε + e − βε

                                                                                
Head office  Branch office 
 
fiziks, H.No. 23, G.F, Jia Sarai,  Anand Institute of Mathematics, 
 
Near IIT, Hauz Khas, New Delhi‐16  28‐B/6, Jia Sarai, Near IIT 
 
Phone: 011‐26865455/+91‐9871145498 Hauz Khas, New Delhi‐16 
                                                   
                                             Website: www.physicsbyfiziks.com                                                                                          
                                                            Email: fiziks.physics@gmail.com                                                                     178 
fiziks
Institute for NET/JRF, GATE, IIT‐JAM, JEST, TIFR and GRE in PHYSICAL SCIENCES 
 
n ae + n2 ae − βε
βε
Nae βε Na
Average n1 aP(− ε ) + n2 aP(ε ) = 1 βε − βε
= βε − βε
=
e +e e +e 1 + e −2βε
Since P(− ε ) >> P(ε ) so n1 ≅ N , n2 ≅ 0 .
Q12. The excitations of a three-dimensional solid are bosonic in nature with their frequency ω
and wave-number k are related by ω ∝ k 2 in the large wavelength limit. If the chemical
potential is zero, the behavior of the specific heat of the system at low temperature is
proportional to
(a) T1/2 (b) T (c) T3/2 (d) T3
Ans: (c)
Solution: If dispersion relation is ω ∝ k s ,
At low temperature specific heat ∝ T 3/ s
Q13. Gas molecules of mass m are confined in a cylinder of radius R and height L
(with R >> L ) kept vertically in the Earth’s gravitational field. The average energy of the
gas at low temperatures (such that mgL >> k BT ) is given by
(a) NkBT / 2 (b) 3NkBT / 2 (c) 2NkBT (d) 5NkBT / 2
Ans: (d)
1 −βH
h3 ∫
Solution: Z = e dpx dp y dpz dxdydz

∞ p x2 ∞ p 2y ∞ − p 2y L mgz
− −
Z= ∫e dpx ∫ e ∫e dpz ∫ dx dy ∫ e
2 mk BT 2 mk BT 2 mk BT k BT
dp y dz
−∞ −∞ −∞ 0

3⎜
⎛ mgL ⎞
3
mgz − ⎟
⎛ mk T ⎞ 2 L −
2 ⎛ mk B T ⎞ ⎜ 1 − e
2
k BT

2 ⎟ ∫
β
Z = πR ⎜
2
⎜ ⎟ e k BT
dz ⇒ Z = πR ⎜ ⎟
⎝ π ⎝ π ⎠ ⎜ mg ⎟
2
⎠ 0 ⎜ ⎟
⎝ k BT ⎠
ZN = ZN ,

∂ ln z 5 Nk BT
⇒ E = k BT 2 = since mgL > > kT .
∂T 2

                                                                                
Head office  Branch office 
 
fiziks, H.No. 23, G.F, Jia Sarai,  Anand Institute of Mathematics, 
 
Near IIT, Hauz Khas, New Delhi‐16  28‐B/6, Jia Sarai, Near IIT 
 
Phone: 011‐26865455/+91‐9871145498 Hauz Khas, New Delhi‐16 
                                                   
                                             Website: www.physicsbyfiziks.com                                                                                          
                                                            Email: fiziks.physics@gmail.com                                                                     179 
fiziks
Institute for NET/JRF, GATE, IIT‐JAM, JEST, TIFR and GRE in PHYSICAL SCIENCES 
 
NET/JRF (JUNE-2012)

Q14. Consider a system of non-interacting particles in d dimensional obeying the dispersion


relation ε = Ak s , where ε is the energy k is the wave vector; s is an integer and A
constant. The density of states, N(ε), is proportional to
s d d s
−1 −1 +1 +1
(a) ε d (b) ε s
(c) ε s
(d) ε d
Ans: (b)
Solution: We can solve this problem with intuition for example ε = Ak 2
1 3
−1
Density of state in 3dimensional N(ε) ∝ ε = ε
2 2

2
−1
Density of state in 2dimensional N(ε) ∝ ε = ε
0 2

−1 1
−1
Density of state in one dimensional N(ε) ∝ ε =ε
2 2

d
−1
Density of state in d dimenstional where ε = Ak ⇒ N ( ε ) ∝ ε
s s

Q15. The number of ways in which N identical bosons can be distributed in two energy levels,
is
N ( N − 1) N ( N + 1)
(a) N + 1 (b) (c) (d) N
2 2
Ans: (a)
Solution: Number of boson = N, Number of energy level = g
N + g −1
So number of ways to distribute N boson into g level is, W = cN = N + 1 since

g = 2.
Q16. The free energy of gas of N particles in a volume V and at a temperature T is
[
F = Nk B T ln a0V (k B T )
5/ 2
]
/ N , where a 0 is a constant and k B denotes the Boltzmann
constant. The internal energy of the gas is
3 5
(a) Nk B T (b) Nk B T
2 2

[
(c) Nk B T ln a 0V (k B T )
5/ 2
/N −] 3
2
Nk B T (d) Nk B T ln a0V / (k BT ) [ 5/ 2
]
                                                                                
Head office  Branch office 
 
fiziks, H.No. 23, G.F, Jia Sarai,  Anand Institute of Mathematics, 
 
Near IIT, Hauz Khas, New Delhi‐16  28‐B/6, Jia Sarai, Near IIT 
 
Phone: 011‐26865455/+91‐9871145498 Hauz Khas, New Delhi‐16 
                                                   
                                             Website: www.physicsbyfiziks.com                                                                                          
                                                            Email: fiziks.physics@gmail.com                                                                     180 
fiziks
Institute for NET/JRF, GATE, IIT‐JAM, JEST, TIFR and GRE in PHYSICAL SCIENCES 
 
Ans: (b)
Solution: F = Nk B T ln a0V (k B T ) [ 5/ 2
]
/ N , F = U − TS , U = F + TS

⎛ ∂F ⎞ ⎛ ∂F ⎞ ⎛ ∂F ⎞
dF = − SdT − pdV ⇒ ⎜ ⎟ = − S or S = −⎜ ⎟ ⇒ U = F −T⎜ ⎟
⎝ ∂T ⎠V ⎝ ∂T ⎠V ⎝ ∂T ⎠V

a 0Vk B5 / 2
F = Nk B T ln (C T 5/ 2
) where C =
N
⎛ ∂F ⎞ ⎛ ∂F ⎞
⎟ = Nk B ln (CT ) + Nk B T ⎟ = Nk B T ln (CT ) + Nk B T
C 5 3/ 2 5

5/ 2
T ⇒ T⎜ 5/ 2

⎝ ∂T ⎠V ⎝ ∂T ⎠V
5/ 2
CT 2 2

⎛ ∂F ⎞ 5 ⎛ ∂F ⎞ 5
T⎜ ⎟ = F + Nk B T ⇒ U = F − T ⎜ ⎟ = − Nk B T .
⎝ ∂T ⎠V 2 ⎝ ∂T ⎠V 2

Q17. A system has two normal modes of vibration, with frequencies ω1 and ω 2 = 2ω1 . What is

the probability that at temperature T, the system has an energy less than 4 ω1 ?

[In the following x = e − β ω1


and Z is the partition function of the system.]
(a) x 3 / 2 (x + 2 x 2 ) / Z (b) x 3 / 2 (1 + x + x 2 ) / Z

(c) x 3 / 2 (1 + 2 x 2 ) / Z (d) x 3 / 2 (1 + x + 2 x 2 ) / Z
Ans: (d)
Solution: There is two normal mode so there is two degree of freedom.
⎛ 1⎞ ⎛ 1⎞
Energy of harmonic oscillator is E = ⎜ n1 + ⎟ ω1 + ⎜ n2 + ⎟ ω 2 .
⎝ 2⎠ ⎝ 2⎠
⎛ 1⎞ ⎛ 1⎞
E = ⎜ n1 + ⎟ ω1 + ⎜ n2 + ⎟ 2ω1 where n1 = 0,1,2,3.... and n 2 = 0,1,2,3....
⎝ 2⎠ ⎝ 2⎠
3 ω1 5 ω1
Ground state energy E = , first excited state energy E = . Second excited state
2 2
7 ω1
energy E = which is doubly degenerate state so g = 2 , other state have more
2
energy than 4 ω1 .
3 β ω1 5 β ω1 7 β ω1

P(E < 4 ω1 ) =
e

2
+e

2
+ 2e

2
=
x
3
2
(1 + x + 2 x )
2
where x = e − β ω1
.
Z Z

                                                                                
Head office  Branch office 
 
fiziks, H.No. 23, G.F, Jia Sarai,  Anand Institute of Mathematics, 
 
Near IIT, Hauz Khas, New Delhi‐16  28‐B/6, Jia Sarai, Near IIT 
 
Phone: 011‐26865455/+91‐9871145498 Hauz Khas, New Delhi‐16 
                                                   
                                             Website: www.physicsbyfiziks.com                                                                                          
                                                            Email: fiziks.physics@gmail.com                                                                     181 
fiziks
Institute for NET/JRF, GATE, IIT‐JAM, JEST, TIFR and GRE in PHYSICAL SCIENCES 
 
4
Q18. Bose condensation occurs in liquid He kept at ambient pressure at 2.17 K. At which
temperature will Bose condensation occur in He4 in gaseous state, the density of which is
1000 times smaller than that of liquid He4? (Assume that it is a perfect Bose gas.)
(a) 2.17 mK (b) 21.7 mK (c) 21.7 μK (d) 2.17 μK
Ans: (b)
2
⎛ N ⎞3
Solution: For bosons T ∝ ⎜ ⎟
⎝V ⎠
Q19. Consider black body radiation contained in a cavity whose walls are at temperature T.
The radiation is in equilibrium with the walls of the cavity. If the temperature of the walls
is increased to 2T and the radiation is allowed to come to equilibrium at the new
temperature, the entropy of the radiation increases by a factor of
(a) 2 (b) 4 (c) 8 (d) 16
Ans: (c)
− 8π 5 k B4T 4 ⎛ ∂F ⎞ ⎛ 32π 5 k B4 ⎞ 3
Solution: For Black Body Energy is given by F = V , S = − ⎜ ⎟ = ⎜ ⎟VT .
⎝ ∂T ⎠V ⎜⎝ 45 C ⎟⎠
3 3
45 2C 3

⇒ S ∝ T 3 , If temperate increase from T to 2T then entropy will incase S to 8S.

NET/JRF (DEC-2012)

Q20. The entropy of a system, S, is related to the accessible phase space volume Γ by
S = k B ln Γ(E , N , V ) where E, N and V are the energy, number of particles and volume
respectively. From this one can conclude that Γ
(a) does not change during evolution to equilibrium
(b) oscillates during evolution to equilibrium
(c) is a maximum at equilibrium
(d) is a minimum at equilibrium
Ans: (c)
Solution: Entropy is maximum at equilibrium.

                                                                                
Head office  Branch office 
 
fiziks, H.No. 23, G.F, Jia Sarai,  Anand Institute of Mathematics, 
 
Near IIT, Hauz Khas, New Delhi‐16  28‐B/6, Jia Sarai, Near IIT 
 
Phone: 011‐26865455/+91‐9871145498 Hauz Khas, New Delhi‐16 
                                                   
                                             Website: www.physicsbyfiziks.com                                                                                          
                                                            Email: fiziks.physics@gmail.com                                                                     182 
fiziks
Institute for NET/JRF, GATE, IIT‐JAM, JEST, TIFR and GRE in PHYSICAL SCIENCES 
 
Q21. Let ΔW be the work done in a quasistatic reversible thermodynamic process. Which of
the following statements about ΔW is correct?
(a) ΔW is a perfect differential if the process is isothermal
(b) ΔW is a perfect differential if the process is adiabatic
(c) ΔW is always a perfect differential
(d) ΔW cannot be a perfect differential
Ans: (b)
Solution: Work done is perfect differential in adiabatic process.
Q22. The free energy difference between the superconducting and the normal states of a
2 β 4
material is given by ΔF = f S − f N = α ψ + ψ where ψ is an order parameter and
2
α and β are constants s.t. α > 0 in Normal and α < 0 in the super conducting state, while
β > 0 always, minimum value of ΔF is
α2 α2 3α 2 5α 2
(a) − (b) − (c) − (d) −
β 2β 2β 2β
Ans: (b)
2 β 4 ΔF 4β 3
Solution: ΔF = α ψ + ψ ⇒ = 2α ψ + ψ
2 ∂ψ 2
3 2 α
2α ψ + 2β ψ = 0 ⇒ ψ =−
β
α2 β α2 α2
Putting the value, ΔF = − + × ⇒ ΔF =−
β 2 β2 min

Q23. A given quantity of gas is taken from the state A → C reversibly, by two paths, A → C
directly and A → B → C as shown in the figure.
P
During the process A → C the work done by the gas is 100 J and the A

heat absorbed is 150 J. If during the process A → B → C the work


done by the gas is 30 J, the heat absorbed is
B C
(a) 20 J (b) 80 J
V
(c) 220 J (d) 280 J

                                                                                
Head office  Branch office 
 
fiziks, H.No. 23, G.F, Jia Sarai,  Anand Institute of Mathematics, 
 
Near IIT, Hauz Khas, New Delhi‐16  28‐B/6, Jia Sarai, Near IIT 
 
Phone: 011‐26865455/+91‐9871145498 Hauz Khas, New Delhi‐16 
                                                   
                                             Website: www.physicsbyfiziks.com                                                                                          
                                                            Email: fiziks.physics@gmail.com                                                                     183 
fiziks
Institute for NET/JRF, GATE, IIT‐JAM, JEST, TIFR and GRE in PHYSICAL SCIENCES 
 
Ans: (b)
Solution: During path AC dU = dQ − dW = 150 − 100 = 50 J
Hence internal energy is point function dU will same in all path
In path ABC dQ = dU + dW = 50 + 30 = 80 J .
NET/JRF (JUNE-2013)

Q24. Ten grams of ice at 0°C is added to a beaker containing 30 grams of water at 25°C. What
is the final temperature of the system when it comes to thermal equilibrium? (The
specific heat of water is 1 cal/gm/°C and latent heat of melting of ice is 80 cal/gm)
(a) 0°C (b) 7.5°C (c) 12.5°C (d) -1.25°C
Ans: (a)
Solution: The amount of heat required to melt the ice of mass 10gm at 00C is
Q = m × L = 10 × 80 = 800Cal . Where L is the latent heat of melting of ice and m is the
mass of the ice. The amount of heat available in water of mass 30gm at 250C is
Q = m × Cv × T = 30 ×1× 25 = 750Cal
Since the heat available is less than the heat required to melt the ice therefore ice will not
melt as a result the temperature of the system will be at 00C only.
Q25. A vessel has two compartments of volume V1 and V2 , containing an ideal gas at pressures
p1 and p 2 , and temperatures T1 and T2 respectively. If the wall separating the
compartments is removed, the resulting equilibrium temperature will be
p1T1 + p 2T2 V1T1 + V2T2
(a) (b)
p1 + p 2 V1 + V2

p1V1 + p 2V2
(d) (T1T2 )
1/ 2
(c)
( p1V1 / T1 ) + ( p 2V2 / T2 )
Ans: (c)

                                                                                
Head office  Branch office 
 
fiziks, H.No. 23, G.F, Jia Sarai,  Anand Institute of Mathematics, 
 
Near IIT, Hauz Khas, New Delhi‐16  28‐B/6, Jia Sarai, Near IIT 
 
Phone: 011‐26865455/+91‐9871145498 Hauz Khas, New Delhi‐16 
                                                   
                                             Website: www.physicsbyfiziks.com                                                                                          
                                                            Email: fiziks.physics@gmail.com                                                                     184 
fiziks
Institute for NET/JRF, GATE, IIT‐JAM, JEST, TIFR and GRE in PHYSICAL SCIENCES 
 
pV p V
Solution: V = V1 + V2 , n = n1 + n2 = 1 1 + 2 2 , U1 + U 2 = U , n1CvT1 + n2CvT2 = nCvT ,
T1 T2

p1V1 + p2V2
n1T1 + n2T2 = nT ⇒ T =
p1V1 p2V2
+
T1 T2

Q26. For temperature T1 > T2 , the qualitative temperature dependence of the probability
distribution F (v ) of the speed v of a molecule in three dimensions is correctly
represented by the following figure:

T2
T1
F(v)

F(v)
T1 T2
(a) (b)

v v

T1
T2 T1
F(v)

F(v)

T2
(c) (d)

v v
Ans: (a)
Solution: Area under the F ( v ) is conserve and the mean velocity shift towards right for higher
temperature.
Q27. A system of non-interacting spin- 1/ 2 charged particles are placed in an external magnetic
field. At low temperature T , the leading behavior of the excess energy above the ground
state energy, depends on T as: ( c is a constant)
(a) cT (b) cT 3 (c) e − c / T (d) c (is independent of T )
Ans: (c)
⎛ μkTB H μ H
− B ⎞
μB H ⎜e − e kT ⎟
Solution: U = − μ B H tanh = −μ B H ⎜ μB H μBH ⎟
kT ⎜ e kT + e − kT ⎟
⎝ ⎠
                                                                                
Head office  Branch office 
 
fiziks, H.No. 23, G.F, Jia Sarai,  Anand Institute of Mathematics, 
 
Near IIT, Hauz Khas, New Delhi‐16  28‐B/6, Jia Sarai, Near IIT 
 
Phone: 011‐26865455/+91‐9871145498 Hauz Khas, New Delhi‐16 
                                                   
                                             Website: www.physicsbyfiziks.com                                                                                          
                                                            Email: fiziks.physics@gmail.com                                                                     185 
fiziks
Institute for NET/JRF, GATE, IIT‐JAM, JEST, TIFR and GRE in PHYSICAL SCIENCES 
 
Excess energy from the ground level

⎛ μkTB H μ H
− B ⎞ ⎡ ⎛ μB H μ H
− B ⎞⎤ ⎛ μ H
− B ⎞
e − e ⎢ e − e ⎥ 2 e
= − μ B H ⎜ μB H ⎟ − (− μ H ) = μ H 1 − ⎜ ⎟ =μ H⎜ ⎟
kT kT kT kT

⎜ μB H ⎟ B B ⎢ ⎜ μB H μB H ⎟⎥ B ⎜ μB H μB H ⎟
⎜ e kT + e− kT ⎟ ⎢ ⎜ e kT + e − kT ⎟⎥ ⎜ e kT + e− kT ⎟
⎝ ⎠ ⎣ ⎝ ⎠⎦ ⎝ ⎠
−C
For lower value ΔU ∝ e T
where C = μ B H .

Q28. Consider a system of two Ising spins S1 and S 2 taking values ± 1 with interaction energy
given by ε = − JS1 S 2 , when it is in thermal equilibrium at temperature T . For large T , the
average energy of the system varies as C / k B T , with C given by

(a) − 2J 2 (b) − J 2 (c) J 2 (d) 4 J


Ans: (b)
Solution: The interaction energy is given by E = − J S1 S 2 where S1 and S 2 taking values ± 1 .
Possible values of the Energy of the system are
E1 = − J 1⋅1 = − J , E 2 = − J (− 1)⋅ (1) = + J

E3 = − J (1)⋅ (− 1) = + J , E 4 = − J (− 1)⋅ (− 1) = − J

Er
⎛ J ⎛ J ⎞⎞
∑ Er g r e kT
−2 Je
J
+ 2 Je

J
⎛ kTJ
e − e

J
kT
⎞ ⎜ 1 + kT − ⎜1 −
⎝ kT ⎠ ⎠
⎟⎟
= −J ⎜ J ⎟ = −J ⎝
kT kT
U = r
= ⎜ J ⎟
E
− r
J

J
⎜ e kT + e − kT ⎟ ⎛ J ⎞ ⎛ J ⎞
∑g e
r
r
kT
2e kT
+ 2e kT
⎝ ⎠ 1+ ⎜
⎝ kT
⎟ +1− ⎜


⎝ kT ⎠
J2 J
⇒ U =− ⇒ C = − J 2 (For large T , << 1 )
kT kT
Q29. Consider two different systems each with three identical non-interacting particles. Both
have single particle states with energies ε 0 ,3ε 0 and 5ε 0 , (ε 0 > 0) . One system is populated

1
by spin − fermions and the other by bosons. What is the value of E F − E B where E F
2
and EB are the ground state energies of the fermionic and bosonic systems respectively?

(a) 6ε 0 (b) 2ε 0 (c) 4ε 0 (d) ε 0


Ans: (b)
Solution: Energy of Fermion = 2 ×1ε 0 + 3ε 0 = 5ε 0

                                                                                
Head office  Branch office 
 
fiziks, H.No. 23, G.F, Jia Sarai,  Anand Institute of Mathematics, 
 
Near IIT, Hauz Khas, New Delhi‐16  28‐B/6, Jia Sarai, Near IIT 
 
Phone: 011‐26865455/+91‐9871145498 Hauz Khas, New Delhi‐16 
                                                   
                                             Website: www.physicsbyfiziks.com                                                                                          
                                                            Email: fiziks.physics@gmail.com                                                                     186 
fiziks
Institute for NET/JRF, GATE, IIT‐JAM, JEST, TIFR and GRE in PHYSICAL SCIENCES 
 
Energy of boson = 3 ×1ε 0 = 3ε 0

E F − E B = 5ε 0 − 3ε 0 = 2ε 0

NET/JRF (DEC-2013)
1
Q30. Three identical spin- fermions are to be distributed in two non-degenerate distinct
2
energy levels. The number of ways this can be done is
(a) 8 (b) 4 (c) 3 (d) 2
Ans: (b)
Solution: Total number of degeneracy
g = (No. of energy state (n)) × (No. of degeneracy due to spin ( 2 s + 1 ))
1 1
n = 2, s = , g = 2 × (2. + 1) = 4
2 2
No. of particle N = 3 so no. of ways g
cN = 4 c3 = 4
Q31. Consider the melting transition of ice into water at constant pressure. Which of the
following thermodynamic quantities does not exhibit a discontinuous change across the
phase transition?
(a) Internal energy (b) Helmholtz free energy (c) Gibbs free energy (d) entropy
Ans: (c)
Solution: Ice to water: 1st order place transition
So Gibbs free energy is continuous so it doesn’t exhibit discontinuous change.
Q32. Two different thermodynamic systems are described by the following equations of state:
1 3RN (1) 1 5 RN (2 )
(1) = (1) and (2 ) = ( 2 ) where T
(1, 2 )
, N (1, 2 ) and U (1, 2 ) are respectively, the
T 2U T 2U
temperatures, the mole numbers and the internal energies of the two systems, and R is
the gas constant. Let U tot denote the total energy when these two systems are put in

U (1)
contact and attain thermal equilibrium. The ratio is
U tot

5 N (2 ) 3N (1) N (1) N (2 )
(a) (b) (c) (1) (d) (1)
3N (1) + 5 N (2 ) 3N (1) + 5 N (2 ) N + N (2 ) N + N (2 )
                                                                                
Head office  Branch office 
 
fiziks, H.No. 23, G.F, Jia Sarai,  Anand Institute of Mathematics, 
 
Near IIT, Hauz Khas, New Delhi‐16  28‐B/6, Jia Sarai, Near IIT 
 
Phone: 011‐26865455/+91‐9871145498 Hauz Khas, New Delhi‐16 
                                                   
                                             Website: www.physicsbyfiziks.com                                                                                          
                                                            Email: fiziks.physics@gmail.com                                                                     187 
fiziks
Institute for NET/JRF, GATE, IIT‐JAM, JEST, TIFR and GRE in PHYSICAL SCIENCES 
 
Ans: (b)
1 3RN (1) 1 5RN (2 )
Solution: = and =
T (1) 2U (1) T (2 ) 2U (2 )
3 5
Now utot = U (1) + U ( 2) = RN (1)T (1) + RN ( 2)T ( 2)
2 2
1⎡
3RN (1)T (1) ⎤⎦
U (1) 2 ⎣ 3 N (1)T (1)
⇒ = =
U tot 1 ⎡3N (1)T (1) + 5 RN ( 2)T ( 2) ⎤ 3N (1)T (1) + 5 N ( 2)T ( 2)
2⎣ ⎦

(1) ( 2) 3N (1)
At thermal equilibrium T = T , thus
3N (1) + 5 N (2 )
Q33. The speed v of the molecules of mass m of an ideal gas obeys Maxwell’s velocity
distribution law at an equilibrium temperature T . Let (v x , v y v z ) denote the components

( )
2
of the velocity and k B the Boltzmann constant. The average value of α vx − βv y , where

α and β are constants, is


(a) (α 2 − β 2 )k B T / m (b) (α 2 + β 2 )k B T / m

(c) (α + β ) k B T / m (d) (α − β ) k B T / m
2 2

Ans: (c)
Solution: Ideal gas obeys Maxwell velocity distribution law at equilibrium temperature. Then

( )
2
average value of α vx − β v y

(α v )
2
Now x − βvy = α 2 v x2 + β 2 v y2 − 2 αβ v x v y

k BT
v x = 0, v y = 0 and vx2 = = v y2 = vz2
m

(α v )
2
Then x − βv y = α 2 v x2 + β 2 v y2 − 2αβ v x v y

(α v x − βv y )
2
= α2
k BT
m
k T
+ β2 B = α 2 + β 2
m m
(
kT
)

                                                                                
Head office  Branch office 
 
fiziks, H.No. 23, G.F, Jia Sarai,  Anand Institute of Mathematics, 
 
Near IIT, Hauz Khas, New Delhi‐16  28‐B/6, Jia Sarai, Near IIT 
 
Phone: 011‐26865455/+91‐9871145498 Hauz Khas, New Delhi‐16 
                                                   
                                             Website: www.physicsbyfiziks.com                                                                                          
                                                            Email: fiziks.physics@gmail.com                                                                     188 
fiziks
Institute for NET/JRF, GATE, IIT‐JAM, JEST, TIFR and GRE in PHYSICAL SCIENCES 
 
Q34. The entropy S of a thermodynamic system as a function of energy E is given by the
following graph S

C
B
A

E
The temperatures of the phases A, B and C , denoted by T A , TB and TC , respectively,
satisfy the following inequalities:
(a) TC > TB > T A (b) T A > TC > TB (c) TB > TC > T A (d) TB > T A > TC
Ans: (c)
Solution: Now temperature of phase TA , TB , TC C
S
⎛ dS ⎞ 1 B
Now ⎜ ⎟=
⎝ dE ⎠ T A
dS
Now will be slope then it will be zero for B - phase
dE →E
So TB = ∞ and in C and A phases internal energy of C phase is more so TC > T A

Now TB > TC > T A

Q35. A system of N classical non-interacting particles, each of mass m , is at a temperature T


1 2
and is confined by the external potential V (r ) = Ar (where A is a constant) in three
2
dimensions. The internal energy of the system is
3 A ⎛ k BT ⎞
(c) N (2mA)
3/ 2
(a) 3 Nk B T (b) Nk B T k BT (d) N ln⎜ ⎟
2 m ⎝ m ⎠
Ans: (a)

                                                                                
Head office  Branch office 
 
fiziks, H.No. 23, G.F, Jia Sarai,  Anand Institute of Mathematics, 
 
Near IIT, Hauz Khas, New Delhi‐16  28‐B/6, Jia Sarai, Near IIT 
 
Phone: 011‐26865455/+91‐9871145498 Hauz Khas, New Delhi‐16 
                                                   
                                             Website: www.physicsbyfiziks.com                                                                                          
                                                            Email: fiziks.physics@gmail.com                                                                     189 
fiziks
Institute for NET/JRF, GATE, IIT‐JAM, JEST, TIFR and GRE in PHYSICAL SCIENCES 
 
1
2
1
( )
Solution: V (r ) = Ar 2 = A x 2 + y 2 + z 2 it is harmonic oscillator so it partition function will
2
3N
1 ⎛ kT ⎞
be z N = ⎜ ⎟
N⎝ ω⎠
∂ ln Z N
Internal energy U = kT 2 then internal energy will be 3 NkT
∂T
Q36. A Carnot cycle operates as a heat engine between two bodies of equal heat capacity until
their temperatures become equal. If the initial temperatures of the bodies are T1 and T2 ,
respectively and T1 > T2 then their common final temperature is
1
(a) T12 / T2 (b) T22 / T1 (c) T1T2 (d) (T1 + T2 )
2
Ans: (c)
Solution: For heat Carnot engine the change in entropy for source and sink
TF dT ⎛T ⎞ TF dT ⎛T ⎞
dS1 = ∫ = log ⎜ F ⎟ and dS 2 = ∫ = log ⎜ F ⎟
T1 T ⎝ T1 ⎠ T1 T
⎝ T2 ⎠
TF T
ΔS = dS1 + dS 2 = log + log F .
T1 T2

(T )
2

Hence carnot engine is reversible in nature log F = 0 ⇒ TF = T1T2


T1T2

                                                                                
Head office  Branch office 
 
fiziks, H.No. 23, G.F, Jia Sarai,  Anand Institute of Mathematics, 
 
Near IIT, Hauz Khas, New Delhi‐16  28‐B/6, Jia Sarai, Near IIT 
 
Phone: 011‐26865455/+91‐9871145498 Hauz Khas, New Delhi‐16 
                                                   
                                             Website: www.physicsbyfiziks.com                                                                                          
                                                            Email: fiziks.physics@gmail.com                                                                     190 
fiziks
Institute for NET/JRF, GATE, IIT‐JAM, JEST, TIFR and GRE in PHYSICAL SCIENCES 
 
NET/JRF (JUNE-2014)

Q37. Which of the graphs below gives the correct qualitative behaviour of the energy density
E r (λ ) of blackbody radiation of wavelength λ at two temperatures T1 and T2 (T1 < T2 ) ?
(a) (b)
T2
T2
Er (λ )
Er (λ )
T1

λ λ
(c) (d)
T2
T2
Er (λ ) Er (λ )
T1
T1

Ans: (c) λ λ
Q38. A system can have three energy levels: E = 0, ± ε . The level E = 0 is doubly
degenerate, while the others are non-degenerate. The average energy at inverse
temperature β is

ε (e βε − e − βε )
(a) − ε tanh (βε ) (b)
(1 + e βε
+ e − βε )

⎛ βε ⎞
(c) zero (d) − ε tanh ⎜ ⎟
⎝ 2 ⎠
Ans: (d)
Solution: E = 0, ± ε , E = 0 doubly degenerate

z = ∑ gi e − β Ei ⇒ 2 × e − β ×0 + e − βε + e βε

z = 2 + e βε + e− βε ⇒ ln z = ln 2 + e βε + e− βε( )

                                                                                
Head office  Branch office 
 
fiziks, H.No. 23, G.F, Jia Sarai,  Anand Institute of Mathematics, 
 
Near IIT, Hauz Khas, New Delhi‐16  28‐B/6, Jia Sarai, Near IIT 
 
Phone: 011‐26865455/+91‐9871145498 Hauz Khas, New Delhi‐16 
                                                   
                                             Website: www.physicsbyfiziks.com                                                                                          
                                                            Email: fiziks.physics@gmail.com                                                                     191 
fiziks
Institute for NET/JRF, GATE, IIT‐JAM, JEST, TIFR and GRE in PHYSICAL SCIENCES 
 
∂ ∂ ⎡ 1 ⎤
Now E = − ln ( z ) = − ln ( 2 + e βε + e − βε ) = − ⎢ × ( ε e βε − ε e − βε ) ⎥
∂β ∂β βε
⎣2+ e + e
− βε

⎡ ⎤ ⎡ ⎛ βε − βε
⎞⎤
⎢ βε ⎥ ⎢⎜ e 2
− e 2
⎟⎥
⎢ e − e − βε ⎥ ⎛ βε ⎞
E = −ε ⎢ = −ε ⎢ ⎝ βε ⎠⎥ ⇒ E = −ε tanh ⎜ ⎟
βε βε 2 ⎥ ⎢⎛ − βε
⎞⎥ ⎝ 2 ⎠
⎢ ⎛ e 2 + e− 2 ⎞ ⎥ ⎢⎜ e + e 2
2
⎟⎥
⎢⎜ ⎟ ⎥
⎢⎣ ⎝ ⎠ ⎥⎦
⎣⎝ ⎠ ⎦
Q39. The free energy F of a system depends on a thermodynamic variable ψ as

F = −aψ 2 + bψ 6
with a, b > 0 . The value of ψ , when the system is in thermodynamic equilibrium, is

(b) ± (a / 6b ) (c) ± (a / 3b ) (d) ± (a / b )


1/ 4 1/ 4 1/ 4
(a) zero
Ans: (c)
Solution: Frequency F = −aψ 2 + bψ 6 a, b > 0

∂2 F
F is equilibrium i.e. >0
∂ψ 2
∂F
Now = −2aψ + 6bψ 5
∂ψ
1/ 4
∂F a ⎛ a ⎞
= 0 ⇒ 2aψ = 6bψ 5 ⇒ =ψ 4 ⇒ψ = ±⎜ ⎟
∂ψ 3b ⎝ 3b ⎠

Q40. For a particular thermodynamic system the entropy S is related to the internal energy U
and volume V by
S = cU 3 / 4V 1 / 4
where c is a constant. The Gibbs potential G = U − TS + pV for this system is
3 pU cU US
(a) (b) (c) zero (d)
4T 3 4V
Ans: (c)

                                                                                
Head office  Branch office 
 
fiziks, H.No. 23, G.F, Jia Sarai,  Anand Institute of Mathematics, 
 
Near IIT, Hauz Khas, New Delhi‐16  28‐B/6, Jia Sarai, Near IIT 
 
Phone: 011‐26865455/+91‐9871145498 Hauz Khas, New Delhi‐16 
                                                   
                                             Website: www.physicsbyfiziks.com                                                                                          
                                                            Email: fiziks.physics@gmail.com                                                                     192 
fiziks
Institute for NET/JRF, GATE, IIT‐JAM, JEST, TIFR and GRE in PHYSICAL SCIENCES 
 
Solution: S = cU V , dU = TdS − pdV
3/4 1/4

⎛ ∂S ⎞ 1 ⎛ ∂S ⎞ 1 c × 3 −1/ 4 1/ 4 4 U 1/ 4
⎜ ⎟ = ⇒ ⎜ ⎟ = = U V ⇒T =
⎝ ∂U ⎠V T ⎝ ∂U ⎠V T 4 3c V 1/ 4
1

⎛ ∂U ⎞ S V −5 / 4U 4 S V −5/ 4 1/ 4
⎜ ⎟ = − P = − ⇒ P = U
⎝ ∂V ⎠ S c 3 c 3

4 U 1/ 4 S V −5/ 4 1/4 4 1
G =U − 1/ 4
× cU 3/ 4 1/ 4
V + U ×V ⇒ G = U − U + U = 0
3c V c 3 3 3
Q41. The pressure of a nonrelativistic free Fermi gas in three-dimensions depends, at T = 0 ,
on the density of fermions n as
(a) n 5 / 3 (b) n1 / 3 (c) n 2 / 3 (d) n 4 / 3
Ans: (a)
2
Pressure P = nEF
3
EF ∝ n 2 / 3 at T = 0

2 2
P= n × n 2 / 3 ⇒ P = n 5 / 3 ⇒ P ∝ n5 / 3
3 3
Q42. The van der Waals’ equation of state for a gas is given by
⎛ a ⎞
⎜ P + 2 ⎟(V − b ) = RT
⎝ V ⎠
where P, V and T represent the pressure, volume and temperature respectively, and a
and b are constant parameters. At the critical point, where all the roots of the above
cubic equation are degenerate, the volume is given by
a a 8a
(a) (b) (c) (d) 3b
9b 27b 2 27bR
Ans: (d)
⎛ a ⎞
Solution: ⎜ P + 2 ⎟ (V − b ) = RT
⎝ V ⎠

                                                                                
Head office  Branch office 
 
fiziks, H.No. 23, G.F, Jia Sarai,  Anand Institute of Mathematics, 
 
Near IIT, Hauz Khas, New Delhi‐16  28‐B/6, Jia Sarai, Near IIT 
 
Phone: 011‐26865455/+91‐9871145498 Hauz Khas, New Delhi‐16 
                                                   
                                             Website: www.physicsbyfiziks.com                                                                                          
                                                            Email: fiziks.physics@gmail.com                                                                     193 
fiziks
Institute for NET/JRF, GATE, IIT‐JAM, JEST, TIFR and GRE in PHYSICAL SCIENCES 
 
⎛ ∂P ⎞ ⎛∂ P⎞
2
For critical volume ⎜ ⎟ = 0, ⎜ 2 ⎟ = 0
⎝ ∂V ⎠ ⎝ ∂V ⎠
a ab
PV + − pb − 2 = RT
V V
∂P a 2ab ∂2 P 2a 6ab 2a 6ab
= 0 ⇒ P − 2 + 3 = 0, = 0 ⇒ 3 − 4 = 0 ⇒ 3 = 4 ⇒ Vc = 3b
∂V V V ∂V 2
V V V V

NET/JRF (DEC-2014)
Q43. The pressure P of a fluid is related to its number density ρ by the equation of state

P = aρ + bρ 2
where a and b are constants. If the initial volume of the fluid is V0 , the work done on
the system when it is compressed so as to increase the number density from an initial
value of ρ 0 to 2 ρ 0 is

(a) aρ 0V0 (b) (a + bρ 0 )ρ 0V0

⎛ 3a 7 ρ b ⎞
(c) ⎜ + 0 ⎟ ρ 0V0 (d) (a ln 2 + bρ 0 )ρ 0V0
⎝ 2 3 ⎠
Ans: (d)
n n2 n
Solution: P = aρ + bρ 2 ⇒ P = a +b 2 ∵ρ =
V V V
V2 dV V2 dV n n
W = ∫ P ⋅ dV = an∫ + bn 2 ∫ where V1 = , V2 =
V1 V V1 V 2 ρ0 2 ρ0

⇒ W = − n ( a ln 2 + b ρ0 ) = − ρ0V0 ( a ln 2 + bρ0 ) ∵ n = ρ0V0

Work done on the system = −W = ( a ln 2 + bρ0 ) ρ0V0

                                                                                
Head office  Branch office 
 
fiziks, H.No. 23, G.F, Jia Sarai,  Anand Institute of Mathematics, 
 
Near IIT, Hauz Khas, New Delhi‐16  28‐B/6, Jia Sarai, Near IIT 
 
Phone: 011‐26865455/+91‐9871145498 Hauz Khas, New Delhi‐16 
                                                   
                                             Website: www.physicsbyfiziks.com                                                                                          
                                                            Email: fiziks.physics@gmail.com                                                                     194 
fiziks
Institute for NET/JRF, GATE, IIT‐JAM, JEST, TIFR and GRE in PHYSICAL SCIENCES 
 
Q44. An ideal Bose gas is confined inside a container that is connected to a particle reservoir.
Each particle can occupy a discrete set of single-particle quantum states. If the probability
that a particular quantum state is unoccupied is 0.1 , then the average number of bosons
in that state is
(a) 8 (b) 9 (c) 10 (d) 11
Ans: (b)
Q45. In low density oxygen gas at low temperature, only the translational and rotational modes
of the molecules are excited. The specific heat per molecule of the gas is
1 3 5
(a) kB (b) k B (c) kB (d) kB
2 2 2
Ans: (d)
Solution: Total D.O.F. = 3 transition + 2 rotation i.e. f = 5
k B T 5k B T ∂U 5
U= f⋅ = ⇒ CV = = kB
2 2 ∂T 2
Q46. When a gas expands adiabatically from volume V1 to V2 by a quasi-static reversible
process, it cools from temperature T1 to T2 . If now the same process is carried out
adiabatically and irreversibly, and T2′ is the temperature of the gas when it has
equilibrated, then
(a) T2′ = T2 (b) T2′ > T2

⎛V −V ⎞ T2V1
(c) T2′ = T2 ⎜⎜ 2 1 ⎟⎟ (d) T2′ =
⎝ V2 ⎠ V2

Ans: (b)
Q47. A random walker takes a step of unit length in the positive direction with probability 2 / 3
and a step of unit length in the negative direction with probability 1 / 3 . The mean
displacement of the walker after n steps is
(a) n / 3 (b) n / 8 (c) 2n / 3 (d) 0
Ans: (a)

                                                                                
Head office  Branch office 
 
fiziks, H.No. 23, G.F, Jia Sarai,  Anand Institute of Mathematics, 
 
Near IIT, Hauz Khas, New Delhi‐16  28‐B/6, Jia Sarai, Near IIT 
 
Phone: 011‐26865455/+91‐9871145498 Hauz Khas, New Delhi‐16 
                                                   
                                             Website: www.physicsbyfiziks.com                                                                                          
                                                            Email: fiziks.physics@gmail.com                                                                     195 
fiziks
Institute for NET/JRF, GATE, IIT‐JAM, JEST, TIFR and GRE in PHYSICAL SCIENCES 
 
2 1
Solution: P ( +1) = ⇒ P ( −1) =
3 3
2 1 1 n
For one step = +1 × − = , for n step =
3 3 3 3
Q48. A collection N of non-interacting spins S i , i = 1, 2, ....., N , (S i = ±1) is kept in an
external magnetic field B at a temperature T . The Hamiltonian of the system is
μB
H = − μBΣ i S i . What should be the minimum value of for which the mean value
k BT
1
Si ≥ ?
3
1 1
(a) N ln 2 (b) 2 ln 2 (c) ln 2 (d) N ln 2
2 2
Ans: (c)
μB μB

e kT
e kT
Solution: P ( Si = +1) = μB μB
, P ( Si = −1) = μB μB
− −
e kT
+e kT e kT
+ e kT
μB μB
− +
+1e kT
−e kT
⎛ μB⎞
Si = ⇒ Si = − ⎜ tanh ⎟

μB μB
⎝ kT ⎠
e kT
+ e kT
μB
For N particle Si = − N tanh
kT
According to question
Si 1 ⎛ μB ⎞ 1 μB 1
≥ ⇒ − tanh ⎜ ⎟= ⇒ = ln 2
N 3 ⎝ kT ⎠ 3 kT 2
NET/JRF (JUNE-2015)
Q49. A system of N non-interacting classical particles, each of mass m is in a two
dimensional harmonic potential of the form V ( r ) = α ( x 2 + y 2 ) where α is a positive

⎛ 1 ⎞
constant. The canonical partition function of the system at temperature T is ⎜ β = ⎟:
⎝ k BT ⎠

                                                                                
Head office  Branch office 
 
fiziks, H.No. 23, G.F, Jia Sarai,  Anand Institute of Mathematics, 
 
Near IIT, Hauz Khas, New Delhi‐16  28‐B/6, Jia Sarai, Near IIT 
 
Phone: 011‐26865455/+91‐9871145498 Hauz Khas, New Delhi‐16 
                                                   
                                             Website: www.physicsbyfiziks.com                                                                                          
                                                            Email: fiziks.physics@gmail.com                                                                     196 
fiziks
Institute for NET/JRF, GATE, IIT‐JAM, JEST, TIFR and GRE in PHYSICAL SCIENCES 
 
N
⎡⎛ α ⎞ 2 π ⎤ ⎛ 2mπ ⎞
2N

(a) ⎢⎜ ⎟ ⎥ (b) ⎜ ⎟
⎢⎣⎝ 2m ⎠ β ⎥⎦ ⎝ αβ ⎠
N N
⎛ απ ⎞ ⎛ 2mπ 2 ⎞
(c) ⎜ ⎟ (d) ⎜ 2 ⎟
⎝ 2m β ⎠ ⎝ αβ ⎠
Ans. (d)
Solution: V ( r ) = α ( x 2 + y 2 )

− px2 − p 2y −α x 2 −α y 2
1 +∞ +∞ +∞ +∞
z1 = 2
h ∫ −∞
e 2 mkT
dpx ∫ e
−∞
2 mkT
dp y ∫ e
−∞
kT
dx ∫ e
−∞
kT
dy

2π mkT 2π mkT 1 1 1 π
⇒ z1 = . .2 x × π .2 ×
h 2
h 2
2 α 2 α
kT kT
N
⎛ 2π 2 m ⎞ ⎛ 2π 2 m ⎞
z1 = ⎜ 2 ⎟ ( kT ) ⇒ z N = ⎜ 2
2

⎝ hα ⎠ ⎝ h αβ ⎠
Q50. A system of N distinguishable particles, each of which can be in one of the two energy
levels 0 and ∈ , has a total energy n ∈ , where n is an integer. The entropy of the system
is proportional to
(a) N ln n (b) n ln N

⎛ N !⎞ ⎛ N! ⎞
(c) ln ⎜ ⎟ (d) ln ⎜
⎝ n! ⎠ ⎜ n !( N − n ) ! ⎟⎟
⎝ ⎠
Ans. (d)
Solution: No of ways for above configuration is = N Cn

N
⇒ W =
n N −n
N
⇒ Entropy=k ln
n N −n

                                                                                
Head office  Branch office 
 
fiziks, H.No. 23, G.F, Jia Sarai,  Anand Institute of Mathematics, 
 
Near IIT, Hauz Khas, New Delhi‐16  28‐B/6, Jia Sarai, Near IIT 
 
Phone: 011‐26865455/+91‐9871145498 Hauz Khas, New Delhi‐16 
                                                   
                                             Website: www.physicsbyfiziks.com                                                                                          
                                                            Email: fiziks.physics@gmail.com                                                                     197 
fiziks
Institute for NET/JRF, GATE, IIT‐JAM, JEST, TIFR and GRE in PHYSICAL SCIENCES 
 
Q51. The condition for the liquid and vapour phases of a fluid to be in equilibrium is given by
dP Q
the approximate equation ≈ 1 (Clausius-Clayperon equation) where vvap is the
dT Tvvap

volume per particle in the vapour phase, and Q1 is the latent heat, which may be taken to
be a constant. If the vapour obeys ideal gas law, which of the following plots is correct?

(a) ln P (b) ln P

O T O T

(c) ln P (d) ln P
O T O T

Ans. (c)
dP Q
Solution: = l
dT Tvap

RT dP Ql P dP Ql dT
vap =
P
⇒ =
dT RT 2

P
=
R ∫T 2

C
ln P = − +α
T
Q52. Consider three Ising spins at the vertices of a triangle which interact with each other with
a ferromagnetic Ising interaction of strength J . The partition function of the system at
⎛ 1 ⎞
temperature T is given by ⎜ β = ⎟:
⎝ k BT ⎠
(a) 2e3β J + 6e − β J (b) 2e −3β J + 6e β J

(d) ( 2 cosh β J )
3
(c) 2e3β J + 6e −3β J + 3e β J + 3e − β J

                                                                                
Head office  Branch office 
 
fiziks, H.No. 23, G.F, Jia Sarai,  Anand Institute of Mathematics, 
 
Near IIT, Hauz Khas, New Delhi‐16  28‐B/6, Jia Sarai, Near IIT 
 
Phone: 011‐26865455/+91‐9871145498 Hauz Khas, New Delhi‐16 
                                                   
                                             Website: www.physicsbyfiziks.com                                                                                          
                                                            Email: fiziks.physics@gmail.com                                                                     198 
fiziks
Institute for NET/JRF, GATE, IIT‐JAM, JEST, TIFR and GRE in PHYSICAL SCIENCES 
 
Ans. (b)
Solution: H = J ( S1S 2 + S1S3 + S 2 S3 )

S1 S2 S3 E
1 1 1 3J

1 −1⎫ 1

1 −1 1 ⎬ − J
−1 1 1 ⎪⎭

−1 −11⎫

−1 1 −1⎬ − J
1 −1 −1⎪⎭

−1 −1 −1} 3J

⇒ z = 2e−3β J + 6e β J
Q53. A large number N of Brownian particles in one dimension start their diffusive motion
from the origin at time t = 0 . The diffusion coefficient is D . The number of particles
crossing a point at a distance L from the origin, per unit time, depends on L and time t
as
− L2 −4 Dt
N NL
(a) e ( 4 Dt ) (b) e L2
4π Dt 4π Dt
− L2 −4 Dt
N ( 4 Dt ) L2
(c) e (d) Ne
16π Dt 3

Ans. (a)
Solution: From Einstein Smoluchowski theory

dx ⎛ − x2 ⎞
p ( x ) dx = exp ⎜ ⎟
4π Dt ⎝ 4 Dt ⎠

N ⎛ − L2 ⎞
No of particle passing from point L at origin = .exp ⎜ ⎟
4π Dt ⎝ 4 Dt ⎠

                                                                                
Head office  Branch office 
 
fiziks, H.No. 23, G.F, Jia Sarai,  Anand Institute of Mathematics, 
 
Near IIT, Hauz Khas, New Delhi‐16  28‐B/6, Jia Sarai, Near IIT 
 
Phone: 011‐26865455/+91‐9871145498 Hauz Khas, New Delhi‐16 
                                                   
                                             Website: www.physicsbyfiziks.com                                                                                          
                                                            Email: fiziks.physics@gmail.com                                                                     199 
fiziks
Institute for NET/JRF, GATE, IIT‐JAM, JEST, TIFR and GRE in PHYSICAL SCIENCES 
 
Q54. An ideal Bose gas in d -dimensions obeys the dispersion relation ∈ k = Ak s , where A ( )
and s are constants. For Bose-Einstein condensation to occur, the occupancy of excited
states
(d −s)

∈ s
Ne = c ∫ β (∈− μ ) −1
d∈
0 e

where c is a constant, should remain finite even for μ = 0 . This can happen if
d 1 1 d 1
(a) < (b) < <
s 4 4 s 2
d 1 d
(c) >1 (d) < <1
s 2 s
Ans. (c)
(d −s)
∞ ∈ s
Solution: Ne = c ∫ d∈
0
e β (∈− μ ) − 1
B.E. condensation is possible in 3-D
1
d −S 1 d 3
For materlistic particle g (∈) ∝∈2 ⇒ = ⇒ =
S 2 S 2
d −S d
For massless particle g (∈) ∝∈2 ⇒ =2⇒ =3
S S
d
In both case >1
S

                                                                                
Head office  Branch office 
 
fiziks, H.No. 23, G.F, Jia Sarai,  Anand Institute of Mathematics, 
 
Near IIT, Hauz Khas, New Delhi‐16  28‐B/6, Jia Sarai, Near IIT 
 
Phone: 011‐26865455/+91‐9871145498 Hauz Khas, New Delhi‐16 
                                                   
                                             Website: www.physicsbyfiziks.com                                                                                          
                                                            Email: fiziks.physics@gmail.com                                                                     200 
fiziks
Institute for NET/JRF, GATE, IIT‐JAM, JEST, TIFR and GRE in PHYSICAL SCIENCES 
 
NET/JRF (DEC-2015)
Q55. The heat capacity of (the interior of a refrigerator is 4.2 kJ / K . The minimum’ work that

must be done to lower the internal temperature from 18o C to 17o C when the outside

temperature is 27o C will be


(a) 2.20 kJ (b) 0.80 kJ (c) 0.30 kJ (d) 0.14 kJ
Ans.: (b)
Q56. For a system of independent non interacting one-dimensional oscillators, the value of the
free energy per oscillator, in the limit T → 0 , is
1 3
(a) ω (b) ω (c) ω (d) 0
2 2
Ans.: (a)
−N
⎡ ω⎤ ⎡ ⎛ ω ⎞⎤
Solution: For the given system Z N = ⎢ 2sinh ⇒ F = −kT ln Z N = NkT ln ⎢ 2sinh ⎜ ⎟⎥
⎣ 2kT ⎥⎦ ⎣ ⎝ 2kT ⎠ ⎦

⎡ ⎛ ω −
ω
⎞⎤
⎢ 2⎜e − e
2 kT 2 kT
⎟⎥ ω ω ω ω
⎛ ⎞ ⎛ ⎞
= NkT ln ⎢ ⎝ ⎠ ⎥ = NkT ln e 2 kT − −
⎜ 1 − e kT
⎟ = NkT ln e 2 kT
+ NkT ln ⎜ 1 − e kT

⎢ 2 ⎥
⎢ ⎥ ⎝ ⎠ ⎝ ⎠
⎢⎣ ⎥⎦
ω
F ω ⎛ − ⎞ F ω
= + kT ln ⎜1 − e kT ⎟ ⇒ = +0 ∵ ( kT → 0 )
N 2 ⎝ ⎠ N 2

F ω
=
N 2
Q57. The partition function of a system of N Ising spins is Z = λ1N + λ2N where λ1 and λ2 are

functions of temperature, but are independent of N . If λ1 > λ2 , the free energy per spin

in the limit N → ∞ is
⎛λ ⎞
(a) − k BT ln ⎜ 1 ⎟ (b) − k BT ln λ2 (c) − k BT ln ( λ1λ2 ) (d) − k BT ln λ1
⎝ λ2 ⎠

                                                                                
Head office  Branch office 
 
fiziks, H.No. 23, G.F, Jia Sarai,  Anand Institute of Mathematics, 
 
Near IIT, Hauz Khas, New Delhi‐16  28‐B/6, Jia Sarai, Near IIT 
 
Phone: 011‐26865455/+91‐9871145498 Hauz Khas, New Delhi‐16 
                                                   
                                             Website: www.physicsbyfiziks.com                                                                                          
                                                            Email: fiziks.physics@gmail.com                                                                     201 
fiziks
Institute for NET/JRF, GATE, IIT‐JAM, JEST, TIFR and GRE in PHYSICAL SCIENCES 
 
Ans.: (d)
Solution: Z = λ1N + λ2N

( )
⇒ F = −kT ln λ1N + λ2N it is given λ1 >> λ2

⎡ ⎛ ⎛ λ ⎞ N ⎞⎤ λ2
⇒ F = −kT ln ⎢λ1N ⎜ 1 + ⎜ 2 ⎟ ⎟ ⎥ ≈0
⎢ ⎜⎝ ⎝ λ1 ⎠ ⎟⎠ ⎥ λ1
⎣ ⎦
F
F = − kT ln λ1N ⇒ F = − NkT ln λ1 ⇒ = − kT ln λ1
N
1
Q58. The Hamiltonian of a system of N non interacting spin - particles is H = − μ0 B ∑i Siz ,
2
where Siz = ±1 are components of i th spin along an external magnetic field B . At a
μ0 B

temperature T such that e kBT = 2 . the specific heat per particle is


16 8 16
(c) k B ( ln 2 ) k B ( ln 2 )
2 2
(a) kB (b) k B ln 2 (d)
25 25 25
Ans.: (d)
μ0 B
Solution: For the given system E = − μ0 B tanh
kT
2
⎛μ B⎞ μB
CV = ⎜ 0 ⎟ Nk sec 2 h 0
⎝ kT ⎠ kT
2 μ0 B
CV ⎛ μ0 B ⎞ 4 μ0 B
=⎜ ⎟ k 2
∵e kT = 2 ⇒ = ln 2
N ⎝ kT ⎠ ⎛ 0 μ B μ B
− 0 ⎞ kT
⎜⎜ e + e kT
kT
⎟⎟
⎝ ⎠
2 2
CV ⎛ μ0 B ⎞ 4 ⎛ μ B ⎞ 16 2 16k
=⎜ ⎟ k 2
=⎜ 0 ⎟ k = ( ln 2 )
N ⎝ kT ⎠ ⎛ 1⎞ ⎝ kT ⎠ 25 25
⎜2 + ⎟
⎝ 2⎠
CV 16
k ( ln 2 )
2
=
N 25

                                                                                
Head office  Branch office 
 
fiziks, H.No. 23, G.F, Jia Sarai,  Anand Institute of Mathematics, 
 
Near IIT, Hauz Khas, New Delhi‐16  28‐B/6, Jia Sarai, Near IIT 
 
Phone: 011‐26865455/+91‐9871145498 Hauz Khas, New Delhi‐16 
                                                   
                                             Website: www.physicsbyfiziks.com                                                                                          
                                                            Email: fiziks.physics@gmail.com                                                                     202 
fiziks
Institute for NET/JRF, GATE, IIT‐JAM, JEST, TIFR and GRE in PHYSICAL SCIENCES 
 
1
Q59. An ensemble of non-interacting spin - particles is in contact with a heat bath at
2
temperature T and is subjected to an external magnetic field. Each particle can be in one
of the two quantum states of energies ± ∈0 . If the mean energy per particle is − ∈0 / 2 ,
then the free energy per particle is

(a) −2 ∈0
(
ln 4 / 3 ) (b) − ∈0 ln ( 3 / 2 ) (c) −2 ∈0 ln 2 (d) − ∈0
ln 2
ln 3 ln 3
Ans.: (a)
Solution: For the given system portion function
∈0
Z n = 2 N cosh
kT
∈0 ∈
mean energy per unit partical is − = − ∈0 tanh 0
2 kT
∈0 1 eα − e −α 1
put =α ⇒ tanh α = ⇒ α =
kT 2 e + e −α 2
1 ∈ 1 2 ∈0
⇒ e 2α = 3 ⇒ α = ln 3 ⇒ 0 = ln 3 ⇒ kT =
2 kT 2 ln 3
F ⎛ ∈ ⎞
it is given = −kT ln ⎜ 2 cosh 0 ⎟
N ⎝ kT ⎠

F ⎛ eα + e −α ⎞
⇒ = − kT ln ( 2 cosh α ) ⎜ 8 ⎟
N ⎝ 8 ⎠

⎛ eα + e −α ⎞
= − kT ln ⎜ 2
2
⎟ = − kT ln ⎡⎣e e + 1 ⎤⎦
−α 2α
( )
⎝ ⎠

= − kT ln ⎡⎣e −α ( 3 + 1) ⎤⎦ = −kT ln ⎡⎣e −α 4 ⎤⎦ = − kT ⎡⎣ln e −α + ln 4 ⎤⎦

= kT α − kT ln 4
F ∈ 2∈
= kT 0 − kT ln 4 ⇒ ∈0 − 30 ln 4
N kT ln

                                                                                
Head office  Branch office 
 
fiziks, H.No. 23, G.F, Jia Sarai,  Anand Institute of Mathematics, 
 
Near IIT, Hauz Khas, New Delhi‐16  28‐B/6, Jia Sarai, Near IIT 
 
Phone: 011‐26865455/+91‐9871145498 Hauz Khas, New Delhi‐16 
                                                   
                                             Website: www.physicsbyfiziks.com                                                                                          
                                                            Email: fiziks.physics@gmail.com                                                                     203 
fiziks
Institute for NET/JRF, GATE, IIT‐JAM, JEST, TIFR and GRE in PHYSICAL SCIENCES 
 
⎡ ⎛ 3 ⎞⎤ ⎡ ⎛ 16 ⎞ ⎤
⎢ ln ⎜ ⎟ ⎥ ⎢ ln ⎜ 3 ⎟ ⎥
⎡ ln 3 − 2 ln 4 ⎤ ⎝ 16 ⎠
⇒ ∈0 ⎢ ⎥ ⇒ ∈0 ⎢ ⎥ ⇒ − ∈0 ⎢ ⎝ ⎠ ⎥
⎣ ln 3 ⎦ ⎢ ln 3 ⎥ ⎢ ln 3 ⎥
⎢⎣ ⎥⎦ ⎢⎣ ⎥⎦

⎡ ⎛ 4 ⎞2 ⎤ ⎛ 4 ⎞
⎢ ln ⎜ ⎟ ⎥ ln ⎜ ⎟
⇒ − ∈0 ⎢ ⎝
3⎠ ⎥
⇒ −2 ∈0 ⎝
3⎠
⎢ ln 3 ⎥ ln 3
⎢ ⎥
⎢⎣ ⎥⎦

⎛ 4 ⎞
ln ⎜ ⎟
F
= −2 ∈0 ⎝
3⎠

N ln 3
Q60. Which of the following graphs shows the qualitative dependence of the free energy
f ( h, T ) of a ferromagnet in an external magnetic field h , and at a fixed temperature

T < TC , where TC is the critical temperature?


f
(a) f (b)
h

h
f f
(c) (d)

h h
Ans.: (c)
Solution: For super conductor state one will find two local minima
f

h Option (c) is correct.


                                                                                
Head office  Branch office 
 
fiziks, H.No. 23, G.F, Jia Sarai,  Anand Institute of Mathematics, 
 
Near IIT, Hauz Khas, New Delhi‐16  28‐B/6, Jia Sarai, Near IIT 
 
Phone: 011‐26865455/+91‐9871145498 Hauz Khas, New Delhi‐16 
                                                   
                                             Website: www.physicsbyfiziks.com                                                                                          
                                                            Email: fiziks.physics@gmail.com                                                                     204 
fiziks
Institute for NET/JRF, GATE, IIT‐JAM, JEST, TIFR and GRE in PHYSICAL SCIENCES 
 
NET/JRF (JUNE-2016)
Q61. The specific heat per molecule of a gas of diatomic molecules at high temperatures is
(a) 8k B (b) 3.5k B (c) 4.5 k B (d) 3k B
Ans: (b)
Solution: For high temperature all number are excited so degree at freedom for diatomic
molecule is 7
fk BT
Internal energy is
2
7 k BT
U=
2
⎛ ∂U ⎞
CV = ⎜ ⎟ = 3.5k B
⎝ ∂T ⎠V
Q62. When an ideal monatomic gas is expanded adiabatically from an initial volume V0 to

T
3V0 , its temperature changes from T0 to T . Then the ratio is
T0
2 1
1 ⎛ 1 ⎞3 ⎛ 1 ⎞3
(a) (b) ⎜ ⎟ (c) ⎜ ⎟ (d) 3
3 ⎝3⎠ ⎝3⎠
Ans: (b)
Solution: For adiabatic process PV γ = k T0V0γ −1 = k
γ −1 v −1
⎛V ⎞ ⎛1⎞
= T ( 3V0 )
γ −1 γ −1
T0V0 ⇒ T = T0 ⎜ 0 ⎟ ⇒ T = T0 ⎜ ⎟
⎝ 3V0 ⎠ ⎝3⎠
5
For monoatomic gas γ =
3
5 2
−1
⎛ 1 ⎞3 ⎛ 1 ⎞3
T = T0 ⎜ ⎟ = T0 ⎜ ⎟
⎝3⎠ ⎝ 3⎠

                                                                                
Head office  Branch office 
 
fiziks, H.No. 23, G.F, Jia Sarai,  Anand Institute of Mathematics, 
 
Near IIT, Hauz Khas, New Delhi‐16  28‐B/6, Jia Sarai, Near IIT 
 
Phone: 011‐26865455/+91‐9871145498 Hauz Khas, New Delhi‐16 
                                                   
                                             Website: www.physicsbyfiziks.com                                                                                          
                                                            Email: fiziks.physics@gmail.com                                                                     205 
fiziks
Institute for NET/JRF, GATE, IIT‐JAM, JEST, TIFR and GRE in PHYSICAL SCIENCES 
 
Q63. A box of volume V containing N molecules of an ideal gas, is divided by a wall with a
V
hole into two compartments. If the volume of the smaller compartment is , the
3
variance of the number of particles in it, is
N 2N N
(a) (b) (c) N (d)
3 9 3
Ans: (b)
V 1 1
Solution: probability that one particle in compartment is so p =
3 3 3
There are only two options either particle is in left half or right half, so for one particle
distribution is Bernoulli for Bernoulli’s distribution
σ 2 = p (1 − p ) . For N particle distribution is Binomial so

1 ⎛ 1⎞ 2N
σ 2 = Np (1 − p ) = N × ⎜1 − ⎟ σ 2 =
3 ⎝ 3⎠ 9
Q64. A gas of non-relativistic classical particles in one dimension is subjected to a potential
⎛ 1 ⎞
V ( x ) = α x (where α is a constant). The partition function is ⎜ β = ⎟
⎝ k BT ⎠

4mπ 2mπ 8mπ 3mπ


(a) (b) (c) (d)
βα 3 2 2
βα 3 2 2
βα3 2 2
β 3α 2 2

Ans: (c)
∞ p2 ∞ αx ∞ αx
1 − x − 1 −
Solution: z = ∫ e 2 mkT dpx ∫ e kT dx = ( 2π mkT ) ∫ e kT dx
1/ 2

h −∞ −∞
h −∞

1/ 2 ∞ αx
⎛ 2π mkT ⎞ −
⇒⎜
⎝ h
2 ⎟
⎠ −∞
∫e kT
dx

∞ αx 0 αx ∞ αx
− + − kT kT 2kT
∫e
−∞
kT
dx = ∫e
−∞
kT
dx + ∫ e
0
kT
dx =
α
+
α
=
α
1/ 2
⎛ 2π mkT ⎞ ⎛ 2kT ⎞ 1
z =⎜ ⎟ ⎜ ⎟ put β =
⎝ α ⎠
2
⎝ h ⎠ kT

                                                                                
Head office  Branch office 
 
fiziks, H.No. 23, G.F, Jia Sarai,  Anand Institute of Mathematics, 
 
Near IIT, Hauz Khas, New Delhi‐16  28‐B/6, Jia Sarai, Near IIT 
 
Phone: 011‐26865455/+91‐9871145498 Hauz Khas, New Delhi‐16 
                                                   
                                             Website: www.physicsbyfiziks.com                                                                                          
                                                            Email: fiziks.physics@gmail.com                                                                     206 
fiziks
Institute for NET/JRF, GATE, IIT‐JAM, JEST, TIFR and GRE in PHYSICAL SCIENCES 
 
1
⎛ 8π m ⎞ 2
z =⎜ 2 3 2 ⎟
⎝h β α ⎠
Q65. The internal energy E (T ) of a system at a fixed volume is found to depend on the

temperature T as E (T ) = aT 2 + bT 4 . Then the entropy S (T ) , as a function of

temperature, is
1 2 1 4
(a) aT + bT (b) 2aT 2 + 4bT 4
2 4
4
(c) 2aT + bT 3 (d) 2aT + 2bT 3
3
Ans: (c)
Solution: From Law of thermodynamics
TdS = dE + PdV dE = TdS − PdV
it is given dV = 0
1
dE = TdS ⇒ dS = dE
T
E = aT 2 + bT 4 ⇒ dE = 2aTdT + 4bT 3 dT
1 4bT 3
dS =
T
( 2 aTdT + 4bT 3
dT ) = 2 adT + 4bT 2
dT = 2 aT +
3
Q66. Consider a gas of Cs atoms at a number density of 1012 atoms/cc. when the typical inter-
particle distance is equal to the thermal de Broglie wavelength of the particles, the
temperature of the gas is nearest to (Take the mass of a Cs atom to be 22.7 ×10−26 kg )

(a) 1× 10−9 K (b) 7 × 10−5 K (c) 1× 10−3 K (d) 2 ×10−8 K


Ans: (d)
Solution: When de Broglie wavelength = thermal wavelength
g3 / 2 ( z ) = 2.61
2/3
N h3 ⎛N⎞ h2
( 2π mkT )3 / 2 = ⇒ 2π mkT = ⎜ ⎟
V 2.61 ⎝V ⎠ ( 2.61)2 / 3
                                                                                
Head office  Branch office 
 
fiziks, H.No. 23, G.F, Jia Sarai,  Anand Institute of Mathematics, 
 
Near IIT, Hauz Khas, New Delhi‐16  28‐B/6, Jia Sarai, Near IIT 
 
Phone: 011‐26865455/+91‐9871145498 Hauz Khas, New Delhi‐16 
                                                   
                                             Website: www.physicsbyfiziks.com                                                                                          
                                                            Email: fiziks.physics@gmail.com                                                                     207 
fiziks
Institute for NET/JRF, GATE, IIT‐JAM, JEST, TIFR and GRE in PHYSICAL SCIENCES 
 
( ) ( )
2/3 2
1 h2 1 1012 6.6 × 10−34
T= (n) 2/3
=
2π mk ( 2.61)2 / 3 2 × 3.14 × 22.7 ×10−26 ×1.38 ×10−23 × ( 2.61)2 / 3
( 6.6 )2 ×108 ×10−64 ×1049 ( 6.6 )2 × 10−7
= =
6.28 × 22.7 × 1.38 × ( 2.61) 6.28 × 22.7 × 1.38 × ( 2.61)
2/3 2/3

0.221× 10−7 0.221


= = × 10−7 = 0.116 × 10−7 = 1.16 × 10−8
( 2.61) 2/3
1.895

                                                                                
Head office  Branch office 
 
fiziks, H.No. 23, G.F, Jia Sarai,  Anand Institute of Mathematics, 
 
Near IIT, Hauz Khas, New Delhi‐16  28‐B/6, Jia Sarai, Near IIT 
 
Phone: 011‐26865455/+91‐9871145498 Hauz Khas, New Delhi‐16 
                                                   
                                             Website: www.physicsbyfiziks.com                                                                                          
                                                            Email: fiziks.physics@gmail.com                                                                     208 
fiziks
Institute for NET/JRF, GATE, IIT‐JAM, JEST, TIFR and GRE in PHYSICAL SCIENCES 
 
ELECTRONICS AND EXPERIMENTAL METHODS
NET/JRF (JUNE-2011)

Q1. A signal of frequency 10 kHz is being digitized by an A/D converter. A possible


sampling time which can be used is
(a) 100 μs (b) 40 μs (c) 60 μs (d) 200 μs
Ans: (b)
1 1
Solution: f S ≥ 2 f ⇒ TS ≤ = = 50 μ s ⇒ TS ≤ 50 μ s
2 f 20 × 103
Q2. Consider the digital circuit shown below in which the input C is always high (1).

A
B Z

C
(high)
The truth table for the circuit can be written as
A B Z
0 0
0 1
1 0
1 1

The entries in the Z column (vertically) are


(a) 1010 (b) 0100 (c) 1111 (d) 1011
Ans: (d)
Solution: Z = A.B + (B ⊕ 1)

                                                                                
Head office  Branch office 
 
fiziks, H.No. 40 D, G.F, Jia Sarai,  Anand Institute of Mathematics, 
 
Near IIT, Hauz Khas, New Delhi‐16  28‐B/6, Jia Sarai, Near IIT 
 
Phone: 011‐26865455/+91‐9871145498 Hauz Khas, New Delhi‐16 
                                                   
                                             Website: www.physicsbyfiziks.com                                                                                          
                                                            Email: fiziks.physics@gmail.com                                                                     209 
fiziks
Institute for NET/JRF, GATE, IIT‐JAM, JEST, TIFR and GRE in PHYSICAL SCIENCES 
 
Q3. A time varying signal Vin is fed to an op-amp circuit with output signal V0 as shown in
the figure below. 10K

The circuit implements a


(a) high pass filter with cutoff frequency 16 Hz 1K

(b) high pass filter with cutoff frequency 100 Hz Vo
Vin +
(c) low pass filter with cutoff frequency 16 Hz 1K 10K
10K
(d) low pass filter with cutoff frequency 100 Hz 1μ F


+

Ans: (c)
Solution: Since circuit has R and C combination, its a Low Pass filter and cutoff frequency
1
= ≈ 16 Hz.
2πRC
NET/JRF (DEC-2011)
Q4. In the operational amplifier circuit below, the voltage at point A is
+ 5V
1K A
1V −
1V +
1K
1K − 5V

(a) 1.0 V (b) 0.5 V (c) 0 V (d) – 5.0 V


Ans: (b)
1
Solution: V A = × 1 = 0.5V .
1+1

                                                                                
Head office  Branch office 
 
fiziks, H.No. 40 D, G.F, Jia Sarai,  Anand Institute of Mathematics, 
 
Near IIT, Hauz Khas, New Delhi‐16  28‐B/6, Jia Sarai, Near IIT 
 
Phone: 011‐26865455/+91‐9871145498 Hauz Khas, New Delhi‐16 
                                                   
                                             Website: www.physicsbyfiziks.com                                                                                          
                                                            Email: fiziks.physics@gmail.com                                                                     210 
fiziks
Institute for NET/JRF, GATE, IIT‐JAM, JEST, TIFR and GRE in PHYSICAL SCIENCES 
 
Q5. A counter consists of four flip-flops connected as shown in the figure:
A0 A1 A2 A3

J Q J Q J Q J Q
CLK
K Q K Q K Q K Q

If the counter is initialized as A0 A1 A2 A3 = 0110, the state after the next clock pulse is
(a) 1000 (b) 0001 (c) 0011 (d) 1100
Ans: (b) A 0= 0 A1= 1 A 2= 1 A 3= 0

0 J Q J Q 0 J Q J Q
CLK
1 K Q K Q 0 1 K Q K Q
1 0 1

Q6. The pins 0, 1, 2 and 3 of part A of a microcontroller are connected with resistors to drive
an LED at various intensities as shown in the figure. For V CC
VCC = 4.2 V and a voltage drop of 1.2 V across the LED, ≈
the range (maximum current) and resolution (step size)
A3
of the drive current are, respectively, 0 . 75 k
A2
(a) 4.0 mA and 1.0 mA μC 1 .5 k
A1
(b) 15.0 mA and 1.0 mA 3k
(c) 7.5 mA and 0.5 mA A0
6k
(d) 4.0 mA and 0.5 mA
Ans: (c)
A3 , A1 , A2 , A0
For Maximum current .
0, 0, 0, 0
                                                                                
Head office  Branch office 
 
fiziks, H.No. 40 D, G.F, Jia Sarai,  Anand Institute of Mathematics, 
 
Near IIT, Hauz Khas, New Delhi‐16  28‐B/6, Jia Sarai, Near IIT 
 
Phone: 011‐26865455/+91‐9871145498 Hauz Khas, New Delhi‐16 
                                                   
                                             Website: www.physicsbyfiziks.com                                                                                          
                                                            Email: fiziks.physics@gmail.com                                                                     211 
fiziks
Institute for NET/JRF, GATE, IIT‐JAM, JEST, TIFR and GRE in PHYSICAL SCIENCES 
 
4.2 − 1.2 4.2 − 1.2 4.2 − 1.2 4.2 − 1.2
Thus I max = + + + = 7.5mA
0.75k 1.5k 3k 6k
A3 , A1 , A2 , A0 4.2 − 1.2
For Step size . Thus I 0 = = 0.5mA
0, 0, 0, 1 6k
Q7. The figure below shows a voltage regulator utilizing a Zener diode of breakdown voltage
5 V and a positive triangular wave input of amplitude 10 V.
500Ω 12
Vi 10
i 8

i(mA)
6
1K 4
2
0
0 1 2 3 4 5 6 7 8
t ( s)
For Vi > 5V, the Zener regulates the output voltage by channeling the excess current
through itself. Which of the following waveforms shows the current i passing through the
Zener diode?
(a) 12 (b) 12
10
10
8
i(mA)

8
6
i(mA)

6
4
4
2
2
0
0 1 2 3 4 5 6 7 8 0
0 1 2 3 4 5 6 7 8
t ( s) t ( s)
(c) 12 (d) 12
10 10
8 8
i(mA)
i(mA)

6 6
4 4
2 2
0 0
0 1 2 3 4 5 6 7 8 0 1 2 3 4 5 6 7 8
t ( s) t ( s)
Ans: (a)
Solution: When zener is OFF zener current is zero when zener is ON zener current will flow.
                                                                                
Head office  Branch office 
 
fiziks, H.No. 40 D, G.F, Jia Sarai,  Anand Institute of Mathematics, 
 
Near IIT, Hauz Khas, New Delhi‐16  28‐B/6, Jia Sarai, Near IIT 
 
Phone: 011‐26865455/+91‐9871145498 Hauz Khas, New Delhi‐16 
                                                   
                                             Website: www.physicsbyfiziks.com                                                                                          
                                                            Email: fiziks.physics@gmail.com                                                                     212 
fiziks
Institute for NET/JRF, GATE, IIT‐JAM, JEST, TIFR and GRE in PHYSICAL SCIENCES 
 
NET/JRF (JUNE-2012)
Q8. In the op-amp circuit shown in the figure below, the input voltage is 1V. The value of the
1K
output V0 is

1K
− 1K Vo
Vi = 1V
+
1K

(a) -0.33 V (b) -0.50 V (c) -1.00 V (d) -0.25 V


Ans: (b)
R F Vin 1 1×1 1
Solution: V0 = − = − V = −0.05 where R F = = K and R1 = 1K .
R1 2 1+1 2
Q9. An LED operates at 1.5 V and 5 mA in forward bias. Assuming an 80% external
efficiency of the LED, how many photons are emitted per second?
(a) 5.0 x 1016 (b) 1.5 x 1016 (c) 0.8 x 1016 (d) 2.5 x 1016
Ans: (d)
i P i 5 × 10 −3
Solution: Pin = η int hf , number of photon = in = η int = .8 × −19
= 2.5 × 1016
e hf e 1.6 × 10
Q10. The transistor in the given circuit has hfe = 35Ω and hie = 1000Ω. If the load resistance
RL = 1000Ω, the voltage and current gain are, respectively.
(a) -35 and + 35 VO
RL
(b) 35 and - 35
+
(c) 35 and – 0.97
VI −

(d) 0.98 and - 35

Ans: (a)

                                                                                
Head office  Branch office 
 
fiziks, H.No. 40 D, G.F, Jia Sarai,  Anand Institute of Mathematics, 
 
Near IIT, Hauz Khas, New Delhi‐16  28‐B/6, Jia Sarai, Near IIT 
 
Phone: 011‐26865455/+91‐9871145498 Hauz Khas, New Delhi‐16 
                                                   
                                             Website: www.physicsbyfiziks.com                                                                                          
                                                            Email: fiziks.physics@gmail.com                                                                     213 
fiziks
Institute for NET/JRF, GATE, IIT‐JAM, JEST, TIFR and GRE in PHYSICAL SCIENCES 
 
Q11. The output, O, of the given circuit in cases I and II, where
Case I: A, B = 1; C, D = 0; E, F = 1 and G = 0
Case II: A, B = 0; C, D = 0: E, F = 0 and G = 1
are respectively Α
Β
(a) 1, 0
(b) 0, 1 C
D
(c) 0, 0
(d) 1, 1
Ans: (d) O

((
Solution: O = AB + CD E + F G ) ) E F
G

NET/JRF (DEC-2012)
Q12. A live music broadcast consists of a radio-wave of frequency 7 MHz, amplitude-
modulated by a microphone output consisting of signals with a maximum frequency of
10 kHz. The spectrum of modulated output will be zero outside the frequency band
(a) 7.00 MHz to 7.01 MHz (b) 6.99 MHz to 7.01 MHz
(c) 6.99 MHz to 7.00 MHz (d) 6.995 MHz to 7.005 MHz
Ans: (b)
Solution: Spectrum consists of f c − f m and f c + f m .
Q13. In the op-amp circuit shown in the figure, Vi is a sinusoidal input signal of frequency 10
Hz and V0 is the output signal. The magnitude of the gain and the phase shift,
0.01μF
respectively, close to the values
(a) 5 2 and π / 2
10 K
(b) 5 2 and − π / 2 1K
Vi −
(c) 10 and zero Vo
+
(d) 10 and π
Ans: (d)

                                                                                
Head office  Branch office 
 
fiziks, H.No. 40 D, G.F, Jia Sarai,  Anand Institute of Mathematics, 
 
Near IIT, Hauz Khas, New Delhi‐16  28‐B/6, Jia Sarai, Near IIT 
 
Phone: 011‐26865455/+91‐9871145498 Hauz Khas, New Delhi‐16 
                                                   
                                             Website: www.physicsbyfiziks.com                                                                                          
                                                            Email: fiziks.physics@gmail.com                                                                     214 
fiziks
Institute for NET/JRF, GATE, IIT‐JAM, JEST, TIFR and GRE in PHYSICAL SCIENCES 
 
v X C RF v
Solution: 0 = − ⇒ 0 ≈ 10
vin R1 (R1 + RF ) vin

Q14. The logic circuit shown in the figure below Implements the Boolean expression
A

HIGH y

(a) y = A ⋅ B (b) y = A ⋅ B (c) y = A ⋅ B (d) y = A + B


Ans: (a)

Solution: Output of each Ex-OR gate is A and B . Thus y = A + B = A ⋅ B


Q15. A diode D as shown in the circuit has an i-v relation that can be approximated by
⎧v 2 + 2v D , for v D > 0
iD = ⎨ D
⎩0, for v D ≤ 0 1Ω

The value of v D in the circuit is iD


+
(
(a) − 1 + 11 V ) (b) 8 V 10 V

D vD

(c) 5 V (d) 2 V
Ans: (d)
Solution: − 10 + (v D2 + 2v D )× 1 + v D = 0 ⇒ v D = 2V
Q16. Band-pass and band-reject filters can be implemented by combining a low pass and a
high pass filter in series and in parallel, respectively. If the cut-off frequencies of the low
pass and high pass filters are ω 0LP and ω 0HP , respectively, the condition required to
implement the band-pass and band-reject filters are, respectively,
(a) ω 0HP < ω 0LP and ω 0HP < ω 0LP (b) ω0HP < ω0LP and ω 0HP > ω 0LP

(c) ω 0HP > ω 0LP and ω0HP < ω0LP (d) ω 0HP > ω 0LP and ω0HP > ω0LP
Ans: (c)

                                                                                
Head office  Branch office 
 
fiziks, H.No. 40 D, G.F, Jia Sarai,  Anand Institute of Mathematics, 
 
Near IIT, Hauz Khas, New Delhi‐16  28‐B/6, Jia Sarai, Near IIT 
 
Phone: 011‐26865455/+91‐9871145498 Hauz Khas, New Delhi‐16 
                                                   
                                             Website: www.physicsbyfiziks.com                                                                                          
                                                            Email: fiziks.physics@gmail.com                                                                     215 
fiziks
Institute for NET/JRF, GATE, IIT‐JAM, JEST, TIFR and GRE in PHYSICAL SCIENCES 
 
NET/JRF (JUNE-2013)

Q17. A silicon transistor with built-in voltage 0.7 V is used in the circuit shown, with
V BB = 9.7V , R B = 300kΩ, VCC = 12V and RC = 2kΩ . Which of the following figures

correctly represents the load line and quiescent Q point?

RC

RB +
+ −
V CC
V BB

iC iC
(μΑ ) ΙΒ = (mA) ΙΒ =
32 35μΑ 6 35μΑ
(a) 32μΑ (b) 32μΑ
Q
30μΑ Q
30μΑ
0 9 .7 VCE (V ) 0 12 VCE (V )

iC iC
(mA) ΙΒ = (μA) ΙΒ =
6 35μΑ 32 Q 35μΑ
(c) Q
32μΑ (d) 32μΑ
30μΑ 30μΑ
0 12 VCE (V ) 0 9 .7 VCE (V )

Ans: (b)
VCC − V BE 9.7 − 0.7 VCC 12
Solution: I B = = = 30 μA and I C , sat = = = 6mA
RB 300 × 10 3
RC 2 × 10 3
Q18. If the analog input to an 8-bit successive approximation ADC is increased from 1.0 V to
2.0 V, then the conversion time will
(a) remain unchanged (b) double
(c) decrease to half its original value (d) increase four times
Ans: (a)

                                                                                
Head office  Branch office 
 
fiziks, H.No. 40 D, G.F, Jia Sarai,  Anand Institute of Mathematics, 
 
Near IIT, Hauz Khas, New Delhi‐16  28‐B/6, Jia Sarai, Near IIT 
 
Phone: 011‐26865455/+91‐9871145498 Hauz Khas, New Delhi‐16 
                                                   
                                             Website: www.physicsbyfiziks.com                                                                                          
                                                            Email: fiziks.physics@gmail.com                                                                     216 
fiziks
Institute for NET/JRF, GATE, IIT‐JAM, JEST, TIFR and GRE in PHYSICAL SCIENCES 
 
Q19. The input to a lock-in amplifier has the form Vi (t ) = Vi sin (ω t + θ i ) where Vi , ω ,θ i are the
amplitude, frequency and phase of the input signal respectively. This signal is multiplied
by a reference signal of the same frequency ω , amplitude Vr and phase θ r . If the
multiplied signal is fed to a low pass filter of cut-off frequency ω , the final output signal
is
1 ⎡ ⎛1 ⎞⎤
(a) ViVr cos(θ i − θ r ) (b) ViVr ⎢cos(θ i − θ r ) − cos⎜ ω t + θ i + θ r ⎟⎥
2 ⎣ ⎝2 ⎠⎦

⎡ ⎛1 ⎞⎤
(c) ViVr sin (θ i − θ r ) (d) ViVr ⎢cos(θ i − θ r ) + cos⎜ ω t + θ i + θ r ⎟⎥
⎣ ⎝2 ⎠⎦
Ans: (a)
Vi V r
Solution: V = Vr sin (ω t + θ r ) × Vi sin (ω t + θ i ) = [cos(θ i − θ r ) − cos(2ω t + θ i + θ r )]
2
Vi V r
Output of low pass filter= cos(θ i − θ r )
2
Q20. Four digital outputs V , P, T and H monitor the speed v , tyre pressure p , temperature t
and relative humidity h of a car. These outputs switch from 0 to 1 when the values of the
parameters exceed 85 km/hr, 2 bar, 40 0 C and 50%, respectively. A logic circuit that is
used to switch ON a lamp at the output E is shown below.
Which of the following condition will switch the lamp ON?
(a) v < 85km / hr , p < 2 bar , t > 40 0 C , h > 50%
V
(b) v < 85km / hr , p < 2 bar , t > 40 C , h < 50% 0

P
(c) v > 85km / hr , p < 2 bar , t > 40 0 C , h < 50% E

(d) v > 85km / hr , p < 2 bar , t > 40 0 C , h > 50% T


Ans: (a)
H

                                                                                
Head office  Branch office 
 
fiziks, H.No. 40 D, G.F, Jia Sarai,  Anand Institute of Mathematics, 
 
Near IIT, Hauz Khas, New Delhi‐16  28‐B/6, Jia Sarai, Near IIT 
 
Phone: 011‐26865455/+91‐9871145498 Hauz Khas, New Delhi‐16 
                                                   
                                             Website: www.physicsbyfiziks.com                                                                                          
                                                            Email: fiziks.physics@gmail.com                                                                     217 
fiziks
Institute for NET/JRF, GATE, IIT‐JAM, JEST, TIFR and GRE in PHYSICAL SCIENCES 
 
JRF/NET-(DEC-2013)
Q21. Consider the op-amp circuit shown in the figure.
If the input is a sinusoidal wave Vi = 5 sin (1000t ) , then 1μ F

the amplitude of the output V0 is


1K
5 1K
(a) (b) 5 Vi − Vo
2 +
5 2
(c) (d) 5 2
2
Ans: (c)
vo X RF X C 10 3 1
= − F , XF = = where R F = 1 × 10 3 Ω, X C =
vin R1 RF + X C (1 + j ) j × 10 × 10 −6
3

vo 10 3 1 1 5 5 2
= × 3 = ⇒ vo = sin ωt = sin ωt
vin 2 10 2 2 2

Q22. If one of the inputs of a J-K flip flop is high and the other is low, then the outputs Q and

Q
(a) oscillate between low and high in race around condition
(b) toggle and the circuit acts like a T flip flop
(c) are opposite to the inputs
(d) follow the inputs and the circuit acts like an R − S flip flop
Ans: (d)
Q23. A sample of Si has electron and hole mobilities of 0.13 and 0.05 m 2 /V- s respectively at
300 K. It is doped with P and Al with doping densities of 1.5 × 10 21 / m 3 and
2.5 × 10 21 / m 3 respectively. The conductivity of the doped Si sample at 300 K is
(a) 8 Ω −1 m −1 (b) 32 Ω −1 m −1 (c) 20.8 Ω −1 m −1 (d) 83.2 Ω −1 m −1
Ans: (a)
Solution: Resulting doped crystal is p-type and p p = (2.5 − 1.5) × 10 21 / m 3 = 1 × 10 21 / m 3

σ = e(n p μ n + p p μ p ) ≈ ep p μ p = 1.6 × 10 −19 × 1 × 10 21 × 0.05 = 8 Ω −1 m −1

                                                                                
Head office  Branch office 
 
fiziks, H.No. 40 D, G.F, Jia Sarai,  Anand Institute of Mathematics, 
 
Near IIT, Hauz Khas, New Delhi‐16  28‐B/6, Jia Sarai, Near IIT 
 
Phone: 011‐26865455/+91‐9871145498 Hauz Khas, New Delhi‐16 
                                                   
                                             Website: www.physicsbyfiziks.com                                                                                          
                                                            Email: fiziks.physics@gmail.com                                                                     218 
fiziks
Institute for NET/JRF, GATE, IIT‐JAM, JEST, TIFR and GRE in PHYSICAL SCIENCES 
 
Q24. Two identical Zener diodes are placed back to back in series and are connected to a
variable DC power supply. The best representation of the I-V characteristics of the circuit
is
(a) I (b) I

V V

(c) I (d) I

V V

Ans: (d)
Q25. A 4-variable switching function is given by f = ∑ (5, 7, 8, 10, 13, 15) + d (0, 1, 2 ) , where

d is the do-not-care-condition. The minimized form of f in sum of products (SOP) form


is
(a) A C + B D (b) AB + CD (c) AD + BC (d) B D + BD

Ans: (d) CD CD CD CD
AB × × × BD

AB 1 1 BD
AB 1 1

AB 1 1

                                                                                
Head office  Branch office 
 
fiziks, H.No. 40 D, G.F, Jia Sarai,  Anand Institute of Mathematics, 
 
Near IIT, Hauz Khas, New Delhi‐16  28‐B/6, Jia Sarai, Near IIT 
 
Phone: 011‐26865455/+91‐9871145498 Hauz Khas, New Delhi‐16 
                                                   
                                             Website: www.physicsbyfiziks.com                                                                                          
                                                            Email: fiziks.physics@gmail.com                                                                     219 
fiziks
Institute for NET/JRF, GATE, IIT‐JAM, JEST, TIFR and GRE in PHYSICAL SCIENCES 
 
NET/JRF (JUNE-2014)
Q26. The inner shield of a triaxial conductor is driven by an (ideal) op-amp follower circuit as
shown. The effective capacitance between the signal-carrying conductor and ground is

Signal


+

(a) unaffected (b) doubled (c) halved (d) made zero


Ans: (a)
Q27. An op-amp based voltage follower
(a) is useful for converting a low impedance source into a high impedance source.
(b) is useful for converting a high impedance source into a low impedance source.
(c) has infinitely high closed loop output impedance
(d) has infinitely high closed loop gain
Ans: (b)
Q28. An RC network produces a phase-shift of 30 o . How many such RC networks should be
cascaded together and connected to a Common Emitter amplifier so that the final circuit
behaves as an oscillator?
(a) 6 (b) 12 (c) 9 (d) 3
Ans: (a)
Solution: Total phase shift must be 0 or 3600. Common Emitter amplifier has phase change of
1800 so we need 6 RC network for next 1800 phase shift.

                                                                                
Head office  Branch office 
 
fiziks, H.No. 40 D, G.F, Jia Sarai,  Anand Institute of Mathematics, 
 
Near IIT, Hauz Khas, New Delhi‐16  28‐B/6, Jia Sarai, Near IIT 
 
Phone: 011‐26865455/+91‐9871145498 Hauz Khas, New Delhi‐16 
                                                   
                                             Website: www.physicsbyfiziks.com                                                                                          
                                                            Email: fiziks.physics@gmail.com                                                                     220 
fiziks
Institute for NET/JRF, GATE, IIT‐JAM, JEST, TIFR and GRE in PHYSICAL SCIENCES 
 
Q29. For the logic circuit shown in the below

X
B

A simplified equivalent circuit is A


(a) A (b) B
B X X
C
C

A A
(c) (d)
B B
X X
C C
Ans: (d)

A A
A.B ( A + B ) AC
B X
B

C
C
ABC

X = ( A + B ) AC + ABC = AC + ABC + ABC = AC + AB = A ( B + C )

                                                                                
Head office  Branch office 
 
fiziks, H.No. 40 D, G.F, Jia Sarai,  Anand Institute of Mathematics, 
 
Near IIT, Hauz Khas, New Delhi‐16  28‐B/6, Jia Sarai, Near IIT 
 
Phone: 011‐26865455/+91‐9871145498 Hauz Khas, New Delhi‐16 
                                                   
                                             Website: www.physicsbyfiziks.com                                                                                          
                                                            Email: fiziks.physics@gmail.com                                                                     221 
fiziks
Institute for NET/JRF, GATE, IIT‐JAM, JEST, TIFR and GRE in PHYSICAL SCIENCES 
 
NET/JRF (DEC-2014)
Q30. Consider the amplifier circuit comprising of the two op-amps A1 and A2 as shown in the
figure. 1M
R
− 10 K

+ r
A1 + V0
A2

If the input ac signal source has an impedance of 50 k Ω , which of the following


statements is true?
(a) A1 is required in the circuit because the source impedance is much greater than r
(b) A1 is required in the circuit because the source impedance is much less than R
(c) A1 can be eliminated from the circuit without affecting the overall gain
(d) A1 is required in the circuit if the output has to follow the phase of the input signal
Ans: (a)
Solution: A1 is required in the circuit because the source impedance is much greater than r
Q31. The I − V characteristics of the diode in the circuit below is given by
⎧(V − 0.7 ) / 500 for V ≥ 0.7⎫
I =⎨ ⎬
⎩ 0 for V < 0.7⎭
where V is measured in volts and I is measured in amperes.
1K I

10 V

The current I in the circuit is


(a) 10.0 mA (b) 9.3 mA (c) 6.2 mA (d) 6.7 mA
Ans: (c)
Solution: Applying K.V.L. −10 + 1000 × I + V = 0 ⇒ −10 + 1000 × (V − 0.7 ) / 500 + V = 0

                                                                                
Head office  Branch office 
 
fiziks, H.No. 40 D, G.F, Jia Sarai,  Anand Institute of Mathematics, 
 
Near IIT, Hauz Khas, New Delhi‐16  28‐B/6, Jia Sarai, Near IIT 
 
Phone: 011‐26865455/+91‐9871145498 Hauz Khas, New Delhi‐16 
                                                   
                                             Website: www.physicsbyfiziks.com                                                                                          
                                                            Email: fiziks.physics@gmail.com                                                                     222 
fiziks
Institute for NET/JRF, GATE, IIT‐JAM, JEST, TIFR and GRE in PHYSICAL SCIENCES 
 
⇒ −10 + 2 (V − 0.7 ) + V = 0 ⇒ 3V = 11.4 ⇒ V = 3.8 Volts

Thus I = (V − 0.7 ) / 500 = ( 3.8 − 0.7 ) / 500 = 3.1/ 500 = 6.2 mA

Q32. In a measurement of the viscous drag force experienced by spherical particles in a liquid,
the force is found to be proportional to V 1 / 3 where V is the measured volume of each
particle. If V is measured to be 30 mm 3 , with an uncertainty of 2.7 mm 3 , the resulting
relative percentage uncertainty in the measured force is
(a) 2.08 (b) 0.09 (c) 6 (d) 3
Ans: (b)
2
⎛ ∂F ⎞ 2
Solution: The relative percentage uncertainty in the measure force is σ = ⎜ σ 2
⎝ ∂V ⎟⎠ V F

⎛ ∂F ⎞
⇒σF = ⎜ ⎟ σ V where σ V is the uncertainty in the measurement of volume.
⎝ ∂V ⎠
∂F 1 −2 / 3
∵ F = V 1/3 ⇒ = V
∂V 3
1 1 1 1
∴σF = × σV = × 2.7 = × 2.7 = × 2.7 ⇒ σ F = 0.09
3 ( 30) 3 × ( 900) 3 × 9.7
2/3 2/3 1/ 3
3V

Q33. Consider a Low Pass (LP) and a High Pass (HP) filter with cut-off frequencies f LP and
f HP , respectively, connected in series or in parallel configurations as shown in the
Figures A and B below. ΗΡ
fHP
(A) Input ΗΡ LΡ Output (B) Input Output
fHP fLP

Which of the following statements is correct? fLP
(a) For f HP < f LP , A acts as a Band Pass filter and B acts as a band Reject filter
(b) For f HP > f LP , A stops the signal from passing through and B passes the signal
without filtering
(c) For f HP < f LP , A acts as a Band Pass filter and B passes the signal without filtering
(d) For f HP > f LP , A passes the signal without filtering and B acts as a Band Reject filter
Ans: (c)

                                                                                
Head office  Branch office 
 
fiziks, H.No. 40 D, G.F, Jia Sarai,  Anand Institute of Mathematics, 
 
Near IIT, Hauz Khas, New Delhi‐16  28‐B/6, Jia Sarai, Near IIT 
 
Phone: 011‐26865455/+91‐9871145498 Hauz Khas, New Delhi‐16 
                                                   
                                             Website: www.physicsbyfiziks.com                                                                                          
                                                            Email: fiziks.physics@gmail.com                                                                     223 
fiziks
Institute for NET/JRF, GATE, IIT‐JAM, JEST, TIFR and GRE in PHYSICAL SCIENCES 
 
Q34. The power density of sunlight incident on a solar cell is 100 mW / cm 2 . Its short circuit

current density is 30 mA / cm 2 and the open circuit voltage is 0.7 V . If the fill factor of
the solar cell decreases from 0.8 to 0.5 then the percentage efficiency will decrease from
(a) 42.0 to 26.2 (b) 24.0 to 16.8 (c) 21.0 to 10.5 (d) 16.8 to 10.5
Ans: (d)
Solution: The efficiency of a solar cell is determined as the fraction of incident power which is
converted to electricity and is defined as
Voc I sc FF
η= and Pmax = Voc I sc FF
Pin

where Voc is the open circuit voltage, I sc is the short circuit current density , FF is the

Fill factor, Pin is the input power and η is the efficiency of the solar cell.

Given Pin = 100 mW / cm 2 , I sc = 30 mA / cm 2 , Voc = 0.7 V

Let η1 is the efficiency of solar cell when FF = 0.8

( 0.7 V ) × ( 30 × 10−3 A / cm2 ) × 0.8 16.8


∴ η1 = −3
= ⇒ η1 = 0.168
100 × 10 W / cm 2
100
Let η2 is the efficiency of solar cell when FF = 0.5

( 0.7 V ) × ( 30 × 10−3 A / cm2 ) × 0.5 10.5


∴ η2 = −3
= ⇒ η2 = 0.105
100 × 10 W / cm 2
100
Thus efficiency decreases from η1 = 16.8% to η2 = 10.5%

                                                                                
Head office  Branch office 
 
fiziks, H.No. 40 D, G.F, Jia Sarai,  Anand Institute of Mathematics, 
 
Near IIT, Hauz Khas, New Delhi‐16  28‐B/6, Jia Sarai, Near IIT 
 
Phone: 011‐26865455/+91‐9871145498 Hauz Khas, New Delhi‐16 
                                                   
                                             Website: www.physicsbyfiziks.com                                                                                          
                                                            Email: fiziks.physics@gmail.com                                                                     224 
fiziks
Institute for NET/JRF, GATE, IIT‐JAM, JEST, TIFR and GRE in PHYSICAL SCIENCES 
 
NET/JRF (JUNE-2015)
Q35. The concentration of electrons, n and holes p , for an intrinsic semiconductor at a
3
⎛ E ⎞
temperature T can be expressed as n = p = AT 2 exp ⎜ − g ⎟ , where Eg is the band
⎝ 2 k BT ⎠
−3
gap and A is a constant. If the mobility of both types of carries is proportional to T 2
,
then the log of the conductivity is a linear function of T −1 , with slope
Eg Eg − Eg − Eg
(a) (b) (c) (d)
( 2k B ) kB ( 2k B ) kB

Ans. (c)
3
⎛ − Eg ⎞ −
⎛ − Eg ⎞
Solution: σ i = ni e ( μe + μ p ) ∝ T 2 exp ⎜ ⎟ × T 2
⇒ σ i = C exp ⎜ ⎟
⎝ 2k B T ⎠ ⎝ 2k B T ⎠
Eg − Eg
ln (σ i ) = + ln C ⇒ slope is
2 k BT 2k B

π Pa 4
Q36. The viscosity η of a liquid is given by Poiseuille’s formula η = . Assume that
8lV
l and V can be measured very accurately, but the pressure P has an rms error of 1% and
the radius a has an independent rms error of 3% . The rms error of the viscosity is
closest to
(a) 2% (b) 4% (c) 12% (d) 13%
Ans. (d)
π pa 4
Solution: η = = k pa 4 (where k is a constant)
8lv
ln η = ln p + 4 ln a + ln k
Δη Δp Δa Δη
⇒ = +4 +0 ⇒ = 1% + 4 × 3% = 13%
η p a η

                                                                                
Head office  Branch office 
 
fiziks, H.No. 40 D, G.F, Jia Sarai,  Anand Institute of Mathematics, 
 
Near IIT, Hauz Khas, New Delhi‐16  28‐B/6, Jia Sarai, Near IIT 
 
Phone: 011‐26865455/+91‐9871145498 Hauz Khas, New Delhi‐16 
                                                   
                                             Website: www.physicsbyfiziks.com                                                                                          
                                                            Email: fiziks.physics@gmail.com                                                                     225 
fiziks
Institute for NET/JRF, GATE, IIT‐JAM, JEST, TIFR and GRE in PHYSICAL SCIENCES 
 
Q37. Consider the circuits shown in figures (a) and (b) below
2K 1K

10 K 10 K
10V 10V

10.7 V 5V

(a) (b)
If the transistors in Figures (a) and (b) have current gain ( β dc ) of 100 and 10

respectively, then they operate in the


(a) active region and saturation region respectively
(b) saturation region and active region respectively
(c) saturation region in both cases
(d) active region in both cases
Ans. (b)
Solution: In both case input section is F.B.
10.7 − 0.7
For figure (a) I B = = 1 mA ⇒ I C = BI B = 100 mA
10
Thus VCB = VC − VB = (10 − 2 × 100 ) = 0.7 = −ve

⇒ output section is F.B.


since both section are F.B. so it is in saturation region.
5 − 0.7
For Figure (b) I B = = 0.43 mA ⇒ I C = BI B = 4.3 mA
10
Thus VCB = VC − VB = (10 − 4.3) − 0.7) = + ve

⇒ out put section is R.B.


Thus it is in active region

                                                                                
Head office  Branch office 
 
fiziks, H.No. 40 D, G.F, Jia Sarai,  Anand Institute of Mathematics, 
 
Near IIT, Hauz Khas, New Delhi‐16  28‐B/6, Jia Sarai, Near IIT 
 
Phone: 011‐26865455/+91‐9871145498 Hauz Khas, New Delhi‐16 
                                                   
                                             Website: www.physicsbyfiziks.com                                                                                          
                                                            Email: fiziks.physics@gmail.com                                                                     226 
fiziks
Institute for NET/JRF, GATE, IIT‐JAM, JEST, TIFR and GRE in PHYSICAL SCIENCES 
 
Q38. In the circuit given below, the thermistor has a resistance 3 k Ω at 250 C . Its resistance

decreases by 150Ω per 0 C upon heating. The output voltage of the circuit at 300 C is
T

−1V

1K
+ Vout

(a) −3.75 V (b) −2.25 V (c) 2.25 V (d) 3.75 V


Ans. (c)
Solution: At 300 C Resistance
= 3000 − 150 × 5 = 2250 Ω
RF −2250
⇒ V0 = − vi = × −1 ⇒ V0 = 2.25 volts
R1 1000
NET/JRF (DEC-2015)
Q39. If the reverse bias voltage of a silicon varactor is increased by a factor of 2 , the
corresponding transition capacitance
(a) increases by a factor of 2 (b) increases by a factor of 2
(c) decreases h a factor of 2 (d) decreases by a factor of 2
Ans.: (c)

1 CT′ V C′ V 1
Solution: CT ∝ ⇒ = ⇒ T = ⇒ CT′ = CT
V CT V′ CT 2V 2

                                                                                
Head office  Branch office 
 
fiziks, H.No. 40 D, G.F, Jia Sarai,  Anand Institute of Mathematics, 
 
Near IIT, Hauz Khas, New Delhi‐16  28‐B/6, Jia Sarai, Near IIT 
 
Phone: 011‐26865455/+91‐9871145498 Hauz Khas, New Delhi‐16 
                                                   
                                             Website: www.physicsbyfiziks.com                                                                                          
                                                            Email: fiziks.physics@gmail.com                                                                     227 
fiziks
Institute for NET/JRF, GATE, IIT‐JAM, JEST, TIFR and GRE in PHYSICAL SCIENCES 
 
Q40. If the parameters y and x are related by y = log ( x ) , then the circuit that can be used to

produce an output voltage V0 varying linearly with x is

(a) y (b) y
− −
Vo Vo
+ +

(c) (d)
y − y −
Vo Vo
+ +

Ans.: (c)
Solution: (1) Integrator (2) Logarithmic Ampere (V0 ∝ log y )

(3) Anti-log (V0 ∝ e y ∝ x ) (4) Differentiator

Q41. The state diagram corresponding to the following circuit is

x D A
y
CLOCK

Flip Flop
00, 01,10 01,11
(a) 11 00 (b) 00,10 00,10

0 1 0 1

01,10,11 01,11

00,11 00, 01,10


(c) 01,10 00,10 (d) 11 11

0 1

00,11
                                                                                 00, 01,10
Head office  Branch office 
 
fiziks, H.No. 40 D, G.F, Jia Sarai,  Anand Institute of Mathematics, 
 
Near IIT, Hauz Khas, New Delhi‐16  28‐B/6, Jia Sarai, Near IIT 
 
Phone: 011‐26865455/+91‐9871145498 Hauz Khas, New Delhi‐16 
                                                   
                                             Website: www.physicsbyfiziks.com                                                                                          
                                                            Email: fiziks.physics@gmail.com                                                                     228 
fiziks
Institute for NET/JRF, GATE, IIT‐JAM, JEST, TIFR and GRE in PHYSICAL SCIENCES 
 
Ans.: (d)
Solution: Let verify option (d)
x y A x y A
0 0 0 →1 0 0 1→ 0
0 1 0 →1 0 1 1→ 0
1 0 0 →1 1 0 1→ 0
1 1 0→0 1 1 1→1
Q42. A sinusoidal signal of peak to peak amplitude 1V and unknown time period is input to

the following circuit for 5 second’s duration. If the counter measures a value ( 3E8 ) H in

hexadecimal then the time period of the input signal is


0.1 μ F
− 10 bit
Vi + counter
1K 10 K

(a) 2.5 ms (b) 4 ms (c) 10 ms (d) 5 ms


Ans.: (d)
Solution: ( 3E8 ) H → 3 ×162 + 14 × 16 + 8 × 1 = (1000 )10

In 5 sec, number of counts is 1000


Then count per sec is = 200 count/sec
1
So T = sec = 5ms
200

                                                                                
Head office  Branch office 
 
fiziks, H.No. 40 D, G.F, Jia Sarai,  Anand Institute of Mathematics, 
 
Near IIT, Hauz Khas, New Delhi‐16  28‐B/6, Jia Sarai, Near IIT 
 
Phone: 011‐26865455/+91‐9871145498 Hauz Khas, New Delhi‐16 
                                                   
                                             Website: www.physicsbyfiziks.com                                                                                          
                                                            Email: fiziks.physics@gmail.com                                                                     229 
fiziks
Institute for NET/JRF, GATE, IIT‐JAM, JEST, TIFR and GRE in PHYSICAL SCIENCES 
 
NET/JRF (JUNE-2016)
Q43. The dependence of current I on the voltage V of a certain device is given by
2
⎛ V ⎞
I = I 0 ⎜1 − ⎟
⎝ V0 ⎠
where I 0 and V0 are constants. In an experiment the current I is measured as the voltage

V applied across the device is increased. The parameters V0 and I 0 can be graphically

determined as
(a) the slope and the y -intercept of the I − V 2 graph
(b) the negative of the ratio of the y -intercept and the slope, and the y -intercept of the

I − V 2 graph
(c) the slope and the y -intercept of the I − V graph
(d) the negative of the ratio of the y -intercept and the slope, and the y -intercept of the

I − V graph
Ans: (d)
2
⎛ V ⎞ ⎛ V ⎞ − I0 I
Solution: I = I 0 ⎜1 − ⎟ ⇒ I = I 0 ⎜1 − ⎟ ⇒ I = V + I0
⎝ V0 ⎠ ⎝ V0 ⎠ V0

− I0 − I0
Slope = ⇒ = V0
V0 − I0 V
V0

Intercept on y -axis = I 0

Q44. In the schematic figure given below, assume that the propagation delay of each logic gate
is tgate . +5 V

                                                                                
Head office  Branch office 
 
fiziks, H.No. 40 D, G.F, Jia Sarai,  Anand Institute of Mathematics, 
 
Near IIT, Hauz Khas, New Delhi‐16  28‐B/6, Jia Sarai, Near IIT 
 
Phone: 011‐26865455/+91‐9871145498 Hauz Khas, New Delhi‐16 
                                                   
                                             Website: www.physicsbyfiziks.com                                                                                          
                                                            Email: fiziks.physics@gmail.com                                                                     230 
fiziks
Institute for NET/JRF, GATE, IIT‐JAM, JEST, TIFR and GRE in PHYSICAL SCIENCES 
 
The propagation delay of the circuit will be maximum when the logic inputs A and B
make the transition
(a) ( 0,1) → (1,1) (b) (1,1) → ( 0,1)

(c) ( 0, 0 ) → (1,1) (d) ( 0, 0 ) → ( 0,1)

Ans: (d)
Solution:
Input Output
A B NOT OR AND OR
0 1 0 0 0 0
× ↓ ↓ ↓ 3t
1 1 0 1 1 1
1 1 0 1 1 1
× ↓ ↓ ↓ 3t
0 1 0 0 0 0
0 0 1 1 1 1
↓ × × × t
1 1 0 1 1 1
0 0 1 1 1 1
↓ ↓ ↓ ↓ 4t
0 1 0 0 0 0

                                                                                
Head office  Branch office 
 
fiziks, H.No. 40 D, G.F, Jia Sarai,  Anand Institute of Mathematics, 
 
Near IIT, Hauz Khas, New Delhi‐16  28‐B/6, Jia Sarai, Near IIT 
 
Phone: 011‐26865455/+91‐9871145498 Hauz Khas, New Delhi‐16 
                                                   
                                             Website: www.physicsbyfiziks.com                                                                                          
                                                            Email: fiziks.physics@gmail.com                                                                     231 
fiziks
Institute for NET/JRF, GATE, IIT‐JAM, JEST, TIFR and GRE in PHYSICAL SCIENCES 
 
Q45. Given the input voltage Vi , which of the following waveforms correctly represents the

output voltage V0 in the circuit shown below?

0.5 10 K

5K
Vi 0 Vi −
t
V0
0.5V +
5K 10 K
−0.5
0

3.0
(a) 2.5
2.0 t
V0 1.5
1.0
0.5
0.0
0

(b)
3.0
2.5
2.0
V0 1.5
1.0 t
0.5
0.0
0

                                                                                
Head office  Branch office 
 
fiziks, H.No. 40 D, G.F, Jia Sarai,  Anand Institute of Mathematics, 
 
Near IIT, Hauz Khas, New Delhi‐16  28‐B/6, Jia Sarai, Near IIT 
 
Phone: 011‐26865455/+91‐9871145498 Hauz Khas, New Delhi‐16 
                                                   
                                             Website: www.physicsbyfiziks.com                                                                                          
                                                            Email: fiziks.physics@gmail.com                                                                     232 
fiziks
Institute for NET/JRF, GATE, IIT‐JAM, JEST, TIFR and GRE in PHYSICAL SCIENCES 
 

3.0
2.5
(c)
2.0
V0 1.5
1.0 t
0.5
0.0
0

3.0
(d)
2.5
2.0 t
V0 1.5
1.0
0.5
0.0
0
Ans: (b)
⎛ 10 ⎞ 10 10
Solution: V0 = ⎜ 1 + ⎟ × × 0.5 − × Vi ⇒ V0 = 1 − 2Vi
⎝ 5 ⎠ 15 5
when Vi = 0 ⇒ V0 = 1V

when Vi = 0.1V ⇒ V0 = 0.8 V

when Vi = 0.5V ⇒ V0 = 0V

Q46. The decay constants f p of the heavy pseudo-scalar mesons, in the heavy quark limit, are

a
related to their masses m p by the relation f p = , where a is an empirical parameter
mp

to be determined. The values m p = 6400 ± 160 MeV and f p = 180 ± 15 MeV correspond

to uncorrelated measurements of a meson. The error on the estimate of a is


3 3 3 3
(a) 175 ( MeV ) 2 (b) 900 ( MeV ) 2 (c) 1200 ( MeV ) 2 (d) 2400 ( MeV ) 2

                                                                                
Head office  Branch office 
 
fiziks, H.No. 40 D, G.F, Jia Sarai,  Anand Institute of Mathematics, 
 
Near IIT, Hauz Khas, New Delhi‐16  28‐B/6, Jia Sarai, Near IIT 
 
Phone: 011‐26865455/+91‐9871145498 Hauz Khas, New Delhi‐16 
                                                   
                                             Website: www.physicsbyfiziks.com                                                                                          
                                                            Email: fiziks.physics@gmail.com                                                                     233 
fiziks
Institute for NET/JRF, GATE, IIT‐JAM, JEST, TIFR and GRE in PHYSICAL SCIENCES 
 
Ans: (c)
Solution: a = f p m1/p 2
2 2
⎛ ∂a ⎞ 2 ⎛ ∂a ⎞ 2 ∂a ∂a f
σ =⎜
2
⎟⎟ σ f p + ⎜⎜ ⎟⎟ σ m p ⇒ = m1/p 2 and = p1
⎜ ∂f p
a
⎝ ⎠ ⎝ ∂m p ⎠ ∂f p ∂m p
2m p2
1

f 2
σ ⎛σ f
2 ⎞
2
⎛ σ mp ⎞
2
⎡⎛ σ f ⎞
2
⎛ σ mp ⎞ ⎤
2

⇒ σ a2 = m pσ 2f p + p
σ m2 ⇒ =⎜ p +⎜ ⇒ σ a = a ⎢⎜ p ⎟⎟ +⎜ ⎥
⎟⎟ ⎜ 2m p ⎟⎟ ⎜ 2m p ⎟⎟ ⎥
a
4m p p
a ⎜ fp
2
⎝ ⎠ ⎝ ⎠ ⎢⎜⎝ f p ⎠ ⎝ ⎠⎦

∵ a = f p m1/p 2 = (180 MeV )( 6400 MeV ) = 180 × 80 ( MeV )


1/ 2 3/ 2

2 2
⎛ σ fp ⎞ ⎛ 15 ⎞ 2 −3
⎛ σm ⎞ ⎛ 160 ⎞ 2 −4
⎜⎜ ⎟⎟ = ⎜ ⎟ = 6.9 × 10 and ⎜ p
⎜ 2m p ⎟⎟ = ⎜ ⎟ = 1.56 × 10
⎝ fp ⎠ ⎝ 180 ⎠ ⎝ ⎠ ⎝ 2 × 6400 ⎠

( ) ( MeV )
1/ 2
σ a = 180 × 80 ( MeV )
3/ 2 1/ 2 3/ 2
⎡⎣6.9 × 10−3 + 1.56 × 10−4 ⎤⎦ = 180 × 80 × 7 × 10−3

⇒ σ a = 1204 ( MeV )
3/ 2

                                                                                
Head office  Branch office 
 
fiziks, H.No. 40 D, G.F, Jia Sarai,  Anand Institute of Mathematics, 
 
Near IIT, Hauz Khas, New Delhi‐16  28‐B/6, Jia Sarai, Near IIT 
 
Phone: 011‐26865455/+91‐9871145498 Hauz Khas, New Delhi‐16 
                                                   
                                             Website: www.physicsbyfiziks.com                                                                                          
                                                            Email: fiziks.physics@gmail.com                                                                     234 
fiziks
Institute for NET/JRF, GATE, IIT‐JAM, JEST, TIFR and GRE in PHYSICAL SCIENCES 
 
ATOMIC AND MOLECULAR PHYSICS
NET/JRF (JUNE-2011)

Q1. Consider the energy level diagram (as shown in the figure below) of a typical three level
ruby laser system with 1.6 × 1019 Chromium ions per cubic centimeter. All the atoms
excited by the 0.4 μm radiation decay rapidly to level E2 which has a lifetime τ = 3 ms.
E3
0.4 μ m

E2
0.7 μ m
E1
A. Assuming that there is no radiation of wavelength 0.7 μm present in the pumping
cycle and that the pumping rate is R atoms per cm3, the population density in the level N2
builds up as:
(a) N2 (t) = Rτ(et/τ –1) (b) N2 (t) = Rτ(1 – e-t/τ)

(c) N 2 (t ) =
Rt 2
τ
(1 − e ) −t / τ
(d) N2 (t) = R t

Ans: (b)
(
N 2 (t ) = Rτ 1 − e t / τ )
B. The minimum pump power required (per cubic centimeter) to bring the system to
transparency, i.e. zero gain, is
(a) 1.52 kW (b) 2.64 kW
(c) 0.76 kW (d) 1.32 kW
Ans: (c)
Solution: The Minimum Power required to achieve zero gain is
N hv N hc 1.6 × 1019 6.6 × 10 −34 × 3 × 10 8
P= = = × = 754 W cm −3
2 τ 2 λτ 2 −6
0.7 × 10 × 3 × 10 −3

P = 0.76kW per cubic centimeter

                                                                                
Head office  Branch office 
 
fiziks, H.No. 40 D, G.F, Jia Sarai,  Anand Institute of Mathematics, 
 
Near IIT, Hauz Khas, New Delhi‐16  28‐B/6, Jia Sarai, Near IIT 
 
Phone: 011‐26865455/+91‐9871145498 Hauz Khas, New Delhi‐16 
                                                   
                                             Website: www.physicsbyfiziks.com                                                                                          
                                                            Email: fiziks.physics@gmail.com                                                                     235 
fiziks
Institute for NET/JRF, GATE, IIT‐JAM, JEST, TIFR and GRE in PHYSICAL SCIENCES 
 
NET/JRF (DEC-2011)
Q2. Given that the ground state energy of the hydrogen atom is –13.6 eV, the ground state
energy of positronium (which is a bound state of an electron and a positron) is
(a) + 6.8 eV (b) – 6.8 eV (c) – 13.6 eV (d) – 27.2 eV
Ans: (b)
13.6
The energy expression for Positronium atom is E n = − (eV )
2n 2
− 13.6
For n = 1, E1 = (eV ) = −6.8eV , ∴ E1 = −6.8 eV
2
Q3. A laser operating at 500 nm is used to excite a molecule. If the Stokes line is observed at
770 cm-1, the approximate positions of the Stokes and the anti-Stokes lines are
(a) 481.5 nm and 520 nm (b) 481.5 nm and 500 nm
(c) 500 nm and 520 nm (d) 500 nm and 600 nm
Ans:
Solution: Given λ 0 = 500 nm = 5 × 10 −5 , v stoke = 770 cm −1 ∴ v 0 = 20,000 cm −1

Raman shift Δv = v 0 − v stoke = 19230 cm −1

Wave number of anti-stokes line is v ami − stoke = Δv + v0 = 39,230 cm-1

In wavelength term λ anti − stoke = 2.549 × 10 −7 = 254.9 nm and λ stoke = 12987 nm

Q4. If the hyperfine interaction in an atom is given by H = a S e ⋅ S p where S e and S p denote

the electron and proton spins, respectively, the splitting between the 3 S1 and 1 S 0 state is

(a) a= 2 / 2 (b) a= 2 (c) a= 2 / 2 (d) 2a= 2


Ans: (b)
G G G G G G G 1
Solution: Total spin is S = S e + S p ⇒ S 2 = S e2 + S p2 + 2 S e ⋅ S p ⇒ S e ⋅ S p = S 2 − S e2 − S p2
2
[ ]
G G
[ ]
⇒ H = aS e ⋅ S p = S 2 − S e2 − S p2 where S e2 = S p2 = S (S + 1)= 2 = = 2
a
2
3
4
a ⎛ 2 3 2 3 2 ⎞ a ⎡ 2 3 2⎤
⇒H = ⎜ S − = − = ⎟ = ⎢S − = ⎥
2⎝ 4 4 ⎠ 2⎣ 2 ⎦

                                                                                
Head office  Branch office 
 
fiziks, H.No. 40 D, G.F, Jia Sarai,  Anand Institute of Mathematics, 
 
Near IIT, Hauz Khas, New Delhi‐16  28‐B/6, Jia Sarai, Near IIT 
 
Phone: 011‐26865455/+91‐9871145498 Hauz Khas, New Delhi‐16 
                                                   
                                             Website: www.physicsbyfiziks.com                                                                                          
                                                            Email: fiziks.physics@gmail.com                                                                     236 
fiziks
Institute for NET/JRF, GATE, IIT‐JAM, JEST, TIFR and GRE in PHYSICAL SCIENCES 
 
For S1 : S = 1 ⇒ S = S (S + 1)= = 2= , For 1 S 0 : S = 0 ⇒ S 2 = S (S + 1)= 2 = 0= 2
3 2 2 2

a⎡ 3⎤ a
∴ H1 = ⎢ 2 − ⎥= 2 = = 2 for 3 S1
2⎣ 2⎦ 4

a⎡ 3⎤ −3 2
and H 2 = ⎢ 0 − ⎥= 2 = a= for 1 S 0
2⎣ 2⎦ 4

⎛1 3⎞
∴ The splitting between 3
S1 and 1 S 0 is ΔH = H 1 − H 2 = ⎜ + ⎟a= 2 = a= 2
⎝4 4⎠
Q5. The ratio of intensities of the D1 and D2 lines of sodium at high temperature is
(a) 1:1 (b) 2:3 (c) 1:3 (d) 1:2
Ans: (d)
Solution: The electronic transition for D2 and D1 line is
3
+1 2×
I ( D2 ) 2 J 2 + 1 2 4 2
D2 : 2 p 3 / 2 → 2 S1 / 2 , D21 : 2 p 3 / 2 → 2 S1 / 2 ∴ = = = =
I (D1 ) 2 J 1 + 1 1 2 1
2× +1
2
Q6. An atom of mass M can be excited to a state of mass (M + Δ ) by photon capture. The
frequency of a photon which can cause this transition is
Δc 2 Δc 2 Δ2 c 2 Δc 2
(a) (b) (c) (d) (Δ + 2M )
2h h 2Mh 2Mh
Ans: (d)
Solution: The conversation law of energy and Momentum give

[
Mc 2 + hν = (M + Δ ) c 4 + p 2 c 2
2
]1/ 2
and

c
= p

M 2 c 4 + h 2ν 2 + 2 Mc 2 hν = M 2 c 4 + Δ2 c 4 + 2 MΔc 4 + h 2ν 2 ⇒ 2Mc 2 hν = Δ2 c 4 + 2MΔc 4

⎛ Δ ⎞ Δc 2 ⎛ Δ ⎞ Δc 2
⇒ 2Mc 2 hν = 2MΔc 4 ⎜1 + ⎟ ⇒ ν = ⎜ 1 + ⎟ ⇒ ν = (Δ + 2M ) .
⎝ 2M ⎠ h ⎝ 2M ⎠ 2Mh

                                                                                
Head office  Branch office 
 
fiziks, H.No. 40 D, G.F, Jia Sarai,  Anand Institute of Mathematics, 
 
Near IIT, Hauz Khas, New Delhi‐16  28‐B/6, Jia Sarai, Near IIT 
 
Phone: 011‐26865455/+91‐9871145498 Hauz Khas, New Delhi‐16 
                                                   
                                             Website: www.physicsbyfiziks.com                                                                                          
                                                            Email: fiziks.physics@gmail.com                                                                     237 
fiziks
Institute for NET/JRF, GATE, IIT‐JAM, JEST, TIFR and GRE in PHYSICAL SCIENCES 
 
NET/JRF (JUNE-2012)
12
Q7. The first absorption spectrum of C16O is at 3.842 cm-1 while that of 13
C16O is at
3.673 cm-1. The ratio of their moments of inertia is
(a) 1.851 (b) 1.286 (c) 1.046 (d) 1.038
Ans: (c)
Solution: For 12
C 16 O : 281 = 3.842 cm −1 ⇒ B1 = 1.921 cm −1

For 13
2 B 2 = 3.673 cm −1
C 16 O : ⇒ B 2 = 1.8365 cm −1
h I 2 B1 1.921
Where, B = 2 ∴ = = = 1.046
8π IC I1 B2 1.8365
Q8. The spin-orbit interaction in an atom is given by H = a L.S, where L and S denote the
orbital and spin angular momenta, respectively, of the electron. The splitting between the
levels 2P3/2 and 2P1/2 is
3 2 1 2 5 2
(a) a= (b) a= (c) 3a= 2 2 (d) a=
2 2 2
Ans:
G G G G G
Solution: Given H = aL ⋅ S where J = L + S .
G G G G 1
2
a
[
∴ J 2 = L2 + S 2 2 L ⋅ S ⇒ L ⋅ S = J 2 − L2 − S 2 ∴ H = J 2 − L2 − S 2
2
] [ ]
For 3 P3 / 2 : S = 1 which gives S 2 = S (S + 1)= 2 = 2= 2

L = 1 which gives L2 = L(L + 1)= 2 = 2= 2

3 15 a ⎡15 ⎤
which gives J 2 = J ( J + 1)= 2 = = 2 ∴ H 1 = ⎢ − 2 − 2⎥ = 2 = − = 2
a
J=
2 4 2⎣4 ⎦ 8
1 3
For 2 P3 / 2 : S = which gives S 2 = S (S + 1)= 2 = = 2
2 4
L = 1 which gives L2 = L(L + 1)= 2 = 2= 2

1 3 a ⎡3 3 ⎤
J= which gives J 2 = J ( J + 1)= 2 = = 2 ∴ H 2 = ⎢ − 2 − = 2 ⎥ = − a= 2
2 4 2 ⎣4 4 ⎦
a ⎛ −1+ 8 ⎞ 2 7 2
∴ ΔH = H 1 − H 2 = − = 2 + a = 2 = ⎜ ⎟ a= ⇒ Δ H = a =
8 ⎝ 8 ⎠ 8
                                                                                
Head office  Branch office 
 
fiziks, H.No. 40 D, G.F, Jia Sarai,  Anand Institute of Mathematics, 
 
Near IIT, Hauz Khas, New Delhi‐16  28‐B/6, Jia Sarai, Near IIT 
 
Phone: 011‐26865455/+91‐9871145498 Hauz Khas, New Delhi‐16 
                                                   
                                             Website: www.physicsbyfiziks.com                                                                                          
                                                            Email: fiziks.physics@gmail.com                                                                     238 
fiziks
Institute for NET/JRF, GATE, IIT‐JAM, JEST, TIFR and GRE in PHYSICAL SCIENCES 
 
Q9. The spectral line corresponding to an atomic transition from J = 1 to J = 0 states splits in
a magnetic field of 1 kG into three components separated by 1.6 × 10-3 Ǻ. If the zero field
spectral line corresponds to 1849 Ǻ, what is the g-factor corresponding to the J = 1 state?
hc
(You may use ≈ 2 × 10 4 cm)
μ0
(a) 2 (b) 3/2 (c) 1 (d) 1/2
Ans: (c)
Solution: The Zeeman splitting is ΔE = ( gM J )(μ B B ) = gμ B B for MJ = +1

λ2
Given, Zeeman splitting separations Δλ = Δν = 1.6 × 10 −3 Å
c
c 3 × 10 8
⇒ Δν = Δλ = × 1.6 × 10 −1 ⇒ Δν = 0.1404 × 1010
λ2 (1849 × 10 ) −10 2

ΔE 6.625 × 10 −34 × 0.1404 × 1010


∴g = = = 1.00 ∴ g = 1 .0
μB B 9.27 × 10 − 24 × 0.1

NET/JRF (DEC-2012)
Q10. Consider the energy level diagram shown below, which corresponds to the molecular
nitrogen laser. 2
R τ 21

1
τ1
0
20 -3 -1
If the pump rate R is 10 atoms cm s and the decay routes are as shown with
τ 21 = 20 ns and τ 1 = 1μs , the equilibrium populations of states 2 and 1 are, respectively,
(a) 1014 cm-3 and 2 × 1012 cm-3 (b) 2× 1012 cm-3 and 1014 cm-3.
(c) 2× 1012 cm-3 and 2 × 10 6 cm-3 (d) zero and 1020 cm-3
Ans: (b)
dN 2 N dN1 N 2 N 1
Solution: = R − 2 and = − .
dt τ 21 dt τ 21 τ 1
                                                                                
Head office  Branch office 
 
fiziks, H.No. 40 D, G.F, Jia Sarai,  Anand Institute of Mathematics, 
 
Near IIT, Hauz Khas, New Delhi‐16  28‐B/6, Jia Sarai, Near IIT 
 
Phone: 011‐26865455/+91‐9871145498 Hauz Khas, New Delhi‐16 
                                                   
                                             Website: www.physicsbyfiziks.com                                                                                          
                                                            Email: fiziks.physics@gmail.com                                                                     239 
fiziks
Institute for NET/JRF, GATE, IIT‐JAM, JEST, TIFR and GRE in PHYSICAL SCIENCES 
 
dN dN1
Under equilibrium condition 2 = =0
dt dt
⇒ N 2 = τ 21 R = 20 × 20 × 10 −9 = 2 × 1012 cm −3

τ 1 N 2 10 −6 × 2 × 1012 cm −3
⇒ N1 = = = 1014 cm −3
τ 21 20 × 10 −9

Q11. Consider a hydrogen atom undergoing a 2 P → 1S transition. The lifetime tsp of the 2P
state for spontaneous emission is 1.6 ns and the energy difference between the levels is
10.2 eV. Assuming that the refractive index of the medium n0 = 1, the ratio of Einstein
coefficients for stimulated and spontaneous emission B21 (ω ) / A21 (ω ) is given by
(a) 0.683 × 1012 m3J-1s-1 (b) 0.146 × 10-12 Jsm-3 .
(c) 6.83 × 1012 m3J-1s-1 (d) 1.463 × 10-12 Jsm-3 .
Ans: (a)
B21 π 2c3 = 2π 2 c 3
Solution: n = 1 , ΔE = 10.2 eV and = = = 0.67 × 1012 .
A21 =ω n 3 3
(ΔE ) n0
3 3

Q12. Consider a He-Ne laser cavity consisting of two mirrors of reflectivities R1 = 1 and
R2 = 0.98. The mirrors are separated by a distance d = 20 cm and the medium in between
has a refractive index n0 = 1 and absorption coefficient α = 0. The values of the
separation between the modes δν and the width Δν p of each mode of the laser cavity

are:
(a) δν = 75kHz, Δν p = 24kHz (b) δν = 100kHz, Δν p = 100kHz

(c) δν = 750 MHz, Δν p = 2.4 MHz (d) δν = 2.4MHz, Δν p = 750 MHz

Ans: (a)
c
Solution: Mode separation δν = = 750 MHz
2dn0

where c = 3 × 10 8 m / sec , c = 20 × 10 −2 m and n0 = 1 .

                                                                                
Head office  Branch office 
 
fiziks, H.No. 40 D, G.F, Jia Sarai,  Anand Institute of Mathematics, 
 
Near IIT, Hauz Khas, New Delhi‐16  28‐B/6, Jia Sarai, Near IIT 
 
Phone: 011‐26865455/+91‐9871145498 Hauz Khas, New Delhi‐16 
                                                   
                                             Website: www.physicsbyfiziks.com                                                                                          
                                                            Email: fiziks.physics@gmail.com                                                                     240 
fiziks
Institute for NET/JRF, GATE, IIT‐JAM, JEST, TIFR and GRE in PHYSICAL SCIENCES 
 
1 2n 0 d
Width of each mode δν p = where t c = .
2πt c ⎛ 1 ⎞
c ln⎜⎜ ⎟
− 2αd ⎟
⎝ 1 2
R R e ⎠
Note: In this question there is no need to calculate δν p since in the given options there is

only one option with δν = 750 MHz and i.e option (a). You can calculate δν p without

calculator so use your common sense.

NET/JRF (JUNE-2013)
Q13. ( )
A muon μ − from cosmic rays is trapped by a proton to form a hydrogen-like atom.
Given that a muon is approximately 200 times heavier than an electron, the longest
wavelength of the spectral line (in the analogue of the Lyman series) of such an atom will
be
o o o o
(a) 5.62 A (b) 6.67 A (c) 3.75 A (d) 13.3 A
Ans: (b)
mμ m p
Solution: In case of muonic atom, the reduce mass is m ' = = 180me
mμ + m p

⎛ m' ⎞ E E
E n' = ⎜⎜ ⎟⎟ 21 = 180 21 where, E1 = −13.6eV
⎝ me ⎠ n n

For ground state of muonic atom n=1, E1' = 180 E1

For first excited state of muonic atom n=2, E 2' = 45 E1


The longest wavelength of the photon corresponds to the transition between first and ground state
of muonic atom.
The energy difference between first excited and ground state is
ΔE = E 2' − E1' = −135E1 = 1836eV = 2938 × 10 −19 J
In term of wavelength

hc hc 6.63 × 10 −34 × 3 × 10 8 o
ΔE = hv = ⇒ λ= = = 6.67 × 10 −10 m = 6.67 A
λ ΔE 2938 × 10 −19

                                                                                
Head office  Branch office 
 
fiziks, H.No. 40 D, G.F, Jia Sarai,  Anand Institute of Mathematics, 
 
Near IIT, Hauz Khas, New Delhi‐16  28‐B/6, Jia Sarai, Near IIT 
 
Phone: 011‐26865455/+91‐9871145498 Hauz Khas, New Delhi‐16 
                                                   
                                             Website: www.physicsbyfiziks.com                                                                                          
                                                            Email: fiziks.physics@gmail.com                                                                     241 
fiziks
Institute for NET/JRF, GATE, IIT‐JAM, JEST, TIFR and GRE in PHYSICAL SCIENCES 
 
Q14. Consider the hydrogen-deuterium molecule HD. If the mean distance between the two
atoms is 0.08 nm and the mass of the hydrogen atom is 938 MeV / c 2 , then the energy
difference ΔE between the two lowest rotational states is approximately
(a) 10 −1 eV (b) 10 −2 eV (c) 2 × 10 −2 eV (d) 10 −3 eV
Ans: (b)
h2
Solution: Rotational energy expression E = J ( J + 1) = BJ ( J + 1)
8π 2 I
Difference between two lowest energy levels is ΔE = 2 B
h2 =2
where B= = and I = μr 2
8π I 2
2I
MHMD M × 2M H 2 2
Here, μ = = H = MH , μ = × 938 MeV/c2
M H + M D M H + 2M H 3 3
1
= = 1.01 × 10 −34 J − sec = 1.01 × 10 −34 × × 1019 eV − sec = 6.3 × 10 −16 eV − sec
1.6

∴ ΔE =
=2
=
(6.3 × 10 −16 eV − sec) 2

, ΔE = 9.2 × 10 −3 eV ≅ 10 −2 eV
× 938 × 10 6 eV / c 2 × (0.08 × 10 −9 m )
I 2 2

3
Q15. The electronic energy levels in a hydrogen atom are given by E n = −13.6 / n 2 eV. If a

selective excitation to the n = 100 level is to be made using a laser, the maximum
allowed frequency line-width of the laser is
(a) 6.5 MHz (b) 6.5 GHz (c) 6.5 Hz (d) 6.5 kHz
Ans: (b)
2 × 13.6 2 × 13.6
Solution: E n = −13.6 / n 2 ⇒ ΔE n = 3
eV ⇒ hΔν = 3
× 1.6 × 10 −19 ⇒ Δν = 6.5 GHz .
n n

                                                                                
Head office  Branch office 
 
fiziks, H.No. 40 D, G.F, Jia Sarai,  Anand Institute of Mathematics, 
 
Near IIT, Hauz Khas, New Delhi‐16  28‐B/6, Jia Sarai, Near IIT 
 
Phone: 011‐26865455/+91‐9871145498 Hauz Khas, New Delhi‐16 
                                                   
                                             Website: www.physicsbyfiziks.com                                                                                          
                                                            Email: fiziks.physics@gmail.com                                                                     242 
fiziks
Institute for NET/JRF, GATE, IIT‐JAM, JEST, TIFR and GRE in PHYSICAL SCIENCES 
 
Q16. Consider the laser resonator cavity shown in the figure.
If I1 is the intensity of the radiation at R1 = 1 R2 = R
l
mirror M 1 and α is the gain coefficient of the
medium between the mirrors, then the energy
x
density of photons in the plane P at a
P
distance x from M 1 is M1 M2

(a) (I 1 / c )e −αx (b) (I 1 / c )eαx

(c) (I 1 / c )(e αx + e −αx ) (d) (I 1 / c )e 2αx


Ans: (c)

NET/JRF (DEC-2013)
Q17. A perturbation V pert = aL2 is added to the Hydrogen atom potential. The shift in the

energy level of the 2 P state, when the effects of spin are neglected up to second order in
a , is
3 2 4
(a) 0 (b) 2a= 2 + a 2 = 4 (c) 2a= 2 (d) a= 2 + a =
2
Ans: (c)
Solution: For 2 P state, L=1
The shift in the energy due to perturbation V pert = aL2 is

ΔE = aL( L + 1)= 2 = a1(1 + 1)= 2 = 2a= 2


Q18. A gas laser cavity has been designed to operate at λ = 0.5μ m with a cavity length of 1 m .
With this set-up, the frequency is found to be larger than the desired frequency by 100
Hz. The change in the effective length of the cavity required to retune the laser is
(a) − 0.334 × 10 −12 m (b) 0.334 × 10 −12 m (c) 0.167 × 10 −12 m (d) − 0.167 × 10 −12 m
Ans: (d)

                                                                                
Head office  Branch office 
 
fiziks, H.No. 40 D, G.F, Jia Sarai,  Anand Institute of Mathematics, 
 
Near IIT, Hauz Khas, New Delhi‐16  28‐B/6, Jia Sarai, Near IIT 
 
Phone: 011‐26865455/+91‐9871145498 Hauz Khas, New Delhi‐16 
                                                   
                                             Website: www.physicsbyfiziks.com                                                                                          
                                                            Email: fiziks.physics@gmail.com                                                                     243 
fiziks
Institute for NET/JRF, GATE, IIT‐JAM, JEST, TIFR and GRE in PHYSICAL SCIENCES 
 
Q19. The spectroscopic symbol for the ground state of 13 Al is 2 P1/ 2 . Under the action of a

strong magnetic field (when L − S coupling can be neglected) the ground state energy
level will split into
(a) 3 levels (b) 4 levels (c) 5 levels (d) 6 levels
Ans: (d)
Solution: In extremely strong magnetic field coupling between L-S breaks down. J is no longer a
valid quantum number.
The ground state energy level will split into (2S+1)(2L+1) = 6

NET/JRF (JUNE-2014)
Q20. A spectral line due to a transition from an electronic state p to an s state splits into three
Zeeman lines in the presence of a strong magnetic field. At intermediate field strengths
the number of spectral lines is
(a) 10 (b) 3 (c) 6 (d) 9
Ans: (a)
Solution: For p state: l = 1, s = 1/ 2 : j = 1/ 2 & 3 / 2 .
This gives two spectral terms 2P3/2 & 2P1/2
For s state: l= 0, s = 1/2: j = 1/2 : This gives spectral terms 2S1/2
The terms 2P3/2 and 2S1/2 corresponding to J = 3/2 & J = 1/2 will break into 2J+1 Zeeman
levels, which is 4 and 2 respectively.
For 2P3/2 Mj = - 3/2 -1/2 +1/2 +3/2
For 2S1/2 Mj = -1/2 +1/2
The selection rule is ∆MJ = 0, ± 1 (MJ = 0 → MJ = 0 If ∆J = 0)
∆MJ = 0 gives π component, ∆MJ = ±1 gives σ component
Number of π component = 2, Numbers of σ+ components = 2
Number of σ- components = 2
The terms 2P1/2 and 2S1/2 corresponding to J = 1/2 & J = 1/2 will break into 2J+1 Zeeman
levels, which is 2 & 2 respectively.

                                                                                
Head office  Branch office 
 
fiziks, H.No. 40 D, G.F, Jia Sarai,  Anand Institute of Mathematics, 
 
Near IIT, Hauz Khas, New Delhi‐16  28‐B/6, Jia Sarai, Near IIT 
 
Phone: 011‐26865455/+91‐9871145498 Hauz Khas, New Delhi‐16 
                                                   
                                             Website: www.physicsbyfiziks.com                                                                                          
                                                            Email: fiziks.physics@gmail.com                                                                     244 
fiziks
Institute for NET/JRF, GATE, IIT‐JAM, JEST, TIFR and GRE in PHYSICAL SCIENCES 
 
2 2
For P1/2 Mj = -1/2 +1/2, For S1/2 Mj = -1/2 +1/2
The selection rule is ∆MJ = 0, ± 1 (MJ = 0 → MJ = 0 If ∆J = 0)
∆MJ = 0 gives π component, ∆MJ = ±1 gives σ component
Number of π component = 2, Numbers of σ+ components = 1
Number of σ- components = 1
Thus, total number of Zeeman component = 10
Q21. A double slit interference experiment uses a laser emitting light of two adjacent
frequencies v1 and v 2 (v1 < v 2 ) . The minimum path difference between the interfering
beams for which the interference pattern disappears is
c c c c
(a) (b) (c) (d)
v 2 + v1 v 2 − v1 2(v 2 − v1 ) 2(v 2 + v1 )
Ans: (c)
Solution: The condition of maximum intensity for interfering laser beam is: d sin θ = nλ
⎛ 1⎞
The condition of dark intensity for interfering laser beam is: d sin θ = ⎜ n + ⎟λ
⎝ 2⎠
For interference pattern to vanish the minimum path difference should be λ/2
cΔλ c λ2
The spectral bandwidth of laser is defined as Δv = whereas =
λ2 Δv Δλ
For two closely spaced line of wavelength λ1 and λ2
c λ2 λλ 1 c c
= = 1 2 = = =
Δv Δλ λ1 − λ 2 1 1 c c ν 2 −ν 1
− −
λ21 λ1 λ21 λ1
Since, for interference pattern to vanish for two closely spaced line of wavelength λ1 and
λ c c
λ2, the minimum path difference should be = = =
2 2Δv 2(ν 2 − ν 1 )

Q22. How much does the total angular momentum quantum number J change in the transition
of Cr (3d 6 ) atom as it ionize to Cr 2+ (3d 4 )?
(a) Increases by 2 (b) Decreases by 2 (c) Decreases by 4 (d) Does not change

                                                                                
Head office  Branch office 
 
fiziks, H.No. 40 D, G.F, Jia Sarai,  Anand Institute of Mathematics, 
 
Near IIT, Hauz Khas, New Delhi‐16  28‐B/6, Jia Sarai, Near IIT 
 
Phone: 011‐26865455/+91‐9871145498 Hauz Khas, New Delhi‐16 
                                                   
                                             Website: www.physicsbyfiziks.com                                                                                          
                                                            Email: fiziks.physics@gmail.com                                                                     245 
fiziks
Institute for NET/JRF, GATE, IIT‐JAM, JEST, TIFR and GRE in PHYSICAL SCIENCES 
 
Ans: (c)
Solution: In Cr 3d 6 state( )
M L = −2 − 1 0 + 1 + 2

1
In this configuration, S = 4 × = 2 and L = 2
2
This is the case of more than half filled subshell, thus state with highest J value will
have the lowest energy.
The hight J - value is J = L + S = y

Thus, the ground state spectral term for this configuration is 5 D4 .

Now in Cr 2+ 3d 4 ( )
M L = −2 − 1 0 + 1 + 2

1
In this configuration, S = 4 × = 2 and L = 2
2
Since this is the case of less than half filled subshell, thus, state with lowest J value will
have the lowest energy.
The lowest J - value is J = L − 5 = 2 − 2 = 0 .
Thus the ground state spectral term for this configuration is the J - value decreases from
J = 4 to J = 0 .
Thus correct answer is option (c).

NET/JRF (DEC-2014)
Q23. An atomic transition 1 P → 1S in a magnetic field 1 Tesla shows Zeeman splitting. Given
that the Bohr magneton μ B = 9.27 × 10 −24 J / T , and the wavelength corresponding to the
transition is 250 nm, the separation in the Zeeman spectral lines is approximately
(a) 0.01 nm (b) 0.1 nm (c) 1.0 nm (d) 10 nm

                                                                                
Head office  Branch office 
 
fiziks, H.No. 40 D, G.F, Jia Sarai,  Anand Institute of Mathematics, 
 
Near IIT, Hauz Khas, New Delhi‐16  28‐B/6, Jia Sarai, Near IIT 
 
Phone: 011‐26865455/+91‐9871145498 Hauz Khas, New Delhi‐16 
                                                   
                                             Website: www.physicsbyfiziks.com                                                                                          
                                                            Email: fiziks.physics@gmail.com                                                                     246 
fiziks
Institute for NET/JRF, GATE, IIT‐JAM, JEST, TIFR and GRE in PHYSICAL SCIENCES 
 
Ans: (a)
Solution: This is the case of Normal Zeeman effect. The Zeeman separation in terms of
frequency is
μB B
Δν = where μ B is Bohr magneton
h
λ2 λ2 μB B
In terms of wavelength it is Δλ = Δν = ×
c c h

( 250 ×10 m ) × ( 9.27 ×10 J / T ) (1T ) ≅ 3 ×10


2
−9 −24

Δλ = −12
m ≅ 0.003 nm
( 3 ×10 m / s )( 6.625 ×10 Js )
8 −34

None of the answer is matching correctly. But best suitable answer is option (a)
Q24. If the leading anharmonic correction to the energy of n -th vibrational level of a diatomic
2
⎛ 1⎞
molecule is − xe ⎜ n + ⎟ =ω with xe = 0.001 , the total number of energy levels possible
⎝ 2⎠
is approximately
(a) 500 (b) 1000 (c) 250 (d) 750
Ans: (a)
2
⎛ 1⎞ ⎛ 1⎞
Solution: The energy of anharmonic oscillator is Ev = ⎜ v + ⎟ =ω − xe ⎜ v + ⎟ =ω v = vmax
⎝ 2⎠ ⎝ 2⎠

where v = 0, 1, 2, .....vmax is vibrational quantum number
E
dEv ⎛ 1⎞
Now, = 0 ⇒ =ω − 2 xe ⎜ vmax + ⎟ =ω = 0
dv v = vmax
⎝ 2⎠
r→
⎛ 1⎞ ⎛ 1⎞
⇒ =ω = 2 xe ⎜ vmax + ⎟ =ω ⇒ 1 = 2 xe ⎜ vmax + ⎟
⎝ 2⎠ ⎝ 2⎠
1 1 1 1
⇒ vmax = − ≅ = = 500
2 xe 2 2 xe 2 × 0.001

                                                                                
Head office  Branch office 
 
fiziks, H.No. 40 D, G.F, Jia Sarai,  Anand Institute of Mathematics, 
 
Near IIT, Hauz Khas, New Delhi‐16  28‐B/6, Jia Sarai, Near IIT 
 
Phone: 011‐26865455/+91‐9871145498 Hauz Khas, New Delhi‐16 
                                                   
                                             Website: www.physicsbyfiziks.com                                                                                          
                                                            Email: fiziks.physics@gmail.com                                                                     247 
fiziks
Institute for NET/JRF, GATE, IIT‐JAM, JEST, TIFR and GRE in PHYSICAL SCIENCES 
 
G G
Q25. The effective spin-spin interaction between the electron spin S e and the proton spin S p
G G
in the ground state of the Hydrogen atom is given by H ′ = aS e ⋅ S p . As a result of this

interaction, the energy levels split by an amount


1 2 3 2
(a) a= (b) 2a= 2 (c) a= 2 (d) a=
2 2
Ans: (c)
G G
Solution: The Hamiltonian is given as H = aSe ⋅ S p where Se and S p are electron

and proton spin. Sp


S
The coupling between Se and S p gives net resultant spin angular

momentum Se
G G G
S = Se + S p
G G G G 1
S 2 = Se2 + S p2 + 2Se ⋅ S p ⇒ Se ⋅ S p = S 2 − Se2 − S p2
2
( )
∴ H=
2
(
a 2
S − Se2 − S p2 )

where S 2 = S ( S + 1) = 2 , Se2 = Se ( Se + 1) = 2 , S p2 = S p ( S p + 1) = 2

1 1
Since Se = and S p = ⇒ S = 0, 1
2 2
For S = 0 (singlet state)
a⎛ 3 3⎞ 2 3 2
H1 = ⎜ 0 − − ⎟ = = − a= F =1
2⎝ 4 4⎠ 4 1 2
a=
4
For S = 1 (Triplet state)
12 s1/2 3
a⎛ 3 3 ⎞ 1 − a= 2
H 2 = ⎜ 2 − = 2 − =2 ⎟ = a= 2 4
2⎝ 4 4 ⎠ 4 F =0
1 2 3 2
∴ ΔH = H 2 − H1 = a= + a= ⇒ ΔH = a= 2
4 4

                                                                                
Head office  Branch office 
 
fiziks, H.No. 40 D, G.F, Jia Sarai,  Anand Institute of Mathematics, 
 
Near IIT, Hauz Khas, New Delhi‐16  28‐B/6, Jia Sarai, Near IIT 
 
Phone: 011‐26865455/+91‐9871145498 Hauz Khas, New Delhi‐16 
                                                   
                                             Website: www.physicsbyfiziks.com                                                                                          
                                                            Email: fiziks.physics@gmail.com                                                                     248 
fiziks
Institute for NET/JRF, GATE, IIT‐JAM, JEST, TIFR and GRE in PHYSICAL SCIENCES 
 
NET/JRF (JUNE-2015)
Q26. Of the following term symbols of the np 2 atomic configurations, 1 S0 ,3 P0 ,3 P1 ,3 P2 and
1
D2 which is the ground state?
(a) 3 P0 (b) 1 S0 (c) 3 P2 (d) 3 P1
Ans. (a)
Solution: According to Hund’s rules
(i) State with highest multiplicity has lowest energy
(ii) State with same multiplicity, the state with highest L will have lowest energy
(iii) State with same multiplicity and L value. The state with lowest J has lowest energy
(only if subshell is less than half filled) from the given states 1 S0 ,3 P0 ,3 P1 ,3 P21 D2

The 3 P0 will have the lowest energy


⎛ 1⎞
Q27. A diatomic molecule has vibrational states with energies Ev = =ω ⎜ v + ⎟ and rotational
⎝ 2⎠

states with energies E j = Bj ( j + 1) , where v and j are non-negative integers . consider

the transitions in which both the initial and final states are restricted to v ≤ 1 and j ≤ 2
and subject to the selection rules Δv = ±1 and Δj = ±1 . Then the largest allowed energy
of transition is
(a) =ω − 3B (b) =ω − B (c) =ω + 4 B (d) 2=ω + B
Ans. (c)
⎛ 1⎞
Solution: E = =ω ⎜ v + ⎟ + BJ ( J + 1)
⎝ 2⎠
For vibrational transition with Δv = ±1 and rotational transition with ΔJ = ± we get
ΔE = Einitial − Efinal = =ω + 2 B ( J + 1)

where J is lowest quantum number


according to question in rotational states two transition is possible one for J = 0 → J = 1
and second for J = 1 → J = 2 but second transition will have photon of higher energy
∴ For J = 1 → J = 2
ΔE = =ω + 2 B (1 + 1) = =ω + 4 B
                                                                                
Head office  Branch office 
 
fiziks, H.No. 40 D, G.F, Jia Sarai,  Anand Institute of Mathematics, 
 
Near IIT, Hauz Khas, New Delhi‐16  28‐B/6, Jia Sarai, Near IIT 
 
Phone: 011‐26865455/+91‐9871145498 Hauz Khas, New Delhi‐16 
                                                   
                                             Website: www.physicsbyfiziks.com                                                                                          
                                                            Email: fiziks.physics@gmail.com                                                                     249 
fiziks
Institute for NET/JRF, GATE, IIT‐JAM, JEST, TIFR and GRE in PHYSICAL SCIENCES 
 
NET/JRF (DEC-2015)
12 13
Q28. The LS configurations of the ground state of Mg , Al , 17Cl and 18
Ar are,
respectively,
(a) 3 S1 , 2 P1/ 2 , 2 P1/ 2 and 1 S0 (b) 3 S1 , 2 P3 / 2 , 2 P3 / 2 and 3 S1

(c) 1 S0 , 2 P1/ 2 , 2 P3 / 2 and 1 S0 (d) 1 S0 , 2 P3 / 2 , 2 P1/ 2 and 3 S1


Ans.: (c)
Solution: 12 Mg :1s 2 2s 2 2 p 6 2s 2

Ground state: 1 s0
13
Al :1s 2 2s 2 2 p 6 3s 2 3 p1
the terms are 2 p1/ 2 and 2 p3/ 2 . Since its less than half filled. Thus

Ground state: 2 p1/ 2


17
Cl :1s 2 2s 2 2 p 6 3s 2 3 p 5
The terms are 2 p1/ 2 and 2 p3/ 2 . Since it shell is more than half filled.

Thus Ground state: 2 p3/ 2


18
Ar :1s 2 2s 2 2 p 6 3s 2 3 p 6

Ground state: 1 s0

Q29. For a two level system, the population of atoms in the upper and lower levels are 3 × 1018
and 0.7 ×1018 , respectively. If the coefficient of stimulated emission is
3.0 × 105 m3 / W -s 3 and the energy density is 9.0 J / m3 -Hz , the rate of stimulated
emission will be
(a) 6.3 ×1016 s −1 (b) 4.1 × 1016 s −1 (c) 2.7 × 1016 s −1 (d) 1.8 × 1016 s −1
Ans.: None of the answer is matching.
Solution: The rate of stimulated emission is
dN 2
= N 2 B21 u ( w )
dt

                                                                                
Head office  Branch office 
 
fiziks, H.No. 40 D, G.F, Jia Sarai,  Anand Institute of Mathematics, 
 
Near IIT, Hauz Khas, New Delhi‐16  28‐B/6, Jia Sarai, Near IIT 
 
Phone: 011‐26865455/+91‐9871145498 Hauz Khas, New Delhi‐16 
                                                   
                                             Website: www.physicsbyfiziks.com                                                                                          
                                                            Email: fiziks.physics@gmail.com                                                                     250 
fiziks
Institute for NET/JRF, GATE, IIT‐JAM, JEST, TIFR and GRE in PHYSICAL SCIENCES 
 
3
m m3
where N 2 = 3 ×1018 , B21 = 3 × 105 = 3 × 10 5

W − s3 J − s2
J −s
and u ( w ) = 9.0 J / n3 − Hz = 9.0
m3
dN 2 m3 J −s
= 3 × 10 × 3 ×10
18 5
× 9 3 = 8.1× 1024 s −1
dt J −s 2
m
Q30. The first ionization potential of K is 4.34 eV , the electron affinity of CI is 3.82 eV
and the equilibrium separation of KCI is 0.3 nm . The required to dissociate
a KCI molecule into a K and a CI atom is
(a) 8.62 eV (b) 8.16 eV (c) 4.28 eV (d) 4.14 eV
Ans.: (c)
Solution: Energy required to dissociate kcl is kcl → k + + cl − ( )
V=
1 q1q2 ⎛
= ⎜ 9 × 10 9 Nm ⎞
2 1.6 ×10−19 c (
= 7.7 × 10−19 J = 4.79 eV
)
2 ⎟
4π ∈0 r12 ⎝
−9
c ⎠ 0.3 × 10 m

The band dissociation energy is the energy required to dissociate a molecule into its
component atom kcl → k + cl
To find the energy required to dissociate kcl into k and cl , we must add an electron to
the k + ion, which releases the atomic potassium ionization energy. Remove one electron
from cl − ion which requires the atomic chlorine electron affinity energy
Given ionization energy of k = Eie = 4.34 eV

Electron affinity of cl = Eai = 3.82 eV


Thus the energy gained in the transformation from ion to atom is
= Eie − Eai = 4.34 − 3.82 = 0.52 eV

Thus dissociation energy = 4.79 − 0.52 = 4.27 eV

                                                                                
Head office  Branch office 
 
fiziks, H.No. 40 D, G.F, Jia Sarai,  Anand Institute of Mathematics, 
 
Near IIT, Hauz Khas, New Delhi‐16  28‐B/6, Jia Sarai, Near IIT 
 
Phone: 011‐26865455/+91‐9871145498 Hauz Khas, New Delhi‐16 
                                                   
                                             Website: www.physicsbyfiziks.com                                                                                          
                                                            Email: fiziks.physics@gmail.com                                                                     251 
fiziks
Institute for NET/JRF, GATE, IIT‐JAM, JEST, TIFR and GRE in PHYSICAL SCIENCES 
 
NET/JRF (JUNE-2016)
Q31. The ground state electronic configuration of 22 Ti is [ Ar ] 3d 2 4s 2 . Which state, in the

standard spectroscopic notations, is not possible in this configuration?


(a) 1 F3 (b) 1 S0 (c) 1 D2 (d) 3 P0
Ans: (a)
Solution: The spectroscopic term for d 2 are obtained as
l1 = 2, l2 = 2 ⇒ L = 4,3, 2,1, 0

1 1
s1 = , s2 = ⇒ S = 0,1
2 2
Now S = 0, L = 4 ⇒ J = 4, 1G4

S = 0, L = 2 ⇒ J = 2, 1 D2

S = 0, L = 0 ⇒ J = 0, 1S0

S = 1, L = 3 ⇒ J = 4,3, 2, 3
G4 ,3 G3 ,3 G2

S = 1, L = 1 ⇒ J = 2,1, 0, 3 P2 , 3 P1 ,3 P0

Thus 1 F3 is not possible spectroscopic term of [ Ar ] 3d 2 4S 2

Q32. In a normal Zeeman Effect experiment using a magnetic field of strength 0.3 T , the
splitting between the components of a 660 nm spectral line is
(a) 12 pm (b) 10 pm (c) 8 pm (d) 6 pm
Ans: (d)

λ 2 eB ( 660 ×10 )
2
−9
1.6 × 10−19 × 0.3
Solution: Δλ = = ×
c 4π m 3 ×108 4π × 9.1×10−31
Δλ = 6.09 ×10−12 m
Δλ = 6 pm

                                                                                
Head office  Branch office 
 
fiziks, H.No. 40 D, G.F, Jia Sarai,  Anand Institute of Mathematics, 
 
Near IIT, Hauz Khas, New Delhi‐16  28‐B/6, Jia Sarai, Near IIT 
 
Phone: 011‐26865455/+91‐9871145498 Hauz Khas, New Delhi‐16 
                                                   
                                             Website: www.physicsbyfiziks.com                                                                                          
                                                            Email: fiziks.physics@gmail.com                                                                     252 
fiziks
Institute for NET/JRF, GATE, IIT‐JAM, JEST, TIFR and GRE in PHYSICAL SCIENCES 
 
Q33. The separation between the energy levels of a two-level atom is 2 eV . Suppose that

4 ×1020 atoms are in the ground state and 7 ×1020 atoms are pumped into the excited state
just before lasing starts. How much energy will be released in a single laser pulse?
(a) 24.6 J (b) 22.4 J (c) 98 J (d) 48 J
Ans: (d)
Solution: N 2 − N1 = 3 × 1020
Energy of laser pulse
⎛ N − N1 ⎞ 3 ×10 20
E =⎜ 2 ⎟ hν = × 2 ×1.6 ×10−19 J ⇒ E = 48 J
⎝ 2 ⎠ 2

                                                                                
Head office  Branch office 
 
fiziks, H.No. 40 D, G.F, Jia Sarai,  Anand Institute of Mathematics, 
 
Near IIT, Hauz Khas, New Delhi‐16  28‐B/6, Jia Sarai, Near IIT 
 
Phone: 011‐26865455/+91‐9871145498 Hauz Khas, New Delhi‐16 
                                                   
                                             Website: www.physicsbyfiziks.com                                                                                          
                                                            Email: fiziks.physics@gmail.com                                                                     253 
fiziks
Institute for NET/JRF, GATE, IIT‐JAM, JEST, TIFR and GRE in PHYSICAL SCIENCES 
 
CONDENSED MATTER PHYSICS

NET/JRF (JUNE-2011)

Q1. A narrow beam of X-rays with wavelength 1.5 Ǻ is reflected from an ionic crystal with an
fcc lattice structure with a density of 3.32 gcm-3. The molecular weight is 108 AMU
(1AMU = 1.66 × 10-24 g)
A. The lattice constant is
(a) 6.00 Ǻ (b) 4.56 Ǻ (c) 4.00 Ǻ (d) 2.56 Ǻ
Ans: (b)
Solution: Given ne f f = 4 , M = 108 kg, ρ = 3.32gm cm -3 = 3320 kgm-3,

N A = 6.023 × 10 +26 atoms kmd

ne f f × M 4 × 108
a3 = = = 6.00 × 10 −30 m 3 = 6.00 × 10 −10 = 6.00 A 0
NA × ρ 6.023 × 10 × 3320
26

B. The sine of the angle corresponding to (111) reflection is


3 3 1 1
(a) (b) (c) (d)
4 8 4 8
Ans: (b)
Solution: According to Bragg’s law
λ a a
2d sin θ = λ , sin θ = where d = = for (111) plane
2d h +k +l
2 2 2
3

3λ 3 × 1 .5 A 0 3 ×3 3
∴ sin θ = = = = .
2a 2 × 6A 0
2 × 62 × 8

                                                                                
Head office  Branch office 
 
fiziks, H.No. 40 D, G.F, Jia Sarai,  Anand Institute of Mathematics, 
 
Near IIT, Hauz Khas, New Delhi‐16  28‐B/6, Jia Sarai, Near IIT 
 
Phone: 011‐26865455/+91‐9871145498 Hauz Khas, New Delhi‐16 
                                                   
                                             Website: www.physicsbyfiziks.com                                                                                          
                                                            Email: fiziks.physics@gmail.com                                                                     254 
fiziks
Institute for NET/JRF, GATE, IIT‐JAM, JEST, TIFR and GRE in PHYSICAL SCIENCES 
 
Q2. A flux quantum (fluxoid) is approximately equal to 2×10-7 gauss-cm2. A type II
superconductor is placed in a small magnetic field, which is then slowly increased till the
field starts penetrating the superconductor. The strength of the field at this point is
2
× 105 gausss.
π
A. The penetrating depth of this superconductor is
(a) 100 Ǻ (b) 10Ǻ (c) 1000Ǻ (d) 314Ǻ
Ans: (a)
Solution: Given Fluxoid (φ )0 = 2 × 10 −7 gauss –cm2

2
First Critical field (H c1 ) = × 10 5 gauss
π
The relation between first critical field and penetration depth is
φ0 φ0 2.10 −7
H c1 = ∴ λ 2
= = = 10 −12 cm 2 ⇒ λ = 10 −6 cm = 100 A 0
πλ2 πH c1 π × 2 × 10 5
π
B. The applied field is further increased till superconductivity is completely destroyed.
8
The strength of the field is now × 105 gauss. The correlation length of the
π
superconductor is
(a) 20 Ǻ (b) 200 Ǻ (c) 628 Ǻ (d) 2000 Ǻ
Ans:
8
Solution: Given second critical field (H c 2 ) = × 10 5 gauss. The relation between second critical
π
φ0
field and correlation length is H c 2 = .
πε 2
φ0 2 × 10 −7 1
∴ε = 2
= = × 10 −12 cm 2
πH c 2 8
π × × 10 5 4
π
1 1 100
⇒ε = × 10 −6 cm = × 10 −6 cm = × 10 −10 m = 50 A 0
2 2 2

                                                                                
Head office  Branch office 
 
fiziks, H.No. 40 D, G.F, Jia Sarai,  Anand Institute of Mathematics, 
 
Near IIT, Hauz Khas, New Delhi‐16  28‐B/6, Jia Sarai, Near IIT 
 
Phone: 011‐26865455/+91‐9871145498 Hauz Khas, New Delhi‐16 
                                                   
                                             Website: www.physicsbyfiziks.com                                                                                          
                                                            Email: fiziks.physics@gmail.com                                                                     255 
fiziks
Institute for NET/JRF, GATE, IIT‐JAM, JEST, TIFR and GRE in PHYSICAL SCIENCES 
 
Q3. The two dimensional lattice of graphene is an arrangement of Carbon atoms forming a
honeycomb lattice of lattice spacing a, as shown below. The Carbon atoms occupy the
vertices.

1
c1

1
(A). The Wigner-Seitz cell has an area of d2

1
c2

1
3 2

1
a1

1 1
(a) 2a 2 (b) a d1 b1
2

1
a2 b2
a
2 3 3 2
(c) 6 3a (d) a
2
Ans:
Area of normal cell
Solution: Area of Wigner-Seitz cell =
Ne f f

1
Where Neff = effective number of lattice paints = 6 × =2
3

3 3 2 3 3 2 3 3 2
Area of normal cell = a ∴ Area of wigner-Seitz cell = a /2 = a
2 2 4
(B). The Bravais lattice for this array is a
G G
(a) rectangular lattice with basis vectors d1 and d 2
G G
(b) rectangular lattice with basis vectors c1 and c2
G G
(c) hexagonal lattice with basis vectors a1 and a2
G G
(d) hexagonal lattice with basis vectors b1 and b2

Ans: (c)
G G
Solution: The Bravaiss lattice for this array is the Hexagonal lattice with basis vectors a1 and a 2
NET/JRF (DEC-2011)
Q4. The potential of a diatomic molecule as a function of the distance r between the atoms is
a b
given by V (r ) = − 6
+ 12 . The value of the potential at equilibrium separation between
r r
the atoms is:
(a) − 4a 2 / b (b) − 2a 2 / b (c) − a 2 / 2b (d) − a 2 / 4b
                                                                                
Head office  Branch office 
 
fiziks, H.No. 40 D, G.F, Jia Sarai,  Anand Institute of Mathematics, 
 
Near IIT, Hauz Khas, New Delhi‐16  28‐B/6, Jia Sarai, Near IIT 
 
Phone: 011‐26865455/+91‐9871145498 Hauz Khas, New Delhi‐16 
                                                   
                                             Website: www.physicsbyfiziks.com                                                                                          
                                                            Email: fiziks.physics@gmail.com                                                                     256 
fiziks
Institute for NET/JRF, GATE, IIT‐JAM, JEST, TIFR and GRE in PHYSICAL SCIENCES 
 
Ans: (d)
a b dV (r )
Solution: Given V (r ) = − 6
+ 12 . At equilibrium radius, =0
r r dr r =r0

dV (r ) 6a 12b r 13 12b 2b 2b
∴ = + 7 − 13 = 0 ⇒ 07 = = ⇒ r06 =
dr r0 r0 r0 6a a a

a b a2 a2 − a2
∴ The value of potential at equilibrium is V (r0 ) = − + = − + = .
r06 r012 2b 4b 4b
Q5. If the number density of a free electron gas in three dimensions is increased eight times,
its Fermi temperature will
(a) increase by a factor of 4 (b) decrease by a factor of 4
(c) increase by a factor of 8 (d) decrease by a factor of 8
Ans: (a)

Solution: The relation between Fermi energy and electron density is E F =


=2
2m
3π 2 n ( ) 2/3
.

⇒E =
=2
'
F
2m
3π 2 × 8n(2/3
= 4EF . )
Q6. The excitations of a three-dimensional solid are bosonic in nature with their frequency ω
and wave-number k are related by ω ∝ k2 in the large wavelength limit. If the chemical
potential is zero, the behaviour of the specific heat of the system at low temperature is
proportional to
(a) T1/2 (b) T (c) T3/2 (d) T3
Ans: (c)
Solution: If the dispersion relation is ω ∝ k s in large wavelength. Than the specific heat is
C v ∝ T 3 / s . Given ω ∝ k 2 ∴ C v ∝ T 3 / 2

                                                                                
Head office  Branch office 
 
fiziks, H.No. 40 D, G.F, Jia Sarai,  Anand Institute of Mathematics, 
 
Near IIT, Hauz Khas, New Delhi‐16  28‐B/6, Jia Sarai, Near IIT 
 
Phone: 011‐26865455/+91‐9871145498 Hauz Khas, New Delhi‐16 
                                                   
                                             Website: www.physicsbyfiziks.com                                                                                          
                                                            Email: fiziks.physics@gmail.com                                                                     257 
fiziks
Institute for NET/JRF, GATE, IIT‐JAM, JEST, TIFR and GRE in PHYSICAL SCIENCES 
 
NET/JRF (JUNE-2012)

Q7. Consider a system of non-interacting particles in d dimensional obeying the dispersion


relation ε = Ak s , where ε is the energy k is the wavevector, s is an integer and A
constant. The density of states, N(ε), is proportional to
s d d s
−1 −1 +1 +1
(a) ε d (b) ε s
(c) ε s
(d) ε d
Ans: (b)
Q8. The experimentally measured transmission spectra of metal, insulator and semiconductor
thin films are shown in the figure. It can be inferred that I, II and III correspond,
respectively, to

Ι ΙΙ

Transmission (%)
(a) insulator, semiconductor and metal
(b) semiconductor, metal and insulator
(c) metal, semiconductor and insulator
(d) insulator, metal and semiconductor ΙΙΙ

Ans: (a) 300 1000


Wavelength (nm)

Q9. The energy required to create a lattice vacancy in a crystal is equal to 1 eV. The ratio of the
number densities of vacancies n(1200 K)/n(300 K) when the crystal is at equilibrium at
1200 K and 300 K, respectively, is approximately
(a) exp(− 30 ) (b) exp(− 15) (c) exp(15) (d) exp(30)
Ans: (d)
Solution: The equation for number density of vacancies n = Ne − E / 2 k BT where E: Energy required
to form vacancies, N: density of lattice sites
+E ⎡ 1 1⎤
⎢ − ⎥ n(1200 K )
E ⎡ 1 1 ⎤ E ⎡ 1 ⎤
n1 e − E / 2 k BT1 2k T T ⎢ − ⎥ ⎢ ⎥
∴ = − E / 2 k BT2 = e B ⎣ 2 1 ⎦ = e 2 k B ⎣ 300 1200 ⎦ = e 2 k B ⎣ 400 ⎦ = e 30
n2 e n(300 K )

                                                                                
Head office  Branch office 
 
fiziks, H.No. 40 D, G.F, Jia Sarai,  Anand Institute of Mathematics, 
 
Near IIT, Hauz Khas, New Delhi‐16  28‐B/6, Jia Sarai, Near IIT 
 
Phone: 011‐26865455/+91‐9871145498 Hauz Khas, New Delhi‐16 
                                                   
                                             Website: www.physicsbyfiziks.com                                                                                          
                                                            Email: fiziks.physics@gmail.com                                                                     258 
fiziks
Institute for NET/JRF, GATE, IIT‐JAM, JEST, TIFR and GRE in PHYSICAL SCIENCES 
 
Q10. The dispersion relation of phonons in a solid is given by
ω 2 (k ) = ω 02 (3 − cos k x a − cos k y a − cos k z a )
The velocity of the phonons at large wavelength is
(a) ω 0 a / 3 (b) ω 0 a (c) 3ω 0 a (d) ω 0 a / 2
Ans: (d)
Solution: For large λ , (k x a, k y a, k z a ) are small.

⎡ ⎛ k 2a2 ⎞ ⎛ kya
2 2
⎞ ⎛ k z2 a 2 ⎞⎤ ω 02 a 2 2
ω (k ) = ω ⎢3 − ⎜⎜1 − x
2 2
0
2
⎟ − ⎜1 −
⎟ ⎜ 2
⎟ − ⎜1 −
⎟ ⎜ 2
⎟⎟⎥ =
2
(
k x + k y2 + k z2 )
⎣⎢ ⎝ ⎠ ⎝ ⎠ ⎝ ⎠⎦⎥

ω 02 a 2 ω0 a dω ω 0 a
ω 2 (k ) == k2 ⇒ ω = k ⇒ vg = = .
2 2 dk 2

NET/JRF (DEC-2012)

Q11. The free energy difference between the superconducting and the normal states of a
2 β 4
material is given by ΔF = FS − FN = α ψ + ψ , where ψ is an order parameter and α
2
and β are constants such that α > 0 in the normal and α < 0 in the superconducting state,
while β > 0 always. The minimum value of ΔF is

(a) − α 2 / β (b) − α 2 / 2β

(c) − 3α 2 / 2β (d) − 5α 2 / 2β
Ans: (b)
2 β 4 β
Solution: ΔF = α ψ + ψ = αx 2 + x 4 where x = ψ .
2 2

(ΔF ) = 2αx + 2βx 3 = 0 ⇒ 2αx + 2βx 3 = 0 ⇒ x 2 = − α since x = 0 is trivial.
∂x β

⎛ β ⎞ α 2
Thus (ΔF )min ⎜ αx 2 + x 4 ⎟ =− .
⎝ 2 ⎠ x2 =−α 2β
β

                                                                                
Head office  Branch office 
 
fiziks, H.No. 40 D, G.F, Jia Sarai,  Anand Institute of Mathematics, 
 
Near IIT, Hauz Khas, New Delhi‐16  28‐B/6, Jia Sarai, Near IIT 
 
Phone: 011‐26865455/+91‐9871145498 Hauz Khas, New Delhi‐16 
                                                   
                                             Website: www.physicsbyfiziks.com                                                                                          
                                                            Email: fiziks.physics@gmail.com                                                                     259 
fiziks
Institute for NET/JRF, GATE, IIT‐JAM, JEST, TIFR and GRE in PHYSICAL SCIENCES 
 
Q12. Consider a one-dimensional Ising model with N spins, at very low temperatures when
almost all spins are aligned parallel to each other. There will be a few spin flips with each
flip costing an energy 2J. In a configuration with r spin flips, the energy of the system is
N
E = - NJ + 2rJ and the number of configuration is C r ; r varies from 0 to N. The
partition function is
N
⎛ J ⎞
(a) ⎜⎜ ⎟⎟ (b) e − NJ / k BT
⎝ k BT ⎠
N N
⎛ J ⎞ ⎛ J ⎞
(c) ⎜⎜ sinh ⎟⎟ (d) ⎜⎜ cosh ⎟⎟
⎝ k B T ⎠ ⎝ k B T ⎠
Ans: (d)
Solution: Let us consider only three energy levels, E r = −2 J + 2rJ i.e. E 0 = −2 J , E1 = 0 and

E2 = 2J .

Q2 =
(Ce
2
0
− β E0
+ 2C1e − βE1 + 2C 2 e − βE2 ) = (e β 2J
+ 2e 0 + e β 2 J
=
) (
e βJ + e βJ )2

2
4 4
∑r =0
2
Cr

2
⎛ e βJ + e βJ ⎞
Q2 = ⎜⎜ ⎟⎟ = (cosh β J )2 ⇒ (cosh β J )2 ⇒ Q N = (cosh β J ) N .
⎝ 2 ⎠
Q13. A magnetic field sensor based on the Hall Effect is to be fabricated by implanting As into
a Si film of thickness 1 μm. The specifications require a magnetic field sensitivity of
500 mV/Tesla at an excitation current of 1 mA. The implantation dose is to be adjusted
such that the average carrier density, after activation, is
(a) 1.25 × 1026 m-3 (b) 1.25 × 1022 m-3
(c) 4.1 × 1021 m-3 (d) 4.1 × 1020 m-3
Ans: (b)
IB 10 3 1 V
Solution: n = = −6 −19
× −3
= 1.25 × 10 22 m −3 where H = 500 × 10 −3 V / T .
teVH 10 × 1.6 × 10 500 × 10 B

                                                                                
Head office  Branch office 
 
fiziks, H.No. 40 D, G.F, Jia Sarai,  Anand Institute of Mathematics, 
 
Near IIT, Hauz Khas, New Delhi‐16  28‐B/6, Jia Sarai, Near IIT 
 
Phone: 011‐26865455/+91‐9871145498 Hauz Khas, New Delhi‐16 
                                                   
                                             Website: www.physicsbyfiziks.com                                                                                          
                                                            Email: fiziks.physics@gmail.com                                                                     260 
fiziks
Institute for NET/JRF, GATE, IIT‐JAM, JEST, TIFR and GRE in PHYSICAL SCIENCES 
 
Q14. In a band structure calculation, the dispersion relation for electrons is found to be
ε k = β (cos k x a + cos k y a + cos k z a ),
where β is a constant and a is the lattice constant. The effective mass at the boundary of
the first Brillouin zone is
2= 2 4= 2 =2 =2
(a) (b) (c) (d)
5β a 2 5β a 2 2 βa 2 3β a 2
Ans: (d)
=2
Solution: ε k = β (cos k x a + cos k y a + cos k z a ), Effective mass m ∗ =
⎛ d 2ε k ⎞
⎜ 2 ⎟
⎜ d k ⎟
⎝ ⎠
π π π
Brilliouin zone boundary is at k x = ± ,ky = ± , kz = ± .
a a a
⎛ d 2ε k ⎞ =2
Hence ⎜⎜ 2 ⎟⎟ = 3β a 2 ⇒ m ∗ = .
⎝ d k ⎠ π ,π ,π 3β a 2
a a a

Q15. The radius of the Fermi sphere of free electrons in a monovalent metal with an fcc
structure, in which the volume of the unit cell is a3, is
1/ 3 1/ 3 1/ 3
⎛ 12π 2 ⎞ ⎛ 3π 2 ⎞ ⎛π 2 ⎞ 1
(a) ⎜⎜ 3 ⎟⎟ (b) ⎜⎜ 3 ⎟⎟ (c) ⎜⎜ 3 ⎟⎟ (d)
⎝ a ⎠ ⎝ a ⎠ ⎝a ⎠ a

Ans: (a)
1/ 3
⎛ 3π 2 N ⎞
Solution: Radius of Fermi sphere is k F = ⎜⎜ ⎟⎟ .
⎝ V ⎠
1/ 3
N 4 ⎛ 12π 2 ⎞
For fcc solid = 3 ⇒ k F = ⎜⎜ 3 ⎟⎟ .
V a ⎝ a ⎠

                                                                                
Head office  Branch office 
 
fiziks, H.No. 40 D, G.F, Jia Sarai,  Anand Institute of Mathematics, 
 
Near IIT, Hauz Khas, New Delhi‐16  28‐B/6, Jia Sarai, Near IIT 
 
Phone: 011‐26865455/+91‐9871145498 Hauz Khas, New Delhi‐16 
                                                   
                                             Website: www.physicsbyfiziks.com                                                                                          
                                                            Email: fiziks.physics@gmail.com                                                                     261 
fiziks
Institute for NET/JRF, GATE, IIT‐JAM, JEST, TIFR and GRE in PHYSICAL SCIENCES 
 
NET/JRF (JUNE-2013)

Q16. Using the frequency-dependent Drude formula, what is the effective kinetic inductance of
a metallic wire that is to be used as a transmission line? [In the following, the electron
mass is m , density of electrons is n , and the length and cross-sectional area of the wire
A and A respectively.]
(a) mA / (ne 2 A ) (b) zero

(c) mA / (ne 2 A) (d) m A / ne 2 A 2( )


Ans: (c)
Q17. The phonon dispersion for the following one-dimensional diatomic lattice with masses
M 1 and M 2 (as shown in the figure)
K

M1 M2 M1 M2
is given by

⎛ 1 1 ⎞⎡ 4M 1 M 2 2 ⎛ qa ⎞

ω 2 (q ) = K ⎜⎜ + ⎟⎟ ⎢1 ± 1 − sin ⎜ ⎟ ⎥
⎝ M 1 M 2 ⎠ ⎢⎣ (M 1 + M 2 )2 ⎝ 2 ⎠ ⎥⎦

where a is the lattice parameter and K is the spring constant. The velocity of sound is
K (M 1 + M 2 ) K
(a) a (b) a
2M 1 M 2 2(M 1 + M 2 )

K (M 1 + M 2 ) KM 1 M 2
(c) a (d) a
2(M 1 + M 2 )
3
M 1M 2

Ans: (b)
Solution: For small value of q (i.e. long wavelength approximation limit), we have

⎛ qa ⎞ qa
sin ⎜ ⎟ ≈
⎝ 2 ⎠ 2

                                                                                
Head office  Branch office 
 
fiziks, H.No. 40 D, G.F, Jia Sarai,  Anand Institute of Mathematics, 
 
Near IIT, Hauz Khas, New Delhi‐16  28‐B/6, Jia Sarai, Near IIT 
 
Phone: 011‐26865455/+91‐9871145498 Hauz Khas, New Delhi‐16 
                                                   
                                             Website: www.physicsbyfiziks.com                                                                                          
                                                            Email: fiziks.physics@gmail.com                                                                     262 
fiziks
Institute for NET/JRF, GATE, IIT‐JAM, JEST, TIFR and GRE in PHYSICAL SCIENCES 
 
⎛ 1 1 ⎞⎡ 4M 1 M 2 2 ⎛ qa ⎞

∴ ω 2 (q ) = Κ ⎜⎜ + ⎟⎟ ⎢1 ± 1 − sin ⎜ ⎟ ⎥
⎝ M 1 M 2 ⎠ ⎢⎣ (M 1 + M 2 )2 ⎝ 2 ⎠ ⎥⎦

1 ⎞⎡ 4M 1 M 2 ⎛ qa ⎞ ⎤
2
⎛ 1
⇒ ω (q ) = Κ ⎜⎜
2
+ ⎟⎟ ⎢1 ± 1 − ⎜ ⎟ ⎥
⎝ M 1 M 2 ⎠ ⎢⎣ (M 1 + M 2 )2 ⎝ 2 ⎠ ⎥⎦

⎛ 1 1 ⎞ ⎡ ⎛⎜ 1 4 M 1 M 2 q 2 a 2 ⎞⎤
⇒ ω (q ) = Κ ⎜⎜
2
+ ⎟⎟ ⎢1 ± 1 − ×
⎜ 2 (M + M )2 4 ⎟⎥

M
⎝ 1 M 2 ⎠⎣ ⎢ ⎝ 1 2 ⎠⎦⎥

⎛ 1 1 ⎞ ⎡ ⎛⎜ M 1 M 2 q 2 a 2 ⎞⎤
⇒ ω 2 (q ) = Κ ⎜⎜ + ⎟⎟ ⎢1 ± 1 −
⎜ (M + M )2 2 ⎟ ⎥

M
⎝ 1 M 2 ⎠⎢ ⎣ ⎝ 1 2 ⎠⎥⎦

⎛ 1 1 ⎞ ⎡ ⎛⎜ M 1 M 2 q 2 a 2 ⎞⎤
For Acoustical branch: ω 2 (q ) = Κ ⎜⎜ + ⎟⎟ ⎢1 − 1 −

⎟⎥

⎝ M 1 M 2 ⎠ ⎢⎣ ⎝ (M 1 + M 2 ) 2 ⎠⎥⎦
2

⎛ M + M2 ⎞⎛ M 1 M 2 q 2 a 2 ⎞ Κa 2
⇒ ω 2 (q ) = Κ ⎜⎜ 1 ⎟⎟⎜ ⎟ =
⎜ (M + M )2 2 ⎟ 2(M + M ) q
2

⎝ M 1M 2 ⎠⎝ 1 2 ⎠ 1 2

Κ
∴ ω (q ) = aq
2(M 1 + M 2 )

ω Κ
Velocity of sound is v g = = a
q 2(M 1 + M 2 )

Q18. The electron dispersion relation for a one-dimensional metal is given by


⎡ ka 1 2 ⎤
ε k = 2ε 0 ⎢sin 2 − sin ka ⎥
⎣ 2 6 ⎦
where k is the momentum, a is the lattice constant, ε 0 is a constant having dimensions of

energy and ka ≤ π . If the average number of electrons per atom in the conduction band

is 1/3, then the Fermi energy is


(a) ε 0 / 4 (b) ε 0 (c) 2ε 0 / 3 (d) 5ε 0 / 3
Ans: (a)

                                                                                
Head office  Branch office 
 
fiziks, H.No. 40 D, G.F, Jia Sarai,  Anand Institute of Mathematics, 
 
Near IIT, Hauz Khas, New Delhi‐16  28‐B/6, Jia Sarai, Near IIT 
 
Phone: 011‐26865455/+91‐9871145498 Hauz Khas, New Delhi‐16 
                                                   
                                             Website: www.physicsbyfiziks.com                                                                                          
                                                            Email: fiziks.physics@gmail.com                                                                     263 
fiziks
Institute for NET/JRF, GATE, IIT‐JAM, JEST, TIFR and GRE in PHYSICAL SCIENCES 
 
Q19. If the energy dispersion of a two-dimensional electron system is E = u=k where u is the
velocity and k is the momentum, then the density of states D(E ) depends on the energy
as
(a) 1 / E (b) E (c) E (d) constant
Ans: (c)
Solution: In two dimensional system, the number of allowed k-states in range k and k + dk is
2
⎛ L ⎞
g (k )dk = ⎜ ⎟ 2πkdk .
⎝ 2π ⎠
E dE
Given dispersion relation is E = u=k ∴ k = ⇒ dk =
u= u=
2 2
⎛ L ⎞ E dE ⎛ L ⎞ 2π
∴ g (E )dE = ⎜ ⎟ 2π × × =⎜ ⎟ EdE
⎝ 2π ⎠ ⎝ 2π ⎠ (u= )
2
u= u=

g (E )dE 1 L2
⇒ ρ (E ) = = E.
dE (u= )2 2π
NET/JRF (DEC-2013)
Q20. The physical phenomenon that cannot be used for memory storage applications is
(a) large variation in magnetoresistance as a function of applied magnetic field
(b) variation in magnetization of a ferromagnet as a function of applied magnetic field
(c) variation in polarization of a ferroelectric as a function of applied electric field
(d) variation in resistance of a metal as a function of applied electric field

Ans: (d)

Q21. The energy of an electron in a band as a function of its wave vector k is given
by E (k ) = E 0 − B (cos k x a + cos k y a + cos k z a ) , where E 0 B and a are constants. The

effective mass of the electron near the bottom of the band is


2= 2 =2 =2 =2
(a) (b) (c) (d)
3Ba 2 3Ba 2 2Ba 2 Ba 2

                                                                                
Head office  Branch office 
 
fiziks, H.No. 40 D, G.F, Jia Sarai,  Anand Institute of Mathematics, 
 
Near IIT, Hauz Khas, New Delhi‐16  28‐B/6, Jia Sarai, Near IIT 
 
Phone: 011‐26865455/+91‐9871145498 Hauz Khas, New Delhi‐16 
                                                   
                                             Website: www.physicsbyfiziks.com                                                                                          
                                                            Email: fiziks.physics@gmail.com                                                                     264 
fiziks
Institute for NET/JRF, GATE, IIT‐JAM, JEST, TIFR and GRE in PHYSICAL SCIENCES 
 
Ans: (d)
Solution: Near the bottom of the band the k → 0

cos k x a ≈ 1 −
1
(k x a )2 , cos k y a ≈ 1 −
1
(k y a )2 , 1
(k z a )2
cos k z a ≈ 1 −
2 2 2
⎛ 1 1 1 2⎞
E ( k ) = E0 − B ( cos k x a + cos k y a + cos k z a ) = E0 − B ⎜ 1 − ( k x a ) + 1 − ( k y a ) + 1 − ( k z a ) ⎟
2 2

⎝ 2 2 2 ⎠
⎛ 1 2⎞ 1
= E0 − B ⎜ 3 − a 2 ( k x + k x + k x ) ⎟ = E0 − 3B − Ba 2 k 2
⎝ 2 ⎠ 2

=2 =2
Effective mass of the electron is m * = =
d 2 E / dk 2 Ba 2

Q22. A DC voltage V is applied across a Josephson junction between two superconductors


with a phase difference φ0 . If I 0 and k are constants that depend on the properties of the
junction, the current flowing through it has the form
⎛ 2eVt ⎞ ⎛ 2eVt ⎞
(a) I 0 sin ⎜ + φ0 ⎟ (b) kV sin ⎜ + φ0 ⎟
⎝ = ⎠ ⎝ = ⎠
(c) kV sin φ 0 (d) I 0 sin φ 0 + kV
Ans: (a)
Q23. A uniform linear monoatomic chain is modeled by a spring-mass system of masses m
separated by nearest neighbour distance a and spring constant mω 02 . The dispersion
relation for this system is
⎛ ⎛ ka ⎞ ⎞ ⎛ ka ⎞
(a) ω (k ) = 2ω 0 ⎜⎜1 − cos⎜ ⎟ ⎟⎟ (b) ω (k ) = 2ω 0 sin 2 ⎜ ⎟
⎝ ⎝ 2 ⎠⎠ ⎝ 2 ⎠

⎛ ka ⎞ ⎛ ka ⎞
(c) ω (k ) = 2ω 0 sin ⎜ ⎟ (d) ω (k ) = 2ω 0 tan ⎜ ⎟
⎝ 2 ⎠ ⎝ 2⎠
Ans: (c)
Solution: The dispersion relation for uniform linear mono-atomic chain of atoms is
⎛ ka ⎞
ω (k ) = 2ω 0 sin ⎜ ⎟
⎝ 2 ⎠
                                                                                
Head office  Branch office 
 
fiziks, H.No. 40 D, G.F, Jia Sarai,  Anand Institute of Mathematics, 
 
Near IIT, Hauz Khas, New Delhi‐16  28‐B/6, Jia Sarai, Near IIT 
 
Phone: 011‐26865455/+91‐9871145498 Hauz Khas, New Delhi‐16 
                                                   
                                             Website: www.physicsbyfiziks.com                                                                                          
                                                            Email: fiziks.physics@gmail.com                                                                     265 
fiziks
Institute for NET/JRF, GATE, IIT‐JAM, JEST, TIFR and GRE in PHYSICAL SCIENCES 
 
NET/JRF (JUNE-2014)
Q24. The pressure of a nonrelativistic free Fermi gas in three-dimensions depends, at T = 0 ,
on the density of fermions n as
(a) n 5 / 3 (b) n1 / 3 (c) n 2 / 3 (d) n 4 / 3
Ans: (a)
Solution: The Fermi energy in three dimension is defined as
2/3
= 2 ⎛ 3π 2 N ⎞
EF = ⎜
2m ⎜⎝ V ⎟⎠
⎟ =
=2
2m
(
3π 2 n )2/3

Where, n is the electron concentration or density of free Fermi gas.


The total energy of free Fermi gas in 3D is
2/3
3 3 = 2 ⎛ 3π 2 N ⎞
E = NE F = N × ⎜ ⎟
5 5 2m ⎜⎝ V ⎟⎠

The pressure of a nonrelativistic free Fermi gas is defined as


⎛ ∂E ⎞
p F = −⎜
3
⎟ =− N×
=2
(3π 2 N ) × ⎜ − ⎟V −5 / 3
2/3 ⎛ 2⎞
⎝ ∂V ⎠ N 5 2m ⎝ 3⎠

2
5
2
= nEF = n ×
5
=2
2m
(3π n ) =
2 2/3 2 =2
5 2m
(3π 2 ) n 5 / 3
2/3

Q25. Consider an electron in b.c.c. lattice with lattice constant a . A single particle
GG
G
wavefunction that satisfies the Bloch theorem will have the form f (r ) exp ik .r , with ( )
G
f (r ) being

⎡ 2π ⎤ ⎡ 2π ⎤ ⎡ 2π ⎤
(a) 1 + cos ⎢ ( x + y − z )⎥ + cos ⎢ (− x + y + z )⎥ + cos ⎢ ( x − y + z )⎥
⎣ a ⎦ ⎣ a ⎦ ⎣ a ⎦
⎡ 2π ⎤ ⎡ 2π ⎤ ⎡ 2π ⎤
(b) 1 + cos ⎢ ( x + y )⎥ + cos ⎢ ( y + z )⎥ + cos ⎢ ( z + x )⎥
⎣ a ⎦ ⎣ a ⎦ ⎣ a ⎦
⎡π ⎤ ⎡π ⎤ ⎡π ⎤
(c) 1 + cos ⎢ ( x + y )⎥ + cos ⎢ ( y + z )⎥ + cos ⎢ ( z + x )⎥
⎣a ⎦ ⎣a ⎦ ⎣a ⎦
⎡π ⎤ ⎡π ⎤ ⎡π ⎤
(d) 1 + cos ⎢ ( x + y − z )⎥ + cos ⎢ (− x + y + z )⎥ + cos ⎢ ( x − y + z )⎥
⎣a ⎦ ⎣a ⎦ ⎣a ⎦
                                                                                
Head office  Branch office 
 
fiziks, H.No. 40 D, G.F, Jia Sarai,  Anand Institute of Mathematics, 
 
Near IIT, Hauz Khas, New Delhi‐16  28‐B/6, Jia Sarai, Near IIT 
 
Phone: 011‐26865455/+91‐9871145498 Hauz Khas, New Delhi‐16 
                                                   
                                             Website: www.physicsbyfiziks.com                                                                                          
                                                            Email: fiziks.physics@gmail.com                                                                     266 
fiziks
Institute for NET/JRF, GATE, IIT‐JAM, JEST, TIFR and GRE in PHYSICAL SCIENCES 
 
Ans: (b)
Solution: The primitive translational vector for BCC is
G a
( G a
) G a
(
a ' = − iˆ + ˆj + kˆ , b ' = iˆ − ˆj + kˆ , b ' = iˆ + ˆj − kˆ
2 2 2
) ( )
Bloch function defined as
G G GG
G GG
ψ k (r ) = u k (r )e ik .r = f (r )e ik .r
G
Here f (r ) is atomic wavefunction, which has the periodicity of the lattice i.e.
G G
u k (r + a ) = u k (r )
Given Bloch function
G ⎡ 2π ⎤ ⎡ 2π ⎤ ⎡ 2π ⎤
f (r ) = 1 + cos ⎢ ( x + y )⎥ + cos ⎢ ( y + z )⎥ + cos ⎢ ( x + z )⎥
⎣ a ⎦ ⎣ a ⎦ ⎣ a ⎦
G G ⎡ 2π ⎛ a a ⎞⎤ ⎡ 2π ⎛ a a ⎞⎤ ⎡ 2π ⎛ a a ⎞⎤
f (r + a ' ) = 1 + cos ⎢ ⎜ x + y − + ⎟⎥ + cos ⎢ ⎜ y + z + + ⎟⎥ + cos ⎢ ⎜ z + x + − ⎟⎥
⎣ a ⎝ 2 2 ⎠⎦ ⎣ a ⎝ 2 2 ⎠⎦ ⎣ a ⎝ 2 2 ⎠⎦

G G ⎡ 2π ⎤ ⎡ 2π ⎤ ⎡ 2π ⎤
f (r + a ' ) = 1 + cos ⎢ ( x + y )⎥ + cos ⎢ ( y + z ) + 2π ⎥ + cos ⎢ ( z + x )⎥
⎣ a ⎦ ⎣ a ⎦ ⎣ a ⎦

G G ⎡ 2π ⎤ ⎡ 2π ⎤ ⎡ 2π ⎤ G
f (r + a ' ) = 1 + cos ⎢ ( x + y )⎥ + cos ⎢ ( y + z )⎥ + cos ⎢ ( z + x )⎥ = f (r )
⎣ a ⎦ ⎣ a ⎦ ⎣ a ⎦
G G G
f (r + a ' ) = f (r )
Similarly
G G G G G G
f (r + b ' ) = f (r ) and f ( r + c ' ) = f ( r )
Other functions do not satisfy the periodicity

                                                                                
Head office  Branch office 
 
fiziks, H.No. 40 D, G.F, Jia Sarai,  Anand Institute of Mathematics, 
 
Near IIT, Hauz Khas, New Delhi‐16  28‐B/6, Jia Sarai, Near IIT 
 
Phone: 011‐26865455/+91‐9871145498 Hauz Khas, New Delhi‐16 
                                                   
                                             Website: www.physicsbyfiziks.com                                                                                          
                                                            Email: fiziks.physics@gmail.com                                                                     267 
fiziks
Institute for NET/JRF, GATE, IIT‐JAM, JEST, TIFR and GRE in PHYSICAL SCIENCES 
 
Q26. The dispersion relation for electrons in an f.c.c. crystal is given, in the tight binding
approximation, by
⎡ kxa kya kya k a k a k a⎤
ε (k ) = −4ε 0 ⎢cos cos + cos cos z + cos z cos x ⎥
⎣ 2 2 2 2 2 2 ⎦

where a is the lattice constant and ε 0 is a constant with the dimension of energy. The x -

⎛π ⎞
component of the velocity of the electron at ⎜ , 0, 0 ⎟ is
⎝a ⎠
(a) − 2ε 0 a / = (b) 2ε 0 a / = (c) − 4ε 0 a / = (d) 4ε 0 a / =

Ans: (d)
Solution: Group velocity of electron in dispersive medium is expressed as

G 1 dε 1 ⎡ dε ˆ dε ˆ dε ⎤ G G G
v= = ⎢ i+ j+ kˆ ⎥ = v x iˆ + v y ˆj + v z kˆ
= dk = ⎢⎣ dk x dk y dk z ⎦⎥

⎡⎛ k x a kya k a k a⎞ ⎛ k a kya kya k a⎞ ⎤


⎢⎜⎜ sin cos + cos z sin x ⎟⎟iˆ + ⎜⎜ cos x sin + sin cos z ⎟⎟ ˆj + ⎥
G 2βa ⎢⎝ 2 2 2 2 ⎠ ⎝ 2 2 2 2 ⎠ ⎥
v=
= ⎢ ⎛ kza kya kxa kza ⎞ ˆ

⎢ ⎜⎜ sin cos + cos sin ⎟ ⎥
⎢⎣ 2 2 2 2 ⎟k ⎥⎦
⎝ ⎠

⎛π ⎞
At ⎜ , 0, 0 ⎟
⎝a ⎠

G 2 β a ⎡⎛ π π⎞ ⎛ π ⎞ ⎛ π⎞ ⎤
v= ⎢⎜ sin cos 0 + cos 0 sin ⎟iˆ + ⎜ cos sin 0 + sin 0 cos 0 ⎟ ˆj + ⎜ cos 0 sin 0 + sin 0 cos ⎟kˆ ⎥
= ⎣⎝ 2 2⎠ ⎝ 2 ⎠ ⎝ 2⎠ ⎦
G 4βa ˆ
v=
=
[ ] [ G G
]
G
i + 0 ˆj + 0kˆ = 0iˆ + 0 ˆj + 0kˆ = v x iˆ + v y ˆj + v z kˆ

G 4βa G G
vx = , v y = 0, vz = 0
=
4βa
The x - component of velocity is v x =
=

                                                                                
Head office  Branch office 
 
fiziks, H.No. 40 D, G.F, Jia Sarai,  Anand Institute of Mathematics, 
 
Near IIT, Hauz Khas, New Delhi‐16  28‐B/6, Jia Sarai, Near IIT 
 
Phone: 011‐26865455/+91‐9871145498 Hauz Khas, New Delhi‐16 
                                                   
                                             Website: www.physicsbyfiziks.com                                                                                          
                                                            Email: fiziks.physics@gmail.com                                                                     268 
fiziks
Institute for NET/JRF, GATE, IIT‐JAM, JEST, TIFR and GRE in PHYSICAL SCIENCES 
 
NET/JRF (JUNE-2014)
Q27. When laser light of wavelength λ falls on a metal scale with 1 mm engravings at a
grazing angle of incidence, it is diffracted to form a vertical chain of diffraction spots on
a screen kept perpendicular to the scale. If the wavelength of the laser is increased by 200
nm, the angle of the first-order diffraction spot changes from 5 0 to
(a) 6.60 0 (b) 5.14 0 (c) 5.018 0 (d) 5.210
Ans: (c)
Solution: The condition of maxima peak in grating is
b sin θ = mλ ; m = 0,1, 2,3,....
where b is the width of slit or width of engraving, whereas ‘ m ’ is the order of
diffraction and θ is the angle of diffraction
For 1st order diffraction: b sin θ = λ (i)
When wavelength of incident light increased to ( λ + 200) nm , let’s assume the 1st order

peak appears at θ ′ ∴ b sin θ ′ = λ + 200 (ii)


Subtracting equation (i) from equation (ii), we get
b sin θ − b sin θ ′ = 200
200 200 × 10−9
⇒ sin θ ′ − sin θ = ⇒ sin θ ′ = + sin θ
b 10 −3
⇒ sin θ ′ = 2 × 10 −4 + sin θ = 2 × 10−4 + sin 50 = 2 × 10 −4 + 0.087196 = 0.08736
⇒ θ ′ = sin −1 ( 0.08736) ⇒ θ ′ = 5.010
Q28. Consider the crystal structure of sodium chloride which is modeled as a set of touching
spheres. Each sodium atom has a radius r1 and each chlorine atom has a radius r2 . The
centres of the spheres from a simple cubic lattice. The packing fraction of this system is
⎡⎛ r 3
⎞ ⎛ r2 ⎞
3
⎤ 2π r13 + r23
(a) π ⎢⎜⎜ 1 ⎟⎟ + ⎜⎜ ⎟⎟ ⎥ (b)
⎢⎣⎝ r1 + r2 ⎠ ⎝ r1 + r2 ⎠ ⎥⎦ 3 (r1 + r2 )3

r13 + r23 r13 + r23


(c) (d) π
(r1 + r2 )3 2(r1 + r2 )
3

                                                                                
Head office  Branch office 
 
fiziks, H.No. 40 D, G.F, Jia Sarai,  Anand Institute of Mathematics, 
 
Near IIT, Hauz Khas, New Delhi‐16  28‐B/6, Jia Sarai, Near IIT 
 
Phone: 011‐26865455/+91‐9871145498 Hauz Khas, New Delhi‐16 
                                                   
                                             Website: www.physicsbyfiziks.com                                                                                          
                                                            Email: fiziks.physics@gmail.com                                                                     269 
fiziks
Institute for NET/JRF, GATE, IIT‐JAM, JEST, TIFR and GRE in PHYSICAL SCIENCES 
 
Ans: (b)
Solution: This question can only be solved by solving each option by assuming r1 = r2 and
π
comparing result with the packing fraction of simple cubic which is .
6
⎡⎛ r ⎞3 ⎛ r ⎞3 ⎤ ⎡ ⎛ 1 ⎞3 ⎛ 1 ⎞3 ⎤ π
Option (a): π ⎢⎜ 1
⎟ +⎜
2
⎟ ⎥ = π ⎢⎜ ⎟ + ⎜ ⎟ ⎥ =
⎢⎣⎝ r1 + r2 ⎠ ⎝ r1 + r2 ⎠ ⎥⎦ ⎣⎢⎝ 2 ⎠ ⎝ 2 ⎠ ⎦⎥ 4
2π r13 + r23 2π 2r 3 2π 1 π
Option (b): = × = × =
3 ( r1 + r2 )3 3 8r 3 3 4 6
r13 + r23 2r 3 1
Option (c): = =
( r1 + r2 )
3
8r 3 4
π r13 + r23 2r 3 π
Option (d): =π =
2 ( r1 + r2 )3 2 × 8r 3
8

Thus, correct option is (b)


Q29. Consider two crystalline solids, one of which has a simple cubic structure, and the other
has a tetragonal structure. The effective spring constant between atoms in the c -direction
is half the effective spring constant between atoms in the a and b directions. At low
temperatures, the behaviour of the lattice contribution to the specific heat will depend as
a function of temperature T as
(a) T 2 for the tetragonal solid, but as T 3 for the simple cubic solid
(b) T for the tetragonal solid, and as T 3 for the simple cubic solid
(c) T for both solids
(d) T 3 for both solids
Ans: (d)
Solution: The specific heat of solid in three dimensions is proportional to T 3 and it is
independent of crystal structure.
In 3D : CV ∝ T 3

In 2D : CV ∝ T 2

In 1D : CV ∝ T

                                                                                
Head office  Branch office 
 
fiziks, H.No. 40 D, G.F, Jia Sarai,  Anand Institute of Mathematics, 
 
Near IIT, Hauz Khas, New Delhi‐16  28‐B/6, Jia Sarai, Near IIT 
 
Phone: 011‐26865455/+91‐9871145498 Hauz Khas, New Delhi‐16 
                                                   
                                             Website: www.physicsbyfiziks.com                                                                                          
                                                            Email: fiziks.physics@gmail.com                                                                     270 
fiziks
Institute for NET/JRF, GATE, IIT‐JAM, JEST, TIFR and GRE in PHYSICAL SCIENCES 
 
G
Q30. A superconducting ring carries a steady current in the presence of a magnetic field B
normal to the plane of the ring. Identify the incorrect statement.
(a) The flux passing through the superconductor is quantized in units of hc / e
(b) The current and the magnetic field in the superconductor are time independent.
G G G G
(c) The current density j and B are related by the equation ∇ × j + Λ2 B = 0 , where Λ
is a constant
(d) The superconductor shows an energy gap which is proportional to the transition
temperature of the superconductor
Ans: (a)
Solution: The flux quantization in superconducting ring is φ = nφo

hc h
where φo = in CGS units and φo = in MKS units.
2e 2e

                                                                                
Head office  Branch office 
 
fiziks, H.No. 40 D, G.F, Jia Sarai,  Anand Institute of Mathematics, 
 
Near IIT, Hauz Khas, New Delhi‐16  28‐B/6, Jia Sarai, Near IIT 
 
Phone: 011‐26865455/+91‐9871145498 Hauz Khas, New Delhi‐16 
                                                   
                                             Website: www.physicsbyfiziks.com                                                                                          
                                                            Email: fiziks.physics@gmail.com                                                                     271 
fiziks
Institute for NET/JRF, GATE, IIT‐JAM, JEST, TIFR and GRE in PHYSICAL SCIENCES 
 
NET/JRF (DEC-2014)
Q31. The Miller indices of a plane passing through the three points having coordinates (0, 0, 1)

(1, 0, 0) ⎛⎜ 1 , 1 , 1 ⎞⎟ are
⎝2 2 4⎠
(a) (212) (b) (111) (c) (121) (d) (211)
Ans: (a)
Q32. The plot of specifies heat versus temperature across the superconducting transition
temperature (Tc ) is most appropriately represented by
(a) (b)

Cp Cp

TC T TC T

(c) (d)

Cp Cp

TC T TC T
Ans: (a)
⎛ Δ ⎞
−⎜ ⎟
Solution: CV ∝ e ⎝ 2kT ⎠

                                                                                
Head office  Branch office 
 
fiziks, H.No. 40 D, G.F, Jia Sarai,  Anand Institute of Mathematics, 
 
Near IIT, Hauz Khas, New Delhi‐16  28‐B/6, Jia Sarai, Near IIT 
 
Phone: 011‐26865455/+91‐9871145498 Hauz Khas, New Delhi‐16 
                                                   
                                             Website: www.physicsbyfiziks.com                                                                                          
                                                            Email: fiziks.physics@gmail.com                                                                     272 
fiziks
Institute for NET/JRF, GATE, IIT‐JAM, JEST, TIFR and GRE in PHYSICAL SCIENCES 
 
Q33. The energy ε k for band electrons as a function of the wave vector k in the first Brillouin

⎛ π π⎞
zone ⎜ − ≤ k ≤ ⎟ of a one dimensional monatomic lattice is shown as ( a is lattice
⎝ a a⎠
εk
constant)

k
− π /a O π /a

The variation of the group velocity vg is most appropriately represented by

(a) vg (b) vg

− π /a k − π /a k
O π /a O π /a

(c) (d)
vg vg

− π /a k − π /a k
O π /a O π /a

Ans: (b)
⎛ ⎛ ka ⎞ ⎞
Solution: E = ⎜ E0 − γβ ⎜ cos ⎟ ⎟
⎝ ⎝ π ⎠⎠
1 dE ka
Vg = = γβ sin
= dk π

                                                                                
Head office  Branch office 
 
fiziks, H.No. 40 D, G.F, Jia Sarai,  Anand Institute of Mathematics, 
 
Near IIT, Hauz Khas, New Delhi‐16  28‐B/6, Jia Sarai, Near IIT 
 
Phone: 011‐26865455/+91‐9871145498 Hauz Khas, New Delhi‐16 
                                                   
                                             Website: www.physicsbyfiziks.com                                                                                          
                                                            Email: fiziks.physics@gmail.com                                                                     273 
fiziks
Institute for NET/JRF, GATE, IIT‐JAM, JEST, TIFR and GRE in PHYSICAL SCIENCES 
 
Q34. For Nickel the number density is 8 × 10 atoms / cm 3 and electronic configuration is
23

1s 2 2 s 2 2 p 6 3s 2 3 p 6 3d 8 4s 2 . The value of the saturation magnetization of Nickel in its

ferromagnetic state is _____________ × 10 9 A / m .

(Given the value of Bohr magneton μ B = 9.21 × 10 −21 Am 2 )


Ans: 4.42
Solution: Saturation magnetization: All magnetic dipoles in a solid material are mutually aligned
to the external field.
M S = (Magnetic dipole moment) μ B N ,

0.6 × 9.21× 10−21 ( Am 2 ) N (For N i :magnetic dipole moment = 0.6

Fe :2.22, For Cu :1.2 )


ρ NA
N= = 8 × 1029 / m3 μ B = 9.21×10−21 A / m (given)
AN

M S = 0.6 × 9.21×10−21 × 8 ×1029 = 4.42 × 109 A / m

NET/JRF (JUNE-2015)
Q35. X -ray of wavelength λ = a is reflected from the (111) plane of a simple cubic lattice. If

the lattice constant is a , the corresponding Bragg angle (in radian) is


π π π π
(a) (b) (c) (d)
6 4 3 8
Ans. (c)
Solution: According to Bragg’s Law 2d sin θ = λ
a a a
Where d = = = for (iii) plane
h +k +l
2 2 2
1+1+1 3

λ a 3 ⎛ 3⎞ π
⇒ sin θ = = = ⇒ θ = sin −1 ⎜⎜ ⎟⎟ =
2d a 2 ⎝ 2 ⎠ 3

3

                                                                                
Head office  Branch office 
 
fiziks, H.No. 40 D, G.F, Jia Sarai,  Anand Institute of Mathematics, 
 
Near IIT, Hauz Khas, New Delhi‐16  28‐B/6, Jia Sarai, Near IIT 
 
Phone: 011‐26865455/+91‐9871145498 Hauz Khas, New Delhi‐16 
                                                   
                                             Website: www.physicsbyfiziks.com                                                                                          
                                                            Email: fiziks.physics@gmail.com                                                                     274 
fiziks
Institute for NET/JRF, GATE, IIT‐JAM, JEST, TIFR and GRE in PHYSICAL SCIENCES 
 
Q36. The critical magnetic fields of a superconductor at temperatures 4 K and 8 K are
11 mA / m and 5.5 mA / m respectively. The transition temperature is approximately
(a) 8.4 K (b) 10.6 K (c) 12.9 K (d) 15.0 K
Ans. (b)
Solution: The relation between critical field and critical temperature is
⎡ ⎛ T ⎞2 ⎤
H C (T ) = H 0 ⎢1 − ⎜ ⎟ ⎥
⎢⎣ ⎝ TC ⎠ ⎥⎦

Let at T = T1 , H C (T1 ) = T = T2 , TC (T ) = H C (T2 )

⎡ ⎛ T ⎞2 ⎤ ⎡ ⎛ T ⎞2 ⎤
Thus we get H C (T1 ) = H 0 ⎢1 − ⎜ ⎟ ⎥ , H C (T2 ) = H 0 ⎢1 − ⎜ 2 ⎟ ⎥
1

⎢⎣ ⎝ TC ⎠ ⎥⎦ ⎢⎣ ⎝ TC ⎠ ⎥⎦

H C (T1 ) 2
2
⎛T ⎞
1− ⎜ 1 ⎟ T2 − T12
H C (T1 ) T H C (T2 )
= ⎝ C ⎠ ⇒T =
H C (T2 ) ⎛T ⎞
2 C
H C (T1 )
1− ⎜ 2 ⎟ −1
H C (T2 )
⎝ TC ⎠

2 (8) − ( 4 )
2 2

⇒ TC = ≈ 10.6 where T1 = 4 k , T2 = 8 k
2 −1
H C (T1 ) = 11 mA / m

H C (T2 ) = 5.5 mA / m

Q37. The low-energy electronic excitations in a two-dimensional sheet of grapheme is given


G
( )
by E k = =vk , where v is the velocity of the excitations. The density of states is

proportional to
3 1
(a) E (b) E 2
(c) E 2
(d) E 2
Ans. (a)
Solution: The number of k - states in range k and k + dk in two dimension is

                                                                                
Head office  Branch office 
 
fiziks, H.No. 40 D, G.F, Jia Sarai,  Anand Institute of Mathematics, 
 
Near IIT, Hauz Khas, New Delhi‐16  28‐B/6, Jia Sarai, Near IIT 
 
Phone: 011‐26865455/+91‐9871145498 Hauz Khas, New Delhi‐16 
                                                   
                                             Website: www.physicsbyfiziks.com                                                                                          
                                                            Email: fiziks.physics@gmail.com                                                                     275 
fiziks
Institute for NET/JRF, GATE, IIT‐JAM, JEST, TIFR and GRE in PHYSICAL SCIENCES 
 
2
⎛ L ⎞
g ( k ) dk = ⎜ ⎟ 2π kdk
⎝ 2π ⎠
∴ E = =vk ⇒ dE = =vdk
2 2
⎛ L ⎞ E dE ⎛ L ⎞ 2π
⇒ g ( E ) dE = ⎜ ⎟ 2π × × =⎜ ⎟ EdE
⎝ 2π ⎠ =v =v ⎝ 2π ⎠ ( =v )
2

The density of state is


g ( E ) dE
2
⎛ L ⎞ 2π
ρ (E) = =⎜ ⎟ E
dE ⎝ 2π ⎠ ( )
=v 2

⇒ ρ (E) ∝ E
Q38. A He − Ne laser operates by using two energy levels of Ne separated by 2.26 eV .
Under steady state conditions of optical pumping, the equivalent temperature of the
system at which the ratio of the number of atoms in the upper state to that in the lower
1
state will be , is approximately (the Boltzmann constant k B = 8.6 × 10−5 eV / K )
20
(a) 1010 K (b) 108 K (c) 106 K (d) 104 K
Ans. (d)
Solution: According to Boltzmann relation
N2 ⎛ ΔE ⎞
= exp ⎜ − ⎟
N1 ⎝ kT ⎠ E2 N2
ΔE
N1 ⎛ kT ⎞ ΔE E1 N1
⇒ = exp ⎜ ⎟⇒T =
N2 ⎝ ΔE ⎠ ⎛N ⎞
k ln ⎜ 1 ⎟
⎝ N2 ⎠
N1
ΔE = 2.26 eV , K = 8.6 × 10−5 eV / k , = 20
N2

T = 8772k ≈ 104 k

                                                                                
Head office  Branch office 
 
fiziks, H.No. 40 D, G.F, Jia Sarai,  Anand Institute of Mathematics, 
 
Near IIT, Hauz Khas, New Delhi‐16  28‐B/6, Jia Sarai, Near IIT 
 
Phone: 011‐26865455/+91‐9871145498 Hauz Khas, New Delhi‐16 
                                                   
                                             Website: www.physicsbyfiziks.com                                                                                          
                                                            Email: fiziks.physics@gmail.com                                                                     276 
fiziks
Institute for NET/JRF, GATE, IIT‐JAM, JEST, TIFR and GRE in PHYSICAL SCIENCES 
 
NET/JRF (DEC-2015)
Q39. The first order diffraction peak of a crystalline solid occurs at a scattering angle of 300
when the diffraction pattern is recorded using an x-ray beam of wavelength 0.15 nm . If

the error in measurements of the wavelength and the angle are 0.01nm and 10
respectively, then the error in calculating the inter-planar spacing will approximately be
(a) 1.1× 10−2 nm (b) 1.3 × 10−4 nm (c) 2.5 × 10−2 nm (d) 2.0 × 10−3 nm
Ans.: (a)
λ ∂d 1 ∂d λ cos θ
Solution: Bragg’s Law for n = 1, λ = 2d sin θ ⇒ d = ⇒ = , =
2sin θ ∂λ 2sin θ ∂θ 2sin 2 θ
Error in d can be calculated as
2 2
⎛ ∂d ⎞ ⎛ ∂d ⎞ 2 ⎛ 1 ⎞ 2 ⎛ −λ cos θ ⎞ 2
σ = ⎜ ⎟ σ λ2 + ⎜
2
⎟σθ = ⎜ ⎟ σλ + ⎜ ⎟σθ
⎝ ∂λ ⎠ ⎝ ∂θ ⎝ 2sin θ ⎠ ⎝ 2sin θ
d 2
⎠ ⎠
2 2
σ d2
1 ⎛ 2sin θ ⎞ 2 ⎛ λ cos θ ⎞ ⎛ 2sin θ ⎞ 2
⇒ 2 = ×⎜ ⎟ σλ + ⎜ × ⎟ ×⎜ ⎟σθ
d 4sin θ ⎝ λ ⎠
2
( )
⎝ 2sin θ sin θ ⎠ ⎝ λ ⎠
1

σ 2
σλ 2
σ 2 ⎡⎛ σ ⎞ ⎛ σ ⎞ ⎤ 2 2 2
⇒ d
= + θ2 ⇒ σ d = d ⎢⎜ λ ⎟ + ⎜ θ ⎟ ⎥
d 2
λ 2
tan θ ⎢⎣⎝ λ ⎠ ⎝ tan θ ⎠ ⎥⎦

where θ = 30o , λ = 1.5 × 10−10 m, σ λ = 0.1× 10−10 m, σ θ = 1o

λ 1.5 ×10−10
d= = = 1.5 × 10−10 m
2sin θ 2sin 30o
1
⎡ ⎛ π ⎞ ⎤
2 2

⎢⎛ 0.1×10−10 ⎞ ⎜ 2

Thus σ d = 1.5 ×10−10 ⎢⎜ + 180 ⎟ ⎥
−10 ⎟ ⎜ ⎟
⎢⎝ 1.5 ×10 ⎠ ⎜ tan 30 ⎟ ⎥
⎢⎣ ⎜ ⎟ ⎥
⎝ ⎠ ⎦
1
⎡ ⎛ 3π ⎞ ⎤
2 2 1

= 1.5 × 10 ⎢( 0.067 ) + ⎜⎜ ⎟ ⎥ = 1.5 × 10 ⎣( 0.067 ) + ( 0.03) ⎦


−10 ⎡ ⎤
−10 2 2 2 2

⎢ ⎝ 180 ⎟⎠ ⎥
⎣ ⎦
1
= 1.5 × 10−10 [ 0.005389] 2 = 1.5 × 10−10 × 0.0734 = 0.11× 10−10
σ d = 1.1× 10−11 m = 1.1× 10−2 nm
                                                                                
Head office  Branch office 
 
fiziks, H.No. 40 D, G.F, Jia Sarai,  Anand Institute of Mathematics, 
 
Near IIT, Hauz Khas, New Delhi‐16  28‐B/6, Jia Sarai, Near IIT 
 
Phone: 011‐26865455/+91‐9871145498 Hauz Khas, New Delhi‐16 
                                                   
                                             Website: www.physicsbyfiziks.com                                                                                          
                                                            Email: fiziks.physics@gmail.com                                                                     277 
fiziks
Institute for NET/JRF, GATE, IIT‐JAM, JEST, TIFR and GRE in PHYSICAL SCIENCES 
 
Q40. The dispersion relation of electrons in a 3-dimensional lattice in the tight binding
approximation is given by,
ε k = α cos k x a + β cos k y a + γ cos k z a
where a is the lattice constant and α , β , γ are constants with dimension of energy. The

⎛π π π ⎞
effective mass tensor at the corner of the first Brillouin zone ⎜ , , ⎟ is
⎝a a a⎠
⎛ 1 ⎞ ⎛ 1 ⎞
⎜− 0 0⎟ ⎜− 0 0 ⎟
⎜ α ⎟ ⎜ α ⎟
= ⎜
2
1 ⎟ = ⎜
2
1 ⎟
(a) 2 ⎜ 0 − 0⎟ (b) 2 ⎜ 0 − 0 ⎟
a ⎜ β ⎟ a ⎜ β ⎟
⎜ 1⎟ ⎜ 1⎟
⎜ 0 0 ⎜ 0 0 − ⎟
⎝ γ ⎟⎠ ⎝ γ⎠

⎛1 ⎞ ⎛1 ⎞
⎜ 0 0⎟ ⎜ 0 0 ⎟
⎜α ⎟ ⎜α ⎟
= ⎜
2
1 ⎟ = ⎜
2
1 ⎟
(c) 2 ⎜ 0 0⎟ (d) 2 ⎜ 0 0 ⎟
a ⎜ β ⎟ a ⎜ β ⎟
⎜ 1⎟ ⎜ 1⎟
⎜0 0 ⎜0 0 − ⎟
⎝ γ ⎟⎠ ⎝ γ⎠
Ans.: (c)
Solution: The effective mass as a tensor quantity can be written as
⎡ m*xx m*xy m*xz ⎤
⎢ * ⎥ =2
mij* = ⎢ m*yx m*yy m yz ⎥ where mij =
*

⎛ ∂2E ⎞
⎢ m*zx m*zy m*zz ⎥⎦ ⎜⎜ ⎟⎟

⎝ ∂ki ∂k j ⎠
since ∈k = α cos k x a + β cos k y a + γ cos k z a

=2 −= 2 =2 −= 2
∴ m*xx = = ⇒ m*
= =
⎛ ∂2 ∈ ⎞ α a 2 cos k x a xx
⎛ ∂ 2 ∈ ⎞ β a 2 cos k y a
⎜ ⎟ ⎜⎜ 2 ⎟⎟
⎝ ∂k x ∂k x ⎠ ⎝ ∂k y ⎠

                                                                                
Head office  Branch office 
 
fiziks, H.No. 40 D, G.F, Jia Sarai,  Anand Institute of Mathematics, 
 
Near IIT, Hauz Khas, New Delhi‐16  28‐B/6, Jia Sarai, Near IIT 
 
Phone: 011‐26865455/+91‐9871145498 Hauz Khas, New Delhi‐16 
                                                   
                                             Website: www.physicsbyfiziks.com                                                                                          
                                                            Email: fiziks.physics@gmail.com                                                                     278 
fiziks
Institute for NET/JRF, GATE, IIT‐JAM, JEST, TIFR and GRE in PHYSICAL SCIENCES 
 
= 2
−= 2
m*zz = 2 = 2 other terms are zero
⎛ ∂ ∈ ⎞ γ a cos k z a
⎜ 2 ⎟
⎝ ∂k z ⎠

⎡1 ⎤
⎢ 0 0⎥
⎢α ⎥
⎛π π π ⎞ = ⎢ ⎥
2 2 2 2
= = = 1
now at ⎜ , , ⎟ ; m*xx = , m*yy = , m*zz = 2 ⇒ mij* = 2 ⎢ 0 0⎥
⎝a a a⎠ αa 2
βa 2
γa a ⎢ β ⎥
⎢ 1⎥
⎢0 0
γ ⎥⎦

Q41. A thin metal film of dimension 2 mm × 2 mm contains 4 × 1012 electrons. The magnitude
of the Fermi wavevector of the system, in the free electron approximation, is
(a) 2 π × 107 cm −1 (b) 2π × 107 cm −1 (c) π × 107 cm −1 (d) 2π × 107 cm −1
Ans.: (b)
Solution: This is the case of two dimensional metal box. The Fermi wave vector of electron in
2 − D is
1
1
⎛ N ⎞2
k F = ( 2π n ) 2 = ⎜ 2π 2 ⎟
⎝ L ⎠
1
⎛ 4 × 1012 ⎞ 2 2
⇒ k F = 2π ⎜ −2 2 ⎟
; L = 2mm × 2mm = 4 × 10−2 cm 2
⎝ 4 × 10 cm ⎠
1
⇒ (
k F = 2π 1014 cm −2 ) 2 ⇒ k F = 2π ×107 cm −1

Q42. For an electron moving through a one-dimensional periodic lattice of periodicity a ,


which of the following corresponds to an energy eigenfunction consistent with Bloch’s
theorem?
⎛ ⎡π x ⎛ π x ⎞⎤ ⎞ ⎛ ⎡π x ⎛ 2π x ⎞ ⎤ ⎞
(a) ψ ( x ) = A exp ⎜ i ⎢ + cos ⎜ ⎟⎥ ⎟ (b) ψ ( x ) = A exp ⎜ i ⎢ + cos ⎜ ⎟⎥ ⎟
⎝ ⎣ a ⎝ 2a ⎠ ⎦ ⎠ ⎝ ⎣ a ⎝ a ⎠⎦ ⎠

⎛ ⎡ 2π x ⎛ 2π x ⎞ ⎤ ⎞ ⎛ ⎡π x π x ⎤⎞
(c) ψ ( x ) = A exp ⎜ i ⎢ + i cosh ⎜ ⎟ ⎥ ⎟ (d) ψ ( x ) = A exp ⎜ i ⎢ +i ⎟
⎝ ⎣ a ⎝ a ⎠⎦ ⎠ ⎝ ⎣ a 2a ⎥⎦ ⎠

                                                                                
Head office  Branch office 
 
fiziks, H.No. 40 D, G.F, Jia Sarai,  Anand Institute of Mathematics, 
 
Near IIT, Hauz Khas, New Delhi‐16  28‐B/6, Jia Sarai, Near IIT 
 
Phone: 011‐26865455/+91‐9871145498 Hauz Khas, New Delhi‐16 
                                                   
                                             Website: www.physicsbyfiziks.com                                                                                          
                                                            Email: fiziks.physics@gmail.com                                                                     279 
fiziks
Institute for NET/JRF, GATE, IIT‐JAM, JEST, TIFR and GRE in PHYSICAL SCIENCES 
 
Ans.: (d)
=2 ∂ 2
Solution: H ψ = E ψ where H = − + V0
2m ∂x 2
⎛ ⎡π x π x ⎤⎞
Let ψ ( x ) = A exp ⎜ i ⎢ +i ⎟
⎝ ⎣ 2a 2a ⎥⎦ ⎠

πx
Put X =
2a

ψ ( x ) = A exp ( i ⎡⎣ X + i X ⎤⎦ ) ⇒ ψ ( x ) = Ae ⎣
i ⎡ X + i X ⎤⎦

∂ψ i ⎡ X + i X ⎦⎤ ⎡ X ⎤
= Ae ⎣ Xi ⎢ X ′ + i X ′⎥ for x > 0, X = X
∂x ⎢⎣ X ⎥⎦
∂ψ
XX ′ [i − 1]
i ⎡ X + i X ⎦⎤
= Ae ⎣
∂x
⎡π x πx ⎤
∂ 2ψ π2 i ⎢ +i ⎥ π
i ⎡⎣ X + i X ⎤⎦
[ ] i ⎡⎣ X + i X ⎤⎦
2
= Ae XX ′ 2
i − 1 = 2 X ′ 2
Ae = Ae ⎣ 2a 2a ⎦
where X ′ =
∂x 2
2a 2
2a
⎡π x πx ⎤
⎛ =2 π 2 ⎞ i ⎢⎣ 2 a +i 2 a ⎥⎦ ⎛ π 2=2 ⎞
∴H ψ = ⎜ − × 2 + V0 ⎟ Ae ⇒ H ψ = ⎜− 2
+ V0 ⎟ ψ = E ψ
⎝ 2m 2a ⎠ ⎝ 4ma ⎠
For other case H ψ ≠ E ψ

                                                                                
Head office  Branch office 
 
fiziks, H.No. 40 D, G.F, Jia Sarai,  Anand Institute of Mathematics, 
 
Near IIT, Hauz Khas, New Delhi‐16  28‐B/6, Jia Sarai, Near IIT 
 
Phone: 011‐26865455/+91‐9871145498 Hauz Khas, New Delhi‐16 
                                                   
                                             Website: www.physicsbyfiziks.com                                                                                          
                                                            Email: fiziks.physics@gmail.com                                                                     280 
fiziks
Institute for NET/JRF, GATE, IIT‐JAM, JEST, TIFR and GRE in PHYSICAL SCIENCES 
 
NET/JRF (JUNE-2016)
Q43. Consider electrons in graphene, which is a planar monatomic layer of carbon atoms. If
the dispersion relation of the electrons is taken to be ε ( k ) = ck (where c is constant)

over the entire k -space, then the Fermi energy ε F depends on the number density of

electrons ρ as
1 2 1
(a) ε F ∝ ρ 2 (b) ε F ∝ ρ (c) ε F ∝ ρ 3 (d) ε F ∝ ρ 3
Ans: (a)
Solution: In 2 D , density of state is
⎛ L ⎞
g ( k ) dk = ⎜ ⎟ 2π kdk
⎝ 2π ⎠
ε dε
where ε = ck ⇒ k = and dk =
c c
2
⎛ L ⎞ ε dε L2
⇒ g (ε ) dε = ⎜ ⎟ × 2π × . = ε dε
⎝ 2π ⎠ c c 2π c 2
Now, number electrons at T = 0 K is
εF L2 εF L2 2 N
N =∫ g (ε ) d ε = ∫ ε dε = ε ⇒ ε F2 = 4π c 2 2 = 4π c 2 ρ
0 2π c 2 0 4π c 2 F
L

⇒ ε F = 4π c 2 ρ 1/ 2 ⇒ ε F ∝ ρ 1/ 2
Q44. Suppose the frequency of phonons in a one-dimensional chain of atoms is proportional to
the wave vector. If n is the number density of atoms and c is the speed of the phonons,
then the Debye frequency is
π cn
(a) 2π cn (b) 2π cn (c) 3π cn (d)
2
Ans: (d)
Solution: Given ω ∝ k ⇒ ω = ck ( c is velocity of phonon)
L dω L
Now g (ω ) dω = = dω
π dω / dk cπ

                                                                                
Head office  Branch office 
 
fiziks, H.No. 40 D, G.F, Jia Sarai,  Anand Institute of Mathematics, 
 
Near IIT, Hauz Khas, New Delhi‐16  28‐B/6, Jia Sarai, Near IIT 
 
Phone: 011‐26865455/+91‐9871145498 Hauz Khas, New Delhi‐16 
                                                   
                                             Website: www.physicsbyfiziks.com                                                                                          
                                                            Email: fiziks.physics@gmail.com                                                                     281 
fiziks
Institute for NET/JRF, GATE, IIT‐JAM, JEST, TIFR and GRE in PHYSICAL SCIENCES 
 
ωD L D ω L
Also N = ∫ g (ω ) d ω = ∫ dω ⇒ N = ωD
0 cπ 0 cπ
N ⎛ N⎞ cn
⇒ ωD = cπ = cπ n, ⎜ n = ⎟ ⇒ f D = . Best answer is (d).
L ⎝ L⎠ 2
Q45. The band energy of an electron in a crystal for a particular k -direction has the form
ε ( k ) = A − B cos 2ka , where A and B are positive constants and 0 < ka < π . The
electron has a hole-like behaviour over the following range of k :
π 3π π
(a) < ka < (b) < ka < π
4 4 2
π π 3π
(c) 0 < ka < (d) < ka <
4 2 4
Ans: (a)
Solution: ε ( k ) = A − B cos 2ka


= 2 Ba sin 2ka
dk
d 2ε
2
= 4 Ba 2 cos 2ka
dk
=2 =2
Effective mass m* = =
d 2ε / dk 2 4 Ba 2 cos 2ka

( )
Effective mass of electron me* and effective mass holes mh* are opposite in sign i.e., ( )
(m *
h = − me* ) .

π
Now, in the range 0 < ka < , m* is positive
4
π 3π
While in the range < ka < , m* is negative
4 4
π 3π
Thus, electron has hole like behaviour in the region < ka <
4 4

                                                                                
Head office  Branch office 
 
fiziks, H.No. 40 D, G.F, Jia Sarai,  Anand Institute of Mathematics, 
 
Near IIT, Hauz Khas, New Delhi‐16  28‐B/6, Jia Sarai, Near IIT 
 
Phone: 011‐26865455/+91‐9871145498 Hauz Khas, New Delhi‐16 
                                                   
                                             Website: www.physicsbyfiziks.com                                                                                          
                                                            Email: fiziks.physics@gmail.com                                                                     282 
fiziks
Institute for NET/JRF, GATE, IIT‐JAM, JEST, TIFR and GRE in PHYSICAL SCIENCES 
 
NUCLEAR AND PARTICLE PHYSICS
NET/JRF (JUNE-2011)
Q1. The radius of a 64
29 Cu nucleus is measured to be 4.8 × 10-13 cm.
27
(A). The radius of a 12 Mg nucleus can be estimated to be
(a) 2.86 × 10-13 cm (b) 5.2× 10-13 cm (c) 3.6× 10-13 cm (d) 8.6× 10-13 cm
Ans: (c)
1/ 3 1/ 3
RMg ⎛ AMg ⎞ ⎛ 27 ⎞
Solution: Since R = R0 ( A) = ⎜⎜ ⎟⎟
1/ 3
⇒ =⎜ ⎟
RCu ⎝ ACu ⎠ ⎝ 64 ⎠
RMg3 3
⇒ ⇒ RMg = × 4.8 × 10 −13 = 3.6 × 10 −13 cm.
=
RCu 4 4
(B). The root-mean-square (r.m.s) energy of a nucleon in a nucleus of atomic number A
in its ground state varies as:
(a) A4 / 3 (b) A1 / 3 (c) A−1/ 3 (d) A−2 / 3
Ans: (c)
Q2. A beam of pions (π+) is incident on a proton target, giving rise to the process
π+p → n + π+ + π+
(A). Assuming that the decay proceeds through strong interactions, the total isospin I and
its third component I3 for the decay products, are
3 3 5 5
(a) I = , I 3 = (b) I = , I 3 =
2 2 2 2
5 3 1 1
(c) I = , I 3 = (d) I = , I 3 = −
2 2 2 2
Ans: (c)
1 5 1 3
Solution: π + + p → n + π + + π + ; I : +1 +1 = , I3 : − +1+1 =
2 2 2 2
(B). Using isospin symmetry, the cross-section for the above process can be related to
that of the process
(a) π n → pπ π (b) π p → n π π
− − − − − −

(c) π n → pπ π (d) π p → nπ π
+ + − + + −

Ans: (c)
                                                                                
Head office  Branch office 
 
fiziks, H.No. 40 D, G.F, Jia Sarai,  Anand Institute of Mathematics, 
 
Near IIT, Hauz Khas, New Delhi‐16  28‐B/6, Jia Sarai, Near IIT 
 
Phone: 011‐26865455/+91‐9871145498 Hauz Khas, New Delhi‐16 
                                                   
                                             Website: www.physicsbyfiziks.com                                                                                          
                                                            Email: fiziks.physics@gmail.com                                                                     283 
fiziks
Institute for NET/JRF, GATE, IIT‐JAM, JEST, TIFR and GRE in PHYSICAL SCIENCES 
 
NET/JRF (DEC-2011)
Q3. According to the shell model the spin and parity of the two nuclei 125 89
51 Sb and 38 Sr are,

respectively,
+ + + +
⎛5⎞ ⎛5⎞ ⎛5⎞ ⎛7⎞
(a) ⎜ ⎟ and ⎜ ⎟ (b) ⎜ ⎟ and ⎜ ⎟
⎝2⎠ ⎝2⎠ ⎝2⎠ ⎝2⎠
+ + + +
⎛7⎞ ⎛5⎞ ⎛7⎞ ⎛7⎞
(c) ⎜ ⎟ and ⎜ ⎟ (d) ⎜ ⎟ and ⎜ ⎟
⎝2⎠ ⎝2⎠ ⎝2⎠ ⎝2⎠
Ans: (d)
125
51 Sb ; Z = 51 and N = 74
Z = 51:
( s1/ 2 ) ( p3/ 2 ) ( p1/ 2 ) ( d5 / 2 ) ( s1/ 2 ) ( d3/ 2 ) ( f 7 / 2 ) ( p3/ 2 ) ( f5 / 2 ) ( p1/ 2 ) ( g9 / 2 ) ( g7 / 2 )
2 4 2 6 2 4 8 4 6 2 10 1

+
7 ⎛7⎞
⇒ j = and l = 4 . Thus spin and parity = ⎜ ⎟
2 ⎝2⎠
89
38 Sr ; Z = 38 and N = 51
N = 51:
( s1/ 2 ) ( p3/ 2 ) ( p1/ 2 ) ( d5 / 2 ) ( s1/ 2 ) ( d3/ 2 ) ( f 7 / 2 ) ( p3/ 2 ) ( f5 / 2 ) ( p1/ 2 ) ( g9 / 2 ) ( g 7 / 2 )
2 4 2 6 2 4 8 4 6 2 10 1

+
7 ⎛7⎞
⇒ j= and l = 4 . Thus spin and parity = ⎜ ⎟
2 ⎝2⎠
49 49
Q4. The difference in the Coulomb energy between the mirror nuclei 24 Cr and 25 Mn is

6.0 MeV . Assuming that the nuclei have a spherically symmetric charge distribution and

that e2 is approximately 1.0 MeV-fm, the radius of the 49


25 Mn nucleus is

(a) 4.9 × 10-13 m (b) 4.9 × 10-15 m


(c) 5.1 × 10-13 m (d) 5.1 × 10-15 m
3 × 1 × 10 −15
Ans: (b) R=
3e 2
5 ⋅ ΔW
(Z1 − Z 2 ) =
2 2

5× 6
(25 2 − 24 2 ) = 4.9 × 10 −15 m .

                                                                                
Head office  Branch office 
 
fiziks, H.No. 40 D, G.F, Jia Sarai,  Anand Institute of Mathematics, 
 
Near IIT, Hauz Khas, New Delhi‐16  28‐B/6, Jia Sarai, Near IIT 
 
Phone: 011‐26865455/+91‐9871145498 Hauz Khas, New Delhi‐16 
                                                   
                                             Website: www.physicsbyfiziks.com                                                                                          
                                                            Email: fiziks.physics@gmail.com                                                                     284 
fiziks
Institute for NET/JRF, GATE, IIT‐JAM, JEST, TIFR and GRE in PHYSICAL SCIENCES 
 
NET/JRF (JUNE-2012)
1−
Q5. The ground state of 12 Pb nucleus has spin-parity J = , while the first excited state
207 p

2
5−
has J = p
.The electromagnetic radiation emitted when the nucleus makes a transition
2
from the first excited state to ground state are
(a) E2 and E3 (b) M2 or E3 (c) E2 or M3 (d) M2 or M3
Ans: (c)
Solution: No parity change; ΔJ = 2,3

For El type, Δπ = (− 1) , (for no parity change l = 2 )


l

For M l type, Δπ = (− 1) , (for no parity change l = 3 )


l +1

ΔJ = 2 , No parity change → E 2 ; ΔJ = 3 , No parity change → M 3

Q6. The dominant interactions underlying the following processes


A. K − + p → ∑ − +π + , B. μ − + μ + → K − + K + , C. Σ + → p + π 0 are

(a) A: strong, B: electromagnetic and; C: weak


(b) A: strong, B: weak and; C: weak
(c) A: weak, B: electromagnetic and; C: strong
(d) A: weak, B: electromagnetic and; C: weak
Ans: (a)
(A) K − + p → ∑ − +π + (Strong interaction)

1 1
I 3 : − + → −1 + 1 (Conserved)
2 2
(B) μ − + μ + → K − + K + (Electromagnetic interaction)

(C) Σ + → p + π 0 (Weak interaction)


1
I3 :1 → +0 (Not conserved)
2

                                                                                
Head office  Branch office 
 
fiziks, H.No. 40 D, G.F, Jia Sarai,  Anand Institute of Mathematics, 
 
Near IIT, Hauz Khas, New Delhi‐16  28‐B/6, Jia Sarai, Near IIT 
 
Phone: 011‐26865455/+91‐9871145498 Hauz Khas, New Delhi‐16 
                                                   
                                             Website: www.physicsbyfiziks.com                                                                                          
                                                            Email: fiziks.physics@gmail.com                                                                     285 
fiziks
Institute for NET/JRF, GATE, IIT‐JAM, JEST, TIFR and GRE in PHYSICAL SCIENCES 
 
NET/JRF (JUNE-2013)
Q7. The binding energy of a light nucleus (Z , A) in MeV is given by the approximate formula

B( A, Z ) ≈ 16 A − 20 A 2/3 3
− Z 2 A −1 / 3 + 30
(N − Z ) 2

4 A
where N = A − Z is the neutron number. The value of Z of the most stable isobar for a
given A is
−1
A⎛ A2 / 3 ⎞ A
(a) ⎜⎜1 − ⎟ (b)
2⎝ 160 ⎟⎠ 2
−1 −1
A⎛ A2 / 3 ⎞ A⎛ A4 / 3 ⎞
(c) ⎜⎜1 − ⎟ (d) ⎜⎜1 + ⎟
2⎝ 120 ⎟⎠ 2⎝ 64 ⎟⎠

Ans: (a)
−1
∂B A⎛ A2 / 3 ⎞
Solution: = 0 ⇒ Z ′ = ⎜⎜1 − ⎟
∂Z Z =Z ′ 2⎝ 160 ⎟⎠

Q8. A spin-1/2 particle A undergoes the delay


A→ B+C + D
where it is known that B and C are also spin-1/2 particles. The complete set of allowed
values of the spin of the particle D is
1 3 5
(a) ,1, , 2, , 3, ... (b) 0, 1
2 2 2
1 1 3 5 7
(c) only (d) , , , ,....
2 2 2 2 2
Ans: (c)
Solution: Spin of the left side and combined spin of the products must be same to conserve the
spin angular momentum conservation law.

                                                                                
Head office  Branch office 
 
fiziks, H.No. 40 D, G.F, Jia Sarai,  Anand Institute of Mathematics, 
 
Near IIT, Hauz Khas, New Delhi‐16  28‐B/6, Jia Sarai, Near IIT 
 
Phone: 011‐26865455/+91‐9871145498 Hauz Khas, New Delhi‐16 
                                                   
                                             Website: www.physicsbyfiziks.com                                                                                          
                                                            Email: fiziks.physics@gmail.com                                                                     286 
fiziks
Institute for NET/JRF, GATE, IIT‐JAM, JEST, TIFR and GRE in PHYSICAL SCIENCES 
 
Q9. Muons are produced through the annihilation of particle a and its antiplarticle, namely
the process
a + a → μ+ + μ−
A muon has a rest mass of 105 MeV/c2 and its proper life time is 2μ s . If the center of
mass energy of the collision is 2.1 GeV in the laboratory frame that coincides with the
center-of-mass frame, then the fraction of muons that will decay before they reach a
detector placed 6 km away from the interaction point is
(a) e −1 (b) 1 − e −1
(c) 1 − e −2 (d) e −10
Ans: (b)
t
− λt N −
Solution: N = N 0 e ⇒ = e −λt = e γτ
N0

2.1 6 × 10 3
where τ = 2 × 10 −6 s , γ = × 10 3 = 20 and t = = 2 × 10 −5 sec .
105 3 × 10 8
1
1 N
t −
Thus = ⇒ = e 2 ≈ 1 − e −1 .
γτ 2 N0

                                                                                
Head office  Branch office 
 
fiziks, H.No. 40 D, G.F, Jia Sarai,  Anand Institute of Mathematics, 
 
Near IIT, Hauz Khas, New Delhi‐16  28‐B/6, Jia Sarai, Near IIT 
 
Phone: 011‐26865455/+91‐9871145498 Hauz Khas, New Delhi‐16 
                                                   
                                             Website: www.physicsbyfiziks.com                                                                                          
                                                            Email: fiziks.physics@gmail.com                                                                     287 
fiziks
Institute for NET/JRF, GATE, IIT‐JAM, JEST, TIFR and GRE in PHYSICAL SCIENCES 
 
JRF/NET-(DEC-2013)
A
Q10. The intrinsic electric dipole moment of a nucleus Z X
(a) increases with Z , but independent of A
(b) decreases with Z , but independent of A
(c) is always zero
(d) increases with Z and A
Ans: (d)
Q11. According to the shell model, the total angular momentum (in units of ) and the parity
of the ground state of the 37 Li nucleus is

3 3
(a) with negative parity (b) with positive parity
2 2
1 7
(c) with positive parity (d) with negative parity
2 2
Ans: (a)
Solution: Z = 3, N = 4

( )( )
For odd Z = 3; s12/ 2 p31 / 2 ⇒ j = 3 / 2, l = 1 and parity = (− 1)1 = −1 .

NET/JRF (JUNE-2014)
Q12. The recently-discovered Higgs boson at the LHC experiment has a decay mode into a
photon and a Z boson. If the rest masses of the Higgs and Z boson are 125 GeV/c 2 and

90 GeV/c 2 respectively, and the decaying Higgs particle is at rest, the energy of the
photon will approximately be
(a) 35 3 GeV (b) 35 GeV (c) 30 GeV (d) 15 GeV
Ans: (c)
Solution: Assume H is symbol of Higgs boson, H → Z +γ

E H2 − E Z2 (125)2 − (90)2
Eγ = = = 30GeV
2EH 2 × 125

                                                                                
Head office  Branch office 
 
fiziks, H.No. 40 D, G.F, Jia Sarai,  Anand Institute of Mathematics, 
 
Near IIT, Hauz Khas, New Delhi‐16  28‐B/6, Jia Sarai, Near IIT 
 
Phone: 011‐26865455/+91‐9871145498 Hauz Khas, New Delhi‐16 
                                                   
                                             Website: www.physicsbyfiziks.com                                                                                          
                                                            Email: fiziks.physics@gmail.com                                                                     288 
fiziks
Institute for NET/JRF, GATE, IIT‐JAM, JEST, TIFR and GRE in PHYSICAL SCIENCES 
 
Q13. In a classical model, a scalar (spin-0) meson consists of a quark and an antiquark bound
by a potential
b
V (r ) = ar +
r
where a = 200 MeV fm -1 and b = 100 MeV fm . If the masses of the quark and antiquark
are negligible, the mass of the meson can be estimated as approximately
(a) 141 MeV/c 2 (b) 283 MeV/c 2 (c) 353 MeV/c 2 (d) 425 MeV/c 2
Ans: (b)
Solution: At equilibrium separation the potential is minimum, thus the equilibrium separation
can be determined as
DV (r ) b
=a− 2 =0
dr r = r0 r0
b 100 MeVfm −1 1
⇒ r0 = = −1
= fm
a 200 MeVfm 2
The equilibrium separation between particles is also estimated by uncertainty principle

r0 = cΔt ⇒ r0 = c ( where, ΔEΔt ≈ )


ΔE
Where, c is the velocity of the virtual meson
200 MeV . fm
r0 = c =
ΔE ΔE (MeV )
200MeV . fm 1
Using above two relation = fm
ΔE (MeV ) 2

ΔE = 200 2 = 283MeV ⇒ ΔE = Δm × c 2
ΔE
the mass of the meson Δm = 2
= 283MeV / c 2
c

                                                                                
Head office  Branch office 
 
fiziks, H.No. 40 D, G.F, Jia Sarai,  Anand Institute of Mathematics, 
 
Near IIT, Hauz Khas, New Delhi‐16  28‐B/6, Jia Sarai, Near IIT 
 
Phone: 011‐26865455/+91‐9871145498 Hauz Khas, New Delhi‐16 
                                                   
                                             Website: www.physicsbyfiziks.com                                                                                          
                                                            Email: fiziks.physics@gmail.com                                                                     289 
fiziks
Institute for NET/JRF, GATE, IIT‐JAM, JEST, TIFR and GRE in PHYSICAL SCIENCES 
 
NET/JRF (DEC-2014)
Q14. Consider the four processes
(i) p + → n + e + + ve (ii) Λ0 → p + + e + + v e

(iii) π + → e + + ve (iv) π 0 → γ + γ
which of the above is/are forbidden for free particles?
(a) only (ii) (b) (ii) and (iv) (c) (i) and (iv) (d) (i) and (ii)
Ans: (d)
Solution: (i) p + → n + e + + ν e [Not allowed]
It violate energy conservation. The mass of proton is less than mass of neutron. Free
proton is stable and can not decay to neutron. Proton can decay to neutron only inside the
nucleus, where energy violation is taken care by Heisenberg uncertainty principle.
(ii) Λ 0 → p + + e + + ν e [Not allowed]. In this decay charge is not conserved

(iii) π + → e + +ν e [allowed through Weak interaction]

(iv) π 0 → γ + γ [allowed through Electromagnetic interaction]


Q15. In deep inelastic scattering electrons are scattered off protons to determine if a proton has
any internal structure. The energy of the electron for this must be at least
(a) 1.25 × 10 9 eV (b) 1.25 × 1012 eV (c) 1.25 × 10 6 eV (d) 1.25 × 10 8 eV
Ans: (b)
Solution: The internal structure of proton can only be determined if the wavelength of the
incoming electron is nearly equal to the size of the proton
i.e. λ = R = 1.2 A1/ 3 ( fm ) = 1.2 fm = 1.2 × 10−15 m

h h
According to de-Broglie relation, λ= =
p 2mE

⎛ 0⎞ 150
This can be also written as λ ⎜ Α⎟ =
⎝ ⎠ E ( eV )

                                                                                
Head office  Branch office 
 
fiziks, H.No. 40 D, G.F, Jia Sarai,  Anand Institute of Mathematics, 
 
Near IIT, Hauz Khas, New Delhi‐16  28‐B/6, Jia Sarai, Near IIT 
 
Phone: 011‐26865455/+91‐9871145498 Hauz Khas, New Delhi‐16 
                                                   
                                             Website: www.physicsbyfiziks.com                                                                                          
                                                            Email: fiziks.physics@gmail.com                                                                     290 
fiziks
Institute for NET/JRF, GATE, IIT‐JAM, JEST, TIFR and GRE in PHYSICAL SCIENCES 
 
150 150
∴ E ( eV ) = = = 1.04 × 1012 ⇒ E = 1.04 × 1012 eV
( )
2 2
⎡ ⎛ 0 ⎞⎤ 1.2 × 10 −5
λ ⎜
⎢ ⎝ ⎠⎥Α ⎟
⎣ ⎦
The bet suitable answer is option (b).
Q16. If the binding energy B of a nucleus (mass number A and charge Z ) is given by

B = aV A − a S A 2/3
− a sym
(2Z − A)2 aC Z 2
− 1/ 3
A A
where aV = 16 MeV , a S = 16 MeV , a sym = 24 MeV and aC = 0.75 MeV , then for the most

stable isobar for a nucleus with A = 216 is


(a) 68 (b) 72 (c) 84 (d) 92
Ans: (c)
dB 2 ( 2Z − A ) × 2 2aC Z
Solution: For the most stable isobar for a nucleus = 0 ⇒ −asym − 1/ 3 = 0
dZ A A
2 ( 2Z − 216 ) × 2 2Z 4 ( 2Z − 216 ) 3 2Z
⇒ 24 + 0.75 =0 ⇒ + =0
( 216 )
1/ 3
216 9 4 6

4 ( 2Z − 216 ) Z
⇒ + = 0 ⇒ 16 ( 2 Z − 216 ) + 9 Z = 0 ⇒ 41Z = 216 × 16 ⇒ Z = 82.3
9 4

NET/JRF (JUNE-2015)
Q17. The reaction 2
1 D +12 D →42 He + π 0 cannot proceed via strong interactions because it
violates the conservation of
(a) angular momentum (b) electric charge
(c) baryon number (d) isospin
Ans. (d)
Solution: 1 D 2 + 1 D 2 → 2 He 4 + π 0 (Not conserved)
I: 0 0 → 0 1
This isopin is not conserved in above reaction.

                                                                                
Head office  Branch office 
 
fiziks, H.No. 40 D, G.F, Jia Sarai,  Anand Institute of Mathematics, 
 
Near IIT, Hauz Khas, New Delhi‐16  28‐B/6, Jia Sarai, Near IIT 
 
Phone: 011‐26865455/+91‐9871145498 Hauz Khas, New Delhi‐16 
                                                   
                                             Website: www.physicsbyfiziks.com                                                                                          
                                                            Email: fiziks.physics@gmail.com                                                                     291 
fiziks
Institute for NET/JRF, GATE, IIT‐JAM, JEST, TIFR and GRE in PHYSICAL SCIENCES 
 
Q18. Let us approximate the nuclear potential in the shell model by a three dimensional
isotropic harmonic oscillator. Since the lowest two energy levels have angular momenta
l = 0 and l = 1 respectively, which of the following two nuclei have magic numbers of
protons and neutrons?
(a) 42 He and 16
8 O (b) 12 D and 84 Be (c) 42 He and 84 Be (d) 42 He and 12
6 C
Ans. (a)
Solution: 2 He 4 has Z = 2, N = 2

and 8 O16 has Z = 8, N = 8 magic numbers ( 2,8, 20, 28,50,82,126 )

Q19. The charm quark s assigned a charm quantum number C = 1 . How should the Gellmann-
Nishijima formula for electric charge be modified for four flavors of quarks?
1 1
(a) I 3 + (B − S −C) (b) I 3 + (B − S + C)
2 2
1 1
(c) I 3 + (B + S −C) (d) I 3 + (B + S + C)
2 2
Ans. (d)
1
Solution: From Gell-Mann-Nishijima formula Q = I 3 + (B + S)
2
1
For Quark it is generalized as Q = I 3 + (B + S + C)
2
NET/JRF (DEC-2015)
Q20. Consider the following processes involving free particles
(i) n → p + e + + ve (ii) p + n → π −

(iii) p + n → π + + π 0 + π 0 (iv) p + ve → n + e +
Which of the following statements is true?
(a) Process (i) obeys all conservation laws
(b) Process (ii) conserves baryon number, but violates energy-momentum conservation
(c) process (iii) is not allowed by strong interaction but is allowed by weak interactions
(d) Process (iv) conserves baryon number, but violates lepton number conservation

                                                                                
Head office  Branch office 
 
fiziks, H.No. 40 D, G.F, Jia Sarai,  Anand Institute of Mathematics, 
 
Near IIT, Hauz Khas, New Delhi‐16  28‐B/6, Jia Sarai, Near IIT 
 
Phone: 011‐26865455/+91‐9871145498 Hauz Khas, New Delhi‐16 
                                                   
                                             Website: www.physicsbyfiziks.com                                                                                          
                                                            Email: fiziks.physics@gmail.com                                                                     292 
fiziks
Institute for NET/JRF, GATE, IIT‐JAM, JEST, TIFR and GRE in PHYSICAL SCIENCES 
 
Ans.: (b)
Solution: (i) n → p + e + + ve

q 0 −1 +1 0 (conserved)
1 1 1 1
spin − − − − (not conserved)
2 2 2 2
Le 0 0 − 1 − 1 (not conserved)
(ii) Baryon number is conserved but energy and momentum conservation violated.
(iii) spin is not conserved
(iv) obeys all conservation laws.
Q21. Of the nuclei of mass number A = 125 , the binding energy calculated from the liquid
drop model (given that the coefficients for the Coulomb and the asymmetry energy are
ac = 0.7 MeV and asym = 22.5 MeV respectively) is a maximum for
125 124 125 125
(a) 54 Xe (b) 53 I (c) 52 Te (d) 51 Sb
Ans.: (c)
4aa + ac A−1/ 3 4a0 A + ac A2 / 3
Solution: Z 0 = =
2ac A−1/ 3 + 8aa A−1 8aa + 2ac A2 / 3

( )
2/3
4 × 22.5 × 125 + 0.7 53 11250 + 17.5 11267.5
⇒ Z0 = ⇒ Z0 = = = 52.4
8 × 22.5 + 2 × 0.7 ( 5 ) 180 + 35
2/3
3 215

⇒ Z 0 ≈ 52

                                                                                
Head office  Branch office 
 
fiziks, H.No. 40 D, G.F, Jia Sarai,  Anand Institute of Mathematics, 
 
Near IIT, Hauz Khas, New Delhi‐16  28‐B/6, Jia Sarai, Near IIT 
 
Phone: 011‐26865455/+91‐9871145498 Hauz Khas, New Delhi‐16 
                                                   
                                             Website: www.physicsbyfiziks.com                                                                                          
                                                            Email: fiziks.physics@gmail.com                                                                     293 
fiziks
Institute for NET/JRF, GATE, IIT‐JAM, JEST, TIFR and GRE in PHYSICAL SCIENCES 
 
NET/JRF (JUNE-2016)
Q22. A radioactive element X decays toY , which in turn decays to a stable element Z . The
decay constant from X to Y is λ1 , and that from Y to Z is λ2 . If, to begin with, there are

1 1
only N 0 atoms of X , at short times ( t as well as ) the number of atoms of Z
λ1 λ2
will be
1 λ1λ2
(a) λ1λ2 N 0t 2 (b) N 0t
2 2 ( λ1 + λ2 )

(c) ( λ1 + λ2 ) N 0t 2 (d) ( λ1 + λ2 ) N 0t
2

Ans: (a)
λ1 λ2
X ⎯⎯ → Y ⎯⎯ →Z
Solution: t = 0 N0 0 0
t N1 N2 N3
dN 2 dN
Rate equations N1 = N 0 e − λ1t , = λ1 N1 − λ2 N 2 , 3 = λ2 N 2
dt dt
⎡ λ1e− λ2t λ2 e − λ1 t ⎤
N 3 = N 0 ⎢1 + − ⎥
⎢⎣ ( λ2 − λ1 ) ( λ2 − λ1 ) ⎥⎦

⎡ λ1 ⎛ λ22t 2 ⎞ λ2 ⎛ λ12t 2 ⎞ ⎤
= N 0 ⎢1 + ⎜ 1 − λ t + ⎟ − ⎜ 1 − λ t + ⎟⎥
⎢⎣ ( λ2 − λ1 ) ⎝ 2 ⎠ ( λ2 − λ1 ) ⎝
2 1
2 ⎠ ⎥⎦

⎡ λ1 λλ t λ1 λ22t 2 λ2 λ2λ1t λ2 λ12t 2 ⎤


= N 0 ⎢1 + − 1 2 + − + − ⎥
⎣⎢ ( λ2 − λ1 ) ( λ2 − λ1 ) ( λ2 − λ1 ) 2 ( λ2 − λ1 ) ( λ2 − λ1 ) ( λ2 − λ1 ) 2 ⎦⎥
⎡ λ1 λ22t 2 λ2 λ12t 2 ⎤ λ1λ2t 2 ⎡ λ λ1 ⎤
= N0 ⎢ × − × ⎥= N0 ⎢ 2 − ⎥
⎣⎢ ( λ2 − λ1 ) 2 ( λ2 − λ1 ) 2 ⎦⎥ 2 ⎣ λ2 − λ1 λ2 − λ1 ⎦
1
⇒ N3 = λ1λ2 N 0t 2
2

                                                                                
Head office  Branch office 
 
fiziks, H.No. 40 D, G.F, Jia Sarai,  Anand Institute of Mathematics, 
 
Near IIT, Hauz Khas, New Delhi‐16  28‐B/6, Jia Sarai, Near IIT 
 
Phone: 011‐26865455/+91‐9871145498 Hauz Khas, New Delhi‐16 
                                                   
                                             Website: www.physicsbyfiziks.com                                                                                          
                                                            Email: fiziks.physics@gmail.com                                                                     294 
fiziks
Institute for NET/JRF, GATE, IIT‐JAM, JEST, TIFR and GRE in PHYSICAL SCIENCES 
 
Q23. In the large hadron collider ( LHC ) , two equal energy proton beams traverse in opposite

directions along a circular path of length 27 km . If the total centre of mass energy of a
proton-proton pair is 14 TeV , which of the following is the best approximation for the
proper time taken by a proton to traverse the entire path?
(a) 12 ns (b) 1.2 μ s (c) 1.2 ns (d) 0.12 μ s
Ans: (a)
Solution: The proton travel at nearly speed of light in LHC , therefore
d 27 ×103
t≈ = ≈ 9 ×10−5 sec
c 3 ×108
since proton is relativistic

v2 t
t0 = t 1 − 2 =
c γ
1 m0 c 2 938 MeV 938 × 106 eV 1
∵ E = γ m0 c 2 ⇒ = = = ⇒ = 1.34 × 10−4
γ E 7 TeV 7 ×10 eV
12
γ
t
Thus t0 = = 9 × 10−5 × 1.34 × 10−4 = 1.2 × 10−8 sec ⇒ t0 = 12 ns
γ
Q24. Let ES denotes the contribution of the surface energy per nucleon in the liquid drop

model. The ratio ES ( 27


13 )
Al : ES ( 64
30 )
Zn is

(a) 2 : 3 (b) 4 : 3 (c) 5 : 3 (d) 3 : 2


Ans: (b)
2
1
B A − 3
Solution: ES = = ∝A3
A A
1 1
ES ( Al ) ( 27 ) ( 64 ) 3

3 4
= = =
ES ( Z n ) 1 1
3
( 64 ) ( 27 )

3 3

                                                                                
Head office  Branch office 
 
fiziks, H.No. 40 D, G.F, Jia Sarai,  Anand Institute of Mathematics, 
 
Near IIT, Hauz Khas, New Delhi‐16  28‐B/6, Jia Sarai, Near IIT 
 
Phone: 011‐26865455/+91‐9871145498 Hauz Khas, New Delhi‐16 
                                                   
                                             Website: www.physicsbyfiziks.com                                                                                          
                                                            Email: fiziks.physics@gmail.com                                                                     295 
fiziks
Institute for NET/JRF, GATE, IIT‐JAM, JEST, TIFR and GRE in PHYSICAL SCIENCES 
 
Q25. According to the shell model, the nuclear magnetic moment of the 13 27 Al nucleus is

(Given that for a proton gl = 1, g s = 5.586 , and for a neutron gl = 0, g s = −3.826 )

(a) −1.913 μ N (b) 14.414 μ N (c) 4.793 μ N (d) 0


Ans: (c)
Solution: As per shell model

13 Al 27 : Z = 13, N = 14

5
for Z = 13, S1/2 2 , P3/4 2 , P1/22 , d5/5 2 ⇒ j = , l = 2
2
Magnetic moment

[ 2 j − 1 + g S ] μ N = ⎡⎢ 2 × − 1 + 5.586⎤⎥ μ N ⇒ μ = 4.793 μ N
1 1 5
μ=
2 2⎣ 2 ⎦

                                                                                
Head office  Branch office 
 
fiziks, H.No. 40 D, G.F, Jia Sarai,  Anand Institute of Mathematics, 
 
Near IIT, Hauz Khas, New Delhi‐16  28‐B/6, Jia Sarai, Near IIT 
 
Phone: 011‐26865455/+91‐9871145498 Hauz Khas, New Delhi‐16 
                                                   
                                             Website: www.physicsbyfiziks.com                                                                                          
                                                            Email: fiziks.physics@gmail.com                                                                     296 
fiziks
Institute for NET/JRF, GATE, IIT‐JAM, JEST, TIFR and GRE in PHYSICAL SCIENCES 
 
MATEMATICAL PHYSICS SOLUTIONS
GATE-2010
Q1. Consider an anti-symmetric tensor Pij with indices i and j running from 1 to 5. The
number of independent components of the tensor is
(a) 3 (b) 10 (c) 9 (d) 6
Ans: (b)
Solution: The number of independent components of the tensor

= (N − N ) = 1 (25 − 5) = 10 ∴N = 5
1 2
2 2
e 2 sin (z )
Q2. The value of the integral ∫ dz , where the contour C is the unit circle: z − 2 = 1 ,
C z2

is
(a) 2πi (b) 4πi (c) πi (d) 0
Ans: (d)
e z sin z
Solution: Pole is at z = 0 , Circle z − 2 = 1 ⇒ ∫ dz = 2πi × 0 = 0 .
C z2
⎛2 3 0⎞
⎜ ⎟
Q3. The eigenvalues of the matrix ⎜ 3 2 0 ⎟ are
⎜0 1 ⎟⎠
⎝ 0

(a) 5, 2, -2 (b) -5, -1, -1 (c) 5, 1, -1 (d) -5, 1, 1


Ans: (c)
Solution: The characteristic equation of the matrix A , A − λI = 0

2−λ 3 0
⇒ A − λI = 3 2−λ 0 = 0 ⇒ λ3 − 5λ2 − λ + 5 = 0 ⇒ λ = 5,1, − 1
0 0 1− λ

⎧0 for x < 3,
Q4. If f ( x ) = ⎨ then the Laplace transform of f(x) is
⎩x − 3 for x ≥ 3

(a) s −2 e sx (b) s 2 e sx (c) s −2 (d) s −2 e − sx

                                                                                
Head office  Branch office 
 
fiziks, H.No. 40 D, G.F, Jia Sarai,  Anand Institute of Mathematics, 
 
Near IIT, Hauz Khas, New Delhi‐16  28‐B/6, Jia Sarai, Near IIT 
 
Phone: 011‐26865455/+91‐9871145498 Hauz Khas, New Delhi‐16 
                                                   
                                             Website: www.physicsbyfiziks.com                                                                                          
                                                            Email: fiziks.physics@gmail.com                                                                     297 
fiziks
Institute for NET/JRF, GATE, IIT‐JAM, JEST, TIFR and GRE in PHYSICAL SCIENCES 
 
Ans: (d)
∞ 3 ∞ ∞
Solution: L{ f ( x )} = ∫ e − sx
f ( x ) dx = ∫ e −s x
f ( x ) dx + ∫ e − sx
f ( x ) dx = ∫ ( x − 3) e − sx dx
0 0 3 3

∞ ∞ ∞ ∞
e − sx ⎛ e − sx ⎞ 1 ⎡ e − sx ⎤
L{ f ( x )} = ( x − 3)
1
− ∫ 1 ⋅ ⎜⎜ ⎟⎟ dx = 0 − ∫ e − sx dx = ⎢ − 2 − sx
⎥ =s e
−s 3 3 ⎝ −s ⎠ s3 s ⎣ − s ⎦3

d2y
Q5. The solution of the differential equation for y (t ) : − y = 2 cosh(t ) , subject to the
dt 2
dy
initial conditions y (0 ) = 0 and = 0 , is
dt t = 0

cosh (t ) + t sinh (t ) (b) − sinh (t ) + t cosh (t )


1
(a)
2
(c) t cosh (t ) (d) t sinh (t )
Ans: (d)
Solution: For C.F (D 2 − 1)y = 0 ⇒ m = ±1 ⇒ C.F . = C1e t + C 2 e −t

⎛ e t + e −t ⎞
P.I . = 2
1
(2 cosh t ) = 2 2⎜⎜
1
⎟⎟ = 2
1
et + 2
1
( ) t t
e −t = e t + − e −t ( ) ( )
D −1 D −1 ⎝ 2 ⎠ D −1 D −1 2 2

t t t −t
⇒ y = C1e t + C 2 e −t + e − e ⇒ y (0 ) = 0 ⇒ C1 + C 2 = 0
2 2
dy t 1 t 1
= C1e t − C 2 e −t + e t + e t + e −t − e −t
dt 2 2 2 2
dy 1 1
= 0 ⇒ C1 − C 2 + 0 + + 0 − = 0 ⇒ C1 − C 2 = 0
dt t =0 2 2

Since C1 + C 2 = 0 and C1 − C 2 = 0 ⇒ C1 = 0, C 2 = 0 .
t t t −t
Thus ⇒ y = e − e ⇒ y = t sinh t
2 2

                                                                                
Head office  Branch office 
 
fiziks, H.No. 40 D, G.F, Jia Sarai,  Anand Institute of Mathematics, 
 
Near IIT, Hauz Khas, New Delhi‐16  28‐B/6, Jia Sarai, Near IIT 
 
Phone: 011‐26865455/+91‐9871145498 Hauz Khas, New Delhi‐16 
                                                   
                                             Website: www.physicsbyfiziks.com                                                                                          
                                                            Email: fiziks.physics@gmail.com                                                                     298 
fiziks
Institute for NET/JRF, GATE, IIT‐JAM, JEST, TIFR and GRE in PHYSICAL SCIENCES 
 
GATE-2011
Q6. Two matrices A and B are said to be similar if B = P-1AP for some invertible matrix P.
Which of the following statements is NOT TRUE?
(a) DetA = DetB (b) Trace of A = Trace of B
(c) A and B have the same eigenvectors (d) A and B have the same eigenvalues
Ans: (c)
Solution: If A and P be square matrices of the same type and if P be invertible then matrices A
and B = P-1AP have the same characteristic roots
Then B − λI = P −1 AP − P −1λIP = P −1 ( A − λI )P where I is identity matrix.

B − λI = P −1 ( A − λI )P = P −1 A − λI P = A − λI P −1 P = A − λI PP −1 = A − λI

Thus the matrices A and B (= P-1AP) have the same characteristic equation and hence
characteristic roots of eigen values. Since the sum of the eigen values of a matrix and
product of eigen values of a matrix is equal to the determinant of matrix hence third
alternative is incorrect.

Q7. If a force F is derivable from a potential function V(r), where r is the distance from the
origin of the coordinate system, it follows that

(a) ∇ × F = 0 (b) ∇ ⋅ F = 0 (c) ∇ V = 0 (d) ∇ 2 V = 0


Ans: (a)

Solution: Since F is derivative from potential V(r) and F = −∇V (r )

⇒ ∇ × F = −∇ × ∇V = 0 . ( )
Q8. A 3×3 matrix has elements such that its trace is 11 and its determinant is 36. The
eigenvalues of the matrix are all known to be positive integers. The largest eigenvalues of
the matrix is
(a) 18 (b) 12 (c) 9 (d) 6
Ans: (c)
Solution: We know that for any matrix
1. The product of eigenvalues is equal to determinant of that matrix.

                                                                                
Head office  Branch office 
 
fiziks, H.No. 40 D, G.F, Jia Sarai,  Anand Institute of Mathematics, 
 
Near IIT, Hauz Khas, New Delhi‐16  28‐B/6, Jia Sarai, Near IIT 
 
Phone: 011‐26865455/+91‐9871145498 Hauz Khas, New Delhi‐16 
                                                   
                                             Website: www.physicsbyfiziks.com                                                                                          
                                                            Email: fiziks.physics@gmail.com                                                                     299 
fiziks
Institute for NET/JRF, GATE, IIT‐JAM, JEST, TIFR and GRE in PHYSICAL SCIENCES 
 
2. λ1 + λ 2 + λ3 + ....... = Trace of matrix
λ1 + λ 2 + λ3 = 11 and λ1λ2 λ3 = 36 . Hence the largest eigen value of the matrix is 9.
Q9. The unit vector normal to the surface x2 + y2 – z = 1 at the point P(1, 1, 1) is
iˆ + ˆj − kˆ 2iˆ + ˆj − kˆ iˆ + 2 ˆj − kˆ 2iˆ + 2 ˆj − kˆ
(a) (b) (c) (d)
3 6 6 3

Ans: (d)
Solution: The equation of the system is f (x, y, z ) ≡ (x 2 + y 2 − z − 1) = 0

⎛∂ ∂ ˆ ∂ ˆ⎞ 2
The gradient of the above function is ∇f = ⎜⎜ iˆ + j + k ⎟⎟(x + y 2 − z − 1)
⎝ ∂x ∂y ∂z ⎠

= 2 xiˆ + 2 yˆj − lˆ

∇f 2iˆ + 2 ˆj − kˆ
Hence unit normal vector at (1, 1, 1) = .
∇f 3

Q10. Consider a cylinder of height h and radius a, closed at both ends, centered at the origin.

Let r = iˆx + ˆjy + kˆz be the position vector and n̂ a unit vector normal to the surface. The

surface integral ∫ r ⋅ nˆ ds over the closed surface of the cylinder is


S
z

O y

x
(a) 2πa2 (a + h) (b) 3πa2h (c) 2 πa2h (d) zero
Ans: (b)

( )
Solution: ∫ r.nˆ ds = ∫ ∇.r dτ = 3∫ dτ = 3πa 2 h
S V V

dy x
Q11. The solutions to the differential equation =− are a family of
dx y +1

(a) circles with different radii


(b) circles with different centres

                                                                                
Head office  Branch office 
 
fiziks, H.No. 40 D, G.F, Jia Sarai,  Anand Institute of Mathematics, 
 
Near IIT, Hauz Khas, New Delhi‐16  28‐B/6, Jia Sarai, Near IIT 
 
Phone: 011‐26865455/+91‐9871145498 Hauz Khas, New Delhi‐16 
                                                   
                                             Website: www.physicsbyfiziks.com                                                                                          
                                                            Email: fiziks.physics@gmail.com                                                                     300 
fiziks
Institute for NET/JRF, GATE, IIT‐JAM, JEST, TIFR and GRE in PHYSICAL SCIENCES 
 
(c) straight lines with different slopes
(d) straight lines with different intercepts on the y-axis
Ans: (a)
dy x x2 y2
Solution: =− ⇒ xdx + ydy + dy = 0 ⇒ + + y = C1 ⇒ x 2 + y 2 + 2 y = 2C1
dx y +1 2 2

⇒ ( x − 0 ) + ( y + 1) = 2C1 + 1 = C
2 2

which is family of circles with different radii.


z sin z
Q12. Which of the following statements is TRUE for the function f ( z ) = ?
( z − π )2
(a) f ( z ) is analytic everywhere in the complex plane

(b) f ( z ) has a zero at z = π

(c) f ( z ) has a pole of order 2 at z = π

(d) f ( z ) has a simple pole at z = π

Ans: (c)
z sin z
Solution: f ( z ) = has a pole of order 2 at z = π
(z −π )
2

Q13. Consider a counterclockwise circular contour z =1 about the origin. Let

z sin z
f (z ) = , then the integral ∫ f (z )dz over this contour is
( z − π )2
(a) –iπ (b) zero (c) iπ (d) 2iπ
Ans: (b)

                                                                                
Head office  Branch office 
 
fiziks, H.No. 40 D, G.F, Jia Sarai,  Anand Institute of Mathematics, 
 
Near IIT, Hauz Khas, New Delhi‐16  28‐B/6, Jia Sarai, Near IIT 
 
Phone: 011‐26865455/+91‐9871145498 Hauz Khas, New Delhi‐16 
                                                   
                                             Website: www.physicsbyfiziks.com                                                                                          
                                                            Email: fiziks.physics@gmail.com                                                                     301 
fiziks
Institute for NET/JRF, GATE, IIT‐JAM, JEST, TIFR and GRE in PHYSICAL SCIENCES 
 
GATE-2012
Q14. Identify the correct statement for the following vectors a = 3iˆ + 2 ˆj and b = iˆ + 2 ˆj

(a) The vectors a and b are linearly independent


(b) The vectors a and b are linearly dependent
(c) The vectors a and b are orthogonal
(d) The vectors a and b are normalized
Ans: (a)
Solution: If a = 3iˆ + 2 ˆj, b = iˆ + 2 ˆj are linearly dependent a + mb = 0, for some values of m but
3 + m = 0 and 2 + 2m = 0 do not have any solution. So they are linearly independent.
a ⋅ b ≠ 0 (Not orthogonal); a × b ≠ 0 (Not normalized)
Q15. The number of independent components of the symmetric tensor Aij with indices
i, j = 1, 2,3 is
(a) 1 (b) 3 (c) 6 (d) 9
⎡ A11 A12 A13 ⎤
Ans: (c) For symmetric tensor Aij = ⎢⎢ A21 A22 A23 ⎥⎥
⎢⎣ A31 A32 A33 ⎥⎦

∵ A12 = A21 , A23 = A32 , A13 = A33 , hence there are six independent components.

⎛0 1 0⎞
Q16. The eigenvalues of the matrix ⎜1 0 1 ⎟ are
⎜0 0 ⎟⎠
⎝ 1

(a) 0, 1, 1 (b) 0,− 2 , 2


1 1
(c) , ,0 (d) 2 , 2 ,0
2 2
Ans: (b)
−λ 1 0
Solution: A − λI = 0 ⇒ 1 −λ ( )
1 = 0 ⇒ −λ λ2 − 1 + λ = 0 ⇒ λ = 0, + 2 , − 2
0 1 −λ

                                                                                
Head office  Branch office 
 
fiziks, H.No. 40 D, G.F, Jia Sarai,  Anand Institute of Mathematics, 
 
Near IIT, Hauz Khas, New Delhi‐16  28‐B/6, Jia Sarai, Near IIT 
 
Phone: 011‐26865455/+91‐9871145498 Hauz Khas, New Delhi‐16 
                                                   
                                             Website: www.physicsbyfiziks.com                                                                                          
                                                            Email: fiziks.physics@gmail.com                                                                     302 
fiziks
Institute for NET/JRF, GATE, IIT‐JAM, JEST, TIFR and GRE in PHYSICAL SCIENCES 
 
GATE-2013

Q17. If A and B are constant vectors, then ∇ A ⋅ B × r ( ( )) is


(a) A ⋅ B (b) A × B (c) r (d) zero
Ans: (d)

Solution: Let A = A0 ( xˆ + yˆ + zˆ ) , B = B0 ( xˆ + yˆ + zˆ ) and r = xxˆ + yyˆ + zzˆ .

B × r = xˆ ( z − y )B0 − yˆ ( z − x )B0 + zˆ ( y − x )B0 ⇒ ∇ A ⋅ B × r = 0 . ( ( ))


16 z
Q18. For the function f ( z ) = , the residue at the pole z = 1 is (your answer
(z + 3)(z − 1)2
should be an integer) ____________.
Ans: 3

1 d 2−1 ⎡ ( z − 1) 16 z ⎤
2

Solution: At z = 1 pole is of order 2 so residue is ⎢ ⎥ =3


2 − 1 dz 2−1 ⎢ ( z + 3)( z − 1)2 ⎥
⎣ ⎦ z =1

⎡ 4 − 1 − 1⎤
Q19. The degenerate eigenvalue of the matrix ⎢⎢− 1 4 − 1⎥⎥ is (your answer should be an
⎢⎣− 1 − 1 4 ⎥⎦

integer) ____________
Ans: 2,5,5

⎡4 − λ −1 −1 ⎤ ⎡ 1 −1 −1 ⎤
⎢ −1 4−λ ⎥ ⎢
−1 ⎥ ⇒ (4 − λ ) ⎢0 5 − λ 0 ⎥⎥ = (4 − λ )(5 − λ )2 = 0 ⇒ λ = 2,5,5 .

⎢⎣ −1 −1 4 − λ ⎥⎦ ⎢⎣0 0 5 − λ ⎥⎦

Q20. The number of distinct ways of placing four indistinguishable balls into five
distinguishable boxes is ___________.
Ans: 120
Solution: 4 × C 45 =120 ways

                                                                                
Head office  Branch office 
 
fiziks, H.No. 40 D, G.F, Jia Sarai,  Anand Institute of Mathematics, 
 
Near IIT, Hauz Khas, New Delhi‐16  28‐B/6, Jia Sarai, Near IIT 
 
Phone: 011‐26865455/+91‐9871145498 Hauz Khas, New Delhi‐16 
                                                   
                                             Website: www.physicsbyfiziks.com                                                                                          
                                                            Email: fiziks.physics@gmail.com                                                                     303 
fiziks
Institute for NET/JRF, GATE, IIT‐JAM, JEST, TIFR and GRE in PHYSICAL SCIENCES 
 
GATE-2014
Q21. The unit vector perpendicular to the surface x 2 + y 2 + z 2 = 3 at the point (1, 1, 1) is
xˆ + yˆ − zˆ xˆ − yˆ − zˆ xˆ − yˆ + zˆ xˆ + yˆ + zˆ
(a) (b) (c) (d)
3 3 3 3
Ans: (d)
Solution: Let f = x 2 + y 2 + z 2 − 3 = 0 ⇒ ∇ f = 2 xxˆ + 2 yyˆ + 2 zzˆ

∇f 2 xˆ + 2 yˆ + 2 zˆ xˆ + yˆ + zˆ
⇒ nˆ = at (1,1,1) = =
∇f 12 3

Q22. The matrix


1 ⎡ 1 1 + i⎤
A= ⎢ ⎥ is
3 ⎣1 − i − 1 ⎦
(a) orthogonal (b) symmetric (c) anti-symmetric (d) Unitary
unitary
unitary A† A = I
Q23. The value of the integral
z2
∫C e z + 1 dz
where C is the circle z = 4 , is

(a) 2π i (b) 2π 2 i (c) 4π 3i (d) 4π 2 i


Ans. (c)
Solution: Pole e z = −1 ⇒ e z = ei( 2 m +1)π where m = 0,1, 2,3.....

φ ( z) π2
Residue Res z =iπ = = − iπ = π 2
φ′( z ) e

Similarly, z = −iπ , Res = π 2

∴I = 2π i (π 2 + π 2 ) = 4π 3i

                                                                                
Head office  Branch office 
 
fiziks, H.No. 40 D, G.F, Jia Sarai,  Anand Institute of Mathematics, 
 
Near IIT, Hauz Khas, New Delhi‐16  28‐B/6, Jia Sarai, Near IIT 
 
Phone: 011‐26865455/+91‐9871145498 Hauz Khas, New Delhi‐16 
                                                   
                                             Website: www.physicsbyfiziks.com                                                                                          
                                                            Email: fiziks.physics@gmail.com                                                                     304 
fiziks
Institute for NET/JRF, GATE, IIT‐JAM, JEST, TIFR and GRE in PHYSICAL SCIENCES 
 
d y
2
Q24. The solution of the differential equation 2 − y = 0 , subject to the boundary conditions
dt

y (0 ) = 1 and y (∞ ) = 0 is
(a) cos t + sin t (b) cosh t + sinh t
(c) cos t − sin t (d) cosh t − sinh t
Ans: (d)
Soluiton:
D 2 − 1 = 0 ⇒ D = ±1 ⇒ y ( t ) = c1et + c2 e − t

Applying boundary condition


y (0 ) = 1 ⇒ 1 = c1 + c2 and y ( ∞ ) = 0 ⇒ 0 = c1e∞ + c2 e −∞

⇒ c1 = 0, c2 = 1

⇒ y ( t ) = e −t ⇒ y ( t ) = cosh t − sinh t

GATE-2015
1
Q25. Consider a complex function f ( z ) = . Which one of the following
⎛ 1⎞
z ⎜ z + ⎟ cos ( zx )
⎝ 2⎠
statements is correct?
(a) f ( z ) has simple poles at z = 0 and z = −
1
2
(b) f ( z ) has second order pole at z = −
1
2
(c) f ( z ) has infinite number of second order poles
(d) f ( z ) has all simple poles
Ans.: (a)
1
Solution: f ( z) =
⎛ 1⎞
z ⎜ z + ⎟ cos ( zπ )
⎝ 2⎠

∵ lim ( z − a ) f ( z ) = finite and ≠ 0


n
For nth order pole
z →a

                                                                                
Head office  Branch office 
 
fiziks, H.No. 40 D, G.F, Jia Sarai,  Anand Institute of Mathematics, 
 
Near IIT, Hauz Khas, New Delhi‐16  28‐B/6, Jia Sarai, Near IIT 
 
Phone: 011‐26865455/+91‐9871145498 Hauz Khas, New Delhi‐16 
                                                   
                                             Website: www.physicsbyfiziks.com                                                                                          
                                                            Email: fiziks.physics@gmail.com                                                                     305 
fiziks
Institute for NET/JRF, GATE, IIT‐JAM, JEST, TIFR and GRE in PHYSICAL SCIENCES 
 
At z = 0
lim zf ( z ) = finite ⇒ z = 0 is a simple pole.
z →0

1
At z = −
2
2
⎛ 1⎞ ⎛ 1⎞
⎜z+ ⎟ ⎜z+ ⎟
lim ⎝ 2⎠
= lim ⎝ 2⎠
= lim
1
⎛ 1⎞ z cos zπ z →− 1.cos zπ + z.π ( − sin zπ )
1 1 1
⎟ cos zπ
z →− z →−
2 z⎜z + 2 2
⎝ 2⎠
1 1 2
= lim1
= = = finite
z →− cos zπ − zπ sin zπ π π
2
2
1
⇒ f ( z ) has second order pole at z = −
2
3
Q26. The value of ∫ t 2δ (3t − 6 )dt is_______________ (upto one decimal place)
0

Ans.: 1.33
3 3 3
1 4
Solution: ∫ t δ ( 3t − 6 ) dt = ∫ t δ ⎡⎣3 ( t − 2 ) ⎤⎦ dt = ∫ t 2δ ( t − 2 ) dt =
2 2

0 0
30 3

If f ( x ) = e − x and g ( x ) = x e − x , then
2 2
Q27.

(a) f and g are differentiable everywhere


(b) f is differentiable everywhere but g is not
(c) g is differentiable everywhere but f is not
(d) g is discontinuous at x = 0
Ans. (b)

Solution: f ( x) = e − x is differentiable but g ( x) = x e − x is not differentiable.


2 2

⎧⎪− xe − x ; x < 0
2

g ( x) = ⎨
− x2
⎪⎩ xe ; x > 0
2
( x −h)
Left hand Limit lim g ( x − h ) = − ( x − h ) e
h →0

                                                                                
Head office  Branch office 
 
fiziks, H.No. 40 D, G.F, Jia Sarai,  Anand Institute of Mathematics, 
 
Near IIT, Hauz Khas, New Delhi‐16  28‐B/6, Jia Sarai, Near IIT 
 
Phone: 011‐26865455/+91‐9871145498 Hauz Khas, New Delhi‐16 
                                                   
                                             Website: www.physicsbyfiziks.com                                                                                          
                                                            Email: fiziks.physics@gmail.com                                                                     306 
fiziks
Institute for NET/JRF, GATE, IIT‐JAM, JEST, TIFR and GRE in PHYSICAL SCIENCES 
 
2
( x+h)
Right hand Limit lim g ( x + h ) = ( x + h ) e
h →0

⇒ lim g ( x − h ) ≠ lim g ( x + h )
h →0 h →o

Q28. Consider w = f ( z ) = u ( x, y ) + iv( x, y ) to be an analytic function in a domain D . Which


one of the following options is NOT correct?
(a) u ( x, y ) satisfies Laplace equation in D
(b) v( x, y ) satisfies Laplace equation in D
z2

(c) ∫ f (z )dz is dependent on the choice of the contour between z and z


z1
1 2 in D

(d) f ( z ) can be Taylor expended in D


Ans.: (c)
Solution: w = f ( z ) = u ( x, y ) + iv ( x, y ) to be an analytic function in a domain D, ∫ zz12 f ( z )dz is

independent on the choice of the contour between z1 and z2 in D .

⎧+ 1 for t > 0
Q29. The Heaviside function is defined as H (t ) = ⎨ and its Fourier transform is
⎩− 1 for t < 0

given by − 2i / ω . The Fourier transform of


1
[H (t + 1 / 2) − H (t − 1 / 2)] is
2
⎛ω ⎞ ⎛ω ⎞
sin ⎜ ⎟ cos⎜ ⎟
(a) ⎝2⎠ (b) ⎝2⎠
ω ω
2 2
⎛ω ⎞
(c) sin ⎜ ⎟ (d) 0
⎝2⎠
Ans.: (a)

Solution: H ( f ) = ∫ h (t ) e
− i 2π ft
dt
−∞

2i
For a function h ( t ) , H ( f ) = −
ω
                                                                                
Head office  Branch office 
 
fiziks, H.No. 40 D, G.F, Jia Sarai,  Anand Institute of Mathematics, 
 
Near IIT, Hauz Khas, New Delhi‐16  28‐B/6, Jia Sarai, Near IIT 
 
Phone: 011‐26865455/+91‐9871145498 Hauz Khas, New Delhi‐16 
                                                   
                                             Website: www.physicsbyfiziks.com                                                                                          
                                                            Email: fiziks.physics@gmail.com                                                                     307 
fiziks
Institute for NET/JRF, GATE, IIT‐JAM, JEST, TIFR and GRE in PHYSICAL SCIENCES 
 
For h ( t − t0 ) , Fourier Transform is e − i 2π ft0
H(f)

Shifting theorem

1 ⎡ ⎛ 1 ⎞ ⎛ 1 ⎞ ⎤ 1 ⎡ i ω2 ω
− i ⎤ −2i 1 ⎡ i ω2 ω
− i ⎤ −2i
For ⎢ h ⎜ t + −
⎟ ⎜h t − ⎟⎥ = ⎢ e − e 2
⎥ = ⎢ e − e 2
⎥ ×i
2 ⎣ ⎝ 2 ⎠ ⎝ 2 ⎠⎦ 2 ⎣ ⎦ ω 2i ⎣ ⎦ ω

⎛ω ⎞
sin ⎜ ⎟
1
The Fourier transform of ⎡⎣ H ( t + 1/ 2 ) − H ( t − 1/ 2 ) ⎤⎦ = ⎝2⎠ .
2 ω
2
m2
A function y ( z ) satisfies the ordinary differential equation y′′ +
1
Q30. y′ − 2 y = 0, where
z z
m = 0, 1, 2, 3, ..... Consider the four statements P, Q, R, S as given below.

P: z m and z − m are linearly independent solutions for all values of m


Q: z m and z − m are linearly independent solutions for all values of m > 0
R: ln z and 1 are linearly independent solutions for m = 0
S: z m and ln z are linearly independent solutions for all values of m
The correct option for the combination of valid statements is
(a) P, R and S only (b) P and R only (c) Q and R only (d) R and S only
Ans.: (c)
1 m2
Solution: y ′′ + y − 2 y = 0 ⇒ z 2 y′′ + zy′ − m 2 y = 0

z z
d
m = 0,1, 2,3,.... z = ex , D =
dx
If m = 0 ; z 2 y′′ + zy′ = 0

⎡⎣ D ( D − 1) + D ⎤⎦ y = 0 ⇒ ⎡⎣ D 2 − D + D ⎤⎦ y = 0 ⇒ D 2 − m 2 y = 0 ( )
y = c1 + c2 x D = ±m

y = c1 + c2 ln z c1e mx + c2 e − mx

R is correct. c1em log z + c2 e− m log z

                                                                                
Head office  Branch office 
 
fiziks, H.No. 40 D, G.F, Jia Sarai,  Anand Institute of Mathematics, 
 
Near IIT, Hauz Khas, New Delhi‐16  28‐B/6, Jia Sarai, Near IIT 
 
Phone: 011‐26865455/+91‐9871145498 Hauz Khas, New Delhi‐16 
                                                   
                                             Website: www.physicsbyfiziks.com                                                                                          
                                                            Email: fiziks.physics@gmail.com                                                                     308 
fiziks
Institute for NET/JRF, GATE, IIT‐JAM, JEST, TIFR and GRE in PHYSICAL SCIENCES 
 
or if m ≠ 0, m > 0

y = c1 cosh ( m log ( z ) ) + ic2 sinh ( m log ( x ) ) ⇒m>0

GATE-2016
dy
Q31. Consider the linear differential equation = xy . If y = 2 at x = 0 , then the value of y at
dx
x = 2 is given by
(a) e −2 (b) 2e −2 (c) e 2 (d) 2e 2
Ans.: (d)
dy 1 x2
= xy ⇒ dy = xdx ⇒ ln y = + ln c ⇒ y = ce x / 2
2
Solution:
dx y 2

If y = 2 at x = 0 ⇒ c = 2 ⇒ y = 2e x
2
/2
.

The value of y at x = 2 is given by y = 2e2


Q32. Which of the following is an analytic function of z everywhere in the complex plane?

(a) z 2 (b) z *( ) 2
(c) z
2
(d) z

Ans.: (a)

Solution: z 2 = ( x + iy ) = x 2 − y 2 + i ( 2 xy ) ⇒ u = x 2 − y 2 and v = 2 xy
2

∂u ∂v ∂v ∂u
Cauchy Riemann equations = = 2 x, =− = 2 y satisfies.
∂x ∂y ∂x ∂y

The direction of ∇f for a scalar field f ( x, y, z ) = x − xy + z 2 at the point P(1,1,2 ) is


1 2 1
Q33.
2 2

(a)
(− ˆj − 2kˆ) (b)
(− ˆj + 2kˆ) (c)
( ˆj − 2kˆ) (d)
( ˆj + 2kˆ)
5 5 5 5
Ans.: (b)
⎛ ∇f ⎞ − ˆj + 2kˆ
Solution: ∇f = ( x − y ) iˆ − xjˆ + zkˆ ⇒ nˆ = ⎜ ⎟ =
⎜ ∇f ⎟ 5
⎝ ⎠1,1,2

                                                                                
Head office  Branch office 
 
fiziks, H.No. 40 D, G.F, Jia Sarai,  Anand Institute of Mathematics, 
 
Near IIT, Hauz Khas, New Delhi‐16  28‐B/6, Jia Sarai, Near IIT 
 
Phone: 011‐26865455/+91‐9871145498 Hauz Khas, New Delhi‐16 
                                                   
                                             Website: www.physicsbyfiziks.com                                                                                          
                                                            Email: fiziks.physics@gmail.com                                                                     309 
fiziks
Institute for NET/JRF, GATE, IIT‐JAM, JEST, TIFR and GRE in PHYSICAL SCIENCES 
 
Q34. A periodic function f ( x ) of period 2π is defined in the interval (− π < x < π )

⎧− 1, − π < x < 0
f (x ) = ⎨
⎩ 1, 0< x<π

The appropriate Fourier series expansion for f ( x ) !is

⎛4⎞
(a) f ( x ) = ⎜ ⎟[sin x + (sin 3x ) / 3 + (sin 5 x ) / 5 + ...]
⎝π ⎠
⎛4⎞
(b) f ( x ) = ⎜ ⎟[sin x − (sin 3x ) / 3 + (sin 5 x ) / 5 − ..]
⎝π ⎠
⎛4⎞
(c) f ( x ) = ⎜ ⎟[cos x + (cos 3x ) / 3 + (cos 5 x ) / 5 + ...]
⎝π ⎠
⎛4⎞
(d) f ( x ) = ⎜ ⎟[cos x − (cos 3x ) / 3 + (cos 5 x ) / 5 − ...]
⎝π ⎠
Ans.: (a)
⎧− 1, − π < x < 0
Solution: f ( x ) = ⎨
⎩ 1, 0< x<π

Let f ( x ) = a0 + ∑ ( an cos nx + bn sin nx )
n =1

1 π
∵ a0 =
2π ∫ π f ( x )dx

π π
∫ π f ( x )dx = 2π ⎡⎢⎣ ∫ π ( −1) dx + ∫ (1) dx ⎤⎥⎦ = 2π ⎡⎣[ − x] π + [ x] ⎤⎦ = 0
1 1 0 1 π
⇒ a0 =
0

2π − − 0 − 0

This can also be seen without integration, since the area under the curve of f (x ) between
−π to π is zero.
1 π
f ( x ) cos nxdx
π∫π
∵ an =

π
1 ⎡ ⎧ sin nx ⎫ ⎧ sin nx ⎫ ⎤
0
1⎡ 0 π

( −1) cos nxdx + ∫0 (1) cos nxdx ⎥⎦ = ⎢ − ⎨
π ⎢⎣ ∫−π
⇒ an = ⎬ +⎨ ⎬ ⎥=0
π ⎢⎣ ⎩ n ⎭−π ⎩ n ⎭0 ⎦⎥

1 π
f ( x ) sin nxdx
π∫π
∵ bn =

                                                                                
Head office  Branch office 
 
fiziks, H.No. 40 D, G.F, Jia Sarai,  Anand Institute of Mathematics, 
 
Near IIT, Hauz Khas, New Delhi‐16  28‐B/6, Jia Sarai, Near IIT 
 
Phone: 011‐26865455/+91‐9871145498 Hauz Khas, New Delhi‐16 
                                                   
                                             Website: www.physicsbyfiziks.com                                                                                          
                                                            Email: fiziks.physics@gmail.com                                                                     310 
fiziks
Institute for NET/JRF, GATE, IIT‐JAM, JEST, TIFR and GRE in PHYSICAL SCIENCES 
 
π
⇒ bn = ⎡ ∫ ( −1) sin nxdx + ∫ (1) sin nxdx ⎤
1 0

π ⎢⎣ −π 0 ⎥⎦

⎧ cos nx ⎫ ⎤ 1 ⎡ 1 ( −1) ( −1) 1 ⎤ 1 ⎡ 2 2 ( −1)n ⎤


π
1 ⎡ ⎧ cos nx ⎫
0 n n

⇒ bn = ⎢ ⎨ ⎬ −⎨ ⎬ ⎥= ⎢ − − + ⎥= ⎢ − ⎥
π ⎢⎣ ⎩ n ⎭−π ⎩ n ⎭0 ⎥⎦ π ⎢⎣ n n n n ⎥⎦ π ⎢⎣ n n ⎥⎦

4
If n is even bn = 0 and If n is odd bn = .

4⎡ 1 1 ⎤
Thus Fourier series is f ( x) = ⎢ sin x + sin 3x + sin 5 x + ...⎥
π⎣ 3 5 ⎦

                                                                                
Head office  Branch office 
 
fiziks, H.No. 40 D, G.F, Jia Sarai,  Anand Institute of Mathematics, 
 
Near IIT, Hauz Khas, New Delhi‐16  28‐B/6, Jia Sarai, Near IIT 
 
Phone: 011‐26865455/+91‐9871145498 Hauz Khas, New Delhi‐16 
                                                   
                                             Website: www.physicsbyfiziks.com                                                                                          
                                                            Email: fiziks.physics@gmail.com                                                                     311 
fiziks
Institute for NET/JRF, GATE, IIT‐JAM, JEST, TIFR and GRE in PHYSICAL SCIENCES 
 
CLASSICAL MECHANICS SOLUTIONS
GATE- 2010
Q1. For the set of all Lorentz transformations with velocities along the x-axis consider the two
statements given below:
P: If L is a Lorentz transformation then, L-1 is also a Lorentz transformation.
Q: If L1 and L2 are Lorentz transformations then, L1L2 is necessarily a Lorentz
transformation.
Choose the correct option
(A) P is true and Q is false (B) Both P and Q are true
(C) Both P and Q are false (D) P is false and Q is true
Ans: (b)
1 2 λ 3
Q2. A particle is placed in a region with the potential V (x ) = kx − x , where k, λ > 0.
2 3
Then,
k
(A) x = 0 and x = are points of stable equilibrium
λ
k
(B) x = 0 is a point of stable equilibrium and x = is a point of unstable equilibrium
λ
k
(C) x = 0 and x = are points of unstable equilibrium
λ
(D) There are no points of stable or unstable equilibrium
Ans: (b)
1 2 λx 3 ∂V k
Solution: V = kx − ⇒ = kx − λx 2 = 0 ⇒ x = 0, x = .
2 3 ∂x λ
∂ 2V
⇒ = k − 2λx
∂x 2
∂ 2V k ∂ 2V
⇒ At x = 0, = + ve (Stable) and ⇒ At x = , = −ve (unstable)
∂x 2 λ ∂x 2

                                                                                
Head office  Branch office 
 
fiziks, H.No. 40 D, G.F, Jia Sarai,  Anand Institute of Mathematics, 
 
Near IIT, Hauz Khas, New Delhi‐16  28‐B/6, Jia Sarai, Near IIT 
 
Phone: 011‐26865455/+91‐9871145498 Hauz Khas, New Delhi‐16 
                                                   
                                             Website: www.physicsbyfiziks.com                                                                                          
                                                            Email: fiziks.physics@gmail.com                                                                     312 
fiziks
Institute for NET/JRF, GATE, IIT‐JAM, JEST, TIFR and GRE in PHYSICAL SCIENCES 
 
0
Q3. A π meson at rest decays into two photons, which move along the x-axis. They are both
detected simultaneously after a time, t = 10s. In an inertial frame moving with a velocity
V = 0.6c in the direction of one of the photons, the time interval between the two
detections is
(A) 15 s (B) 0 s (C) 10 s (D) 20 s
Ans: (a)
v v
1+ 1−
c 1 + 0.6 c
Solution: t1 = t 0 = 10 = 10 × 2 = 20sec , t 2 = t 0
v 1 − 0.6 v
1− 1+
c c

1 − 0.6 1
= 10 = 10 × = 5sec
1 + 0.6 2
⇒ t1 − t 2 = 15sec
Statement for Linked Answer Questions 4 and 5:
1 2 2
The Lagrangian for a simple pendulum is given by L = ml θ − mgl (1 − cos θ )
2
Q4. Hamilton’s equations are then given by
pθ pθ
(A) pθ = −mgl sin θ ; θ = (B) pθ = mgl sin θ ; θ =
ml 2 ml 2
pθ ⎛g⎞ pθ
(C) pθ = −mθ ; θ = (D) pθ = −⎜ ⎟θ ; θ=
m ⎝l⎠ ml
Ans: (b)
Pθ2 ∂H ∂H P
Solution: H = + mgl (1 − cos θ ) ⇒ = −Pθ ⇒ Pθ = mglsin θ; = θ ⇒ θ = θ 2 .Q5. The
2ml 2
∂θ ∂Pθ ml

Poisson bracket between θ and θ is

{ }
(A) θ ,θ = 1 { }
(B) θ ,θ =
1
ml 2

{ }
(C) θ ,θ =
1
m
{ }
(D) θ ,θ =
g
l
Ans: (b)
                                                                                
Head office  Branch office 
 
fiziks, H.No. 40 D, G.F, Jia Sarai,  Anand Institute of Mathematics, 
 
Near IIT, Hauz Khas, New Delhi‐16  28‐B/6, Jia Sarai, Near IIT 
 
Phone: 011‐26865455/+91‐9871145498 Hauz Khas, New Delhi‐16 
                                                   
                                             Website: www.physicsbyfiziks.com                                                                                          
                                                            Email: fiziks.physics@gmail.com                                                                     313 
fiziks
Institute for NET/JRF, GATE, IIT‐JAM, JEST, TIFR and GRE in PHYSICAL SCIENCES 
 

{ } ⎧ P ⎫ P 1 ⎛ ∂θ ∂θ
θ ,θ = ⎨θ , θ 2 ⎬ where θ = θ 2 . ⇒ 2 ⎜⎜ −
∂θ ∂Pθ ⎞ 1 1
⎟⎟ = 1 ⋅ 2 − 0 = 2 .
⎩ ml ⎭ ml ml ⎝ ∂θ ∂Pθ ∂Pθ ∂θ ⎠ ml ml

GATE- 2011
1+ q
Q6. A particle is moving under the action of a generalized potential V (q, q ) = . The
q2

magnitude of the generalized force is


2(1 + q ) 2(1 − q ) 2 q
(A) (B) (C) (D)
q3 q3 q3 q3

Ans: (c)
d ⎛ ∂V ⎞ ∂V 2
Solution: ⎜ ⎟− = Fq ⇒ Fq = 3 .
dt ⎝ ∂q ⎠ ∂q q
Q7. Two bodies of mass m and 2m are connected by a spring constant k. The frequency of the
normal mode is
(A) 3k / 2m (B) k/m (C) 2k / 3m (D) k / 2m

Ans: (a)

k k 3k 2mm 2m
Solution: m k
2m ω = = = where reduce mass μ = = .
μ 2m 2m 2m + m 3
3
Q8. Let (p, q) and (P, Q) be two pairs of canonical variables. The transformation
Q = q α cos(βp ) , P = q α sin(βp )

is canonical for
(A) α = 2, β = 1/2 (B) α = 2, β =2 (C) α = 1, β = 1 (D) α = 1/2, β = 2
Ans: (d)
∂Q ∂P ∂Q ∂P
Solution: ⋅ − ⋅ =1
∂q ∂p ∂p ∂q

⇒ αq α −1 cos(βp ) × q α β cos(βp ) − q α β (− sin (βp )) × αq α −1 sin (βp ) = 1

αq 2α −1 β (cos 2 βp + sin 2 βp ) = 1 ⇒ αβq 2α −1 = 1 ⇒ α = , β = 2 .


1
2

                                                                                
Head office  Branch office 
 
fiziks, H.No. 40 D, G.F, Jia Sarai,  Anand Institute of Mathematics, 
 
Near IIT, Hauz Khas, New Delhi‐16  28‐B/6, Jia Sarai, Near IIT 
 
Phone: 011‐26865455/+91‐9871145498 Hauz Khas, New Delhi‐16 
                                                   
                                             Website: www.physicsbyfiziks.com                                                                                          
                                                            Email: fiziks.physics@gmail.com                                                                     314 
fiziks
Institute for NET/JRF, GATE, IIT‐JAM, JEST, TIFR and GRE in PHYSICAL SCIENCES 
 
Q9. Two particles each of rest mass m collide head-on and stick together. Before collision, the
speed of each mass was 0.6 times the speed of light in free space. The mass of the final
entity is
(A) 5m / 4 (B) 2m (C) 5m / 2 (D) 25 m / 8
Ans: (c)
Solution: From conservation of energy
mc 2 mc 2 2mc 2
+ = m1c 2 ⇒ = m1c 2
2
v2
v2
v
1− 1− 1−
c2 c2 c2

Since v = 0.6c ⇒ m1 = 5m / 2

GATE- 2012
Q10. In a central force field, the trajectory of a particle of mass m and angular momentum L in
plane polar coordinates is given by,
1 m
= (1 + ε cos θ )
r L2
where, ε is the eccentricity of the particle’s motion. Which one of the following choice
for ε gives rise to a parabolic trajectory?
(a) ε = 0 (b) ε = 1 (c) 0 < ε < 1 (d) ε > 1
Ans: (b)
l m
Solution: = (1 + ε cos θ ) for parabolic trajectory ε = 1 .
r l2
Q11. A particle of unit mass moves along the x-axis under the influence of a potential,
V ( x ) = x( x − 2) . The particle is found to be in stable equilibrium at the point x = 2. The
2

time period of oscillation of the particle is


π 3π
(a) (b) π (c) (d) 2π
2 2
Ans: (b)

                                                                                
Head office  Branch office 
 
fiziks, H.No. 40 D, G.F, Jia Sarai,  Anand Institute of Mathematics, 
 
Near IIT, Hauz Khas, New Delhi‐16  28‐B/6, Jia Sarai, Near IIT 
 
Phone: 011‐26865455/+91‐9871145498 Hauz Khas, New Delhi‐16 
                                                   
                                             Website: www.physicsbyfiziks.com                                                                                          
                                                            Email: fiziks.physics@gmail.com                                                                     315 
fiziks
Institute for NET/JRF, GATE, IIT‐JAM, JEST, TIFR and GRE in PHYSICAL SCIENCES 
 
∂V 2
V ( x ) = x( x − 2 ) ⇒ = ( x − 2 ) + 2 x( x − 2 ) = 0 ⇒ x = 2, x =
2 2

∂x 3

∂ 2V ∂ 2V
= 2( x − 2 )2
+ 2( x − 2 ) + 2 x ⇒ = 2× 2 = 4
∂x 2 ∂x 2

∂ 2V 2π
⇒ω = ⇒ω = =2 ⇒T =π .
∂x 2 x=2
T

Q12. A rod of proper length l0 oriented parallel to the x-axis moves with speed 2c/3 along the
x-axis in the S-frame, where c is the speed of the light in free space. The observer is also
moving along the x-axis with speed c/2 with respect to the S-frame. The length of the rod
as measured by the observer is
(a) 0.35l0 (b) 0.48l0 (c) 0.87l0 (d) 0.97l0
Ans: (d)

u2x
Solution: l = l0 1 − = 0.97 l0
c2
Q13. A particle of mass m is attached to fixed point O by a weightless inextensible string of
length a. It is rotating under the gravity as shown in the figure. The
z
Lagrangian of the particle is
θ
1
( )
L(θ , φ ) = ma 2 θ 2 + sin 2 θφ 2 − mga cos θ where θ and φ are the
2 aθ

polar angles. The Hamiltonian of the particles is m

1 ⎛ 2 pφ2 ⎞ g
(a) H = ⎜ pθ + ⎟ − mga cos θ (b)
2ma 2 ⎜ sin 2
θ ⎟
⎝ ⎠

1 ⎛ 2 pφ2 ⎞
H= ⎜ pθ + ⎟ + mga cosθ
2ma 2 ⎜ sin 2
θ ⎟
⎝ ⎠

(c) H =
1
2ma 2
( pθ2 + pφ2 ) − mga cos θ (d) H =
1
2ma 2
( pθ2 + pφ2 ) + mga cos θ

Ans: (b)
                                                                                
Head office  Branch office 
 
fiziks, H.No. 40 D, G.F, Jia Sarai,  Anand Institute of Mathematics, 
 
Near IIT, Hauz Khas, New Delhi‐16  28‐B/6, Jia Sarai, Near IIT 
 
Phone: 011‐26865455/+91‐9871145498 Hauz Khas, New Delhi‐16 
                                                   
                                             Website: www.physicsbyfiziks.com                                                                                          
                                                            Email: fiziks.physics@gmail.com                                                                     316 
fiziks
Institute for NET/JRF, GATE, IIT‐JAM, JEST, TIFR and GRE in PHYSICAL SCIENCES 
 
1
Solution: H = Pθθ + Pφφ − L = Pθ θ + Pφ φ − ma 2 θ 2 + sin 2 θφ 2 + mga cos θ
2
( )
∂L P ∂L Pφ
= Pθ ⇒ ma 2 θ = Pθ ⇒ θ = θ 2 and Pφ = = ma 2 sin 2 θφ ⇒ φ =
∂θ ma ∂φ ma 2 sin 2 θ

Put the value of θ and φ

⎛ 2
⎞ ⎞⎟
2
2 ⎛
Pθ Pφ 1 2 ⎜ ⎛ Pθ ⎞

H = Pθ × + Pφ × − − ma ⎜ ⎟ + sin θ ⎜⎜ ⎟ + mga cosθ
ma 2 ma 2 sin 2 θ 2 ⎜ ⎝ ma 2 ⎠ ⎝ ma 2 sin 2 θ ⎟⎠ ⎟
⎝ ⎠

Pθ2 Pθ2 Pφ2 Pφ2


H= − + − + mga cosθ
ma 2 2ma 2 ma 2 sin 2 θ 2ma 2 sin 2 θ

1 ⎛ 2 Pθ2 ⎞
H= ⎜ Pθ + ⎟ + mga cos θ
2ma 2 ⎜ sin 2 θ ⎟
⎝ ⎠

Statement for Linked Answer Questions 14 and 15:


Q14. A particle of mass m slides under the gravity without friction along the parabolic path
y = ax 2 axis shown in the figure. Here a is a constant.
y

x
The Lagrangian for this particle is given by

m(1 + 4a 2 x 2 )x 2 − mgax 2
1 2 1
(a) L = mx − mgax 2 (b) L =
2 2

(c) L =
1 2
2
mx + mgax 2 (d) L =
1
2
( )
m 1 + 4a 2 x 2 x 2 + mgax 2

Ans: (d)

                                                                                
Head office  Branch office 
 
fiziks, H.No. 40 D, G.F, Jia Sarai,  Anand Institute of Mathematics, 
 
Near IIT, Hauz Khas, New Delhi‐16  28‐B/6, Jia Sarai, Near IIT 
 
Phone: 011‐26865455/+91‐9871145498 Hauz Khas, New Delhi‐16 
                                                   
                                             Website: www.physicsbyfiziks.com                                                                                          
                                                            Email: fiziks.physics@gmail.com                                                                     317 
fiziks
Institute for NET/JRF, GATE, IIT‐JAM, JEST, TIFR and GRE in PHYSICAL SCIENCES 
 
1
Solution: Equation of constrain is given by y = ax 2 , K.E T = m ( x 2 + y 2 )
2
1 1
y = 2axx ⇒ T = m ( x 2 + 4ax 2 x 2 ) = mx 2 (1 + 4ax 2 )
2 2

V = − mgy = − mgax 2 . Since particle is moving downward direction so potential V is

negative.

∵ L = T −V ⇒ L =
1
2
( )
m 1 + 4a 2 x 2 x 2 + mgax 2

Q15. The Lagrange’s equation of motion of the particle for above question is given by
(a) x = 2 gax (b) m (1 + 4a 2 x 2 ) x = −2mgax − 4ma 2 xx 2

( )
(c) m 1 + 4a 2 x 2 x = 2mgax + 4ma 2 xx 2 (d) x = −2 gax
Ans: (c)
d ⎛ dL ⎞ dL
Solution: ⎜ ⎟= ⇒ m(1 + 4a 2 x 2 ) x = 4ma 2 xx 2 + 2mgax
dt ⎝ dx ⎠ dx

GATE- 2013
Q16. In the most general case, which one of the following quantities is NOT a second order
tensor?
(a) Stress (b) Strain
(c) Moment of inertia (d) Pressure
Ans: (b)
Solution: Strain is not a tensor.
Q17. An electron is moving with a velocity of 0.85c in the same direction as that of a moving
photon. The relative velocity of the electron with respect to photon is
(a) c (b) − c
(c) 0.15c (d) − 0.15c
Ans: (b)

                                                                                
Head office  Branch office 
 
fiziks, H.No. 40 D, G.F, Jia Sarai,  Anand Institute of Mathematics, 
 
Near IIT, Hauz Khas, New Delhi‐16  28‐B/6, Jia Sarai, Near IIT 
 
Phone: 011‐26865455/+91‐9871145498 Hauz Khas, New Delhi‐16 
                                                   
                                             Website: www.physicsbyfiziks.com                                                                                          
                                                            Email: fiziks.physics@gmail.com                                                                     318 
fiziks
Institute for NET/JRF, GATE, IIT‐JAM, JEST, TIFR and GRE in PHYSICAL SCIENCES 
 
Q18. The Lagrangian of a system with one degree of freedom q is given by L = αq 2 + βq 2 ,
where α and β are non-zero constants. If p q denotes the canonical momentum

conjugate to q then which one of the following statements is CORRECT?


(a) p q = 2 β q and it is a conserved quantity.

(b) p q = 2 β q and it is not a conserved quantity.

(c) p q = 2α q and it is a conserved quantity.

(d) p q = 2αq and it is not a conserved quantity.

Ans: (d)
∂L ∂L
Solution: = pq but ≠0
∂q ∂q
Q19. The relativistic form of Newton’s second law of motion is

mc dv m c 2 − v 2 dv
(a) F = (b) F =
c 2 − v 2 dt c dt

mc 2 dv c 2 − v 2 dv
(c) F = (d) F = m
c 2 − v 2 dt c2 dt
Ans:
mv dP dv 1 ⎛ 1⎞ 1 −2v dv
Solution: P = ⇒F= =m ⋅ + mv ⎜ − ⎟ ⋅ 3/ 2
⋅ 2
v2 dt dt v2 ⎝ 2 ⎠ ⎛ v2 ⎞ c dt
1− 1− 2 ⎜ 1− 2 ⎟
c2 c ⎝ c ⎠

⎛ ⎞ ⎛ ⎞
⎜ v 2 ⎟
2 ⎜ 2 ⎟
dv 1 ⎜ 1 dv ⎜ 1− v ⎟
⇒F =m 1+ c ⎟=m 2c 2
⎜ ⎟
v 2 ⎜ 2 ⎛1 − v ⎞ ⎟ dt ⎜ ⎛ v 2 ⎞ 3 2
2
dt ⎟
1 − 2 ⎜⎜ ⎜ ⎟
2 ⎟⎟
c ⎝ ⎝ c ⎠⎠ ⎜ ⎜1 − c 2 ⎟ ⎟
⎝⎝ ⎠ ⎠

                                                                                
Head office  Branch office 
 
fiziks, H.No. 40 D, G.F, Jia Sarai,  Anand Institute of Mathematics, 
 
Near IIT, Hauz Khas, New Delhi‐16  28‐B/6, Jia Sarai, Near IIT 
 
Phone: 011‐26865455/+91‐9871145498 Hauz Khas, New Delhi‐16 
                                                   
                                             Website: www.physicsbyfiziks.com                                                                                          
                                                            Email: fiziks.physics@gmail.com                                                                     319 
fiziks
Institute for NET/JRF, GATE, IIT‐JAM, JEST, TIFR and GRE in PHYSICAL SCIENCES 
 
Q20. Consider two small blocks, each of mass M, attached to two identical springs. One of the
springs is attached to the wall, as shown in the figure. The spring constant of each spring
is k . The masses slide along the surface and the friction is negligible. The frequency of
one of the normal modes of the system is,

3+ 2 k
(a)
2 M

3+ 3 k
(b)
2 M
k k
3+ 5 k M M
(c)
2 M

3+ 6 k
(d)
2 M
Ans: (c)
1 2 1 2
Solution: T = mx1 + mx 2 ,
2 2

V =
1 2 1
2
1 1 1
(
kx1 + k ( x 2 − x1 ) = kx12 + k x 22 + x12 − 2 x 2 x1 = k 2 x 2 + x 2 − 2 x 2 x1
2
2

2 2 2
) ( )
⎛m 0 ⎞ ⎛ 2k −k ⎞
T =⎜ ⎟; V = ⎜ ⎟
⎝ 0 m⎠ ⎝ −k k ⎠

2k − ω 2 m −k 3+ 5 k
= 0 ⇒ ( 2 k − ω 2 m )( k − ω 2 m ) − k 2 = 0 ⇒ ω =
−k k −ω m2
2 m

GATE- 2014
Q21. If the half-life of an elementary particle moving with speed 0.9c in the laboratory frame is
5 × 10 −8 s, then the proper half-life is _______________ ×10 −8 s. c = 3 × 10 8 m / s ( )
Ans: 2.18

                                                                                
Head office  Branch office 
 
fiziks, H.No. 40 D, G.F, Jia Sarai,  Anand Institute of Mathematics, 
 
Near IIT, Hauz Khas, New Delhi‐16  28‐B/6, Jia Sarai, Near IIT 
 
Phone: 011‐26865455/+91‐9871145498 Hauz Khas, New Delhi‐16 
                                                   
                                             Website: www.physicsbyfiziks.com                                                                                          
                                                            Email: fiziks.physics@gmail.com                                                                     320 
fiziks
Institute for NET/JRF, GATE, IIT‐JAM, JEST, TIFR and GRE in PHYSICAL SCIENCES 
 
t0 v2
Solution: t = , t0 = t × 1 − 2 = t0 = 5 × 10 −8 × .19 ⇒ 2.18 ×10−8 s
v2 c
1−
c2
Q22. Two masses m and 3m are attached to the two ends of a massless spring with force
constant K . If m = 100 g and K = 0.3 N / m , then the natural angular frequency of
oscillation is ________ Hz .
Ans: 0.318

1 k m1.m2 3m.m 3m 4k
Solution: f = μ= = = ω= = 2 = 0.318 Hz
2π μ m1 + m2 4m 4 3m
Q23. The Hamilton’s canonical equation of motion in terms of Poisson Brackets are
(a) q = {q, H }; p = {p, H } (b) q = {H , q}; p = {H , p}

(c) q = {H , p}; p = {H , p} (d) q = {p, H }; p = {q, H }


Ans: (a)
df ∂f ∂q ∂f ∂p ∂f
Solution: = . + . +
dt ∂q ∂t ∂p ∂t ∂t
df ∂f ∂H ∂f ∂H ∂f df ∂f
= . − . + ⇒ = { f , H} +
dt ∂q ∂p ∂p ∂q ∂t dt ∂t
dq dp
= {q, H } and = { p, H }
dt dt
Q24. A bead of mass m can slide without friction along a mass less rod kept at 45 o with the
vertical as shown in the figure. The rod is rotating about the vertical axis with a constant
angular speed ω . At any instant r is the distance of the bead from the origin. The

momentum conjugate to r is
ω
(a) mr
1
(b) mr
2
1 m
(c) mr
2 45 o
r
(d) 2mr

                                                                                
Head office  Branch office 
 
fiziks, H.No. 40 D, G.F, Jia Sarai,  Anand Institute of Mathematics, 
 
Near IIT, Hauz Khas, New Delhi‐16  28‐B/6, Jia Sarai, Near IIT 
 
Phone: 011‐26865455/+91‐9871145498 Hauz Khas, New Delhi‐16 
                                                   
                                             Website: www.physicsbyfiziks.com                                                                                          
                                                            Email: fiziks.physics@gmail.com                                                                     321 
fiziks
Institute for NET/JRF, GATE, IIT‐JAM, JEST, TIFR and GRE in PHYSICAL SCIENCES 
 
Ans: (a)
1
Solution: L = m(r 2 + r 2θ 2 + r 2 sin 2 θφ 2 ) − mgr cos θ
2
π
equation of constrain is θ = and it is given φ = ω
4
1 1 1
L= m( r 2 + r 2 ω 2 ) − mgr
2 2 2
∂L
the momentum conjugate to r is p r = = p r = mr
∂r
Q25. A particle of mass m is in a potential given by
a ar02
V (r ) = − +
r 3r 3
when a and r0 are positive constants. When disturbed slightly from its stable equilibrium
position it undergoes a simple harmonic oscillation. The time period of oscillation is
3 3
mr03 m r0 2m r0 mr03
(a) 2π (b) 2π (c) 2π (d) 4π
2a a a a
Ans: (a)
a ar02 ∂V a 3ar02
Solution: V ( r ) = − + 3 for equilibrium = − =0 r = ± r0
r 3r ∂r r 2 3r 4

∂ 2V 2a 4ar02 2a 4ar02 2a
=− 3 + 5 =− 3 + 5 = 3
∂r 2 r r r0
r0 r0 r0

∂ 2V
∂r 2 mr03
ω= ⇒ T = 2π
r0

m 2a
Q26. A planet of mass m moves in a circular orbit of radius r0 in the gravitational potential

k
V (r ) = − , where k is a positive constant. The orbit angular momentum of the planet is
r
(a) 2r0 km (b) 2r0 km (c) r0 km (d) r0 km

Ans: (d)
                                                                                
Head office  Branch office 
 
fiziks, H.No. 40 D, G.F, Jia Sarai,  Anand Institute of Mathematics, 
 
Near IIT, Hauz Khas, New Delhi‐16  28‐B/6, Jia Sarai, Near IIT 
 
Phone: 011‐26865455/+91‐9871145498 Hauz Khas, New Delhi‐16 
                                                   
                                             Website: www.physicsbyfiziks.com                                                                                          
                                                            Email: fiziks.physics@gmail.com                                                                     322 
fiziks
Institute for NET/JRF, GATE, IIT‐JAM, JEST, TIFR and GRE in PHYSICAL SCIENCES 
 
J 2
k dVeffect J 2
k
Solution: Veffctive = 2
− ⇒ = − 3 + 2 =0 at r = r0
2mr r dr mr r
so J = r0 km

Q27. Given that the linear transformation of a generalized coordinate q and the corresponding
momentum p ,
Q = q + 4ap
P = q + 2p
is canonical, the value of the constant a is _________________
Ans: 0.5
∂Q ∂P ∂Q ∂P
Solution: . − . = 0 ⇒ 1.2 − 4a.1 = 0 ⇒ a = 0.5
∂q ∂p ∂p ∂q

p2 α q2
Q28. The Hamiltonian of particle of mass m is given by H = − .which one of the
2m 2
following figure describes the motion of the particle in phase space?
(a) (b)
p p

q q

(c) (d)
p p

q q

Ans: (d)

                                                                                
Head office  Branch office 
 
fiziks, H.No. 40 D, G.F, Jia Sarai,  Anand Institute of Mathematics, 
 
Near IIT, Hauz Khas, New Delhi‐16  28‐B/6, Jia Sarai, Near IIT 
 
Phone: 011‐26865455/+91‐9871145498 Hauz Khas, New Delhi‐16 
                                                   
                                             Website: www.physicsbyfiziks.com                                                                                          
                                                            Email: fiziks.physics@gmail.com                                                                     323 
fiziks
Institute for NET/JRF, GATE, IIT‐JAM, JEST, TIFR and GRE in PHYSICAL SCIENCES 
 
GATE- 2015
Q29. A satellite is moving in a circular orbit around the Earth. If T ,V and E are its average
kinetic, average potential and total energies, respectively, then which one of the
following options is correct?
(a) V = −2T ; E = −T (b) V = −T ; E = 0
T T − 3T −T
(c) V = − ;E = (d) V = ;E =
2 2 2 2
Ans.: (a)
n +1
Solution: From Virial theorem T = V where V ∝ r n +1
2
−k 1
∵V = ⇒ V ∝ ⇒ n = −2 ⇒ V = −2 T
r r
Q30. In an inertial frame S , two events A and B take place at (ct A = 0, rA = 0) and
(ct B = 0, rB = 2 yˆ ) , respectively. The times at which these events take place in a frame
S ′ moving with a velocity 0.6cyˆ with respect to S are given by
3
(a) ct ′A = 0; ct B′ = − (b) ct ′A = 0; ct ′B = 0
2
3 1
(c) ct ′A = 0; ct B′ = (d) ct ′A = 0; ct B′ =
2 2
Ans.: (a)
Solution: Velocity of S ' with respect to S is v = .6c
v
tA − y
t A' = c2 for event A t A = 0, y = 0 so ct A' = 0
v2
1− 2
c
v
tB − y
c2 3
t B' = for event B t B = 0, y = 2 so ct B' = −
v2 2
1− 2
c

                                                                                
Head office  Branch office 
 
fiziks, H.No. 40 D, G.F, Jia Sarai,  Anand Institute of Mathematics, 
 
Near IIT, Hauz Khas, New Delhi‐16  28‐B/6, Jia Sarai, Near IIT 
 
Phone: 011‐26865455/+91‐9871145498 Hauz Khas, New Delhi‐16 
                                                   
                                             Website: www.physicsbyfiziks.com                                                                                          
                                                            Email: fiziks.physics@gmail.com                                                                     324 
fiziks
Institute for NET/JRF, GATE, IIT‐JAM, JEST, TIFR and GRE in PHYSICAL SCIENCES 
 
Q31. The Lagrangian for a particle of mass m at a position r moving with a velocity v is given
m
by L = v 2 + Cr .v − V (r ) , where V (r ) is a potential and C is a constant. If pc is the
2
canonical momentum, then its Hamiltonian is given by
1 1
(a) ( pc + Cr )2 + V (r ) (b) ( pc − Cr )2 + V (r )
2m 2m
pc2 1 2
(c) + V (r ) (d) pc + C 2 r 2 + V (r )
2m 2m
Ans.: (b)
m 2
Solution: L = v + Cr.v − V ( r ) where v = r
2
m 2
H = ∑ r pc − L = rpc − L where L = r + Cr.r − V ( r )
2
∂L p − Cr
⇒ = pc = ( mr + Cr ) ⇒ r = c
∂r m
2
⎛ p − Cr ⎞ m ⎛ pc − Cr ⎞ ⎛ pc − Cr ⎞
⇒ H =⎜ c ⎟ pc − ⎜ ⎟ − cr ⎜ ⎟ +V (r )
⎝ m ⎠ 2⎝ m ⎠ ⎝ m ⎠
2
⎛ p − Cr ⎞ m ⎛ pc − Cr ⎞
⇒ H =⎜ c ⎟ ( pc − Cr ) − ⎜ ⎟ +V (r )
⎝ m ⎠ 2⎝ m ⎠

( p − Cr ) ( p − Cr )
2 2
1
+V (r ) ( pc − Cr ) + V ( r )
2
⇒H = c − c ⇒H =
m 2m 2m
Q32. The Hamiltonian for a system of two particles of masses m1 and m2 at r1 and r2 having
1 1 C
velocities v1 and v2 is given by H = m1v12 + m2v22 + zˆ ⋅ ( r1 × r2 ) , wrong where
( r1 − r2 )
2
2 2

C is constant. Which one of the following statements is correct?


(a) The total energy and total momentum are conserved
(b) Only the total energy is conserved
(c) The total energy and the z - component of the total angular momentum are conserved
(d) The total energy and total angular momentum are conserved
Ans.: (c)

                                                                                
Head office  Branch office 
 
fiziks, H.No. 40 D, G.F, Jia Sarai,  Anand Institute of Mathematics, 
 
Near IIT, Hauz Khas, New Delhi‐16  28‐B/6, Jia Sarai, Near IIT 
 
Phone: 011‐26865455/+91‐9871145498 Hauz Khas, New Delhi‐16 
                                                   
                                             Website: www.physicsbyfiziks.com                                                                                          
                                                            Email: fiziks.physics@gmail.com                                                                     325 
fiziks
Institute for NET/JRF, GATE, IIT‐JAM, JEST, TIFR and GRE in PHYSICAL SCIENCES 
 
Solution: Lagrangian is not function of time so energy is conserve and component of ( r1 × r2 ) are

Only in ẑ direction means potential is symmetric under φ so Lz is conserve.

Q33. A particle of mass 0.01 kg falls freely in the earth’s gravitational field with an initial

velocity (0) = 10ms −1 . If the air exerts a frictional force of the form, f = −kv , then for

k = 0.05 Nm −1 s , the velocity (in ms −1 ) at time t = 0.2 s is _________ (upto two decimal

places). (use g = 10 ms −2 and e = 2.72 )


Ans.: Data given is incorrect
u 0.2
dv dv k dv dv
Solution: m = mg − kv ⇒ =g− v ⇒ = dt ⇒ ∫ = ∫ dt
dt dt m k k
g− v 10 g− v 0
m m
u
m⎡ ⎡ k ⎤⎤ m ⎧⎡ ⎛ k ⎞⎤ ⎛ 10k ⎞ ⎫
⇒ − ⎢ ln ⎢ g − v ⎥ ⎥ = [t ]0 ⇒ − ⎨ ⎢ln ⎜ g − u ⎟ ⎥ − ln ⎜ g −
0.2
⎟ ⎬ = 0.2
k ⎣ ⎣ m ⎦ ⎦10 k ⎩⎣ ⎝ m ⎠⎦ ⎝ m ⎠⎭

m⎧ ⎛ 0.05 ⎞ ⎛ .05 ⎞ ⎫
⇒− ⎨ln ⎜10 − u ⎟ − ln ⎜10 − 10 × ⎟ ⎬ = 0.2
k⎩ ⎝ 0.01 ⎠ ⎝ .01 ⎠ ⎭
m
⇒−
k
{ln (10 − 5u ) − ln ( −40 )} = 0.2
∵ ln ( −40 ) can not be defined. So given data are not correct.

Q34. Consider the motion of the Sun with respect to the rotation of the Earth about its axis. If
Fc and FCo denote the centrifugal and the Coriolis forces, respectively, acting on the
Sun, then
(a) Fc is radially outward and FCo = Fc

(b) Fc is radially inward and FCo = −2 Fc

(c) Fc is radially outward and FCo = −2 Fc

(d) Fc is radially outward and FCo = 2 Fc


Ans.: (b)

                                                                                
Head office  Branch office 
 
fiziks, H.No. 40 D, G.F, Jia Sarai,  Anand Institute of Mathematics, 
 
Near IIT, Hauz Khas, New Delhi‐16  28‐B/6, Jia Sarai, Near IIT 
 
Phone: 011‐26865455/+91‐9871145498 Hauz Khas, New Delhi‐16 
                                                   
                                             Website: www.physicsbyfiziks.com                                                                                          
                                                            Email: fiziks.physics@gmail.com                                                                     326 
fiziks
Institute for NET/JRF, GATE, IIT‐JAM, JEST, TIFR and GRE in PHYSICAL SCIENCES 
 
Q35. A particle with rest mass M is at rest and decays into two particles of equal rest masses
3
M which move along the z axis. Their velocities are given by
10
(a) v1 = v 2 = (0.8c )zˆ (b) v1 = −v 2 = (0.8c )zˆ
(c) v1 = −v 2 = (0.6c )zˆ (d) v1 = (0.6c )zˆ; v 2 = (− 0.8c )zˆ
Ans.: (b)
3 3
Solution: M→ M+ M
10 10
From momentum conservation
0 = P1 + P 2 ⇒ P1 = − P 2 ⇒ P1 = P2
From energy conservation E = E1 + E2

3 Mc 2 3 Mc 2 3 Mc 2
⇒ Mc 2 = + ⇒ Mc 2 =
10 v 2 10 v2 5 v2
1− 2 1− 2 1− 2
c c c

⎛ v2 ⎞ 9 v 2 16
⎜1 − 2 ⎟ = ⇒ 2 = ⇒ v = 0.8c
⎝ v ⎠ 25 v 25

GATE-2016
Q36. The kinetic energy of a particle of rest mass m0 is equal to its rest mass energy. Its

momentum in units of m0 c , where c is the speed of light in vacuum, is _______.


(Give your answer upto two decimal places
Ans. : 1.73

m0c 2
Solution: = 2m0 c 2 ⇒ E
2
v
1−
c2
E 2 = p 2 c 2 + m02 c 4 ⇒ 4m02c 4 − m02c 4 = p 2 c 2 ⇒ p = 3m0 c = 1.732m0 c

                                                                                
Head office  Branch office 
 
fiziks, H.No. 40 D, G.F, Jia Sarai,  Anand Institute of Mathematics, 
 
Near IIT, Hauz Khas, New Delhi‐16  28‐B/6, Jia Sarai, Near IIT 
 
Phone: 011‐26865455/+91‐9871145498 Hauz Khas, New Delhi‐16 
                                                   
                                             Website: www.physicsbyfiziks.com                                                                                          
                                                            Email: fiziks.physics@gmail.com                                                                     327 
fiziks
Institute for NET/JRF, GATE, IIT‐JAM, JEST, TIFR and GRE in PHYSICAL SCIENCES 
 
Q37. In an inertial frame of reference S , an observer finds two events occurring at the same
time at coordinates x1 = 0 and x 2 = d A different inertial frame S ′ moves with velocity
v with respect to S along the positive x -axis. An observer in S ′ also notices these two
events and finds them to occur at times t1′ and t 2′ and at positions x1′ and x2′ respectively.
1
If Δt ′ = t 2′ − t1′ , Δx ′ = x 2′ − x1′ and γ = , which of the following statements is true?
v2
1− 2
c
d
(a) Δt ′ = 0, Δx ′ = γd (b) Δt ′ = 0, Δx ′ =
γ
− γvd − γvd d
(c) Δt ′ = , Δx ′ = γd (d) Δt ′ = , Δx ′ =
c2 c 2
γ
Ans.: (c)
⎛ vx ⎞ ⎛ vx ⎞
⎜ t2 − 22 ⎟ ⎜ t1 − 21 ⎟
Solution: t2' − t1' = ⎜ c ⎟−⎜ c ⎟ ⇒ Δt ' = γΔt − γ vΔx it is given Δt = 0, Δx = d
⎜ v2 ⎟ ⎜ v2 ⎟ c2
⎜⎜ 1 − 2 ⎟⎟ ⎜⎜ 1 − 2 ⎟⎟
⎝ c ⎠ ⎝ c ⎠
γ vΔx
⇒ Δt ' = −
c2
⎛ ⎞ ⎛ ⎞
⎜ ⎟ ⎜ ⎟
x − vt2 ⎟ − ⎜ x1 − vt1
x2 − x1 = ⎜ 2
' ' ⎟ ⇒ Δx ' = γ ( Δx − vΔt ) it is given Δt = 0, Δx = d
⎜ v2 ⎟ ⎜ v2 ⎟
⎜⎜ 1 − 2 ⎟⎟ ⎜⎜ 1 − 2 ⎟⎟
⎝ c ⎠ ⎝ c ⎠
⇒ Δx ' = γ d
Q38. The Lagrangian of a system is given by

L=
1 2 2
2
[ ]
ml θ + sin 2 θϕ 2 − mgl cos θ , where m, l and g are constants.

Which of the following is conserved?


ϕ ϕ
(a) ϕ sin 2 θ (b) ϕ sin θ (c) (d)
sin θ sin 2 θ

                                                                                
Head office  Branch office 
 
fiziks, H.No. 40 D, G.F, Jia Sarai,  Anand Institute of Mathematics, 
 
Near IIT, Hauz Khas, New Delhi‐16  28‐B/6, Jia Sarai, Near IIT 
 
Phone: 011‐26865455/+91‐9871145498 Hauz Khas, New Delhi‐16 
                                                   
                                             Website: www.physicsbyfiziks.com                                                                                          
                                                            Email: fiziks.physics@gmail.com                                                                     328 
fiziks
Institute for NET/JRF, GATE, IIT‐JAM, JEST, TIFR and GRE in PHYSICAL SCIENCES 
 
Ans.: (a)
∂L
Solution: ϕ is cyclic coordinate so = pϕ ⇒ ml 2 sin 2 ϕ is constant hence m, l and g are
∂ϕ

constants. Then ϕ sin 2 θ


Q39. A particle of rest mass M is moving along the positive x -direction. It decays into two
photons γ 1 and γ 2 as shown in the figure. The energy of γ 1 is 1 GeV and the energy of
GeV
γ 2 is 0.82 GeV . The value of M (in units of ) is ________. (Give your answer
c2
γ1
upto two decimal places)

M 450
600

γ2
Ans.: 1.40

Solution: p 2 c 2 + M 2 c 4 = E1 + E2 = 1.82GeV

E1 E 1GeV 1 .82GeV 1 1.11GeV


p= cos θ1 + 2 cos θ 2 = + =
c c c 2 c 2 c

⇒ p 2 c 2 + m 2c 4 = 3.312

⇒ m 2 c 4 = 3.312 − 1.21 = 2.077


⇒ m = 2.076 = 1.40

                                                                                
Head office  Branch office 
 
fiziks, H.No. 40 D, G.F, Jia Sarai,  Anand Institute of Mathematics, 
 
Near IIT, Hauz Khas, New Delhi‐16  28‐B/6, Jia Sarai, Near IIT 
 
Phone: 011‐26865455/+91‐9871145498 Hauz Khas, New Delhi‐16 
                                                   
                                             Website: www.physicsbyfiziks.com                                                                                          
                                                            Email: fiziks.physics@gmail.com                                                                     329 
fiziks
Institute for NET/JRF, GATE, IIT‐JAM, JEST, TIFR and GRE in PHYSICAL SCIENCES 
 
ELECTROMAGNETIC THEORY SOLUTIONS
GATE- 2010
Q1. An insulating sphere of radius a carries a charge density

()
ρ r = ρ 0 (a 2 − r 2 )cos θ ; r < a .
The leading order term for the electric field at a distance d, far away from the charge
distribution, is proportional to
(a) d-1 (b) d-2 (c) d-3 (d) d-4
Ans: (c)
⎡1 1 ⎤
Solution: V (r ) = ⎢ ∫ ρdτ + 2 ∫ ρ cos θdτ + ⎥,
⎣r V r ⎦
a π 2π
Ist term, ∫ ρ dτ = ∫ ∫ ∫ ρ0 ( a − r ) cos θ × r sin θ drdθ dφ = 0
2 2 2

0 0 0

a π 2π
IInd term, ∫ ρ cos θ dτ = ∫ ∫ ∫ ρ0 ( a − r ) cos θ × r sin θ drdθ dφ ≠ 0 .
2 2 2 2

0 0 0

1 1
⇒ Vα 2
⇒ Eα 3
r r
Q2. Two magnetic dipoles of magnitude m each are placed in a plane as shown in figure.
m
The energy of interaction is given by
45 o 2
μ0 m 2
(a) Zero (b)
4πd 3 d
3μ 0 m 2
3μ 0 m 2 45 o
(c) (d) −
2πd 3 8πd 3 m 1
Ans: (d)
μ0
Solution: U = [m1 ⋅ m2 − 3(m1 ⋅ rˆ )(m2 ⋅ rˆ )],
4πr 3
μ0
Since m1 ⊥ m2 ⇒ m1 ⋅ m2 = 0 ⇒ U =
4πd 3
[− 3 × m cos 45 0 × m cos 45 0 ]

                                                                                
Head office  Branch office 
 
fiziks, H.No. 40 D, G.F, Jia Sarai,  Anand Institute of Mathematics, 
 
Near IIT, Hauz Khas, New Delhi‐16  28‐B/6, Jia Sarai, Near IIT 
 
Phone: 011‐26865455/+91‐9871145498 Hauz Khas, New Delhi‐16 
                                                   
                                             Website: www.physicsbyfiziks.com                                                                                          
                                                            Email: fiziks.physics@gmail.com                                                                     330 
fiziks
Institute for NET/JRF, GATE, IIT‐JAM, JEST, TIFR and GRE in PHYSICAL SCIENCES 
 
3μ m 2
⇒U = − 0 3 .
8π d
Statement for Linked Answer Questions 3 and 4:
Consider the propagation of electromagnetic waves in a linear, homogenous and isotropic
material medium with electric permittivity ε and magnetic permeability μ.

Q3. For a plane wave of angular frequency ω and propagation vector k propagating in the
medium Maxwell’s equations reduce to

(a) k ⋅ E = 0; k ⋅ H = 0; k × E = ωε H ; k × H = −ωμ E

(b) k ⋅ E = 0; k ⋅ H = 0; k × E = −ωε H ; k × H = ωμ E

(c) k ⋅ E = 0; k ⋅ H = 0; k × E = −ωμ H ; k × H = ωε E

(d) k ⋅ E = 0; k ⋅ H = 0; k × E = ωμ H ; k × H = −ωε E
Ans: (d)
Q4. If ε and μ assume negative values in a certain frequency range, then the directions of the

propagation vector k and the Poynting vector S in that frequency range are related as

(a) k and S are parallel

(b) k and S are anti-parallel

(c) k and S are perpendicular to each other

(d) k and S makes an angle that depends on the magnitude of |ε| and |μ|
Ans: (a)

                                                                                
Head office  Branch office 
 
fiziks, H.No. 40 D, G.F, Jia Sarai,  Anand Institute of Mathematics, 
 
Near IIT, Hauz Khas, New Delhi‐16  28‐B/6, Jia Sarai, Near IIT 
 
Phone: 011‐26865455/+91‐9871145498 Hauz Khas, New Delhi‐16 
                                                   
                                             Website: www.physicsbyfiziks.com                                                                                          
                                                            Email: fiziks.physics@gmail.com                                                                     331 
fiziks
Institute for NET/JRF, GATE, IIT‐JAM, JEST, TIFR and GRE in PHYSICAL SCIENCES 
 
Q5. Consider a conducting loop of radius a and total loop resistance R placed in a region with
a magnetic field B thereby enclosing a flux φ0. The loop is connected to an electronic
circuit as shown, the capacitor being initially uncharged

×××××××××× C
××××××××××
××××××××××
×××××××××× −
××××××××××
× × × ×Β
×××××× Vout
××××××××××
×××××××××× +
××××××××××
××××××××××

If the loop is pulled out of the region of the magnetic field at a constant speed u, the final
output voltage Vout is independent of
(a) φ0 (b) u (c) R (d) C
Ans: (a)
GATE-2011

Q6. If a force F is derivable from a potential function V(r), where r is the distance from the
origin of the coordinate system, it follows that

(a) ∇ × F = 0 (b) ∇ ⋅ F = 0 (c) ∇V = 0 (d) ∇ 2 V = 0


Ans: (a)
Q7. Tow charges q and 2q are placed along the x-axis in front of a grounded, infinite
conducting plane, as shown in the figure. They
are located respectively at a distance of 0.5 m and
1.5 m from the plane. The force acting on the
0 .5 m q 2q
charge q is • • x
1 .5 m
7q 2
1 1 2
(a) (b) 2q
4πε 0 2 4πε 0

1 1 q2
(c) q2 (d)
4πε 0 4πε 0 2
Ans: (a)
Solution: Using method of Images we can draw equivalent figure as shown below:

                                                                                
Head office  Branch office 
 
fiziks, H.No. 40 D, G.F, Jia Sarai,  Anand Institute of Mathematics, 
 
Near IIT, Hauz Khas, New Delhi‐16  28‐B/6, Jia Sarai, Near IIT 
 
Phone: 011‐26865455/+91‐9871145498 Hauz Khas, New Delhi‐16 
                                                   
                                             Website: www.physicsbyfiziks.com                                                                                          
                                                            Email: fiziks.physics@gmail.com                                                                     332 
fiziks
Institute for NET/JRF, GATE, IIT‐JAM, JEST, TIFR and GRE in PHYSICAL SCIENCES 
 
− 2q −q 0 .5 m 0 .5 m q 2q
• • • • x
1 .5 m 1 .5 m

q ⎡ 2q q 2q ⎤ q 7q 1 7q 2
F= ⎢ + + ⎥ = × =
4πε 0 ⎢⎣ (1)2 (1)2 ( 2 )2 ⎥⎦ 4πε 0 2 4πε 0 2

Q8. A uniform surface current is flowing in the positive y-direction over an infinite sheet
lying in x-y plane. The direction of the magnetic field is
(a) along iˆ for z > 0 and along − iˆ for z < 0
(b) along k̂ for z > 0 and along − k̂ for z < 0
(c) along − iˆ for z > 0 and along iˆ for z < 0
(d) along − k̂ for z > 0 and along k̂ for z < 0
Ans: (a)
Q9. A magnetic dipole of dipole moment m is placed in a non-uniform magnetic field B . If

the position vector of the dipole is r , the torque acting on the dipole about the origin is
(
(a) r × m × B ) (b) r × ∇ m ⋅ B( )
(c) m × B (d) m × B + r × ∇ m ⋅ B ( )
Ans: (c)

Q10. A spherical conductor of radius a is placed in a uniform electric field E = E 0 kˆ . The


potential at a point P(r, θ) for r > a, is given by
E0 a 3
Φ(r, θ) = constant – E 0 r cos θ + cos θ
r2
where r is the distance of P from the centre O of the sphere and θ is the angle OP makes
with the z-axis P
The charge density on the sphere at θ = 30o is r
(a) 3 3ε 0 E 0 / 2 (b) 3ε 0 E 0 / 2 θ
O k̂

                                                                                
Head office  Branch office 
 
fiziks, H.No. 40 D, G.F, Jia Sarai,  Anand Institute of Mathematics, 
 
Near IIT, Hauz Khas, New Delhi‐16  28‐B/6, Jia Sarai, Near IIT 
 
Phone: 011‐26865455/+91‐9871145498 Hauz Khas, New Delhi‐16 
                                                   
                                             Website: www.physicsbyfiziks.com                                                                                          
                                                            Email: fiziks.physics@gmail.com                                                                     333 
fiziks
Institute for NET/JRF, GATE, IIT‐JAM, JEST, TIFR and GRE in PHYSICAL SCIENCES 
 
(c) 3ε 0 E 0 / 2 (d)) ε 0 E 0 / 2
Ans: (a)

∂V ⎡ 2E a 3 ⎤
Solution: σ = −ε 0 = −ε 0 ⎢− E 0 cos θ − 03 cos θ ⎥ .
∂r r =a ⎣ r ⎦ r =a

3 3
σ = −ε 0 [− E 0 cos θ − 2 E 0 cos θ ] ⇒ σ = +3E 0 ε 0 cos θ = +3E 0 ε 0 cos 30 0 = ε 0 E0
2
Q11. Which of the following expressions for a vector potential A DOES NOT represent a
uniform magnetic field of magnitude B0 along the z-direction?
(a) A = (0, B0 x,0) (b) A = (− B0 y,0,0 )

⎛ B0 x B0 y ⎞ ⎛ B0 y B0 x ⎞
(c) A = ⎜ , ,0 ⎟ (d) A = ⎜ − , ,0 ⎟
⎝ 2 2 ⎠ ⎝ 2 2 ⎠
Ans: (c)

Solution: B ≠ ∇ × A .

Statement for Linked Questions 12 and 13:


A plane electromagnetic wave has the magnetic field given by
⎡ k ⎤
B( x, y, z , t ) = B0 sin ⎢(x + y ) + ω t ⎥ kˆ
⎣ 2 ⎦

where k is the wave number and iˆ, ˆj and kˆ are the Cartesian unit vectors in x, y and z
directions respectively.

Q12. The electric field E ( x, y, z , t ) corresponding to the above wave is given by


(a) cB0 sin ⎢( x + y )
k ⎤ iˆ − ˆj
+ ω t⎥
( ) ⎡
(b) cB0 sin ⎢( x + y )
k ⎤ iˆ + ˆj
+ ω t⎥
( )
⎣ 2 ⎦ 2 ⎣ 2 ⎦ 2
⎡ k ⎤ ⎡ k ⎤
(c) cB0 sin ⎢( x + y ) + ω t ⎥ iˆ (d) cB0 sin ⎢( x + y ) + ω t ⎥ ˆj
⎣ 2 ⎦ ⎣ 2 ⎦
Ans: (a)

                                                                                
Head office  Branch office 
 
fiziks, H.No. 40 D, G.F, Jia Sarai,  Anand Institute of Mathematics, 
 
Near IIT, Hauz Khas, New Delhi‐16  28‐B/6, Jia Sarai, Near IIT 
 
Phone: 011‐26865455/+91‐9871145498 Hauz Khas, New Delhi‐16 
                                                   
                                             Website: www.physicsbyfiziks.com                                                                                          
                                                            Email: fiziks.physics@gmail.com                                                                     334 
fiziks
Institute for NET/JRF, GATE, IIT‐JAM, JEST, TIFR and GRE in PHYSICAL SCIENCES 
 
c ⎡ k ( xˆ + yˆ ) ⎧ ( x + y )k
c
Solution: E = − k × B = − ⎢−
k k⎣
( ) × B0 sin ⎨
⎫ ⎤
+ ωt ⎬ zˆ ⎥
2 ⎩ 2 ⎭ ⎦

⎡ k ⎤ (xˆ − yˆ )
E = cB0 sin ⎢( x + y ) + ωt ⎥
⎣ 2 ⎦ 2

Q13. The average Poynting vector is given by

(a)
(
cB02 iˆ − ˆj ) (b) −
cB02 iˆ − ˆj ( ) (c)
cB02 iˆ + ˆj( ) (d) −
cB02 iˆ + ˆj ( )
2μ 0 2 2μ 0 2 2μ 0 2 2μ 0 2
Ans: (d)
cB02 ˆ cB02 ⎛ xˆ + yˆ ⎞ − cB02 ⎛ xˆ + yˆ ⎞
Solution: S = k= × −⎜ ⎟= ×⎜ ⎟
2μ 0 2μ 0 ⎝ 2 ⎠ 2μ 0 ⎝ 2 ⎠

GATE-2012
Q14. The space-time dependence of the electric field of a linearly polarized light in free space
is given by xE0 cos(ωt − kz ) where E0, ω and k are the amplitude, the angular frequency
and the wavevector, respectively. The time average energy density associated with the
electric field is
1 1
(a) ε 0 E 02 (b) ε 0 E 02 (c) ε 0 E 02 (d) 2ε 0 E 02
4 2
Ans: (a)
1 1 1
Solution: u E = ε 0 E 2 = ε 0 E 2 cos 2 (wt − kz ) ⇒< u E >= ε 0 E 02
2 2 4
Q15. A plane electromagnetic wave traveling in free space is incident normally on a glass plate
of refractive index 3/2. If there is no absorption by the glass, its reflectivity is
(a) 4% (b) 16% (c) 20% (d) 50%
Ans: (a)

                                                                                
Head office  Branch office 
 
fiziks, H.No. 40 D, G.F, Jia Sarai,  Anand Institute of Mathematics, 
 
Near IIT, Hauz Khas, New Delhi‐16  28‐B/6, Jia Sarai, Near IIT 
 
Phone: 011‐26865455/+91‐9871145498 Hauz Khas, New Delhi‐16 
                                                   
                                             Website: www.physicsbyfiziks.com                                                                                          
                                                            Email: fiziks.physics@gmail.com                                                                     335 
fiziks
Institute for NET/JRF, GATE, IIT‐JAM, JEST, TIFR and GRE in PHYSICAL SCIENCES 
 
2 2
⎛ n − n2 ⎞ ⎛1− 3/ 2 ⎞ 1 4
Solution: R = ⎜⎜ 1 ⎟⎟ = ⎜ ⎟ = × = .04 or 4%
⎝ n1 + n2 ⎠ ⎝1+ 3/ 2 ⎠ 4 25

Q16. The electric and the magnetic field E ( z , t ) and B ( z , t ) , respectively corresponding to the

scalar potential φ ( z , t ) = 0 and vector potential A( z , t ) = iˆtz are

(a) E = iˆz and B = -ĵt (b) E = iˆz and B = ĵt

(c) E = −iˆz and B = -ĵt (d) E = −iˆz and B = -ĵt


Ans: (d)
∂A ∂A
Solution: E = −∇φ − =− = −iˆz , B = ∇ × A = + ˆjt .
∂t ∂t
Q17. A plane polarized electromagnetic wave in free space at time t=0 is given
by E ( x, y ) = 10 ˆj exp[i(6 x + 8 z )] . The magnetic field B( x, z , t ) is given by

(a) B( x, z , t ) =
c
(
1 ˆ ˆ
)
6k − 8i exp[i (6 x + 8 z − 10ct )]

(b) B( x, z , t ) =
c
(
1 ˆ ˆ
)
6k + 8i exp[i (6 x + 8 z − 10ct )]

(c) B( x, z , t ) =
c
(
1 ˆ ˆ
)
6k − 8i exp[i (6 x + 8 z − ct )]

(d) B( x, z , t ) =
c
(
1 ˆ ˆ
)
6k + 8i exp[i (6 x + 8 z + ct )]

Ans: (a)
⎛ ⎞
1 ˆ 1⎜ k
(⎟
Solution: B = k × E = ⎜ × E ⎟ =
c c⎜ k
) 1 ⎛⎜ 6iˆ + 8kˆ ⎞⎟
c ⎜⎝ 10 ⎟⎠
[(
× 10 ˆjexp i k .r − ωt )]

⎝ ⎠

B=
c
(
1 ˆ ˆ
)
6k − 8i exp[i (6 x + 8 z − 10ct )], ω = 10c.

                                                                                
Head office  Branch office 
 
fiziks, H.No. 40 D, G.F, Jia Sarai,  Anand Institute of Mathematics, 
 
Near IIT, Hauz Khas, New Delhi‐16  28‐B/6, Jia Sarai, Near IIT 
 
Phone: 011‐26865455/+91‐9871145498 Hauz Khas, New Delhi‐16 
                                                   
                                             Website: www.physicsbyfiziks.com                                                                                          
                                                            Email: fiziks.physics@gmail.com                                                                     336 
fiziks
Institute for NET/JRF, GATE, IIT‐JAM, JEST, TIFR and GRE in PHYSICAL SCIENCES 
 

Q18. Two infinitely extended homogeneous isotopic dielectric media (medium-1and medium-2
ε1 ε
with dielectric constant = 2 and 2 = 5 , respectively)
ε0 ε0
medium - 1
meet at the z = 0 plane as shown in the figure. A uniform
electric field exists everywhere. For z ≥ 0, the electric field
medium - 2 z=0
is given by E1 = 2iˆ − 3 ˆj + 5kˆ . The interface separating the
two media is charge free. The electric displacement vector
in the medium-2 is given by

[
(a) D 2 = ε 0 10iˆ + 15 ˆj + 10kˆ ] [
(b) D 2 = ε 0 10iˆ − 15 ˆj + 10kˆ ]
(c) D 2 = ε [4iˆ − 6 ˆj + 10kˆ ]
0 (d) D 2 = ε [4iˆ + 6 ˆj + 10kˆ ]
0

Ans: (b)
∵ E1 = E 2 ⇒ E 2 = 2iˆ − 3 ˆj

ε1 ⊥ 2 × 5 ˆ
and σ f = 0 ⇒ D1⊥ = D2⊥ ⇒ E 2⊥ = E1 = k = 2kˆ ⇒ E 2 = 2iˆ − 3 ˆj + 2kˆ
ε2 5

[
⇒ D2 = ε 2 E 2 = ε 0 10iˆ − 15 ˆj + 10kˆ . ]
GATE-2013
Q19. At a surface current, which one of the magnetostatic boundary condition is NOT
CORRECT?
(a) Normal component of the magnetic field is continuous.
(b) Normal component of the magnetic vector potential is continuous.
(c) Tangential component of the magnetic vector potential is continuous.
(d) Tangential component of the magnetic vector potential is not continuous.
Ans: (d)

                                                                                
Head office  Branch office 
 
fiziks, H.No. 40 D, G.F, Jia Sarai,  Anand Institute of Mathematics, 
 
Near IIT, Hauz Khas, New Delhi‐16  28‐B/6, Jia Sarai, Near IIT 
 
Phone: 011‐26865455/+91‐9871145498 Hauz Khas, New Delhi‐16 
                                                   
                                             Website: www.physicsbyfiziks.com                                                                                          
                                                            Email: fiziks.physics@gmail.com                                                                     337 
fiziks
Institute for NET/JRF, GATE, IIT‐JAM, JEST, TIFR and GRE in PHYSICAL SCIENCES 
 
Q20. Interference fringes are seen at an observation plane z = 0 , by the superposition of two
plane waves A1 exp i k1 ⋅ r − ωt [( )] and A 2 [( )]
exp i k 2 ⋅ r − ωt , where A1 and A2 are real
amplitudes. The condition for interference maximum is
( )
(a) k1 − k 2 ⋅ r = (2m + 1)π ( )
(b) k1 − k 2 ⋅ r = 2mπ

(c) (k 1 + k2 )⋅ r = (2m + 1)π (d) (k 1 + k2 )⋅ r = 2mπ


Ans: (b)

Q21. For a scalar function ϕ satisfying the Laplace equation, ∇ϕ has


(a) zero curl and non-zero divergence
(b) non-zero curl and zero divergence
(c) zero curl and zero divergence
(d) non-zero curl and non-zero divergence
Ans: (c)

( )
∇ 2ϕ = 0 ⇒ ∇. ∇ϕ = 0 and ⇒ ∇ × ∇ϕ = 0 . ( )
Q22. A circularly polarized monochromatic plane wave is incident on a dielectric interface at
Brewaster angle. Which one of the following statements is correct?
(a) The reflected light is plane polarized in the plane of incidence and the transmitted
light is circularly polarized.
(b) The reflected light is plane polarized perpendicular to the plane of incidence and the
transmitted light is plane polarized in the plane of incidence.
(c) The reflected light is plane polarized perpendicular to the plane of incidence and the
transmitted light is elliptically polarized.
(d) There will be no reflected light and the transmitted light is circularly polarized.
Ans: (c)

                                                                                
Head office  Branch office 
 
fiziks, H.No. 40 D, G.F, Jia Sarai,  Anand Institute of Mathematics, 
 
Near IIT, Hauz Khas, New Delhi‐16  28‐B/6, Jia Sarai, Near IIT 
 
Phone: 011‐26865455/+91‐9871145498 Hauz Khas, New Delhi‐16 
                                                   
                                             Website: www.physicsbyfiziks.com                                                                                          
                                                            Email: fiziks.physics@gmail.com                                                                     338 
fiziks
Institute for NET/JRF, GATE, IIT‐JAM, JEST, TIFR and GRE in PHYSICAL SCIENCES 
 
Q23. A charge distribution has the charge density given by ρ = Q{δ (x − x 0 ) − δ ( x + x0 )} . For

this charge distribution the electric field at (2 x0 ,0,0)

2Qxˆ Qxˆ Qxˆ Qxˆ


(a) (b) (c) (d)
9πε 0 x02 4πε 0 x03 4πε 0 x 02 16πε 0 x02
Ans:

Solution: Potential V ( r ) =
1 ⎡ ρ x
⎢∫
a '

dx′ + ∫
a ρ x'
( )a ρ x'
( ) ⎤ ( )
′ ′ + ∫a x3 ′ ′ + ⎥
2
x dx x dx ....
4πε 0 ⎢ − a x x2 ⎥⎦
⎣ − a −

First term, total charge


x0 x0

QT = ∫ ρ ( x ′)dx ′ =Q ∫ δ ( x ′ − x 0 )dx ′ − Q ∫ δ ( x ′ + x 0 )dx ′ = Q − Q = 0


− x0 − x0

Second term, dipole moment


x0 x0

p = ∫ x ′ρ ( x ′)dx ′ =Q ∫ x ′δ ( x ′ − x 0 )dx ′ − Q ∫ x ′δ (x ′ + x 0 )dx ′ = Qx 0 − Q × − x 0 = 2Qx 0


− x0 − x0

2Qx 0 ∂V 4Qx 0 4Qx 0 Q


V= ⇒E=− xˆ = xˆ = xˆ = xˆ
4πε 0 x ∂x 4πε 0 x 4πε 0 (2 x 0 ) 8πε 0 x 20
2 3 3

Q24. A monochromatic plane wave at oblique incidence undergoes reflection at a dielectric


interface. If kˆi , kˆr and n̂ are the unit vectors in the directions of incident wave, reflected
wave and the normal to the surface respectively, which one of the following expressions
is correct?
( )
(a) kˆi − kˆr × nˆ ≠ 0 ( )
(b) kˆi − kˆr ⋅ nˆ = 0

(c) (kˆ × nˆ )⋅ kˆ
i r =0 (d) (kˆ × nˆ )⋅ kˆ
i r ≠0
Ans: (c)

                                                                                
Head office  Branch office 
 
fiziks, H.No. 40 D, G.F, Jia Sarai,  Anand Institute of Mathematics, 
 
Near IIT, Hauz Khas, New Delhi‐16  28‐B/6, Jia Sarai, Near IIT 
 
Phone: 011‐26865455/+91‐9871145498 Hauz Khas, New Delhi‐16 
                                                   
                                             Website: www.physicsbyfiziks.com                                                                                          
                                                            Email: fiziks.physics@gmail.com                                                                     339 
fiziks
Institute for NET/JRF, GATE, IIT‐JAM, JEST, TIFR and GRE in PHYSICAL SCIENCES 
 
Q25. In a constant magnetic field of 0.6 Tesla along the z direction, find the value of the path

integral ∫ A ⋅ dl in the units of (Tesla m 2 ) on a square loop of side length 1 / 2 meters. ( )


The normal to the loop makes an angle of 60 0 to the z-axis, as shown in the figure. The
answer should be up to two decimal places. ___________

60 o

Ans: 0.15

∫ A ⋅ dl = ∫ ( )
2
⎛ 1 ⎞ 1
Solution: ∇ × A .d a = ∫ B.d a = BA cos 60 = 0.6 × ⎜⎜ ⎟⎟ × = 0.15T .m 2
0

S S ⎝ 2⎠ 2

GATE-2014

Q26. Which one of the following quantities is invariant under Lorentz transformation?
(a) Charge density (b) Charge (c) Current (d) Electric field
Ans: (b)
Q27. An unpolarized light wave is incident from air on a glass surface at the Brewster angle.
The angle between the reflected and the refracted wave is
(a) 0 o (b) 45 o (c) 90 o (d) 120 o
Ans: (c)

                                                                                
Head office  Branch office 
 
fiziks, H.No. 40 D, G.F, Jia Sarai,  Anand Institute of Mathematics, 
 
Near IIT, Hauz Khas, New Delhi‐16  28‐B/6, Jia Sarai, Near IIT 
 
Phone: 011‐26865455/+91‐9871145498 Hauz Khas, New Delhi‐16 
                                                   
                                             Website: www.physicsbyfiziks.com                                                                                          
                                                            Email: fiziks.physics@gmail.com                                                                     340 
fiziks
Institute for NET/JRF, GATE, IIT‐JAM, JEST, TIFR and GRE in PHYSICAL SCIENCES 
 
Q28. The electric field of a uniform plane wave propagating in a dielectric non-conducting
medium is given by E = xˆ 10 cos ( 6π ×107 t − 0.4π z ) V / m . The phase velocity of the

wave is _________ 10 8 m / s
Ans: 1.5
ω
6π × 107
v= = = 1.5 ×108 m / sec
k 0.4π
Q29. If the vector potential A = αxxˆ + 2 yyˆ − 3zzˆ , satisfies the Coulomb gauge, the value of the
constant α is _______
Ans: 1
Solution: Coulomb gauge condition ∇. A = 0 ⇒ α + 2 − 3 = 0 ⇒ α = 1
Q30. A ray of light inside Region 1 in the xy -plane is incident at the semicircular boundary

⎛ π⎞
that carries no free charges. The electric field at the point P ⎜ r0 , ⎟ in plane polar
⎝ 4⎠

coordinates is E1 = 7eˆr − 3eˆϕ where êr and êϕ are the unit vectors. The emerging ray in

Region 2 has the electric field E 2 parallel to x -axis. If ε 1 and ε 2 are the dielectric

ε2
constants of Region-1 and Region-2 respectively then is ________
ε1
y

P (r0 , π / 4 )

O
ε1 ε2 x
Region 1 Region 2

                                                                                
Head office  Branch office 
 
fiziks, H.No. 40 D, G.F, Jia Sarai,  Anand Institute of Mathematics, 
 
Near IIT, Hauz Khas, New Delhi‐16  28‐B/6, Jia Sarai, Near IIT 
 
Phone: 011‐26865455/+91‐9871145498 Hauz Khas, New Delhi‐16 
                                                   
                                             Website: www.physicsbyfiziks.com                                                                                          
                                                            Email: fiziks.physics@gmail.com                                                                     341 
fiziks
Institute for NET/JRF, GATE, IIT‐JAM, JEST, TIFR and GRE in PHYSICAL SCIENCES 
 
Ans: 2.32
y
Solution: ∵ E1 = 7er − 3eϕ
ˆ ˆ P (r0 , π / 4 )
10 θ2 E 2
⇒ Ex = ( 7eˆr − 3eˆϕ ) .xˆ = 7 cos 45 + 3sin 45 = θ1
2 E1
α
O
4 ε1 x
⇒ E y = ( 7eˆr − 3eˆϕ ) . yˆ = 7 sin 45 − 3sin 45 = ε2
2 Region 1 Region 2

⎛E ⎞ ⎛ 4⎞
Thus E1 makes an angle α = tan −1 ⎜ y ⎟ = tan −1 ⎜ ⎟ = 21.80
⎝ Ex ⎠ ⎝ 10 ⎠
tan θ 2 ε 2 ε tan 45
∵ = ⇒ 2 = = 2.32 where θ1 = α − 450 and θ 2 = 450
tan θ1 ε 2 ε 2 tan 23.2
Q31. The value of the magnetic field required to maintain non-relativistic protons of energy
1MeV in a circular orbit of radius 100 mm is _______Tesla
(Given: m p = 1.67 × 10−27 kg , e = 1.6 × 10−19 C )

Ans: 1.44

(1.6 × 10−19 ) B 2 ( 0.1) 1.6 × 10−13 × 2 (1.67 × 10−27 )


2 2
q2 B2 R2 −13
E= ⇒ 1.6 × 10 = ⇒B =
2

2 (1.67 × 10−27 ) (1.6 ×10−19 ) ( 0.1)2


2
2m p

10−13 × 2 (1.67 × 10−27 ) 3.34 × 10−40


⇒B = 2
= = 2.08 ⇒ B = 2.08 Tesla = 1.44Tesla
(1.6 ×10 ) ( 0.01)
−38
1.6 ×10−40

Q32. In an interference pattern formed by two coherent sources, the maximum and minimum
of the intensities are 9 I 0 and I 0 respectively. The intensities of the individual wave are

(a) 3I 0 and I 0 (b) 4 I 0 and I 0

(c) 5I 0 and 4 I 0 (d) 9 I 0 and I 0


Ans: (b)

( ) ( )
2 2
Solution: I max = I1 + I 2 and I min = I1 − I 2

( ) ( )
2 2
9I0 = I1 + I 2 and I 0 = I1 − I 2 ⇒ I1 = 4 I 0 and I 2 = I 0

                                                                                
Head office  Branch office 
 
fiziks, H.No. 40 D, G.F, Jia Sarai,  Anand Institute of Mathematics, 
 
Near IIT, Hauz Khas, New Delhi‐16  28‐B/6, Jia Sarai, Near IIT 
 
Phone: 011‐26865455/+91‐9871145498 Hauz Khas, New Delhi‐16 
                                                   
                                             Website: www.physicsbyfiziks.com                                                                                          
                                                            Email: fiziks.physics@gmail.com                                                                     342 
fiziks
Institute for NET/JRF, GATE, IIT‐JAM, JEST, TIFR and GRE in PHYSICAL SCIENCES 
 
Q33. The intensity of a laser in free space is 150mW / m 2 . The corresponding amplitude of the

electric field of the laser is _________


V
m
(ε 0 = 8.854 × 10 −12 C 2 / N .m 2 )
Ans: 10.6

1 2I 2 × 150 ×10−3
Solution: I = cε 0 E02 ⇒ E0 = = = 10.6 V / m
2 cε 0 3 × 108 × 8.854 × 10−12

                                                                                
Head office  Branch office 
 
fiziks, H.No. 40 D, G.F, Jia Sarai,  Anand Institute of Mathematics, 
 
Near IIT, Hauz Khas, New Delhi‐16  28‐B/6, Jia Sarai, Near IIT 
 
Phone: 011‐26865455/+91‐9871145498 Hauz Khas, New Delhi‐16 
                                                   
                                             Website: www.physicsbyfiziks.com                                                                                          
                                                            Email: fiziks.physics@gmail.com                                                                     343 
fiziks
Institute for NET/JRF, GATE, IIT‐JAM, JEST, TIFR and GRE in PHYSICAL SCIENCES 
 
GATE-2015
Q34. A point charge is placed between two semi-infinite conducting plates which are inclined
at an angle of 30 o with respect to each other. The number of image charges
is___________.
Ans.: 11
360 360
Solution: n = −1 = − 1 = 11
θ 30
R
Q35. Given that the magnetic flux through the closed loop PQRSP is φ . If ∫ A ⋅ dl = φ along
P
1

R
PQR , the value of ∫ A ⋅ dl along PSR is
P
Q
R

S
(a) φ − φ1 (b) φ1 − φ (c) − φ1 (d) φ1
Ans.: (b)
R P
Solution: φ = ∫ s B.d a = ∫ A.dl = ∫ A ⋅ dl + ∫ A ⋅ dl
P R

R R
⇒ φ = φ1 − ∫ A ⋅ dl ⇒ ∫ A ⋅ dl = φ1 − φ
P P

Q15. The space between two plates of a capacitor carrying charges + Q and − Q is filled with
two different dielectric materials, as shown in the figure. Across the interface of the two
dielectric materials, which one of the following statements is correct?
(a) E and D are continuous
(b) E is continuous and D is discontinuous
(c) D is continuous and E is discontinuous +Q −Q

(d) E and D are discontinuous


Ans.: (d)

                                                                                
Head office  Branch office 
 
fiziks, H.No. 40 D, G.F, Jia Sarai,  Anand Institute of Mathematics, 
 
Near IIT, Hauz Khas, New Delhi‐16  28‐B/6, Jia Sarai, Near IIT 
 
Phone: 011‐26865455/+91‐9871145498 Hauz Khas, New Delhi‐16 
                                                   
                                             Website: www.physicsbyfiziks.com                                                                                          
                                                            Email: fiziks.physics@gmail.com                                                                     344 
fiziks
Institute for NET/JRF, GATE, IIT‐JAM, JEST, TIFR and GRE in PHYSICAL SCIENCES 
 
Q22. Four forces are given below in Cartesian and spherical polar coordinates
⎛ − r2 ⎞
(i) F1 = K exp⎜⎜ 2 ⎟⎟rˆ (
(ii) F2 = K x 3 yˆ − y 3 zˆ )
⎝ R ⎠

⎛ φˆ ⎞
(iii) F3 = K (x 3 xˆ + y 3 yˆ ) (iv) F4 = K ⎜⎜ ⎟⎟
⎝r⎠
where K is a constant Identify the correct option
(a) (iii) and (iv) are conservative but (i) and (ii)are not
(b) (i) and (ii) are conservative but (iii) and (iv) are not
(c) (ii) and (iii) are conservative but (i) and (iv) are not
(d) (i) and (iii) are conservative but (ii) and (iv) are not
Ans.: (d)

r rθˆ r sin θφ
1 ∂ ∂ ∂
Solution: ∇ × F 1 = =0
r sin θ
2
∂r ∂θ ∂φ
⎛ r2 ⎞
k exp ⎜ − 2 ⎟ 0 0
⎝ R ⎠

x y z
∂ ∂ ∂
∇× F2 = = x ( −3ky 2 − 0 ) = −3ky 2 x
∂x ∂y ∂z
0 kx 3 − ky 3

x y z
∂ ∂ ∂
∇× F3 = =0
∂x ∂y ∂z
kx3 ky 3 0

                                                                                
Head office  Branch office 
 
fiziks, H.No. 40 D, G.F, Jia Sarai,  Anand Institute of Mathematics, 
 
Near IIT, Hauz Khas, New Delhi‐16  28‐B/6, Jia Sarai, Near IIT 
 
Phone: 011‐26865455/+91‐9871145498 Hauz Khas, New Delhi‐16 
                                                   
                                             Website: www.physicsbyfiziks.com                                                                                          
                                                            Email: fiziks.physics@gmail.com                                                                     345 
fiziks
Institute for NET/JRF, GATE, IIT‐JAM, JEST, TIFR and GRE in PHYSICAL SCIENCES 
 

r rθ r sin θφ
1 ∂ ∂ ∂
∇× F4 = 2 = r [ k cos θ ]
r sin θ ∂r ∂θ ∂φ
k
0 0 r sin θ ×
r
Q23. A monochromatic plane wave (wavelength = 600 nm ) E 0 exp[i (kz − ω t )] is incident

normally on a diffraction grating giving rise to a plane wave E1 exp i k1 ⋅ r − ω t in the [( )]


⎡1 3 ⎤
first order of diffraction. Here E1 < E 0 and k1 = k1 ⎢ xˆ + zˆ ⎥ . The period (in μm ) of
⎣2 2 ⎦

the diffraction grating is ______________ (upto one decimal place)


Ans.: 1.2
λ
Solution: d sin θ = nλ ⇒ d = ∵n =1
sin θ
⎡1 3 ⎤ 1
∵ k1 = k1 ⎢ xˆ + zˆ ⎥ ⇒ sin θ = ⇒ θ = 300
⎣2 2 ⎦ 2

600
⇒d = nm = 1200 nm = 1.2 μ m
sin 30
Q24. A long solenoid is embedded in a conducting medium and is insulated from the medium.
If the current through the solenoid is increased at a constant rate, the induced current in
the medium as a function of the radial distance r from the axis of the solenoid is
proportional to
1 1
(a) r 2 inside the solenoid and outside (b) r inside the solenoid and outside
r r2
1 1
(c) r 2 inside the solenoid and outside (d) r inside the solenoid and outside
r2 r
Ans.: (d)

∂B
Solution: ∫ E ⋅ dl = −∫
∂t
⋅ da ;

                                                                                
Head office  Branch office 
 
fiziks, H.No. 40 D, G.F, Jia Sarai,  Anand Institute of Mathematics, 
 
Near IIT, Hauz Khas, New Delhi‐16  28‐B/6, Jia Sarai, Near IIT 
 
Phone: 011‐26865455/+91‐9871145498 Hauz Khas, New Delhi‐16 
                                                   
                                             Website: www.physicsbyfiziks.com                                                                                          
                                                            Email: fiziks.physics@gmail.com                                                                     346 
fiziks
Institute for NET/JRF, GATE, IIT‐JAM, JEST, TIFR and GRE in PHYSICAL SCIENCES 
 
For r < R

dI 2π r 2
r
dI 1 dI
⇒ E × 2π r = − μ0 n
dt ∫
r ′= 0
2π r ′dr ′ = − μ0 n ×
dt
×
2
⇒ E = − μ0 n r
2 dt

For r > R

dI 2π R 2
R
dI 1 dI
⇒ E × 2π r = − μ0 n
dt ∫
r ′= 0
2π r ′dr ′ = − μ0 n ×
dt
×
2
⇒ E = − μ0 n R 2
2r dt

Q25. A plane wave (xˆ + iyˆ )E 0 exp[i(kz − ωt )] after passing through an optical element emerges

as (xˆ − iyˆ )E0 exp[i(kz − ωt )] , where k and ω are the wavevector and the angular
frequency, respectively. The optical element is a
(a) quarter wave plate (b) half wave plate
(c) polarizer (d) Faraday rotator
Ans.: (b)

( )
Solution: Incident wave: x + i y E0 eiθ = ⎡ E0 cos θ x − E0 sin θ y ⎤
⎣ ⎦
Left circular polarization with phase angle φ1 = −θ = θ eiπ

( )
Emergent wave: x − i y E0 eiθ = ⎡ E0 cos θ + E0 sin θ y ⎤
⎣ ⎦
Right circular polarization with phase angle φ1 = +θ = θ ei 0

λ
Thus there is phase change of π and hence path difference of .
2

                                                                                
Head office  Branch office 
 
fiziks, H.No. 40 D, G.F, Jia Sarai,  Anand Institute of Mathematics, 
 
Near IIT, Hauz Khas, New Delhi‐16  28‐B/6, Jia Sarai, Near IIT 
 
Phone: 011‐26865455/+91‐9871145498 Hauz Khas, New Delhi‐16 
                                                   
                                             Website: www.physicsbyfiziks.com                                                                                          
                                                            Email: fiziks.physics@gmail.com                                                                     347 
fiziks
Institute for NET/JRF, GATE, IIT‐JAM, JEST, TIFR and GRE in PHYSICAL SCIENCES 
 
Q26. A charge − q is distributed uniformly over a sphere, with a positive charge q at its center
in (i). Also in (ii), a charge − q is distributed uniformly over an ellipsoid with a positive
charge q at its center. With respect to the origin of the coordinate system, which one of
the following statements is correct?

X X

Z Z

Y Y
(i ) (ii )
(a) The dipole moment is zero in both (i) and (ii)
(b) The dipole moment is non-zero in (i) but zero in (ii)
(c) The dipole moment is zero in (i) but non-zero in (ii)
(d) The dipole moment is non-zero in both (i) and (ii)
Ans.: (a)
Solution: p = ∑ qi ri = 0 in both cases.

GATE-2016
Q27. Which of the following magnetic vector potentials gives rise to a uniform magnetic field
B0 kˆ ?

(a) B0 z kˆ (b) − B0 x ˆj (c)


B0
2
(
− yiˆ + xˆj ) (d)
2
(
B0 ˆ ˆ
yi + xj )
Ans.: (c)
Solution: (a) ∇ × A = 0
(b) ∇ × A = − B0 kˆ

(c) ∇ × A = B0 kˆ

(d) ∇ × A = 0

                                                                                
Head office  Branch office 
 
fiziks, H.No. 40 D, G.F, Jia Sarai,  Anand Institute of Mathematics, 
 
Near IIT, Hauz Khas, New Delhi‐16  28‐B/6, Jia Sarai, Near IIT 
 
Phone: 011‐26865455/+91‐9871145498 Hauz Khas, New Delhi‐16 
                                                   
                                             Website: www.physicsbyfiziks.com                                                                                          
                                                            Email: fiziks.physics@gmail.com                                                                     348 
fiziks
Institute for NET/JRF, GATE, IIT‐JAM, JEST, TIFR and GRE in PHYSICAL SCIENCES 
 
Q28. The magnitude of the magnetic dipole moment associated with a square shaped loop
carrying a steady current I is m . If this loop is changed to a circular shape with the same
pm
current I passing through it, the magnetic dipole moment becomes . The value of p
π
is ______.
Ans.: 4
Solution: Magnetic dipole moment associated with a square shaped loop (let side is a) carrying a
steady current I is m = Ia 2 .
Magnetic dipole moment associated with a circular shaped loop (let radius is r) carrying a
steady current I is m′ = I π r 2 .
2
2a ⎛ 2a ⎞ 4 Ia 2 4m
Here 4a = 2π r ⇒ r = ⇒ m′ = I π r = I π ⎜ ⎟ = = 2

π ⎝π ⎠ π π
Q29. In a Young’s double slit experiment using light, the apparatus has two slits of unequal
widths. When only slit- 1 is open, the maximum observed intensity on the screen is 4 I 0 .

When only slit- 2 is open, the maximum observed intensity is I 0 . When both the slits are
open, an interference pattern appears on the screen. The ratio of the intensity of the
principal maximum to that of the nearest minimum is ________.
Ans.: 9

( ) =( ) = ( 2 I + I ) = 9I
2 2
I max I1 + I 2 4I0 + I0 0 0
Solution: = 0
=9
( ) ( ) (2 I − I ) I
2 2 2
I min I1 − I 2 4I0 − I0 0
0 0

                                                                                
Head office  Branch office 
 
fiziks, H.No. 40 D, G.F, Jia Sarai,  Anand Institute of Mathematics, 
 
Near IIT, Hauz Khas, New Delhi‐16  28‐B/6, Jia Sarai, Near IIT 
 
Phone: 011‐26865455/+91‐9871145498 Hauz Khas, New Delhi‐16 
                                                   
                                             Website: www.physicsbyfiziks.com                                                                                          
                                                            Email: fiziks.physics@gmail.com                                                                     349 
fiziks
Institute for NET/JRF, GATE, IIT‐JAM, JEST, TIFR and GRE in PHYSICAL SCIENCES 
 
Q30. An infinite, conducting slab kept in a horizontal plane carries a uniform charge density σ .
Another infinite slab of thickness t, made of a linear dielectric material of dielectric
constant k , is kept above the conducting slab. The bound charge density on the upper
surface of the dielectric slab is
σ σ σ (k − 2) σ (k − 1)
(a) (b) (c) (d)
2k k 2k k

Ans.: (d) +σ 1
k −σ 1 z
Solution: +σ
σ σ
Electric field due to infinite, conducting slab inside the dielectric is E = zˆ = zˆ
ε ε 0k

σ σ ( k − 1) σ ( k − 1)
Polarisation P = ε 0 χ e E = ε 0 ( k − 1) zˆ = zˆ ⇒ σ 1 = P.zˆ =
ε 0k k k
Q31. The electric field component of a plane electromagnetic wave travelling in vacuum is
given by E ( z , t ) = E 0 cos(kz − ωt )iˆ . The Poynting vector for the wave is

⎛ cε ⎞ ⎛ cε ⎞
(a) ⎜ 0 ⎟ E 02 cos 2 (kz − ωt ) ˆj (b) ⎜ 0 ⎟ E 02 cos 2 (kz − ωt )kˆ
⎝ 2 ⎠ ⎝ 2 ⎠

(c) cε 0 E 02 cos 2 (kz − ωt ) ˆj (d) cε 0 E 02 cos 2 (kz − ωt )kˆ


Ans.: (d)
1 E
Solution: E ( z , t ) = E 0 cos(kz − ωt )iˆ ⇒ B = zˆ × E ( z , t ) = 0 cos ( kz − ωt ) ˆj
c c
The Poynting vector for the wave is
E02
S=
1
μ0
( E×B = ) μ0 c
cos 2 ( kz − ωt ) kˆ = cε 0 E02 cos 2 ( kz − ωt ) kˆ

                                                                                
Head office  Branch office 
 
fiziks, H.No. 40 D, G.F, Jia Sarai,  Anand Institute of Mathematics, 
 
Near IIT, Hauz Khas, New Delhi‐16  28‐B/6, Jia Sarai, Near IIT 
 
Phone: 011‐26865455/+91‐9871145498 Hauz Khas, New Delhi‐16 
                                                   
                                             Website: www.physicsbyfiziks.com                                                                                          
                                                            Email: fiziks.physics@gmail.com                                                                     350 
fiziks
Institute for NET/JRF, GATE, IIT‐JAM, JEST, TIFR and GRE in PHYSICAL SCIENCES 
 
Q32. The x − y plane is the boundary between free space and a magnetic material with relative

permeability μ r . The magnetic field in the free space is Bx iˆ + Bz kˆ . The magnetic field in
the magnetic material is
(a) B x iˆ + B z kˆ (b) B x iˆ + μ r B z kˆ

1
(c) B x iˆ + B z kˆ (d) μ r B x iˆ + B z kˆ
μr
Ans.: (d)
B B
Solution: B1⊥ = Bz kˆ = B2⊥ and H1 = H1 ⇒ 1 = 2 ⇒ B2 = μr B1 = μr Bx iˆ
μ0 μ0 μr

The magnetic field in the magnetic material is μ r B x iˆ + B z kˆ

                                                                                
Head office  Branch office 
 
fiziks, H.No. 40 D, G.F, Jia Sarai,  Anand Institute of Mathematics, 
 
Near IIT, Hauz Khas, New Delhi‐16  28‐B/6, Jia Sarai, Near IIT 
 
Phone: 011‐26865455/+91‐9871145498 Hauz Khas, New Delhi‐16 
                                                   
                                             Website: www.physicsbyfiziks.com                                                                                          
                                                            Email: fiziks.physics@gmail.com                                                                     351 
fiziks
Institute for NET/JRF, GATE, IIT‐JAM, JEST, TIFR and GRE in PHYSICAL SCIENCES 
 
QUANTUM MECHANICS SOLUTIONS
GATE- 2010
Q1. Which of the following is an allowed wavefunction for a particle in a bound state? N is
a constant and α, β > 0.
e −αr
(a) ψ = N (b) ψ = N (1 − e −αr )
r3
(c) ψ = Ne −αx e − β (x ) (d) ψ = ⎧⎨0non - zero constant if r < R
2
+ y2 +z2
⎩ if r > R
Ans: (c)
Q2. A particle of mass m is confined in the potential
⎧⎪ 1
V ( x ) = ⎨ 2 mω x
2 2
for x > 0
⎪⎩∞ for x ≤ 0
Let the wavefunction of the particle be given by
1 2
ψ (x ) = − ψ 0 + ψ1 ,
5 5
where ψ 0 and ψ 1 are the eigenfunctions of the ground state and the first excited state

respectively. The expectation value of the energy is


31 25 13 11
(a) ω (b) ω (c) ω (d) ω
10 10 10 10
Ans: (a)
Solution: For half parabolic potential
3 7 1 3 4 7 31
E0 = ω , E1 = ω ⇒ E = × + × = ω.
2 2 5 2 5 2 10
2
Q3. For a spin-s particle, in the eigen basis of S , Sz the expectation value sm S z2 sm is

(a)
2
{s(s + 1) − m } 2
(b) 2
{s(s + 1) − 2m } 2

2
(c) 2
{
s (s + 1) − m 2 } (d) 2
m2

Ans: (a)
1 1
sm (S + + S − ) sm = sm S +2 + S −2 + S + S − + S − S + sm
2
Solution: sm S z2 sm =
2 2
[ ]
2
1
= sm S + S − + S − S + sm = s (s + 1) − m 2
2 2
                                                                                
Head office  Branch office 
 
fiziks, H.No. 40 D, G.F, Jia Sarai,  Anand Institute of Mathematics, 
 
Near IIT, Hauz Khas, New Delhi‐16  28‐B/6, Jia Sarai, Near IIT 
 
Phone: 011‐26865455/+91‐9871145498 Hauz Khas, New Delhi‐16 
                                                   
                                             Website: www.physicsbyfiziks.com                                                                                          
                                                            Email: fiziks.physics@gmail.com                                                                     352 
fiziks
Institute for NET/JRF, GATE, IIT‐JAM, JEST, TIFR and GRE in PHYSICAL SCIENCES 
 
Q4. A particle of mass m is confined in an infinite potential well:

V (x ) = ⎧⎨0 if 0 < x < L,


⎩∞ otherwise.
⎛ 2πx ⎞
It is subjected to a perturbing potential V p ( x ) = Vo sin ⎜ ⎟
⎝ L ⎠ V (x )
within the well. Let E(a) and E(2) be corrections to the ground
state energy in the first and second order in V0, respectively. V p (x )
0 L
Which of the following are true?
(a) E(a) = 0; E(2) < 0 (b) E(a) >
0; E(2) = 0
(c) E(a) = 0; E(2) depends on the sign of V0 (d) E(a) < 0; E(2) < 0
Ans: (a)
2
2
L
2πx ψ m VP ψ 1
Solution: E = ∫ V0 sin
1
1 dx = 0 ; E12 = ∑ ∵ E1 < E m so E12 = −ve .
L0 L m ≠1 E1 − Em

GATE- 2011
Q5. The quantum mechanical operator for the momentum of a particle moving in one
dimension is given by
2
d d ∂ d2
(a) i (b) − i (c) i (d) −
dx dx ∂t 2m dx 2
Ans: (b)
Q6. An electron with energy E is incident from left on a potential barrier, given by
V ( x) = 0 for x < 0 V (x )
= V0 for x > 0
V0
as shown in the figure. For E < V0, the space part of the
E
wavefunction for x > 0 is of the form
(a) eax (b) e-ax (c) eiax x
0
(d)e-iax
Ans: (b)
Solution: ∵ E < V0 , so there is decaying wave function.

                                                                                
Head office  Branch office 
 
fiziks, H.No. 40 D, G.F, Jia Sarai,  Anand Institute of Mathematics, 
 
Near IIT, Hauz Khas, New Delhi‐16  28‐B/6, Jia Sarai, Near IIT 
 
Phone: 011‐26865455/+91‐9871145498 Hauz Khas, New Delhi‐16 
                                                   
                                             Website: www.physicsbyfiziks.com                                                                                          
                                                            Email: fiziks.physics@gmail.com                                                                     353 
fiziks
Institute for NET/JRF, GATE, IIT‐JAM, JEST, TIFR and GRE in PHYSICAL SCIENCES 
 
Q7. If Lx, Ly and Lz are respectively the x, y and z components of angular momentum operator
L. The commutator [Lx Ly, Lz] is equal to
(a) i (L2x + L2y ) (b) 2i Lz (c) i (L2x − L2y ) (d) 0

Ans: (c)
[ ]
Solution: Lx L y , Lz = Lx [Ly Lz ] + [Lx , Lz ]Ly = i (L2x − L2y )

Q8. The normalized ground state wavefunciton of a hydrogen atom is given by


1 2
ψ (r ) = e −r / a , where a is the Bohr radius and r is the distance of the electron
4π a
3/ 2

1
from the nucleus, located at the origin. The expectation value is
r2
8π 4π 4 2
(a) (b) (c) (d)
a2 a2 a2 a2
Ans: (d)
∞ π 2π
1 1 1 2 − 2ar 2
3 ∫ 2
Solution: = r e dr ∫ ∫ sin θ dθ dφ = 2
r2 4πa 0 r 0 0
a

Q9. The normalized eigenstates of a particle in a one-dimensional potential well

V (x ) = ⎧⎨0 if 0 ≤ x ≤ a
⎩∞ otherwise

2 ⎛ nπx ⎞
are given by ψ n ( x ) = sin ⎜ ⎟ , where n = 1, 2, 3,….
a ⎝ a ⎠
The particle is subjected to a perturbation
⎛ πx ⎞ a
V ' ( x ) = Vo cos⎜ ⎟ for 0 ≤ x ≤
⎝ a⎠ 2
=0 otherwise
Q10. The shift in the ground state energy due to the perturbation, in the first order perturbation
theory,
2Vo Vo Vo 2Vo
(a) (b) (c) − (d) −
3π 3π 3π 3π

                                                                                
Head office  Branch office 
 
fiziks, H.No. 40 D, G.F, Jia Sarai,  Anand Institute of Mathematics, 
 
Near IIT, Hauz Khas, New Delhi‐16  28‐B/6, Jia Sarai, Near IIT 
 
Phone: 011‐26865455/+91‐9871145498 Hauz Khas, New Delhi‐16 
                                                   
                                             Website: www.physicsbyfiziks.com                                                                                          
                                                            Email: fiziks.physics@gmail.com                                                                     354 
fiziks
Institute for NET/JRF, GATE, IIT‐JAM, JEST, TIFR and GRE in PHYSICAL SCIENCES 
 
Ans: (a)
a/2
3 πx
sin
⎛ πx ⎞ ⎛ πx ⎞
a/2 a/2
2 2 2V0
Solution: E11 = ∫ψ 1 V ′(x )ψ 1dx = ∫ = V0 a =
*
sin 2 ⎜ ⎟V0 cos⎜ ⎟dx
a ⎝ a⎠ ⎝ a⎠ a π 3π
0 0 3
a 0

Common data questions Q-10 and Q-11


Q11. In a one-dimensional harmonic oscillator, φ0, φ1 and φ2 are respectively the ground, first
and the second excited states. These three states are normalized and are orthogonal to one
another. ψ1 and ψ2 are two states defined by
ψ1 = φ0 - 2φ1 + 3φ2 , ψ2 = φ0 – φ1 + αφ2
where α is a constant

Q12. The value of α which ψ2 is orthogonal to ψ1 is


(a) 2 (b) 1 (c) – 1 (d) – 2
Ans: (c)
Solution: For orthogonal condition scalar product (ψ2, ψ1)=0 so 1 + 2 + 3α = 0 ⇒ α = −1 .

Q13. For the value of α determined in Q30, the expectation value of energy of the oscillator in
the state ψ2 is
(a) ħω (b) 3 ħω/2 (c) 3 ħω (d) 9 ħω/2
Ans: (b)
ω 3 ω 5 ω
ψ2 H ψ2 + +
Solution: ψ 2 = φ0 − φ1 + αφ2 put α = −1 , H = = 2 2 2 =3 ω
ψ2 ψ2 3 2

                                                                                
Head office  Branch office 
 
fiziks, H.No. 40 D, G.F, Jia Sarai,  Anand Institute of Mathematics, 
 
Near IIT, Hauz Khas, New Delhi‐16  28‐B/6, Jia Sarai, Near IIT 
 
Phone: 011‐26865455/+91‐9871145498 Hauz Khas, New Delhi‐16 
                                                   
                                             Website: www.physicsbyfiziks.com                                                                                          
                                                            Email: fiziks.physics@gmail.com                                                                     355 
fiziks
Institute for NET/JRF, GATE, IIT‐JAM, JEST, TIFR and GRE in PHYSICAL SCIENCES 
 
GATE- 2012

Q14. A particle of mass m is confined in a two dimensional square well potential of


dimension a. This potential V(x, y) is given by
V(x, y) = 0 for –a < x < a and –a < y < a
= ∞ elsewhere
The energy of the first excited state for this particle is given by,
π2 2
2π 2 2 5π 2 2 4π 2 2
(a) (b) (c) (d)
ma 2 ma 2 8ma 2 ma 2
Ans: (c)
π2 2
5π 2 2
π2 2
Solution: E = ( n + n 2 2
) = (n + n )
2
x
2
y = ∵ n x = 1, n y = 2 .
2m ( 2a )
x y 2
8ma 2 8ma 2

Q15. Consider the wavefunction ψ = ψ (r1 , r2 )χ s for a fermionic system consisting of two spin-
half particles. The spatial part of the wavefunction is given by
1
ψ (r1 , r2 ) = [φ1 (r1 )φ 2 (r2 ) + φ 2 (r1 )φ1 (r2 )]
2
where φ1andφ 2 are single particle states. The spin part χs of the wavefunction with spin
states α (+ 1 / 2 )andβ (- 1/2 ) should be
1 1
(a) (αβ + βα ) (b) (αβ − βα ) (c) αα (d) ββ
2 2
Ans: (b)
Solution: Since ψ (r1 , r2 ) is symmetric the total wavefunction must be antisymmetric for fermions
so spin part must be antisymmetric.

                                                                                
Head office  Branch office 
 
fiziks, H.No. 40 D, G.F, Jia Sarai,  Anand Institute of Mathematics, 
 
Near IIT, Hauz Khas, New Delhi‐16  28‐B/6, Jia Sarai, Near IIT 
 
Phone: 011‐26865455/+91‐9871145498 Hauz Khas, New Delhi‐16 
                                                   
                                             Website: www.physicsbyfiziks.com                                                                                          
                                                            Email: fiziks.physics@gmail.com                                                                     356 
fiziks
Institute for NET/JRF, GATE, IIT‐JAM, JEST, TIFR and GRE in PHYSICAL SCIENCES 
 
Q16. A particle is constrained to move in a truncated harmonic potential well (x > 0) as shown
in the figure. Which one of the following statements is CORRECT?
(a) The parity of the first excited state is even
(b) The parity of the ground state is even V(x)
1
(c) the ground state energy is ω
2
7
(d) The first excited state energy is ω
2
Ans: (d) x
3 7
Solution: There is only odd parity. Ground state is ω and first excited = ω
2 2

Q17. Consider a system in the unperturbed state described by the Hamiltonian, H0 = ⎛⎜1 0⎞ .
⎝0 1 ⎟⎠

The system is subjected to a perturbation of the form H' = ⎛⎜δ δ⎞


⎝δ δ ⎟⎠ , when δ<< 1. The
energy eigenvalues of the perturbed system using the first order perturbation
approximation are
(a) 1 and (1 + 2δ) (b) (1 + δ) and (1 - δ)
(c) (1+ 2δ) and (1 - 2δ) (d) (1+ δ) and (1 - 2δ)
Ans: (a)
Solution: H 0 + H ′ , H0 is degenerate so after using degenerate perturbation through diagonalized

⎛ 2 0⎞ ⎛1 0⎞ ⎛ 2 0⎞
H ′ one will get H′ = δ ⎜ ⎟, H = ⎜ ⎟ + δ⎜ ⎟.
⎝0 0⎠ ⎝0 1⎠ ⎝ 0 0⎠
So E = 1 + 2δ and 1+ 0δ .

                                                                                
Head office  Branch office 
 
fiziks, H.No. 40 D, G.F, Jia Sarai,  Anand Institute of Mathematics, 
 
Near IIT, Hauz Khas, New Delhi‐16  28‐B/6, Jia Sarai, Near IIT 
 
Phone: 011‐26865455/+91‐9871145498 Hauz Khas, New Delhi‐16 
                                                   
                                             Website: www.physicsbyfiziks.com                                                                                          
                                                            Email: fiziks.physics@gmail.com                                                                     357 
fiziks
Institute for NET/JRF, GATE, IIT‐JAM, JEST, TIFR and GRE in PHYSICAL SCIENCES 
 
Q18. The ground state wavefunction for the hydrogen atom is given by
3/ 2
1 ⎛ 1 ⎞
ψ 100 = ⎜ ⎟⎟ e − r / a0 , where a 0 is the Bohr radius. The plot of the radial probability
4π ⎜⎝ a 0 ⎠
density, P(r) for the hydrogen atom in the ground state is
(a) (b)

P(r) P(r)

r/a 0 r/a 0
(c) (d)

P(r) P(r)

r/a 0 r/a 0
Ans: (d)
3/ 2
1 ⎛1⎞
Solution: The ground state is given by ψ 100 = ⎜ ⎟ e− r / a0
4π ⎝ a0 ⎠
Radial probability function
P(r )
1 1 2 − 2 r / a0
P(r ) = ψ r =
2 2
r e
4π a0 r / a0
Common Data for Questions 19–20
The wavefunction of particle moving in free space is given by, ψ = e ikx + 2e − ikx
Q19. The energy of the particle is
5 2k 2 3 2k 2 k2 2 2
k2
(a) (b) (c) (d)
2m 4m 2m m
Ans: (c)

                                                                                
Head office  Branch office 
 
fiziks, H.No. 40 D, G.F, Jia Sarai,  Anand Institute of Mathematics, 
 
Near IIT, Hauz Khas, New Delhi‐16  28‐B/6, Jia Sarai, Near IIT 
 
Phone: 011‐26865455/+91‐9871145498 Hauz Khas, New Delhi‐16 
                                                   
                                             Website: www.physicsbyfiziks.com                                                                                          
                                                            Email: fiziks.physics@gmail.com                                                                     358 
fiziks
Institute for NET/JRF, GATE, IIT‐JAM, JEST, TIFR and GRE in PHYSICAL SCIENCES 
 
− ∂ψ −
2 2 2
Solution: Hψ = Eψ , Hψ = = (ik )(ik )e ikx + 2(− ik )(− ik )e −ikx
2m ∂x 2
2m
2
k 2 ikx 2 2
k
⇒ Hψ = (e + 2e −2ikx ) = ψ
2m 2m

Q20. The probability current density for the real part of the wavefunction is
k k
(a) 1 (b) (c) (d) 0
m 2m
Ans: (d)
Solution: The real part of the wave functionψ real = cos kx + 2 cos kx
Current density for real part of wave function = 0

GATE- 2013
Q21. Which one of the following commutation relations is NOT CORRECT? Here, symbols
have their usual meanings.
[
(a) L2 , L z = 0 ] [ ]
(b) L x , L y = i L z

(c) [L z , L+ ] = L+ (d) [L z , L− ] = L−
Ans: (d)
Q22. A proton is confined to a cubic box, whose sides have length 10 −12 m . What is the
minimum kinetic energy of the proton? The mass of proton is 1.67 × 10 −27 kg and

Planck’s constant is 6.63 × 10 −34 Js .


(a) 1.1 × 10 −17 J (b) 3.3 × 10 −17 J (c) 9.9 × 10 −17 J (d) 6.6 × 10 −17 J
Ans: (c)
3π 2 2
Solution: 2
= 9.9 ×10−17
2ma

                                                                                
Head office  Branch office 
 
fiziks, H.No. 40 D, G.F, Jia Sarai,  Anand Institute of Mathematics, 
 
Near IIT, Hauz Khas, New Delhi‐16  28‐B/6, Jia Sarai, Near IIT 
 
Phone: 011‐26865455/+91‐9871145498 Hauz Khas, New Delhi‐16 
                                                   
                                             Website: www.physicsbyfiziks.com                                                                                          
                                                            Email: fiziks.physics@gmail.com                                                                     359 
fiziks
Institute for NET/JRF, GATE, IIT‐JAM, JEST, TIFR and GRE in PHYSICAL SCIENCES 
 
Q23. A spin-half particle is in a linear superposition 0.8 ↑ + 0.6 ↓ of its spin-up and spin-

down states. If ↑ and ↓ are the eigenstates if σ z then what is the expectation value

up to one decimal place, of the operator 10σ z + 5σ x ? Here, symbols have their usual
meanings. _______________
Ans: 76
⎛1⎞ ⎛ 0 ⎞ ⎛ .8 ⎞
Solution: ψ = .8 ↑ + .6 ↓ = .8⎜⎜ ⎟⎟ + .6⎜⎜ ⎟⎟ = ⎜⎜ ⎟⎟
⎝ 0⎠ ⎝ 1 ⎠ ⎝ .6 ⎠
⎛1 0⎞ ⎛0 1⎞ ⎛10 5 ⎞
Operator A = 10σ z + 5σ x = 10⎜⎜ ⎟⎟ + 5⎜⎜ ⎟⎟ ⇒ A = ⎜⎜ ⎟⎟
⎝0 1⎠ ⎝1 0⎠ ⎝ 5 10 ⎠
⎛10 5 ⎞⎛ .8 ⎞
A = ψ Aψ = (.8 .6)⎜⎜ ⎟⎟⎜⎜ ⎟⎟ = (88 − 12 ) = 76
⎝ 5 10 ⎠⎝ .6 ⎠
Q24. Consider the wave function Ae i k r (r0 / r ) , where A is the normalization constant.

For r = 2r0 , the magnitude of probability current density up to two decimal places, in

( )
units of A 2 k / m is. _____________
Ans:
2 2
2 k 2 r k 2 r k 2 k
Solution: J = ψ = A 0 ⇒J= A 0 ⇒J= A
m r m 2r0 m 4m

Common data questions 25 and 26


⎡5 2 0 ⎤
To the given unperturbed Hamiltonian ⎢⎢2 5 0⎥⎥
⎢⎣0 0 2⎥⎦

⎡1 1 1⎤
we add a small perturbation given by ε ⎢1 1 − 1⎥⎥ where ε is small quantity.

⎢⎣1 − 1 1 ⎥⎦

                                                                                
Head office  Branch office 
 
fiziks, H.No. 40 D, G.F, Jia Sarai,  Anand Institute of Mathematics, 
 
Near IIT, Hauz Khas, New Delhi‐16  28‐B/6, Jia Sarai, Near IIT 
 
Phone: 011‐26865455/+91‐9871145498 Hauz Khas, New Delhi‐16 
                                                   
                                             Website: www.physicsbyfiziks.com                                                                                          
                                                            Email: fiziks.physics@gmail.com                                                                     360 
fiziks
Institute for NET/JRF, GATE, IIT‐JAM, JEST, TIFR and GRE in PHYSICAL SCIENCES 
 
Q25. The ground state eigenvector of the unperturbed Hamiltonian is
(
(a) 1 / 2 ,1 2 ,0 ) (
(b) 1 / 2 ,−1 / 2 ,0 )
(c) (0,0,1) (d) (1,0,0 )
Ans: (c)
⎡5 2 0⎤ ⎛1 1 1⎞
⎢ ⎥ ⎜ ⎟
H 0 = ⎢ 2 5 0 ⎥ , H P = ε ⎜ 1 1 − 1⎟
⎢⎣0 0 2⎥⎦ ⎜1 − 1 1 ⎟
⎝ ⎠
Eigen value of H 0 is E1 = 2, E2 = 3, E3 = 7 and the Eigen vector corresponds

⎛0⎞ ⎛1⎞ ⎛1⎞


⎜ ⎟ 1 ⎜ ⎟ 1 ⎜ ⎟
to φ1 = ⎜ 0 ⎟ , φ2 = −1 , φ3 = 1 .
⎜1⎟ 2 ⎜⎜ ⎟⎟ 2 ⎜⎜ ⎟⎟
⎝ ⎠ ⎝0⎠ ⎝0⎠
Q26. A pair of eigenvalues of the perturbed Hamiltonian, using first order perturbation theory,
is
(a) 3 + 2ε ,7 + 2ε (b) 3 + 2ε ,+2 + ε
(c) 3, 7 + 2ε (d) 3, 2 + 2ε
Ans: (c)
Solution: E1′ = φ1 H P φ1 = 1ε ⇒ E1 = 2 + 1ε

⎛1 1 1⎞ ⎛1⎞ ⎛1⎞
1 ⎜ ⎟ 1 ⎜ ⎟ 1 ⎜ ⎟
E2′ = φ2 H P φ2 = (1 − 1 0).ε ⎜1 1 − 1⎟. ⎜ − 1⎟ = (0 0 1)⎜ − 1⎟ = 0
2 ⎜1 − 1 1 ⎟ 2 ⎜ 0 ⎟ 2 ⎜0⎟
⎝ ⎠ ⎝ ⎠ ⎝ ⎠

⎛1 1 1 ⎞ ⎛1⎞ ⎛1⎞
1 ⎜ ⎟ 1 ⎜ ⎟ 1 ⎜ ⎟
E3′ = φ3 H P φ3 = (1 1 0 ) .ε ⎜1 1 −1⎟ . ⎜ 1 ⎟ = ε . (2 2 0).⎜ 1 ⎟
2 ⎜1 −1 1 ⎟ 2 ⎜ 0 ⎟ 2 ⎜ 0⎟
⎝ ⎠ ⎝ ⎠ ⎝ ⎠
1
⇒ E 3′ = (4)ε = 2ε
2
E1 = 2 + 1ε , E 2 = 3 + 0ε , E3 = 7 + 2ε .

                                                                                
Head office  Branch office 
 
fiziks, H.No. 40 D, G.F, Jia Sarai,  Anand Institute of Mathematics, 
 
Near IIT, Hauz Khas, New Delhi‐16  28‐B/6, Jia Sarai, Near IIT 
 
Phone: 011‐26865455/+91‐9871145498 Hauz Khas, New Delhi‐16 
                                                   
                                             Website: www.physicsbyfiziks.com                                                                                          
                                                            Email: fiziks.physics@gmail.com                                                                     361 
fiziks
Institute for NET/JRF, GATE, IIT‐JAM, JEST, TIFR and GRE in PHYSICAL SCIENCES 
 
GATE- 2014
Q27. The recoil momentum of an atom is p A when it emits an infrared photon of wavelength
1500 nm , and it is p B when it emits a photon of visible wavelength 500 nm . The ratio

pA
is
pB

(a) 1 : 1 (b) 1 : 3 (c) 1 : 3 (d) 3 : 2


Ans: (c)
h p A λB λ B 500
Solution: p = , = , = =1 : 3
λ pB λ A λ A 1500
Q28. The ground state and first excited state wave function of a one dimensional infinite
potential well are ψ 1 and ψ 2 respectively. When two spin-up electrons are placed in this

potential which one of the following with x1 and x2 denoting the position of the two
electrons correctly represents the space part of the ground state wave function of the
system?
1 1
(a) [ψ 1 (x1 )ψ 2 (x1 ) − ψ 1 (x2 )ψ 2 (x2 )] (b) [ψ 1 (x1 )ψ 2 (x2 ) + ψ 1 (x2 )ψ 2 (x1 )]
2 2
1 1
(c) [ψ 1 (x1 )ψ 2 (x1 ) + ψ 1 (x2 )ψ 2 (x2 )] (d [ψ 1 (x1 )ψ 2 (x 2 ) − ψ 1 (x 2 )ψ 2 (x1 )] )
2 2
Ans: (d)
Solution: From the given information only possible spin configuration is symmetric in nature so
space part will anti symmetric
1
[ψ 1 (x1 )ψ 2 (x 2 ) − ψ 1 (x 2 )ψ 2 (x1 )]
2

Q29. If L is the orbital angular momentum and S is the spin angular momentum, then L.S
does not commute with

(a) S z (b) L2 (c) S 2 (


(d) L + S )2

Ans: (d)

                                                                                
Head office  Branch office 
 
fiziks, H.No. 40 D, G.F, Jia Sarai,  Anand Institute of Mathematics, 
 
Near IIT, Hauz Khas, New Delhi‐16  28‐B/6, Jia Sarai, Near IIT 
 
Phone: 011‐26865455/+91‐9871145498 Hauz Khas, New Delhi‐16 
                                                   
                                             Website: www.physicsbyfiziks.com                                                                                          
                                                            Email: fiziks.physics@gmail.com                                                                     362 
fiziks
Institute for NET/JRF, GATE, IIT‐JAM, JEST, TIFR and GRE in PHYSICAL SCIENCES 
 
Q30. An electron in the ground state of the hydrogen atom has the wave function
⎛ r ⎞
− ⎜⎜ ⎟⎟
1
ψ (r ) = e ⎝ a0 ⎠

πa 03

where a0 is constant. The expectation value of the operator Qˆ = z 2 − r 2 , where

z = r cos θ is
∞ n ( n − 1)! )
(Hint: ∫ 0
e− ar r n dr =
a n +1
=
a n +1
− a 02 − 3a 02
(a) (b) − a 02 (c) (d) − 2a02
2 2
Ans: (d)

Solution: Qˆ = z 2 − r 2 ⇒ a02 − 3a02 = −2a02

Q31. A particle of mass m is subjected to a potential

V ( x, y ) =
1
2
( )
mω 2 x 2 + y 2 , − ∞ ≤ x ≤ ∞,−∞ ≤ y ≤ ∞

The state with energy 4 ω is g − fold degenerate. The value of g is______


Ans: 4
Solution: This is two isotropic dimensional harmonic oscillator the energy eigen value for nth
state is E n = (n + 1) ω with degeneracy g n = (n + 1) so degeneracy for 4 ω is 4 .
Q32. A hydrogen atom is in the state

8 3 4
ψ= ψ 200 − ψ 310 + ψ 321
21 7 21
where n, l , m in ψ nl m denote the principal. Orbit and magnetic quantum numbers,

respectively. If L is the angular momentum operator, the average value of L2 is


2
_______
Ans: 2

                                                                                
Head office  Branch office 
 
fiziks, H.No. 40 D, G.F, Jia Sarai,  Anand Institute of Mathematics, 
 
Near IIT, Hauz Khas, New Delhi‐16  28‐B/6, Jia Sarai, Near IIT 
 
Phone: 011‐26865455/+91‐9871145498 Hauz Khas, New Delhi‐16 
                                                   
                                             Website: www.physicsbyfiziks.com                                                                                          
                                                            Email: fiziks.physics@gmail.com                                                                     363 
fiziks
Institute for NET/JRF, GATE, IIT‐JAM, JEST, TIFR and GRE in PHYSICAL SCIENCES 
 
Solution: If L will measure on state ψ the measurement is 0 2 , 2 2 and 6 2 with probability
2

8 3 4 3 4
, and so L2 = 2 2
× +6 2
× = 2 2

21 7 21 7 21
1
Q33. ψ 1andψ 2 are two orthogonal states of a spin system. It is given that
2
1 ⎛1⎞ 2 ⎛0⎞
ψ1 = ⎜⎜ ⎟⎟ + ⎜ ⎟,
3 ⎝0⎠ 3 ⎜⎝ 1 ⎟⎠

⎛1⎞ ⎛ 0⎞
where ⎜⎜ ⎟⎟ and ⎜⎜ ⎟⎟ represent the spin-up and spin-down states, respectively. When the
⎝0⎠ ⎝1⎠
system is in the state ψ 2 its probability to be in the spin-up state is _______
2
Ans:
3
1 ⎛1⎞ 2 ⎛0⎞ 2 ⎛1⎞ 1 ⎛ 0⎞
Solution: If is ψ 1 = ⎜⎜ ⎟⎟ + ⎜ ⎟, then ψ 2 = ⎜⎜ ⎟⎟ + ⎜ ⎟,
3 ⎝0⎠ 3 ⎜⎝ 1 ⎟⎠ 3 ⎝ 0⎠ 3 ⎜⎝ 1 ⎟⎠

2
so probability that ψ 2 is in up state is
3
Q34. A particle is confined to a one dimensional potential box, with the potential
V (x ) = 0, 0< x<a
= ∞, otherwise
If particle is subjected to a perturbation within the box. W = β x . Where β is small
constant, the first order correction to the ground state energy is
(a) 0 (b) aβ / 4 (c) aβ / 2 (d) aβ
Ans: (c)
Solution: First order energy correction is W = β x the average value position in ground state

a
is x = so ans is aβ / 2
2

                                                                                
Head office  Branch office 
 
fiziks, H.No. 40 D, G.F, Jia Sarai,  Anand Institute of Mathematics, 
 
Near IIT, Hauz Khas, New Delhi‐16  28‐B/6, Jia Sarai, Near IIT 
 
Phone: 011‐26865455/+91‐9871145498 Hauz Khas, New Delhi‐16 
                                                   
                                             Website: www.physicsbyfiziks.com                                                                                          
                                                            Email: fiziks.physics@gmail.com                                                                     364 
fiziks
Institute for NET/JRF, GATE, IIT‐JAM, JEST, TIFR and GRE in PHYSICAL SCIENCES 
 
Q35. A one dimensional harmonic oscillator is in the superposition of number state n given

by
1 3
ψ = 2 + 3
2 2
The average energy of the oscillator in the given state is______ ω .
Ans: 3.25
1 5 ω 3 7 ω
. + .
Solution: Average energy will 4 2 4 2 = 3.25 ω
1 3
+
4 4
Q36. If L+ and L− are the angular momentum ladder operators then the expectation value of

(L+ L− + L− L+ ) in the state l = 1, m = 1 of an atom is _____ 2

Ans: 2
Solution: (L+ L− + L− L+ ) = 2( L2 − L2z ) = 2(l.(l + 1) − m 2 ) 2
= 2 2

GATE- 2015
1
Q37. An operator for a spin − particle is given by  = λσ ⋅ B ,where
2
B
B=( xˆ + yˆ ) , σ denotes Pauli matrices and λ is a constant. The eigenvalues of  are
2
λB
(a) ± (b) ± λB (c) 0, λB (d) 0, − λ B
2
Ans.: (b)
B
Solution: Aˆ = λσ ⋅ β , B= ( xˆ + yˆ )
2

Aˆ = λ (σ x Bx + σ y By + σ z Bz ) ⇒ Aˆ = λ ⎡⎣σ x Bx + σ y By ⎤⎦

⎡⎛ 0 1 ⎞ B ⎛ 0 −i ⎞ B ⎤ ˆ λB ⎡ 0 1− i⎤
Aˆ = λ ⎢⎜ ⎟ +⎜ ⎟ ⎥ ⇒ A= ⎢ ⎥
⎣⎝ 1 0 ⎠ 2 ⎝ i 0 ⎠ 2 ⎦ 2 ⎣1 + i 0 ⎦

λ B ⎡ −λ 1 − i ⎤
A − λ ′I = 0 ⇒ ⎢ ⎥ = 0 ⇒ λ ′ = ±λ B
2 ⎣1 + i −λ ⎦

                                                                                
Head office  Branch office 
 
fiziks, H.No. 40 D, G.F, Jia Sarai,  Anand Institute of Mathematics, 
 
Near IIT, Hauz Khas, New Delhi‐16  28‐B/6, Jia Sarai, Near IIT 
 
Phone: 011‐26865455/+91‐9871145498 Hauz Khas, New Delhi‐16 
                                                   
                                             Website: www.physicsbyfiziks.com                                                                                          
                                                            Email: fiziks.physics@gmail.com                                                                     365 
fiziks
Institute for NET/JRF, GATE, IIT‐JAM, JEST, TIFR and GRE in PHYSICAL SCIENCES 
 
1
Q38. The Pauli matrices for three spin − particles are σ 1 , σ 2 and σ 3 , respectively. The
2
dimension of the Hilbert space required to define an operator Oˆ = σ 1 ⋅ σ 2 × σ 3 is_______
Ans.: 8
Solution: σ 2 × σ 3 has dimension of 4 and σ 1. σ 2 × σ 3 has dimension of 2 × 4 = 8

Q39. Let L and p be the angular and linear momentum operators, respectively, for a a particle.
The commutator L x , p y gives [ ]
(a) −i pz (b) 0 (c) i px (d) i pz
Ans.: (d)
Solution: ⎡⎣ Lx , p y ⎤⎦ = ⎡⎣ ypz − zp y , p y ⎤⎦ = ⎡⎣ ypz , p y ⎤⎦ − ⎡⎣ zp y , p y ⎤⎦ = ⎡⎣ y, p y ⎤⎦ pz

∵ ⎡⎣ p y , p y ⎤⎦ = 0 and ⎡⎣ z , p y ⎤⎦ = 0

⇒ ⎡⎣ Lx , p y ⎤⎦ = i pz ∵ ⎣⎡ y, p y ⎦⎤ = i
3
Q40. Consider a system of eight non-interacting, identical quantum particles of spin − in a
2
one dimensional box of length L . The minimum excitation energy of the system, in units
π2 2
of is ________
2mL2
Ans.: 5
3 ⎛ 3 ⎞
Solution: spin ⇒ degeneracy = ( 2S + 1) = ⎜ 2 × + 1⎟ = 4
2 ⎝ 2 ⎠
π2 2
4π 2 2 20π 2 2
Eground = 4 × + 4× =
2mL2 2mL2 2mL2
st π2 2
π2 2
π2 2
π2 2
I
Eexcited = 4× + 3× 4 + 1× 9 = 25
2mL2 2mL2 2mL2 2mL2
st π2 2
π2 2
π2 2
Now minimum excitation energy ΔE = Eexcited
I
− Eground = 25 − 20 =5
2mL2 2mL2 2mL2

                                                                                
Head office  Branch office 
 
fiziks, H.No. 40 D, G.F, Jia Sarai,  Anand Institute of Mathematics, 
 
Near IIT, Hauz Khas, New Delhi‐16  28‐B/6, Jia Sarai, Near IIT 
 
Phone: 011‐26865455/+91‐9871145498 Hauz Khas, New Delhi‐16 
                                                   
                                             Website: www.physicsbyfiziks.com                                                                                          
                                                            Email: fiziks.physics@gmail.com                                                                     366 
fiziks
Institute for NET/JRF, GATE, IIT‐JAM, JEST, TIFR and GRE in PHYSICAL SCIENCES 
 
Q41. A particle is confined in a box of length L as shown below.

V0
L/2
If the potential V0 is treated as a perturbation, including the first order correction, the
ground state energy is
2
π2 2
π2 V0
(a) E = 2
+ V0 (b) E = 2

2mL 2mL 2
2
π2 V0 2
π2 V0
(c) E = 2
+ (d) E = 2
+
2mL 4 2mL 2
Ans.: (d)
L
2
πx L
πx
Solution: E01 = ∫ V0 sin 2 dx + ∫ 0 × sin 2 dx
0
L L L
2

L L
V0 ⎛ 2
2π x ⎞ V0 ⎡ ⎛ 2π x ⎞ L ⎤ 2
⇒ E01 = ∫
2 0⎝
⎜1 − cos
L ⎠
⎟dx =
2 ⎢ x − sin ⎜ L ⎟ 2π x ⎥
⎣ ⎝ ⎠ ⎦0

V0 π
2 2
V
⇒ E01 = ⇒E= 2
+ 0
2 2mL 2
Q42. Let the Hamiltonian for two spin-½ particles of equal masses m , momenta p1 and p2

and positions r1 and r2 be H =


1 2
2m
p1 +
1 2 1
2m 2
(
p 2 + mω 2 r12 + r22 + kσ 1 ⋅ σ 2 , where σ 1 )
and σ 2 denote the corresponding Pauli matrices, ω = 0.1eV and k = 0.2 eV . If the
ground state has net spin zero, then the energy (in eV ) is ___________
Ans.: -0.3

                                                                                
Head office  Branch office 
 
fiziks, H.No. 40 D, G.F, Jia Sarai,  Anand Institute of Mathematics, 
 
Near IIT, Hauz Khas, New Delhi‐16  28‐B/6, Jia Sarai, Near IIT 
 
Phone: 011‐26865455/+91‐9871145498 Hauz Khas, New Delhi‐16 
                                                   
                                             Website: www.physicsbyfiziks.com                                                                                          
                                                            Email: fiziks.physics@gmail.com                                                                     367 
fiziks
Institute for NET/JRF, GATE, IIT‐JAM, JEST, TIFR and GRE in PHYSICAL SCIENCES 
 
1 2 1 2 1
Solution: H = p1 + p2 + mω 2 ( r12 + r22 ) + kσ 1.σ 2
2m 2m 2
2 2
σ = σ 1 + σ 2 ⇒ σ = σ 12 + σ 22 + 2σ 1.σ 2 ⇒ 2σ 1.σ 2 = σ − σ 12 − σ 22

⇒ 2σ 1.σ 2 = 0 − 3I − 3I = −6 I ⇒ σ 1.σ 2 = −3
3
Now energy E = 2 × ω + k ( −3) = 3 × ( 0.1) + ( 0.2 )( −3) = −0.3 eV
2
Q43. Suppose a linear harmonic oscillator of frequency ω and mass m is in the state
π
1 ⎡ i ⎤
ψ = ψ
⎢ 0 + e 2
ψ 1 ⎥ at t = 0 where ψ 0 and ψ 1 are the ground and the first
2⎣ ⎦

excited states, respectively. The value of ψ x ψ in the units of at t = 0 is _____



Ans. : 0
1 ⎡ π ⎤
Solution: Ψ = ⎢ Ψ 0 + ei Ψ1 ⎥
2⎣ 2 ⎦

Ψ xΨ = Ψ
2mω
(a + a ) Ψ +
⇒ Ψ xΨ =
2mω
(ΨaΨ + Ψ a+ Ψ )
1 i π2 1 ⎛ i
π

∵a Ψ = e Ψ0 and a + Ψ = Ψ
⎜ 0 + 2 e 2
Ψ2 ⎟
2 2⎝ ⎠
⎛ 1 1 i π2 1 1 − i π2 ⎞
⇒ Ψ xΨ = ⎜ e Ψ Ψ + e Ψ1 Ψ1 ⎟
2mω ⎝ 2 2
0 0
2 2 ⎠

⇒ Ψ xΨ = (0) = 0
2mω
GATE-2016
Q44. Which of the following operators is Hermitian?
d d2 d2 d3
(a) (b) (c) i (d)
dx dx 2 dx 2 dx 3
Ans.: (b)

                                                                                
Head office  Branch office 
 
fiziks, H.No. 40 D, G.F, Jia Sarai,  Anand Institute of Mathematics, 
 
Near IIT, Hauz Khas, New Delhi‐16  28‐B/6, Jia Sarai, Near IIT 
 
Phone: 011‐26865455/+91‐9871145498 Hauz Khas, New Delhi‐16 
                                                   
                                             Website: www.physicsbyfiziks.com                                                                                          
                                                            Email: fiziks.physics@gmail.com                                                                     368 
fiziks
Institute for NET/JRF, GATE, IIT‐JAM, JEST, TIFR and GRE in PHYSICAL SCIENCES 
 

Q45. The scattering of particles by a potential can be analyzed by Born approximation. In


particular, if the scattered wave is replaced by an appropriate plane wave, the
corresponding Born approximation is known as the first Born approximation. Such an
approximation is valid for
(a) large incident energies and weak scattering potentials.
(b) large incident energies and strong scattering potentials.
(c) small incident energies and weak scattering potentials.
(d) small incident energies and strong scattering potentials.
Ans.: (a)
Q46. Consider an elastic scattering of particles in l = 0 states. If the corresponding phase shift
δ 0 is 900 and the magnitude of the incident wave vector is equal to 2π fm −1 then the

total scattering cross section in units of fm 2 is _______.


Ans.: 2

Solution: σ = ∑ ( 2l + 1) sin δ l for l = 0 it is given δ 0 = 90 0 and k = 2π fm −1
k 2 l =0

σ= sin 90 = 2

Q47. A hydrogen atom is in its ground state. In the presence of a uniform electric field

E = E0 zˆ , the leading order change in its energy is proportional to ( E0 ) . The value of


n

the exponent n is _______.


Ans.: 2
Solution: First order energy correction is zero ψ 1,0,0 E0 r cos θ ψ 1,0,0 = 0
2
ψ n,l ,m E0 r cos θ ψ 1,0,0
So one need to find correction of second ∑ ∝ E02
n ≠1 E10 − Em0

So value of n = 2

                                                                                
Head office  Branch office 
 
fiziks, H.No. 40 D, G.F, Jia Sarai,  Anand Institute of Mathematics, 
 
Near IIT, Hauz Khas, New Delhi‐16  28‐B/6, Jia Sarai, Near IIT 
 
Phone: 011‐26865455/+91‐9871145498 Hauz Khas, New Delhi‐16 
                                                   
                                             Website: www.physicsbyfiziks.com                                                                                          
                                                            Email: fiziks.physics@gmail.com                                                                     369 
fiziks
Institute for NET/JRF, GATE, IIT‐JAM, JEST, TIFR and GRE in PHYSICAL SCIENCES 
 
Q48. If s1 and s2 are the spin operators of the two electrons of a He atom, the value of

s1 .s 2 for the ground state is

3 3 1
(a) − 2
(b) − 2
(c) 0 (d) 2

2 4 4
Ans.: (b)
1 1 s (s + 1) 2
− s1 (s1 + 1) 2
− s 2 (s 2 + 1) 2
Solution: s = s1 + s2 s1 = , s1 = , s = 0,1 , s1 .s 2 =
2 2 2
2 3 2 3 2
2 − −
4 4 3 2
For s = 1, s1.s2 = =
2 4
2 3 2 3 2
0 − −
4 4 3 2
s = 0, s1.s2 = =−
2 4
Q49. A two-dimensional square rigid box of side L contains six non-interacting electrons at
T = 0 K . The mass of the electron is m . The ground state energy of the system of
π2 2
electrons, in units of is _________.
2mL2
Ans.: 24

Solution: 2 ×
(1 2
)
+ 12 π 2 2

+ 4×
(2 2
+ 12 π 2 ) 2

=
24π 2 2
2mL2 2mL2 2mL2
Q50. σ x , σ y and σ z are the Pauli matrices. The expression 2σ xσ y + σ yσ x is equal to

(a) − 3iσ z (b) − iσ z (c) iσ z (d) 3iσ z


Ans.: (c)
Solution: 2σ xσ y + σ yσ x ⇒ σ xσ y + σ xσ y + σ yσ x ⇒ σ xσ y = iσ z

Q51. If x and p are the x components of the position and the momentum operators of a

particle respectively, the commutator x 2 , p 2 is [ ]


(a) i ( xp − px ) (b) 2i ( xp − px ) (c) i ( xp + px ) (d) 2i ( xp + px )
Ans.: (d)

                                                                                
Head office  Branch office 
 
fiziks, H.No. 40 D, G.F, Jia Sarai,  Anand Institute of Mathematics, 
 
Near IIT, Hauz Khas, New Delhi‐16  28‐B/6, Jia Sarai, Near IIT 
 
Phone: 011‐26865455/+91‐9871145498 Hauz Khas, New Delhi‐16 
                                                   
                                             Website: www.physicsbyfiziks.com                                                                                          
                                                            Email: fiziks.physics@gmail.com                                                                     370 
fiziks
Institute for NET/JRF, GATE, IIT‐JAM, JEST, TIFR and GRE in PHYSICAL SCIENCES 
 
2 2
[ 2
] [2
] [ ]
Solution: x , p = p x , p + x p p = 2i px + 2i xp ⇒ 2i ( xp + px )

Q52. Let l , m be the simultaneous eigenstates of L2 and L z . Here L is the angular

momentum operator with Cartesian components (L x , L y , L z ), l is the angular momentum

quantum number and m is the azimuthal quantum number. The value of

1, 0 ( Lx + iLy ) 1, −1 is

(a) 0 (b) (c) 2 (d) 3


Ans.: (c)

Solution: 1, 0 ( Lx + iLy 1, −1 = 1, 0 L+ 1, −1 = 2 1, 0 1, 0 = 2

Q53. For the parity operator P , which of the following statements is NOT true?
(a) P† = P (b) P 2 = − P (c) P 2 = I (d) P † = P −1
Ans.: (b)
Q54. The state of a system is given by
ψ = φ1 + 2 φ 2 + 3 φ3

Where φ1 , φ2 and φ3 form an orthonormal set. The probability of finding the system

in the state φ 2 is ________. (Give your answer upto two decimal places)

22 4 4 2
Ans probability that ψ in state φ2 = = = = = 0.28
2 2
1 +2 +32
1 + 4 + 9 14 7
Q55. A particle of mass m and energy E , moving in the positive x
V0
direction, is incident on a step potential at x = 0 , as indicated in the
E
figure. The height of the potential is V0 , where V0 > E . At x = x0 ,

1
where x0 > 0 , the probability of finding the electron is times the
e x=0 x = x0
2m(V0 − E )
probability of finding it at x = 0 . If α = 2
, the value of x0 is

2 1 1 1
(a) (b) (c) (d)
α α 2α 4α
                                                                                
Head office  Branch office 
 
fiziks, H.No. 40 D, G.F, Jia Sarai,  Anand Institute of Mathematics, 
 
Near IIT, Hauz Khas, New Delhi‐16  28‐B/6, Jia Sarai, Near IIT 
 
Phone: 011‐26865455/+91‐9871145498 Hauz Khas, New Delhi‐16 
                                                   
                                             Website: www.physicsbyfiziks.com                                                                                          
                                                            Email: fiziks.physics@gmail.com                                                                     371 
fiziks
Institute for NET/JRF, GATE, IIT‐JAM, JEST, TIFR and GRE in PHYSICAL SCIENCES 
 
Ans.: (c)
1 1
Solution: = e −2α x0 = e −1 = e−2α x0 ⇒ x0 =
e 2α

                                                                                
Head office  Branch office 
 
fiziks, H.No. 40 D, G.F, Jia Sarai,  Anand Institute of Mathematics, 
 
Near IIT, Hauz Khas, New Delhi‐16  28‐B/6, Jia Sarai, Near IIT 
 
Phone: 011‐26865455/+91‐9871145498 Hauz Khas, New Delhi‐16 
                                                   
                                             Website: www.physicsbyfiziks.com                                                                                          
                                                            Email: fiziks.physics@gmail.com                                                                     372 
fiziks
Institute for NET/JRF, GATE, IIT‐JAM, JEST, TIFR and GRE in PHYSICAL SCIENCES 
 
THERMODYNAMICS AND STATISTICAL PHYSICS SOLUTIONS
GATE 2010
Q1. A system of N non-interacting classical point particles is constrained to move on the two-
dimensional surface of a sphere. The internal energy of the system is
3 1 5
(a) Nk BT (b) Nk BT (c) Nk BT (d) Nk BT
2 2 2
Ans: (c)
Solution: There are 2 N degree of freedom.
Nk BT Nk BT
The internal energy of the system is + = Nk BT
2 2
Q2. Which of the following atoms cannot exhibit Bose-Einstein condensation, even in
principle?
(a) 1H1 (b) 4H2 (c) 23Na11 (d) 30K19
Ans: (d)
Solution: For Bose-Einstein condensation:
Number of electron + number of proton + number of neutron = Even
For 30 K19
Number of proton = 19, Number of electron = 19, Number of neutron = 11.
19 + 19 + 11 = 49 this is odd. So it will not exhibit Bose-Einstein condensation.
Q3. For a two-dimensional free electron gas, the electronic density n, and the Fermi energy
EF, are related by

(2mE F ) 2
3
mE F
(a) n = (b) n =
3π 2 3
π 2

mE F 23
2
(mE F ) 3 2
(c) n = (d) n =
2π 2 π
Ans: (c)
EF
2m f (E ) = 1 if E < E F
Solution: n = ∫ g(E)f (E)dE ,
0
g(E)dE =
h2
dE , at T=0
=0 if E > E F

                                                                                
Head office  Branch office 
 
fiziks, H.No. 40 D, G.F, Jia Sarai,  Anand Institute of Mathematics, 
 
Near IIT, Hauz Khas, New Delhi‐16  28‐B/6, Jia Sarai, Near IIT 
 
Phone: 011‐26865455/+91‐9871145498 Hauz Khas, New Delhi‐16 
                                                   
                                             Website: www.physicsbyfiziks.com                                                                                          
                                                            Email: fiziks.physics@gmail.com                                                                     373 
fiziks
Institute for NET/JRF, GATE, IIT‐JAM, JEST, TIFR and GRE in PHYSICAL SCIENCES 
 
2mE F mE F
⇒n= =
h 2
2π 2
Q4. Which among the following sets of Maxwell relations is correct? (U-internal energy, H-
enthalpy, A-Helmholtz free energy and G-Gibbs free energy)
⎛ ∂U ⎞ ⎛ ∂U ⎞ ⎛ ∂H ⎞ ⎛ ∂H ⎞
(a) T = ⎜ ⎟ and P = ⎜ ⎟ (b) V = ⎜ ⎟ and T = ⎜ ⎟
⎝ ∂V ⎠ S ⎝ ∂S ⎠V ⎝ ∂P ⎠ S ⎝ ∂S ⎠ P

⎛ ∂G ⎞ ⎛ ∂G ⎞ ⎛ ∂A ⎞ ⎛ ∂A ⎞
(c) P = −⎜ ⎟ and V = ⎜ ⎟ (d) P = −⎜ ⎟ and S = ⎜ ⎟
⎝ ∂V ⎠T ⎝ ∂P ⎠ S ⎝ ∂S ⎠T ⎝ ∂P ⎠V
Ans: (b)
⎛ ∂H ⎞ ⎛ ∂H ⎞
Solution: dH = TdS + VdP ⇒ ⎜ ⎟ = T,⎜ ⎟ =V
⎝ ∂S ⎠ P ⎝ ∂P ⎠ S

π 2V (k 0T )3
Q5. Partition function for a gas of photons is given as ln Z = . The specific heat of
45 3 C 3
the photon gas varies with temperature as
(a) (b)

CV CV

T T
(c) (d)

CV
CV

T
T
Ans: (a)
∂ ln z ⎛ ∂U ⎞
Solution: U = K B T 2 , CV = ⎜ ⎟ ⇒ CV ∝ T .
3

∂T ⎝ ∂T ⎠ v

                                                                                
Head office  Branch office 
 
fiziks, H.No. 40 D, G.F, Jia Sarai,  Anand Institute of Mathematics, 
 
Near IIT, Hauz Khas, New Delhi‐16  28‐B/6, Jia Sarai, Near IIT 
 
Phone: 011‐26865455/+91‐9871145498 Hauz Khas, New Delhi‐16 
                                                   
                                             Website: www.physicsbyfiziks.com                                                                                          
                                                            Email: fiziks.physics@gmail.com                                                                     374 
fiziks
Institute for NET/JRF, GATE, IIT‐JAM, JEST, TIFR and GRE in PHYSICAL SCIENCES 
 
Q6. From Q. no. 5, the pressure of the photon gas is
π 2 (k B T )3 π 2 (k B T )4 π 2 (k B T )4 π 2 (k B T )3 / 2
(a) (b) (c) (d)
15 3 C 3 8 3C 3 45 3 C 3 45 3 C 3
Ans: (c)

π2 ( k 0 T )
4
⎛ ∂ ln z ⎞
Solution: P = KT ⎜ ⎟ =
⎝ ∂V ⎠T 45 3C3
GATE 2011

Q7. A Carnot cycle operates on a working substance between two reservoir at temperatures T1
and T2 with T1 > T2. During each cycle, an amount of heat Q1 is extracted from the
reservoir at T1 and an amount Q2 is delivered in the reservoir at T2. Which of the
following statements is INCORRECT?
(a) Work done in one cycle is Q1 – Q2
Q1 Q2
(b) =
T1 T2

(c) Entropy of the hotter reservoir decreases


(d) Entropy of the universe (consisting of the working substance and the two reservoirs)
increases
Ans: (a)
Solution: Entropy of hotter reservoirs decreases.
Q8. In a first order phase transition, at the transition temperature, specific heat of the system
(a) diverges and its entropy remains the same
(b) diverges and its entropy has finite discontinuity
(c) remains unchanged and its entropy has finite discontinuity
(d) has finite discontinuity and its entropy diverges
Ans: (b)
Q9. A system of N non-interacting and distinguishable particle of spin 1 is in thermodynamic
equilibrium. The entropy of the system is
(a) 2kB ln N (b) 3kB ln N (c) NkB ln 2 (d) NkB ln 3

                                                                                
Head office  Branch office 
 
fiziks, H.No. 40 D, G.F, Jia Sarai,  Anand Institute of Mathematics, 
 
Near IIT, Hauz Khas, New Delhi‐16  28‐B/6, Jia Sarai, Near IIT 
 
Phone: 011‐26865455/+91‐9871145498 Hauz Khas, New Delhi‐16 
                                                   
                                             Website: www.physicsbyfiziks.com                                                                                          
                                                            Email: fiziks.physics@gmail.com                                                                     375 
fiziks
Institute for NET/JRF, GATE, IIT‐JAM, JEST, TIFR and GRE in PHYSICAL SCIENCES 
 
Ans: (d)
Solution: S = k B ∑ ln Ω , Ω =3 is number of microstate. S = 1; Sz = −1, 0, 1
i

The entropy of the system is NkB ln 3.


Q10. A system has two energy levels with energies ε and 2ε. The lower level is 4-fold
degenerate while the upper level is doubly degenerate. If there are N non-interacting
classical particles in the system, which is in thermodynamic equilibrium at a temperature
T, the fraction of particles in the upper level is
1 1
(a) (b)
1 + e ε / k BT 1 + 2e ε / k BT
1 1
(c) ε / k BT
(d)
2e + 4e 2 ε / k B T 2e ε / k BT
− 4e 2 ε / k B T
Ans: (b)
Solution: Partition function Z = 4e −∈/ kT + 2e −∈/ kT
2e −2∈/ kT
P(2ε ) =
1
−∈/ kT − 2∈/ kT
=
4e + 2e 1 + 2e∈/ kT
GATE 2012

Q11. The isothermal compressibility, κ of an ideal gas at temperature T0, and V0, is given by
1 ∂V 1 ∂V ∂P ∂P
(a) − (b) (c) − V0 (d) V0
V0 ∂P T0 V0 ∂P T0 ∂V T0 ∂V T0

Ans: (c)
⎛ ∂P ⎞
Solution: Isothermal compressibility κ = −V ⎜ ⎟
⎝ ∂V ⎠T
Q12. For an ideal Fermi gas in three dimensions, the electron velocity VF at the Fermi surface
is related to electron concentration n as,
(a) VF ∝ n 2 / 3 (b) VF ∝ n (c) VF ∝ n1 / 2 (d) V F ∝ n1 / 3
Ans: (d)
1
Solution: E F = mV F2 ∵ E F ∝ n 2 / 3 ⇒ VF2 ∝ n 2 / 3 ⇒ VF ∝ n1 / 3 .
2

                                                                                
Head office  Branch office 
 
fiziks, H.No. 40 D, G.F, Jia Sarai,  Anand Institute of Mathematics, 
 
Near IIT, Hauz Khas, New Delhi‐16  28‐B/6, Jia Sarai, Near IIT 
 
Phone: 011‐26865455/+91‐9871145498 Hauz Khas, New Delhi‐16 
                                                   
                                             Website: www.physicsbyfiziks.com                                                                                          
                                                            Email: fiziks.physics@gmail.com                                                                     376 
fiziks
Institute for NET/JRF, GATE, IIT‐JAM, JEST, TIFR and GRE in PHYSICAL SCIENCES 
 
Q13. A classical gas of molecules, each of mass m, is in thermal equilibrium at the
absolute temperature T. The velocity components of the molecules along the Cartesian

axes are v x , v y and v z . The mean value of (v x + v y ) is


2

k BT 3 k BT 1 k BT 2k B T
(a) (b) (c) (d)
m 2 m 2 m m
Ans: (d)

Solution: (Vx + Vy )
2k B T
= V x2 + V y2 + 2 V x ⋅ V y = Vx2 + Vy2 + 2 Vx 〉 ⋅ 〈 Vy =
2

m
2k B T
∵ Vx 〉 = 〈 Vy = 0 and Vx2 + V y2 = .
m
Q14. The total energy, E of an ideal non-relativistic Fermi gas in three dimensions is given by
N 5/3
E∝ where N is the number of particles and V is the volume of the gas. Identify the
V 2/3
CORRECT equation of state (P being the pressure),
1 2 5
(a) PV = E (b) PV = E (c) PV = E (d) PV = E
3 3 3
Ans: (b)
5 5
⎛ ∂E ⎞ 2 ⎛ N ⎞3 2N 3 2
Solution: P = − ⎜ ⎟ = ⎜ ⎟ ⇒ PV = = E.
⎝ ∂V ⎠ N 3 ⎝ V ⎠ 3 V23 3
Q15. Consider a system whose three energy levels are given by 0, ε and 2ε. The energy level ε
is two-fold degenerate and the other two are non-degenerate. The partition function of the
1
system with β = is given by
k BT

(a) 1 + 2e − βε (b) 2e − βε + e −2 βε (c) (1 + e − βε ) 2 (d) 1 + e − βε + e −2 βε


Ans: (b)
Solution: E1 = 0, E 2 = ε , E 3 = 2ε ; g1 = 1, g 2 = 2, g 3 = 1 where g1 , g 2 and g 3 are degeneracy.

The partition function Z = g1e − β ⋅E1 + g 2 e − β ⋅E2 + g 3 e − β ⋅E3 = 1 + 2e − βε + e − β 2ε = 1 + e − βε ( )2

                                                                                
Head office  Branch office 
 
fiziks, H.No. 40 D, G.F, Jia Sarai,  Anand Institute of Mathematics, 
 
Near IIT, Hauz Khas, New Delhi‐16  28‐B/6, Jia Sarai, Near IIT 
 
Phone: 011‐26865455/+91‐9871145498 Hauz Khas, New Delhi‐16 
                                                   
                                             Website: www.physicsbyfiziks.com                                                                                          
                                                            Email: fiziks.physics@gmail.com                                                                     377 
fiziks
Institute for NET/JRF, GATE, IIT‐JAM, JEST, TIFR and GRE in PHYSICAL SCIENCES 
 
GATE 2013

Q16. If Planck’s constant were zero, then the total energy contained in a box filled with
radiation of all frequencies at temperature T would be ( k is the Boltzmann constant and
T is nonzero)
3
(a) zero (b) Infinite (c) kT (d) kT
2
Ans: (d)
Solution: If Planck’s constant were zero, then the system behaved as a classical system and the
energy is kT .
Q17. Across a first order phase transition, the free energy is
(a) proportional to the temperature
(b) a discontinuous function of the temperature
(c) a continuous function of the temperature but its first derivative is discontinuous
(d) such that the first derivative with respect to temperature is continuous
Ans: (c)
Q18. Two gases separated by an impermeable but movable partition are allowed to freely
exchange energy. At equilibrium, the two sides will have the same
(a) pressure and temperature (b) volume and temperature
(c) pressure and volume (d) volume and energy
Ans: (a)
Q19. The entropy function of a system is given by S (E ) = aE(E0 − E ) where a and E0 are
positive constants. The temperature of the system is
(a) negative for some energies (b) increases monotonically with energy
(c) decreases monotonically with energy (d) Zero
Ans: (a)
Solution: From first and second law of thermodynamics
1 ⎛ ∂S ⎞ 1
TdS = dU − PdV ⇒ dS = ( dU − PdV ) ⇒ ⎜ ⎟ = ∵ E = U
T ⎝ ∂E ⎠V T

                                                                                
Head office  Branch office 
 
fiziks, H.No. 40 D, G.F, Jia Sarai,  Anand Institute of Mathematics, 
 
Near IIT, Hauz Khas, New Delhi‐16  28‐B/6, Jia Sarai, Near IIT 
 
Phone: 011‐26865455/+91‐9871145498 Hauz Khas, New Delhi‐16 
                                                   
                                             Website: www.physicsbyfiziks.com                                                                                          
                                                            Email: fiziks.physics@gmail.com                                                                     378 
fiziks
Institute for NET/JRF, GATE, IIT‐JAM, JEST, TIFR and GRE in PHYSICAL SCIENCES 
 
⎛ ∂S ⎞
S (E ) = aE (E0 − E ) ⇒ ⎜
1
⎟ = α ( E0 − E ) − α E = α ( E0 − 2 E ) ⇒ T = .
⎝ ∂E ⎠V α ( E0 − 2 E )
Q20. Consider a linear collection of N independent spin ½ particles, each at a fixed location.
The entropy of this system is ( k is the Boltzmann constant)

(d) Nk ln (2 )
1
(a) zero (b) Nk (c) Nk
2
Ans: (d)
Solution: There are two microstates possible for one so entropy is given by Nk ln (2 )
Q21. Consider a gas of atoms obeying Maxwell-Boltzmann statistics. The average value of
e a⋅ p over all the moments p of each of the particles (where a is a constant vector and a
is the magnitude, m is the mass of each atom, T is temperature and k is Boltzmann’s
constant) is,
1 3
− a 2 mkT − a 2 mkT
(a) one (b) zero (c) e 2
(d) e 2

Ans: (c)
∞ ∞ ∞
Solution: 〈 e p.a 〉 = ∫ ∫ ∫ f ( p , p , p )e dpx dp y dpz where f ( px , p y , pz ) is Maxwell probability
p .a
x y z
−∞ −∞ −∞

distribution at temperature T.
∞ p x2 ∞ p 2y ∞ p z2
− − −
∫ Ae ∫ Ae ∫ Ae
py ay
〈e 〉 =
p.a
x
2 mkT
e px ax
dpx y
2 mkT
e dp y z
2 mkT
e pz az dpz
−∞ −∞ −∞

− ( a x2 + a 2y + a z2 ) mkT ∞ ( px − mkTax )2 ∞ ( p y − mkTa y )2 ∞ ( pz − mkTa z )2


− − −
〈e 〉 = e ∫ Ae dx ∫ Ay e dy ∫ Ax e
p.a 2 2 mkT 2 mkT 2 mkT
x
−∞ −∞ −∞

− ( ax2 + a 2y + a z2 ) mkT 1
− a 2 mkT
〈e p .a
〉=e 2
.1.1.1 = e 2

Common Data for Questions 22 and 23: There are four energy levels E , 2E , 3 E and
4 E (where E > 0 ). The canonical partition function of two particles is, if these particles
are
Q22. Two identical fermions

                                                                                
Head office  Branch office 
 
fiziks, H.No. 40 D, G.F, Jia Sarai,  Anand Institute of Mathematics, 
 
Near IIT, Hauz Khas, New Delhi‐16  28‐B/6, Jia Sarai, Near IIT 
 
Phone: 011‐26865455/+91‐9871145498 Hauz Khas, New Delhi‐16 
                                                   
                                             Website: www.physicsbyfiziks.com                                                                                          
                                                            Email: fiziks.physics@gmail.com                                                                     379 
fiziks
Institute for NET/JRF, GATE, IIT‐JAM, JEST, TIFR and GRE in PHYSICAL SCIENCES 
 
−2 βE −4 βE −6 βE −8 βE
(a) e +e +e +e
(b) e −3 βE + e −4 βE + e −5 βE + e −6 βE + e −7 βE

(c) (e − βE + e − 2 βE + e −3 βE + e − 4 βE )
2

(d) e −2 βE − e −4 βE + e −6 βE − e −8 βE
Ans: (b)
Solution: The possible value of Energy for two Fermions
E1 = 3E, E2 = 4 E , E3 = 5E, E4 = 6 E , E5 = 7 E

The partition function is Z = e −3βE + e −4 βE + 2e −5 βE + e −6 βE + e −7 βE


Q23. Two distinguishable particles
(a) e −2 βE + e −4 βE + e −6 βE + e −8 βE
(b) e −3 βE + e −4 βE + e −5 βE + e −6 βE + e −7 βE

(c) (e − βE + e − 2 βE + e −3 βE + e − 4 βE )
2

(d) e −2 βE − e −4 βE + e −6 βE − e −8 βE
Ans: (c)
Solution: When two particles are distinguishable then minimum value of Energy is 2 E and
maximum value is 8 E .

So from checking all four options (Z = e − βE + e −2 βE + e −3 βE + e −4 βE )


2

GATE 2014

Q24. For a gas under isothermal condition its pressure p varies with volume V as P ∝ V −5 / 3 .
The bulk modules B is proportional to
(a) V −1 / 2 (b) V −2 / 3 (c) V −3 / 5 (d) V −5 / 3
Ans: (d)
dP
Solution: P = KV −5 / 3 , B = −V B ∝ V −5 / 3
dV
Q25. At a given temperature T , the average energy per particle of a non-interacting gas of
two-dimensional classical harmonic oscillators is _________ k B T

                                                                                
Head office  Branch office 
 
fiziks, H.No. 40 D, G.F, Jia Sarai,  Anand Institute of Mathematics, 
 
Near IIT, Hauz Khas, New Delhi‐16  28‐B/6, Jia Sarai, Near IIT 
 
Phone: 011‐26865455/+91‐9871145498 Hauz Khas, New Delhi‐16 
                                                   
                                             Website: www.physicsbyfiziks.com                                                                                          
                                                            Email: fiziks.physics@gmail.com                                                                     380 
fiziks
Institute for NET/JRF, GATE, IIT‐JAM, JEST, TIFR and GRE in PHYSICAL SCIENCES 
 
( k B is the Boltzmann constant)

Ans: 2k B T
Q26. Which one of the following is a fermion?
(a) α particle (b) 4 Be 2 nucleus
(c) Hydrogen atom (d) deuteron
Ans (d)
Solution: Total number of particles: P + N + E = 3
Q27. For a free electron gas in two dimensions the variations of the density of states. N (E ) as a
function of energy E , is best represented by
(a) (b)

N (E ) N (E )

E E

(c) (d)

N (E ) N (E )

E E

ans (c)
N (E ) ∝ E 0
Q28. For a system of two bosons each of which can occupy any of the two energy levels 0 and
1
ε the mean energy of the system at temperature T with β = is given by
kβ T

                                                                                
Head office  Branch office 
 
fiziks, H.No. 40 D, G.F, Jia Sarai,  Anand Institute of Mathematics, 
 
Near IIT, Hauz Khas, New Delhi‐16  28‐B/6, Jia Sarai, Near IIT 
 
Phone: 011‐26865455/+91‐9871145498 Hauz Khas, New Delhi‐16 
                                                   
                                             Website: www.physicsbyfiziks.com                                                                                          
                                                            Email: fiziks.physics@gmail.com                                                                     381 
fiziks
Institute for NET/JRF, GATE, IIT‐JAM, JEST, TIFR and GRE in PHYSICAL SCIENCES 
 
−β ε −2 β ε
εe + 2ε e 1 + ε e −β ε
(a) (b)
1 + 2e − β ε + e − 2 β ε 2e − β ε + e − 2 β ε
2ε e − βε + ε e −2 β ε ε e − β ε + 2ε e −2 β ε
(c) (d)
2 + e −β ε + e −2 β ε 2 + e − βε + e − 2 β ε
if both particle will in ground state the energy will 0 which is non degenerate
if one particle is in ground state and other is in first excited state then energy is ε and
non degenerate
if both particle will in first excited state the energy will 2ε which is non degenerate
then partition function is Z = 1 + exp− βε + exp− 2βε
exp − βε + 2ε exp − 2 βε
average value of energy
1 + exp − βε + exp − 2 βε
no one ans. is correct .
Q29. Consider a system of 3 fermions which can occupy any of the 4 available energy states
with equal probability. The entropy of the system is
(a) k B ln 2 (b) 2k B ln 2 (c) 2k B ln 4 (d) 3k B ln 4
Ans: (b)
Solution: Number of ways that 3 fermions will adjust in 4 available energy is 4
C3 = 4 so

entropy is k B ln 4 = 2k B ln 2
GATE 2015
Q30. In Boss-Einstein condensates, the particles
(a) have strong interparticle attraction
(b) condense in real space
(c) have overlapping wavefunctions
(d) have large and positive chemical potential
Ans.: (c)
Solution: In Bose- Einstein condensates, the particles have overlapping wave function.

                                                                                
Head office  Branch office 
 
fiziks, H.No. 40 D, G.F, Jia Sarai,  Anand Institute of Mathematics, 
 
Near IIT, Hauz Khas, New Delhi‐16  28‐B/6, Jia Sarai, Near IIT 
 
Phone: 011‐26865455/+91‐9871145498 Hauz Khas, New Delhi‐16 
                                                   
                                             Website: www.physicsbyfiziks.com                                                                                          
                                                            Email: fiziks.physics@gmail.com                                                                     382 
fiziks
Institute for NET/JRF, GATE, IIT‐JAM, JEST, TIFR and GRE in PHYSICAL SCIENCES 
 
Q31. For a black body radiation in a cavity, photons are created and annihilated freely as a
result of emission and absorption by the walls of the cavity. This is because
(a) the chemical potential of the photons is zero
(b) photons obey Pauli exclusion principle
(c) photons are spin-1 particles
(d) the entropy of the photons is very large
Ans.: (a)
Solution: The chemical potential of photon is zero
1
Q32. Consider a system of N non-interacting spin − particles, each having a magnetic
2
moment μ , is in a magnetic field B = Bzˆ . If E is the total energy of the system, the
number of accessible microstates Ω is given by
⎛ E ⎞
⎜⎜ N − ⎟!
N! μB ⎟⎠
(a) Ω = (b) Ω = ⎝
1⎛ E ⎞ 1⎛ E ⎞ ⎛ E ⎞
⎜⎜ N − ⎟⎟ ! ⎜⎜ N + ⎟! ⎜⎜ N + ⎟!
2⎝ μB ⎠ 2 ⎝ μB ⎟⎠ ⎝ μB ⎟⎠

1⎛ E ⎞ 1⎛ E ⎞ N!
(c) Ω = ⎜⎜ N − ⎟⎟! ⎜⎜ N + ⎟! (d) Ω =
2⎝ μB ⎠ 2 ⎝ μB ⎟⎠ ⎛ E ⎞
⎜⎜ N + ⎟!
⎝ μB ⎟⎠
Ans.: (a)
1
Solution: Number of microstate is N
Cn1 where n1 is number of particle in + state and
2
n2 = N − n1 is

1
Number of state in − state.
2
1⎛ E ⎞ 1⎛ E ⎞
n1 = ⎜ N − ⎟ , n2 = ⎜ N +
2⎝ μB ⎠ 2⎝ μ B ⎟⎠
N
So number of microstate is
1⎛ E ⎞ 1⎛ E ⎞
⎜ N− ⎟ ⎜ N+
2⎝ μB ⎠ 2 ⎝ μ B ⎟⎠

                                                                                
Head office  Branch office 
 
fiziks, H.No. 40 D, G.F, Jia Sarai,  Anand Institute of Mathematics, 
 
Near IIT, Hauz Khas, New Delhi‐16  28‐B/6, Jia Sarai, Near IIT 
 
Phone: 011‐26865455/+91‐9871145498 Hauz Khas, New Delhi‐16 
                                                   
                                             Website: www.physicsbyfiziks.com                                                                                          
                                                            Email: fiziks.physics@gmail.com                                                                     383 
fiziks
Institute for NET/JRF, GATE, IIT‐JAM, JEST, TIFR and GRE in PHYSICAL SCIENCES 
 
Q33. The average energy U of a one dimensional quantum oscillator of frequency ω and in
contact with a heat bath at temperature T is given by
1 ⎛1 ⎞ 1 ⎛1 ⎞
(a) U = ω coth⎜ β ω ⎟ (b) U = ω sinh ⎜ β ω ⎟
2 ⎝2 ⎠ 2 ⎝2 ⎠
1 ⎛1 ⎞ 1 ⎛1 ⎞
(c) U = ω tanh⎜ β ω ⎟ (d) U = ω cosh⎜ β ω ⎟
2 ⎝2 ⎠ 2 ⎝2 ⎠
Ans.: (a)
∞ ⎛ 1⎞
β Ei − β ⎜ n+ ⎟ ω
⎛ 1⎞ 1
Solution: ∵ Z = ∑ e = ∑e ⎝ 2⎠
where E = ⎜ n + ⎟ ω ⇒ Z =
0 ⎝ 2⎠ ⎛β ω⎞
2sinh ⎜ ⎟
⎝ 2 ⎠
⎡ ⎤
−∂ ∂ ⎢ 1 ⎥ ω ⎛β ω⎞
∵U = ln Z ⇒ U = − ln ⎢ ⎥ ⇒U = coth ⎜ ⎟
∂β ∂β ⎢ ⎛ β ω ⎞⎥ 2 ⎝ 2 ⎠
2sinh ⎜ ⎟
⎢⎣ ⎝ 2 ⎠ ⎥⎦
Q34. The entropy of a gas containing N particles enclosed in a volume V is given by
⎛ aVE 3 / 2 ⎞
S = Nk B ln⎜⎜ ⎟⎟ , where E is the total energy, a is a constant and k B is the
⎝ N
5/ 2

Boltzmann constant. The chemical potential μ of the system at a temperature T is given
by
⎡ ⎛ aVE 3 / 2 ⎞ 5⎤ ⎡ ⎛ aVE 3 / 2 ⎞ 3⎤
(a) μ = − k B T ⎢ln⎜⎜ ⎟⎟ − ⎥ (b) μ = − k B T ⎢ln⎜⎜ ⎟⎟ − ⎥
⎣ ⎝ N ⎣ ⎝ N
5/ 2 5/ 2
⎠ 2⎦ ⎠ 2⎦

⎡ ⎛ aVE 3/ 2 ⎞ 5 ⎤ ⎡ ⎛ aVE 3 / 2 ⎞ 3⎤
(c) μ = −k BT ⎢ln ⎜ 3/ 2 ⎟
− ⎥ (d) μ = − k B T ⎢ln⎜⎜ ⎟⎟ − ⎥
⎣ ⎝ N ⎣ ⎝ N
3/ 2
⎠ 2⎦ ⎠ 2⎦
Ans.: (a)
⎛ 3
⎞ ⎛ 3

⎛ ∂G ⎞ ⎜ aVE 2
⎟ ∵ S = Nk ln ⎜ aVE 2

Solution: ⎜ ⎟ = − S = − Nk ln
⎝ ∂T ⎠ P
B ⎜ 5 ⎟ B ⎜ 5 ⎟
⎜ N2 ⎟ ⎜ N2 ⎟
⎝ ⎠ ⎝ ⎠

                                                                                
Head office  Branch office 
 
fiziks, H.No. 40 D, G.F, Jia Sarai,  Anand Institute of Mathematics, 
 
Near IIT, Hauz Khas, New Delhi‐16  28‐B/6, Jia Sarai, Near IIT 
 
Phone: 011‐26865455/+91‐9871145498 Hauz Khas, New Delhi‐16 
                                                   
                                             Website: www.physicsbyfiziks.com                                                                                          
                                                            Email: fiziks.physics@gmail.com                                                                     384 
fiziks
Institute for NET/JRF, GATE, IIT‐JAM, JEST, TIFR and GRE in PHYSICAL SCIENCES 
 
⎛ 3

⎜ aVE ⎟
2

⇒ G = − Nk BT ln ⎝ 5
⎠ + ln A
N2
⎡ ⎛ 3
⎞ ⎛ 5⎞ ⎤
⎢ ⎜ aVE 2
⎟ 5
− 3⎥
⎛ ∂G ⎞ ⎢ ⎝ ⎠ N 2 ⎝⎜ 2 ⎠⎟
⇒μ =⎜ ⎟ = − ⎢ k BT ln + Nk BT . aVE 2 ⎥
⎝ ∂N ⎠
5 3 7 ⎥
⎢ N2 aVE 2 N 2 ⎥
⎢⎣ ⎥⎦

⎡ ⎛ 3
⎞ ⎤
aVE
⇒ μ = − k BT ⎢⎢ln ⎜ ⎟ − 5⎥
2

⎜ 5 ⎟ 2⎥
⎜ ⎟
⎣⎢ ⎝ N ⎠ ⎦⎥
2

GATE-2016
Q35. The total power emitted by a spherical black body of radius R at a temperature T is P1 .
R
Let P2 be the total power emitted by another spherical black body of radius kept at
2
P1
temperature 2T . The ratio, is _______. (Give your answer upto two decimal places)
P2
Ans.: 0.25

p1 R12T14 R 2T 4 4 1
Solution: p ∝ AT ⇒ = 2 4= 4
= = = 0.25
p2 R2 T2 ⎛ R ⎞ 2
16 4
⎜ ⎟ ( 2T )
4

⎝2⎠
Q36. The entropy S of a system of N spins, which may align either in the upward or in the
downward direction, is given by S = − k B N ⎡⎣ p ln p + (1 − p ) In(1 − p) ⎤⎦ . Here k B is the

Boltzmann constant. The probability of alignment in the upward direction is p. The value
of p, at which the entropy is maximum, is _______. (Give your answer upto one decimal
place)
Ans.: 0.5
Solution: S = − k B N ⎡⎣ p ln p + (1 − p ) In(1 − p) ⎤⎦

                                                                                
Head office  Branch office 
 
fiziks, H.No. 40 D, G.F, Jia Sarai,  Anand Institute of Mathematics, 
 
Near IIT, Hauz Khas, New Delhi‐16  28‐B/6, Jia Sarai, Near IIT 
 
Phone: 011‐26865455/+91‐9871145498 Hauz Khas, New Delhi‐16 
                                                   
                                             Website: www.physicsbyfiziks.com                                                                                          
                                                            Email: fiziks.physics@gmail.com                                                                     385 
fiziks
Institute for NET/JRF, GATE, IIT‐JAM, JEST, TIFR and GRE in PHYSICAL SCIENCES 
 
dS 1 1
For maximum entropy = 0 ⇒ ln p + p × − ln (1 − p ) + (1 − p ) × ( −1) = 0
dp p 1− p

⎛ p ⎞
ln p + 1 − ln (1 − p ) − 1 = 0 ⇒ ln ⎜ ⎟ = 0 ⇒ p = 1 − p ⇒ p = 0.5
⎝ 1− p ⎠
Q37. For a system at constant temperature and volume, which of the following statements is
correct at equilibrium?
(a) The Helmholtz free energy attains a local minimum.
(b) The Helmholtz free energy attains a local maximum.
(c) The Gibbs free energy attains a local minimum.
(d) The Gibbs free energy attains a local maximum.
Ans.: (a)
Solution: dF = − SdT − PdV
Q38. N atoms of an ideal gas are enclosed in a container of volume V . The volume of the
container is changed to 4V , while keeping the total energy constant. The change in the
entropy of the gas, in units of Nk B ln 2 , is _______, where k B is the Boltzmann constant.
Ans.: 2
1
Solution: S1 = − Nk B ln1 S2 = − Nk B ln
4
ΔS = S2 − S1 = Nk B ln 4 = 2 Nk B ln 2

Q39. Consider a system having three energy levels with energies 0, 2ε and 3ε ,with
respective degeneracies of 2,2 and 3 . Four bosons of spin zero have to be
accommodated in these levels such that the total energy of the system is 10 ε . The
number of ways in which it can be done is ______.
Ans.: 18
Solution: The system have energy 10 ε if out of four boson two boson are in energy level
2ε and two boson are in energy level 3ε and
ni + gi − 1
W =∏ n1 = 2, g1 = 2 and n2 = 2, g 2 = 3
i ni gi − 1

                                                                                
Head office  Branch office 
 
fiziks, H.No. 40 D, G.F, Jia Sarai,  Anand Institute of Mathematics, 
 
Near IIT, Hauz Khas, New Delhi‐16  28‐B/6, Jia Sarai, Near IIT 
 
Phone: 011‐26865455/+91‐9871145498 Hauz Khas, New Delhi‐16 
                                                   
                                             Website: www.physicsbyfiziks.com                                                                                          
                                                            Email: fiziks.physics@gmail.com                                                                     386 
fiziks
Institute for NET/JRF, GATE, IIT‐JAM, JEST, TIFR and GRE in PHYSICAL SCIENCES 
 
2 + 2 −1 2 + 3 −1
× = 3 × 6 = 18
2 2 −1 2 3 −1
Q40. A two-level system has energies zero and E . The level with zero energy is non-
degenerate, while the level with energy E is triply degenerate. The mean energy of a
classical particle in this system at a temperature T is
−E −E −E −E
k BT k BT k BT k BT
Ee Ee 3Ee 3Ee
(a) −E
(b) −E
(c) −E
(d) −E

1 + 3e k BT
1+ e k BT
1+ e k BT
1 + 3e k BT
Ans.: (d)
Ei
− 0 E −E
∑ gi Ei e kT − −
k BT
0×e kT + 3× E × e kT 3Ee
Solution: E = i
E
= 0 E
= −E
− i − −
∑ gi e kT e kT + 3 × e kT 1 + 3e k BT
i

                                                                                
Head office  Branch office 
 
fiziks, H.No. 40 D, G.F, Jia Sarai,  Anand Institute of Mathematics, 
 
Near IIT, Hauz Khas, New Delhi‐16  28‐B/6, Jia Sarai, Near IIT 
 
Phone: 011‐26865455/+91‐9871145498 Hauz Khas, New Delhi‐16 
                                                   
                                             Website: www.physicsbyfiziks.com                                                                                          
                                                            Email: fiziks.physics@gmail.com                                                                     387 
fiziks
Institute for NET/JRF, GATE, IIT‐JAM, JEST, TIFR and GRE in PHYSICAL SCIENCES 
 
ELECTRONICS SOLUTIONS

GATE-2010
Q1. The voltage resolution of a 12-bit digital to analog converter (DAC), whose output varies
from − 10 V to + 10 V is, approximately
(a) 1 mV (b) 5 mV (c) 20 mV (d) 100 mV
Ans: (b)
20V
Solution: Voltage resolution= = 4.8 mV
212 − 1
Q2. The figure shows a constant current source charging a capacitor that is initially
uncharged. Vout

If the switch is closed at t = 0, which of the following plots depicts correctly the output
voltage of the circuit as a function of time?
(a) (b)

Vout Vout

t t
(c) (d)

Vout Vout

t t

Ans: (d)
CdV0 I
Solution: I 0 = ⇒ V0 = 0 t
dt C

                                                                                
Head office  Branch office 
 
fiziks, H.No. 40 D, G.F, Jia Sarai,  Anand Institute of Mathematics, 
 
Near IIT, Hauz Khas, New Delhi‐16  28‐B/6, Jia Sarai, Near IIT 
 
Phone: 011‐26865455/+91‐9871145498 Hauz Khas, New Delhi‐16 
                                                   
                                             Website: www.physicsbyfiziks.com                                                                                          
                                                            Email: fiziks.physics@gmail.com                                                                     388 
fiziks
Institute for NET/JRF, GATE, IIT‐JAM, JEST, TIFR and GRE in PHYSICAL SCIENCES 
 
Q3. In one of the following circuits, negative feedback does not operate for a negative input.
Which one is it? The opamps are running from ± 15 V supplies.
(a) (b)
− −

+ +

(c) (d)
− −

+ +

Ans: (c)
Q4. For any set of inputs, A and B, the following circuits give the same output, Q, except one.
Which one is it?
(a) Α (b) Α
Q
Β Q Β

(c) Α (d) Α
Q
Β Q Β

Ans: (d)

                                                                                
Head office  Branch office 
 
fiziks, H.No. 40 D, G.F, Jia Sarai,  Anand Institute of Mathematics, 
 
Near IIT, Hauz Khas, New Delhi‐16  28‐B/6, Jia Sarai, Near IIT 
 
Phone: 011‐26865455/+91‐9871145498 Hauz Khas, New Delhi‐16 
                                                   
                                             Website: www.physicsbyfiziks.com                                                                                          
                                                            Email: fiziks.physics@gmail.com                                                                     389 
fiziks
Institute for NET/JRF, GATE, IIT‐JAM, JEST, TIFR and GRE in PHYSICAL SCIENCES 
 
GATE-2011

Q5. Which of the following statements is CORRECT for a common emitter amplifier
circuit?
(a) The output is taken from the emitter
(b) There is 180o phase shift between input and output voltages
(c) There is no phase shift between input and output voltages
(d) Both p-n junctions are forward biased
Ans: (b)
Q6. For an intrinsic semiconductor, me* and mh* are respectively the effective masses of
electrons and holes near the corresponding band edges. At a finite temperature the
position of the Fermi level
(a) depends on me* but not on mh* (b) depends on mh* but not on me*
(c) depends on both me* and mh* (d) depends neither on me* nor on mh*
Ans: (c)
Q7. In the following circuit, the voltage across and the current through the 2 kΩ resistance are

500Ω 1kΩ

20V 10V
2kΩ
30V

(a) 20 V, 10 mA (b) 20 V, 5 mA (c) 10 V, 10 mA (d) 10 V, 5 mA


Ans: (d)

                                                                                
Head office  Branch office 
 
fiziks, H.No. 40 D, G.F, Jia Sarai,  Anand Institute of Mathematics, 
 
Near IIT, Hauz Khas, New Delhi‐16  28‐B/6, Jia Sarai, Near IIT 
 
Phone: 011‐26865455/+91‐9871145498 Hauz Khas, New Delhi‐16 
                                                   
                                             Website: www.physicsbyfiziks.com                                                                                          
                                                            Email: fiziks.physics@gmail.com                                                                     390 
fiziks
Institute for NET/JRF, GATE, IIT‐JAM, JEST, TIFR and GRE in PHYSICAL SCIENCES 
 
Q8. In the following circuit, Tr1 and Tr2 are identical transistors having VBE = 0.7 V. The
current passing through the transistor Tr2 is

100Ω
Tr2
+5V

Tr1

(a) 57 mA (b) 50 mA (c) 48 mA (d) 43 mA


Ans: (d)
5 − 0.7
Solution: Current through 100 Ω, I = = 43 mA
100
I = I C + 2 I B ≈ I C = 43 mA .
Q9. The following Boolean expression
Y = A ⋅ B ⋅ C ⋅ D + A ⋅ B ⋅ C ⋅ D + A ⋅ B ⋅ C ⋅ D + A ⋅ B ⋅ C ⋅ D + A ⋅ B ⋅ C ⋅ D + A ⋅ B ⋅ C ⋅ D can
be simplified to
(a) A • B • C + A • D (b) A • B • C + A • D
(c) A • B • C + A • D (d) A • B • C + A • D
Ans: (c)
CD CD CD CD
AD
AB 1 1
AB 1 1
AB
AB 1 1

AB C

                                                                                
Head office  Branch office 
 
fiziks, H.No. 40 D, G.F, Jia Sarai,  Anand Institute of Mathematics, 
 
Near IIT, Hauz Khas, New Delhi‐16  28‐B/6, Jia Sarai, Near IIT 
 
Phone: 011‐26865455/+91‐9871145498 Hauz Khas, New Delhi‐16 
                                                   
                                             Website: www.physicsbyfiziks.com                                                                                          
                                                            Email: fiziks.physics@gmail.com                                                                     391 
fiziks
Institute for NET/JRF, GATE, IIT‐JAM, JEST, TIFR and GRE in PHYSICAL SCIENCES 
 
Q10. Consider the following circuit 1kΩ 4kΩ
+ 10V
+
− Vout
Vin
− 10V
Which of the following correctly represents the output Vout corresponding to the input
Vin?
+ 5V + 5V
(a) (b)
+ 2V + 2V
Vin Vin
− 2V time − 2V time
− 5V − 5V

+ 10V
+ 10V Vout
Vout time
- 10V
time
- 10V

(c) + 5V (d)
+ 5V
+ 2V
Vin + 2V
− 2V time Vin
− 2V time
− 5V
− 5V

+ 10V
+ 10V
Vout
Vout
time
time - 10V
- 10V

Ans: (a)
⎛ 1 ⎞ ⎛ 1 ⎞
Solution: Vut = ⎜ ⎟ × 10 = +2V , Vlt = ⎜ ⎟ × −10 = −2V .
⎝1+ 4 ⎠ ⎝1+ 4 ⎠
                                                                                
Head office  Branch office 
 
fiziks, H.No. 40 D, G.F, Jia Sarai,  Anand Institute of Mathematics, 
 
Near IIT, Hauz Khas, New Delhi‐16  28‐B/6, Jia Sarai, Near IIT 
 
Phone: 011‐26865455/+91‐9871145498 Hauz Khas, New Delhi‐16 
                                                   
                                             Website: www.physicsbyfiziks.com                                                                                          
                                                            Email: fiziks.physics@gmail.com                                                                     392 
fiziks
Institute for NET/JRF, GATE, IIT‐JAM, JEST, TIFR and GRE in PHYSICAL SCIENCES 
 
GATE-2012
Q11. If the peak output voltage of a full wave rectifier is 10 V, its d.c. voltage is
(a) 10.0 V (b) 7.07 V (c) 6.36 V (d) 3.18 V
Ans: (c)
2Vm 2 × 10 14 × 10 70
Solution: Vdc = = = = = 6.36V
π 22 / 7 22 11
Q12. A Ge semiconductor is doped with acceptor impurity concentration of 1015 atoms/cm3.
For the given hole mobility of 1800 cm2/V-s, the resistivity of the material is
(a) 0.288 Ω cm (b) 0.694 Ω cm (c) 3.472 Ω cm (d) 6.944 Ω cm
Ans: (c)
1 1 1
Solution: ρ = = = 15 = 3.47Ω cm
σ N A e u h 10 × 1.6 × 10 −19 × 1800
Q13. Identify the CORRECT energy band diagram for silcon doped with Arsenic. Here CB,
VB, ED and EF are conduction band, valence band, impurity level and Fermi level,
respectively.
(a) (b)
CB CB
ED
ED
EF
EF

VB VB

(c) (d)
CB CB

EF EF
ED ED
VB VB

Ans: (b)
Solution: N-type material ( Si doped with AS ).
                                                                                
Head office  Branch office 
 
fiziks, H.No. 40 D, G.F, Jia Sarai,  Anand Institute of Mathematics, 
 
Near IIT, Hauz Khas, New Delhi‐16  28‐B/6, Jia Sarai, Near IIT 
 
Phone: 011‐26865455/+91‐9871145498 Hauz Khas, New Delhi‐16 
                                                   
                                             Website: www.physicsbyfiziks.com                                                                                          
                                                            Email: fiziks.physics@gmail.com                                                                     393 
fiziks
Institute for NET/JRF, GATE, IIT‐JAM, JEST, TIFR and GRE in PHYSICAL SCIENCES 
 
+ 10V
Q14. Consider the following OP-AMP circuit
Which one of the following correctly represents the Vin +

output Vout corresponding to the input Vin? +5V Vout



4kΩ
- 10V
1kΩ

(a) (b)
+ 5V + 5V
Vin Vin
+ 1V + 1V
0V t→ 0V t→

+ 10V + 10V
Vout Vout

t→ t→

− 10V − 10V
(c) (d)
+ 5V + 5V
Vin Vin

0V t→ 0V t→

+ 10V + 10V
Vout Vout

t→ t→

− 10V − 10V

Ans: (a)
⎛ 1 ⎞
Solution: Voltage at inverting input V2 = ⎜ ⎟ × 5 = +1V .
⎝1+ 4 ⎠
When vin > +1V , v0 = +VCC and when vin < +1V , v0 = −VCC
                                                                                
Head office  Branch office 
 
fiziks, H.No. 40 D, G.F, Jia Sarai,  Anand Institute of Mathematics, 
 
Near IIT, Hauz Khas, New Delhi‐16  28‐B/6, Jia Sarai, Near IIT 
 
Phone: 011‐26865455/+91‐9871145498 Hauz Khas, New Delhi‐16 
                                                   
                                             Website: www.physicsbyfiziks.com                                                                                          
                                                            Email: fiziks.physics@gmail.com                                                                     394 
fiziks
Institute for NET/JRF, GATE, IIT‐JAM, JEST, TIFR and GRE in PHYSICAL SCIENCES 
 
Q15. In the following circuit, for the output voltage to be V0 = (− V1 + V2 / 2 ) the ratio R1/R2 is
(a) 1/2 R

(b) 1 + VCC
R
(c) 2 V1 −

Vo
(d) 3 +
V2
R1
- VCC
Ans: (d) R2

Solution: When V2 = 0, v01 = −V1

⎛ R ⎞ ⎛ R2 ⎞
when V1 = 0, v02 = ⎜1 + ⎟ ⎜⎜ ⎟⎟ V2
⎝ R ⎠ ⎝ R1 + R2 ⎠
V2 R2 1 R
Since V0 = −V1 + ⇒ 2⋅ = ⇒ 1 =3
2 R1 + R2 2 R2
Q16. In the following circuit, the voltage drop across the ideal diode in forward bias condition
is 0.7V. The current passing through the diode is
(a) 0.5 mA
12kΩ
(b) 1.0 mA
+ 24 Volt

(c) 1.5 mA −

(d) 2.0 mA 6kΩ 3.3 kΩ

Ans: (b)
Solution: Let current through 12kΩ is I and through diode is I D
Then 0 .7 + I D × 3 .3 = ( I − I D ) × 6 (1)
and − 24 + I × 12 + (I − I D ) × 6 = 0 (2)
From (1) and (2) I D ≈ 1mA.

                                                                                
Head office  Branch office 
 
fiziks, H.No. 40 D, G.F, Jia Sarai,  Anand Institute of Mathematics, 
 
Near IIT, Hauz Khas, New Delhi‐16  28‐B/6, Jia Sarai, Near IIT 
 
Phone: 011‐26865455/+91‐9871145498 Hauz Khas, New Delhi‐16 
                                                   
                                             Website: www.physicsbyfiziks.com                                                                                          
                                                            Email: fiziks.physics@gmail.com                                                                     395 
fiziks
Institute for NET/JRF, GATE, IIT‐JAM, JEST, TIFR and GRE in PHYSICAL SCIENCES 
 
Q17. Consider the following circuit in which the current gain βdc of the transistor is 100.
+ 15 V

100 kΩ 900 Ω

100 Ω

Which one of the following correctly represents the load line (collector current IC with
respect to collector-emitter voltage VCE) and Q-point of this circuit?
(a) (b)
15 mA Q - point 13 mA Q - point
(2 V, 13 mA) (2 V, 10 mA)
IC →

IC →

VCE → 15 V VCE → 15 V

(c) (d)
15 mA 13 mA
Q - point Q - point
(7.5 V, 7.5 mA) (7.5 V, 6.5 mA)
IC →

IC →

VCE → 15 V VCE → 15 V

Ans: (a)
VCC − V BE 15 − 0.7 14.3
Solution: I B = = ≈ mA.
RB + RE 100 × 10 + 100 100
3

I C ≈ βI B ≈ 14.3mA ≈ 13mA , VCE = VCC − I C (RC + RE ) = 15 − (900 + 100) × 13 × 10 −3 = 2V .

VCC 15
I C , Sat = = = 15 mA.
R C + R E 1000
                                                                                
Head office  Branch office 
 
fiziks, H.No. 40 D, G.F, Jia Sarai,  Anand Institute of Mathematics, 
 
Near IIT, Hauz Khas, New Delhi‐16  28‐B/6, Jia Sarai, Near IIT 
 
Phone: 011‐26865455/+91‐9871145498 Hauz Khas, New Delhi‐16 
                                                   
                                             Website: www.physicsbyfiziks.com                                                                                          
                                                            Email: fiziks.physics@gmail.com                                                                     396 
fiziks
Institute for NET/JRF, GATE, IIT‐JAM, JEST, TIFR and GRE in PHYSICAL SCIENCES 
 
GATE-2013
Q18. What should be the clock frequency of a 6 − bit A / D converter so that its maximum
conserved time is 32μs ?
(a) 1 MHz (b) 2 MHz (c) 0.5 MHz (d) 4 MHz
Ans: (c)
Q19. A phosphorous doped silicon semiconductor (doping density: 1017/cm3) is heated from
100°C to 200°C. Which one of the following statements is CORRECT?
(a) Position of Fermi level moves towards conduction band
(b) Position of dopant level moves towards conduction band
(c) Position of Fermi level moves towards middle of energy gap
(d) Position of dopant level moves towards middle of energy gap
Ans: (c)

Statement for Linked Answer Questions 20 and 21:


Consider the following circuit
Q20. For this circuit the frequency above which the gain will decrease by 20 dB per decade is
(a) 15.9 kHz (b) 1.2kHz 10 k Ω
V(in ) +
(c) 5.6 kHz (d) 22.5 kHz
V(out )
1000pF −
Ans: (a)
1 1 kΩ
Solution: f H = = 16kHz
2πRC

2 kΩ
Q21. At 1.2 kHz the closed loop gain is
(a) 1 (b) 1.5 (c) 3 (d) 0.5
⎛1 + R F ⎞
v0 ⎜ R1 ⎟⎠

Ans: (b) = = 1 .5
v in 2
1 + ⎛⎜ f ⎞

⎝ fH ⎠

                                                                                
Head office  Branch office 
 
fiziks, H.No. 40 D, G.F, Jia Sarai,  Anand Institute of Mathematics, 
 
Near IIT, Hauz Khas, New Delhi‐16  28‐B/6, Jia Sarai, Near IIT 
 
Phone: 011‐26865455/+91‐9871145498 Hauz Khas, New Delhi‐16 
                                                   
                                             Website: www.physicsbyfiziks.com                                                                                          
                                                            Email: fiziks.physics@gmail.com                                                                     397 
fiziks
Institute for NET/JRF, GATE, IIT‐JAM, JEST, TIFR and GRE in PHYSICAL SCIENCES 
 
GATE-2014
Q22. The input given to an ideal OP-AMP integrator circuit is
V

V0

t
t0
The correct output of the integrator circuit is
(a) V (b) V

V0 V0

t0 t t0 t

(c) V (d) V

V0
V0
t0 t
t0 t
Ans: (a)
Q23. The minimum number of flip-flops required to construct a mod-75 counter is
__________
Ans: 7
Q24. The donor concentration in a sample of n -type silicon is increased by a factor of 100.
The shift in the position of the Fermi level at 300K, assuming the sample to non
degenerate is ________ meV
(k BT = 25meV at 300 K )
Ans: 115.15
⎛N ⎞ ⎛ Nc ⎞ ⎛ Nc ⎞
Solution: EC − EF = kT ln ⎜ c ⎟ and EC − EF′ = kT ln ⎜ ⎟ = kT ln ⎜ ⎟ − kT ln (100 )
⎝ Nd ⎠ ⎝ 100 N d ⎠ ⎝ Nd ⎠
Thus shift is ΔE = kT ln (100 ) = 25ln (100 ) meV = 115.15 meV

                                                                                
Head office  Branch office 
 
fiziks, H.No. 40 D, G.F, Jia Sarai,  Anand Institute of Mathematics, 
 
Near IIT, Hauz Khas, New Delhi‐16  28‐B/6, Jia Sarai, Near IIT 
 
Phone: 011‐26865455/+91‐9871145498 Hauz Khas, New Delhi‐16 
                                                   
                                             Website: www.physicsbyfiziks.com                                                                                          
                                                            Email: fiziks.physics@gmail.com                                                                     398 
fiziks
Institute for NET/JRF, GATE, IIT‐JAM, JEST, TIFR and GRE in PHYSICAL SCIENCES 
 
Q25. The current gain of the transistor in the following circuit is β dc = 100 . The value of

collector current I C is_________ mA 12V

3k Ω 20 μ F
V0
150 k Ω
Vi
20 μ F

3k Ω

Ans: 1.6
VCC − VBE 12 − 0
Solution: I B = = = 0.016 mA ⇒ I C = β I B = 1.6 mA
RB + β ( RC + RE ) 150 + 100 ( 3 + 3)

Q26. In order to measure a maximum of 1V with a resolution of 1mV using a n − bit


A
converter working under the principle of ladder network the minimum value of n is
D
___________
Ans: 10
1
Solution: 1×10−3 = ⇒ n ≈ 10
2 −1 n

Q27. A low pass fliter is formed by a resistance R and a capacitance C . At the cut-off angular
1
frequency ωC = the voltage gain and the phase of the output voltage relative to the
RC
input voltage respectively are
(a) 0.71and 45 o (b) 0.71and − 45 o (c) 0.5 and − 90 o (d) 0.5 and 90 o
Ans: (b)
v0 XC 1 1
Solution: = = =
+ 1 1 + jωCR
vin R + X C R
XC
1 v 1 1 1 − j 450
At ω = ωC = ⇒ 0 == = 0 = e
RC vin 1+ j 2e j 45 2

                                                                                
Head office  Branch office 
 
fiziks, H.No. 40 D, G.F, Jia Sarai,  Anand Institute of Mathematics, 
 
Near IIT, Hauz Khas, New Delhi‐16  28‐B/6, Jia Sarai, Near IIT 
 
Phone: 011‐26865455/+91‐9871145498 Hauz Khas, New Delhi‐16 
                                                   
                                             Website: www.physicsbyfiziks.com                                                                                          
                                                            Email: fiziks.physics@gmail.com                                                                     399 
fiziks
Institute for NET/JRF, GATE, IIT‐JAM, JEST, TIFR and GRE in PHYSICAL SCIENCES 
 
GATE-2015
Q28. The band gap of an intrinsic semiconductor is E g = 0.72 eV and mn* = 6m*g . At 300 K ,

the Fermi level with respect to the edge of the valence band (in eV ) is at _______(upto
three decimal places) k B = 1.38 × 10 −23 JK −1
Ans.: 0.395
Ec + Eν 3 ⎛ m* ⎞
Solution: Ei = + kT ln ⎜ h* ⎟
2 4 ⎝ mn ⎠
− ( Ei − Ev ) / kT − Eg / 2 kT
ni = NV e = Nc Nv e

− ( Ei − Ev ) / kT N c − Eg / 2 kT N v Eg / 2 kT
⇒ e( i v ) =
E − E / kT
⇒e = e e
Nv Nc
3
Ei − Ev ⎛ N v ⎞ Eg ⎛ mh* ⎞ 4 Eg 3 E
= ln ⎜⎜ ⎟⎟ + = ln ⎜ *⎟ + ⇒ Ei − Ev = kT ln ( 6 ) + g
kT ⎝ N c ⎠ 2kT ⎝ me ⎠ 2kT 4 2

3 0.72
⇒ Ei − Ev = × 0.026 × 1.7917 + = 0.3949eV ≈ 0.395 eV
4 2
Q29. Which one of the following DOES NOT represent an exclusive OR operation for inputs
A and B ?
(a) ( A + B )AB (b) AB + BA

(c) ( A + B )(A + B ) (d) ( A + B )AB


Ans.: (d)
Solution: (a) ( A + B) AB = ( A + B)( A + B) = AB + AB

(b) AB + AB
(c) AB + AB
(d) ( A + B ) AB = AB

                                                                                
Head office  Branch office 
 
fiziks, H.No. 40 D, G.F, Jia Sarai,  Anand Institute of Mathematics, 
 
Near IIT, Hauz Khas, New Delhi‐16  28‐B/6, Jia Sarai, Near IIT 
 
Phone: 011‐26865455/+91‐9871145498 Hauz Khas, New Delhi‐16 
                                                   
                                             Website: www.physicsbyfiziks.com                                                                                          
                                                            Email: fiziks.physics@gmail.com                                                                     400 
fiziks
Institute for NET/JRF, GATE, IIT‐JAM, JEST, TIFR and GRE in PHYSICAL SCIENCES 
 
Q30. Consider the circuit shown in the figure, where RC = 1 . For an input signal Vi shown

below, choose the correct V0 from the options:


R
Vi
C
Vi −
V0 1
+
R

1 2 3 t

V0 V0
(a) (b)
1 1

1 2 3 t 1 2 3 t
−1 −1

(c) V0 (d) V
i

0 .1
1
1 2 3 t

− 0 .1 3 t
1 2

Ans.: (b)
dvi 0 − v0 dv dv
Solution: C = ⇒ v0 = − RC in = − in
dt R dt dt
vin = +t ⇒ v0 = −1V and vin = −t ⇒ v0 = +1V

                                                                                
Head office  Branch office 
 
fiziks, H.No. 40 D, G.F, Jia Sarai,  Anand Institute of Mathematics, 
 
Near IIT, Hauz Khas, New Delhi‐16  28‐B/6, Jia Sarai, Near IIT 
 
Phone: 011‐26865455/+91‐9871145498 Hauz Khas, New Delhi‐16 
                                                   
                                             Website: www.physicsbyfiziks.com                                                                                          
                                                            Email: fiziks.physics@gmail.com                                                                     401 
fiziks
Institute for NET/JRF, GATE, IIT‐JAM, JEST, TIFR and GRE in PHYSICAL SCIENCES 
 
Q31. In the simple current source shown in the figure, Q1 and Q2 are identical transistors with
current gain β = 100 and VBE = 0.7 V
Vice = 30 V

5 kΩ I0

Q1 Q2

The current I 0 (in mA) is __________ (upto two decimal places)


Ans.: 5.86
Solution: −VCC + I C RC + VBE = 0

30 − 0.7 29.3
IC = = = 5.86 mA
5 5
Q32. In the given circuit, if the open loop gain A = 10 5 the feedback configurations and the
closed loop gain A f are Vi +
V0

9 kΩ
1 kΩ RL

(a) series-shunt, A f = 9 (b) series-series, A f = 10

(c) series-shunt, A f = 10 (d) shunt-shunt, A f = 10

Ans.: (c)
⎛ R ⎞
Solution: AF = ⎜1 + F ⎟ = (1 + 9 ) = 10.
⎝ R1 ⎠

                                                                                
Head office  Branch office 
 
fiziks, H.No. 40 D, G.F, Jia Sarai,  Anand Institute of Mathematics, 
 
Near IIT, Hauz Khas, New Delhi‐16  28‐B/6, Jia Sarai, Near IIT 
 
Phone: 011‐26865455/+91‐9871145498 Hauz Khas, New Delhi‐16 
                                                   
                                             Website: www.physicsbyfiziks.com                                                                                          
                                                            Email: fiziks.physics@gmail.com                                                                     402 
fiziks
Institute for NET/JRF, GATE, IIT‐JAM, JEST, TIFR and GRE in PHYSICAL SCIENCES 
 
Q33. In the given circuit, the voltage across the source resistor is 1 V . The drain voltage (in V )
is ___________ 25V

5k Ω

2 MΩ
500 Ω

Ans.: 15
1 1
Solution: VS = I D RS ⇒ I D = A ⇒ VD = VDD − I D RD = 25 − × 5000 ⇒ VD = 15V
500 500

GATE-2016
Q34. The number density of electrons in the conduction band of a semiconductor at a given
temperature is 2 × 1019 m −3 . Upon lightly doping this semiconductor with donor
impurities, the number density of conduction electrons at the same temperature
becomes 4 × 10 20 m −3 . The ratio of majority to minority charge carrier concentration is
________.
Ans : 400
Solution: Intrinsic carrier concentration is ni = 2 × 1019 m −3

Majority carrier concentration is n = 4 ×1020 m −3

ni2 ( 2 × 10 )
19 2

Minority carrier concentration is p = = = 1018 m −3


n 4 × 10 20

n 4 × 1020
The ratio of majority to minority charge carrier concentration is = = 400
p 1018

                                                                                
Head office  Branch office 
 
fiziks, H.No. 40 D, G.F, Jia Sarai,  Anand Institute of Mathematics, 
 
Near IIT, Hauz Khas, New Delhi‐16  28‐B/6, Jia Sarai, Near IIT 
 
Phone: 011‐26865455/+91‐9871145498 Hauz Khas, New Delhi‐16 
                                                   
                                             Website: www.physicsbyfiziks.com                                                                                          
                                                            Email: fiziks.physics@gmail.com                                                                     403 
fiziks
Institute for NET/JRF, GATE, IIT‐JAM, JEST, TIFR and GRE in PHYSICAL SCIENCES 
 
Q35. For the digital circuit given below, the output X is

X
B
C

(a) A + B.C (b) A . (B + C ) (c) A . (B + C ) (d) A + (B.C )


Ans.: (b)
Q36. For the transistor shown in the figure, assume V BE = 0.7V and β dc = 100 . If Vin = 5V , Vout
(in Volts) is _________. (Give your answer upto one decimal place)

10V

3 kΩ

Vin Vout

200 k Ω

1k Ω

Ans.: 5.7
Vin − VBE 5 − 0.7 4.3
Solution: I B = = = mA I C = β I B = 1.433 mA
RB + β RE 200 + 100 300

Vout = VCC − I C RC ⇒ Vout = 10 − 1.433 × 3 = 5.7 V

                                                                                
Head office  Branch office 
 
fiziks, H.No. 40 D, G.F, Jia Sarai,  Anand Institute of Mathematics, 
 
Near IIT, Hauz Khas, New Delhi‐16  28‐B/6, Jia Sarai, Near IIT 
 
Phone: 011‐26865455/+91‐9871145498 Hauz Khas, New Delhi‐16 
                                                   
                                             Website: www.physicsbyfiziks.com                                                                                          
                                                            Email: fiziks.physics@gmail.com                                                                     404 
fiziks
Institute for NET/JRF, GATE, IIT‐JAM, JEST, TIFR and GRE in PHYSICAL SCIENCES 
 
ATOMIC AND MOLECULAR PHYSICS SOLUTIONS

GATE-2010

Q1. To detect trace amounts of gaseous species in a mixture of gases, the preferred probing
tool is
(a) Ionization spectroscopy with X-rays (b) NMR spectroscopy
(c) ESR spectroscopy (d) Laser spectroscopy
Ans: (a)
Q2. A collection of N atoms is exposed to a strong resonant electromagnetic radiation with Ng
atoms in the ground state and Ne atoms in the excited state, such that
Ng+Ne=N. This collection of two-level atoms will have the following population
distribution:
(a) Ng << Ne (b) Ng >> Ne (c) Ng ≈ Ne ≈ N/2 (d) Ng – Ne ≈ N/2
Ans: (c)
Solution: In two level lair population inversion is possible to achieve at any power level. The
N
maximum possible situation can be N g ≈ N e ≈
2
Q3. Two states of an atom have definite parities. An electric dipole transition between these
states is
(a) Allowed if both the sates have even parity
(b) Allowed if both the states have odd parity
(c) Allowed if the two states have opposite parities
(d) Not allowed unless a static electric field is applied
Ans: (c)
Q4. The spectrum of radiation emitted by a black body at a temperature 1000 K peaks in the
(a) Visible range of frequencies (b) Infrared range of frequencies
(c) Ultraviolet range of frequencies (d) Microwave range of frequencies
Ans: (a)

                                                                                
Head office  Branch office 
 
fiziks, H.No. 40 D, G.F, Jia Sarai,  Anand Institute of Mathematics, 
 
Near IIT, Hauz Khas, New Delhi‐16  28‐B/6, Jia Sarai, Near IIT 
 
Phone: 011‐26865455/+91‐9871145498 Hauz Khas, New Delhi‐16 
                                                   
                                             Website: www.physicsbyfiziks.com                                                                                          
                                                            Email: fiziks.physics@gmail.com                                                                     405 
fiziks
Institute for NET/JRF, GATE, IIT‐JAM, JEST, TIFR and GRE in PHYSICAL SCIENCES 
 
Q5. The three principal moments of inertia of a methanol (CH3OH) molecule have the
property Ix = Iy = I and Iz ≠ I. The rotation energy eigenvalues are
2 2
m12 ⎛ 1 1 ⎞ 2
(a) l (l + 1) + ⎜ − ⎟ (b) l (l + 1)
2I 2 I ⎜⎝ I z I ⎟⎠ 2I
2
m12 ⎛ 1 1 ⎞ 2 2
m12 ⎛ 1 1 ⎞
(c) ⎜ − ⎟ (d) l (l + 1) + ⎜ + ⎟
2 I ⎜⎝ I z I ⎟⎠ 2I 2 ⎜⎝ I z I ⎟⎠
Ans: (a)
Solution: CH 3 OH is example of symmetric rotar where I x = I y ≠ I z ( I x + I y = I and I z ≠ I )

The classical expression for energy is E =


1 2
2I
(
J x + J y2 +
1 2
2I z
)
Jz

This can be expressed in term of J 2 = J x2 + J y2 + J z2 by adding and subtracting J z2

1 2 ⎛ 1 1 ⎞
E= J + ⎜⎜ − ⎟⎟ J z2 .
2I ⎝ 2I z 2I ⎠
m J2 ⎛ 1 1 ⎞
2 2
Quantum mechanically E = ⎜ − ⎟ J (J + 1) +
2I 2 ⎜⎝ I z I ⎟⎠
Q6. Match the typical spectra of stable molecules with the corresponding wave-number range
1. Electronic spectra (i) 106 cm-1 and above
2. Rotational spectra (ii) 105 – 106 cm-1
3. Molecule dissociation (iii) 108 – 102 cm-1
(a) 1 – ii, 2 – i, 3 – iii (b) 1 – ii, 2 – iii, 3 – i
(b) 1 – iii, 2 – ii, 3 – i (d) 1 – i, 2 – ii, 3 – iii
Ans: (b)

Q7. Consider the operations P : r → − r (parity) and T: t → - t (time reversal). For the electric

and magnetic fields E and B , which of the following set of transformations is correct?

(a) P : E → − E , B → B; (b) P : E → E , B → B;

T : E → E, B → − B T : E → E, B → B

(c) P : E → − E , B → B; (d) P : E → E , B → − B;

                                                                                
Head office  Branch office 
 
fiziks, H.No. 40 D, G.F, Jia Sarai,  Anand Institute of Mathematics, 
 
Near IIT, Hauz Khas, New Delhi‐16  28‐B/6, Jia Sarai, Near IIT 
 
Phone: 011‐26865455/+91‐9871145498 Hauz Khas, New Delhi‐16 
                                                   
                                             Website: www.physicsbyfiziks.com                                                                                          
                                                            Email: fiziks.physics@gmail.com                                                                     406 
fiziks
Institute for NET/JRF, GATE, IIT‐JAM, JEST, TIFR and GRE in PHYSICAL SCIENCES 
 
T : E → − E, B → − B T : E → −E, B → B

Ans: (b)
Common Data Questions 8 and 9:
In the presence of a weak magnetic field, atomic hydrogen undergoes the transition:
2
P1 / 2 →1S1 / 2 , by emission of radiation

Q8. The number of distinct spectral lines that are observed in the resultant Zeeman spectrum
is
(a) 2 (b) 3 (c) 4 (d) 6
Ans: (c)
Solution: 2 p1 / 2 − 2 S 1 / 2 is sodium D1 lines and it has total 4 zeeman components.
Q9. The spectral line corresponding to the transition
⎛ 1⎞ ⎛ 1⎞
2
P1 ⎜ m j = + ⎟→1S1 / 2 ⎜ m j = − ⎟
2⎝ 2⎠ ⎝ 2⎠
is observed along the direction of the applied magnetic field. The emitted electromagnetic
field is
(a) Circularly polarized (b) Linearly polarized
(c) Unpolarized (d) Not emitted along the magnetic field direction
Ans: (a)
⎛ 1⎞ ⎛ 1⎞
Solution: For 2
P1 ⎜ m j = + ⎟→1S1 / 2 ⎜ m j = − ⎟
2⎝ 2⎠ ⎝ 2⎠

Here Δm j = +1 gives σ + component. In longitudinal observation σ + is circularly

polarized.
GATE-2011

Q10. The population inversion in a two layer material CANNOT be achieved by optical
pumping because
(a) the rate of upward transitions is equal to the rate of downward transitions
(b) the upward transitions are forbidden but downward transitions are allowed
(c) the upward transitions are allowed but downward transitions are forbidden
                                                                                
Head office  Branch office 
 
fiziks, H.No. 40 D, G.F, Jia Sarai,  Anand Institute of Mathematics, 
 
Near IIT, Hauz Khas, New Delhi‐16  28‐B/6, Jia Sarai, Near IIT 
 
Phone: 011‐26865455/+91‐9871145498 Hauz Khas, New Delhi‐16 
                                                   
                                             Website: www.physicsbyfiziks.com                                                                                          
                                                            Email: fiziks.physics@gmail.com                                                                     407 
fiziks
Institute for NET/JRF, GATE, IIT‐JAM, JEST, TIFR and GRE in PHYSICAL SCIENCES 
 
(d) the spontaneous decay rate of the higher level is very low
Ans: (a)
Q11. A heavy symmetrical top is rotating about its own axis of symmetry (the z-axis). If I1, I2
and I3 are the principal moments of inertia along x, y and z axes respectively, then
(a) I2 = I3; I1 ≠ I2 (b) I1 = I3; I1 ≠ I2 (c) I1 = I2; I1 ≠ I3 (d) I1 ≠ I2 ≠ I3
Ans: (c)
Q12. A neutron passing through a detector is detected because of
(a) the ionization it produces (b) the scintillation light it produces
(c) the electron-hole pairs it produces
(d) the secondary particles produced in a nuclear reaction in the detector medium
Ans: (b)
Q13. An atom with one outer electron having orbital angular momentum l is placed in a weak
magnetic field. The number of energy levels into which the higher total angular
momentum state splits, is
(a) 2l + 2 (b) 2l + 1 (c) 2l (d) 2l – 1
Ans: (b)
Q14. For a multi-electron atom l, L and S specify the one-electron orbital angular momentum,
total orbital angular momentum and total spin angular momentum, respectively. The
selection rules for electric dipole transition between the two electronic energy levels,
specified by l, L and S are
(a) ∆L = 0, ±1; ∆S = 0; ∆l = 0, ±1 (b) ∆L = 0, ±1; ∆S = 0; ∆l = ±1
(c) ∆L = 0, ±1; ∆S = ±1; ∆l = 0, ±1 (d) ∆L = 0, ±1; ∆S = ±1; ∆l = ±1
Ans: (b)
Q15. The lifetime of an atomic state is 1 nanosecond. The natural line width of the spectral line
in the emission spectrum of this state is of the order of
(a) 10-10 eV (b) 10-9 eV (c) 10-6 eV (d) 10-4 eV
Ans: (c)
h 6.625 × 10 −34 J − S 6.625 × 10 −25
Solution: ΔE = hΔν = = −9
= −19
eV = 4.14 × 10 −6 eV
Δt 10 1.6 × 10

                                                                                
Head office  Branch office 
 
fiziks, H.No. 40 D, G.F, Jia Sarai,  Anand Institute of Mathematics, 
 
Near IIT, Hauz Khas, New Delhi‐16  28‐B/6, Jia Sarai, Near IIT 
 
Phone: 011‐26865455/+91‐9871145498 Hauz Khas, New Delhi‐16 
                                                   
                                             Website: www.physicsbyfiziks.com                                                                                          
                                                            Email: fiziks.physics@gmail.com                                                                     408 
fiziks
Institute for NET/JRF, GATE, IIT‐JAM, JEST, TIFR and GRE in PHYSICAL SCIENCES 
 
Q16. The degeneracy of an excited state of nitrogen atom having electronic configuration
1s22s22p23d1 is
(a) 6 (b) 10 (c) 15 (d) 150
Ans: (b)
Solution: Degeneracy = 2 (2l + 1)
Q17. The far infrared rotational absorption spectrum of a diatomic molecule shows equilibrium
lines with spacing 20 cm-1. The position of the first Stokes line in the rotational Raman
spectrum of this molecule is
(a) 20 cm-1 (b) 40 cm-1 (c) 60 cm-1 (d) 120 cm-1
Ans: (c)
Solution: Given 2B = 20 cm-1 ⇒ B = 10 cm-1
The position of the first stokes line in the rotational Raman spectrum = 6B
∴ Δν = 6 B = 6 × 10 = 60 cm −1 .
GATE-2012
Q18. The ground state of sodium atom ( 11 Na ) is a 2 S1 / 2 state. The difference in energy levels
arising in the presence of a weak external magnetic field B, given in terms of Bohr
magnet on, μ B , is
(a) μ B B (b) 2 μ B B (c) 4 μ B B (d) 6 μ B B
Ans: (b)
Solution: The energy separation in the Zeeman level is ΔE = gM J (μ B B )

1
For 2 S1 / 2 state; g = 2 and M J = ± . Therefore ΔE1 = +(μ B B ) and ΔE 2 = −(μ B B ) .
2
Thus ΔE = 2 μ B B
Q19. The first Stokes line of a rotational Raman spectrum is observed at 12.96 cm-1.
Considering the rigid rotor approximation, the rotational constant is given by
(a) 6.48 cm-1 (b) 3.24 cm-1 (c) 2.16 cm-1 (d) 1.62 cm-1
Ans: (c)
Solution: The first Stoke line of the Rotational Raman spectrum lies at = 6B

                                                                                
Head office  Branch office 
 
fiziks, H.No. 40 D, G.F, Jia Sarai,  Anand Institute of Mathematics, 
 
Near IIT, Hauz Khas, New Delhi‐16  28‐B/6, Jia Sarai, Near IIT 
 
Phone: 011‐26865455/+91‐9871145498 Hauz Khas, New Delhi‐16 
                                                   
                                             Website: www.physicsbyfiziks.com                                                                                          
                                                            Email: fiziks.physics@gmail.com                                                                     409 
fiziks
Institute for NET/JRF, GATE, IIT‐JAM, JEST, TIFR and GRE in PHYSICAL SCIENCES 
 
−1 −1
Thus 6 B = 12.96 cm ⇒ B = 1.16 cm .
Q20. Match the typical spectroscopic regions specified in Group I with the corresponding type
of transitions in Group II.
Group I Group II
(P) Infra-red region (i) electronic transitions involving valence electrons
(Q) Ultraviolet-visible region (ii) nuclear transitions
(R) X-ray region (iii) vibrational transitions of molecules
(S) γ-ray region (iv) transitions involving inner shell electrons
(a) (P, i); (Q, iii); (R, ii); (S, iv) (b) (P, ii); (Q, iv); (R, i); (S, iii)
(c) (P, iii); (Q, i); (R, iv); (S, ii) (d) (P, iv); (Q, i); (R, ii); (S, iii)
Ans: (c)
Q21. The term { j1 , j 2 }J arising from 2s1 3d 1 electronic in j-j coupling scheme are

⎧1 3 ⎫ ⎧1 5 ⎫ ⎧1 1 ⎫ ⎧1 3⎫
(a) ⎨ , ⎬ and ⎨ , ⎬ (b) ⎨ , ⎬ and ⎨ , ⎬
⎩ 2 2 ⎭ 2,1 ⎩ 2 2 ⎭ 3, 2 ⎩ 2 2 ⎭1, 0 ⎩ 2 2 ⎭ 2,1

⎧1 1 ⎫ ⎧1 5 ⎫ ⎧3 1⎫ ⎧1 5 ⎫
(c) ⎨ , ⎬ and ⎨ , ⎬ (d) ⎨ , ⎬ and ⎨ , ⎬
⎩ 2 2 ⎭1,0 ⎩ 2 2 ⎭ 3, 2 ⎩ 2 2 ⎭ 2,1 ⎩ 2 2 ⎭ 3, 2
Ans: (c)
Q22. The equilibrium vibration frequency for an oscillator is observed at 2990 cm-1. The ratio
of the frequencies corresponding to the first and the fundamental spectral lines is 1.96.
Considering the oscillator to be anharmonic, the anharmonicity constant is
(a) 0.005 (b) 0.02 (c) 0.05 (d) 0.1
Ans: (b)
2ω e (1 − 3 xe ) (1 − 3xe )
Solution: ω e (1 − 2 xe ) = 2990cm −1 and = 1.96cm −1 ⇒ = 0.98cm −1
ω e (1 − 2 xe ) (1 − 2 xe )
⇒ xe = 0.02 .

                                                                                
Head office  Branch office 
 
fiziks, H.No. 40 D, G.F, Jia Sarai,  Anand Institute of Mathematics, 
 
Near IIT, Hauz Khas, New Delhi‐16  28‐B/6, Jia Sarai, Near IIT 
 
Phone: 011‐26865455/+91‐9871145498 Hauz Khas, New Delhi‐16 
                                                   
                                             Website: www.physicsbyfiziks.com                                                                                          
                                                            Email: fiziks.physics@gmail.com                                                                     410 
fiziks
Institute for NET/JRF, GATE, IIT‐JAM, JEST, TIFR and GRE in PHYSICAL SCIENCES 
 
GATE-2013
Q23. The number of spectral lines allowed in the spectrum for the 3 2 D → 3 2 P transition in
sodium is _____________.
Ans: 28
Solution: The numbers of Zeeman components for 2D5/2 → 2P3/2 transition = 12
The numbers of Zeeman components for 2D3/2 → 2P3/2 transition = 10
The numbers of Zeeman components for 2D3/2 → 2P1/2 transition = 6
Q24. In a normal Zeeman Effect experiment, spectral splitting of the line at the wavelength
643.8 nm corresponding to the transition 5 1 D2 → 5 1 P1 of cadmium atoms is to be
observed. The spectrometer has a resolution of 0.01 nm. Minimum magnetic field needed
(
to observe this is me = 9.1 × 10 −31 kg , e = 1.6 × −19 C , c = 3 × 10 8 m / s )
(a) 0.26T (b) 0.52T (c) 2.6T (d) 5.2T
Ans: (b)
Solution: Separation of Zeeman Components
eB λ2 λ2 eB
Δν = ⇒ Δλ = Δν =
4πm c c 4πm
4πmc Δλ 4 × 3.14 × 9.1 × 10 −31 × 3 × 10 8 0.01 × 10 −9
B= = = 0.514T
e λ2 1.6 × 10 −19 643.8 × 10 −9
2
( )
Q25. The spacing between vibrational energy levels in CO molecule is found to
be 8.44 × 10 −2 eV . Given that the reduced mass of CO is 1.14 × 10 −26 kg , Planck’s constant

is 6.626 × 10 −34 Js and 1 eV = 1.6 × 10 −19 J . The force constant of the bond in CO
molecule is
(a) 1.87 N/m (b) 18.7 N/m (c) 187 N/m (d) 1870 N/m
Ans: (c)
Solution: The energy of the quantum harmonic oscillator is
⎛ 1⎞
E = hν ⎜ n + ⎟, n = 0,1,2,........
⎝ 2⎠
1 k
The frequency of oscillation is ν = .
2π μ
                                                                                
Head office  Branch office 
 
fiziks, H.No. 40 D, G.F, Jia Sarai,  Anand Institute of Mathematics, 
 
Near IIT, Hauz Khas, New Delhi‐16  28‐B/6, Jia Sarai, Near IIT 
 
Phone: 011‐26865455/+91‐9871145498 Hauz Khas, New Delhi‐16 
                                                   
                                             Website: www.physicsbyfiziks.com                                                                                          
                                                            Email: fiziks.physics@gmail.com                                                                     411 
fiziks
Institute for NET/JRF, GATE, IIT‐JAM, JEST, TIFR and GRE in PHYSICAL SCIENCES 
 
Where k = Spring constant and μ = reduced mass
h k
The energy levels are equally spaced with energy separation of ΔE = hν =
2π μ
2 2
⎛ 2π ⎞ ⎛ 2 × 3.14 ⎞
k =⎜ ΔE ⎟ μ = ⎜ −34
× 8.44 × 10 − 2 × 1.6 × 10 −19 ⎟ × 1.14 × 10 − 26 = 186.7 N / m
⎝ h ⎠ ⎝ 6.626 × 10 ⎠
GATE-2014
Q26. The number of normal Zeeman splitting components of 1 P → 1D transition is
(a) 3 (b) 4 (c) 8 (d) 9
Ans: (a)
Solution: This is singlet transition.
Q27. The moment of inertia of a rigid diatomic molecule A is 6 times that of another rigid
diatomic molecule B . If the rotational energies of the two molecules are equal, then the
corresponding values of the rotational quantum numbers J A and J B are
(a) J A = 2, J B = 1 (b) J A = 3, J B = 1
(c) J A = 5, J B = 0 (d) J A = 6, J B = 1
Ans: (b)
J A ( J A + 1) I B I
Solution: = ⇒ B
J B ( J B + 1) I A 6I B

J A = 6, J B = 1
Q28. The value of the magnetic field required to maintain non-relativistic protons of energy
1MeV in a circular orbit of radius 100mm is_______Tesla
(Given: m p = 1.67 × 10 −27 kg . e = 1.6 × 10 −19 C )

Ans: 1.44

mv 2 1 2mE
Solution: = qvB, E = mv 2 ⇒ B = = 1.44
r 2 qr

                                                                                
Head office  Branch office 
 
fiziks, H.No. 40 D, G.F, Jia Sarai,  Anand Institute of Mathematics, 
 
Near IIT, Hauz Khas, New Delhi‐16  28‐B/6, Jia Sarai, Near IIT 
 
Phone: 011‐26865455/+91‐9871145498 Hauz Khas, New Delhi‐16 
                                                   
                                             Website: www.physicsbyfiziks.com                                                                                          
                                                            Email: fiziks.physics@gmail.com                                                                     412 
fiziks
Institute for NET/JRF, GATE, IIT‐JAM, JEST, TIFR and GRE in PHYSICAL SCIENCES 
 
3
Q29. Neutrons moving with speed 10 m / s are used for the determination of crystal structure.

If the Bragg angle for the first order diffraction is 30 o the interplannar spacing of the
0
crystal is ______ A
(Given: mn = 1.675 × 10−27 kg h = 6.626 × 10−34 J .s )
Ans: 4
h 0
Solution: 2d sin θ = λ = ∴d = 4 A
mv
Q30. The emission wavelength for the transition D2 → F3 is 3122 Ǻ. The ratio of population

of the final to the initial states at a temperature 5000 K is


(h = 6.626 × 10 −34
J .s, c = 3 × 10 8 m / s k B = 1.380 × 10 −23 J / K )

(a) 2.03 × 10 −5 (b) 4.02 × 10 −5 (c) 7.02 × 10 −5 (d) 9.83 × 10 −5


Ans: (c)

N F ( 2 J f + 1) − λ kBT 5 −9.227641144
hc

Solution: = e = e = 7.02 ×10−5


NI ( 2 J i + 1) 7

GATE-2015
Q31. In a rigid rotator of mass M , if the energy of the first excited state is ( 1 meV ), then the
fourth excited state energy (in meV ) is ____________.
Ans.: 6
Solution: ∵ E ∝ J ( J + 1) where J = 0,1, 2,3..

E4 3 ( 3 + 1)
⇒ = ⇒ E4 = 6 E1 = 6 meV where J = 0,1, 2,3..
E1 1(1 + 1)
Q32. The binding energy per molecule of NaCl (lattice parameter is 0.563 nm ) is 7.956 eV .
K
The repulsive term of the potential is of the form , where K is a constant. The value
r9
of the Madelung constant is ___________ (upto three decimal places)
(Electron charge e = −1.6 × 10 −19 C ; ε 0 = 8.854 × 10 −12 C 2 N −1 m −2 )
Ans.: 1.68
                                                                                
Head office  Branch office 
 
fiziks, H.No. 40 D, G.F, Jia Sarai,  Anand Institute of Mathematics, 
 
Near IIT, Hauz Khas, New Delhi‐16  28‐B/6, Jia Sarai, Near IIT 
 
Phone: 011‐26865455/+91‐9871145498 Hauz Khas, New Delhi‐16 
                                                   
                                             Website: www.physicsbyfiziks.com                                                                                          
                                                            Email: fiziks.physics@gmail.com                                                                     413 
fiziks
Institute for NET/JRF, GATE, IIT‐JAM, JEST, TIFR and GRE in PHYSICAL SCIENCES 
 
Solution: The total energy of one ion due to the presence of all others in NaCl crystal is
(considering univalent ions)
Ae2 K
U (r ) = − + n where A is Modelung Constant.
4πε 0 r r

The potential energy will be minimum at the equilibrium spacing r0 .

⎡ dU ⎤ ⎡ Ae 2 Kn ⎤ Ae2 r0n −1
Thus ⎢ = ⎢ − n +1 ⎥
= 0 ⇒ K =
⎣ dr ⎥⎦ r = r0 ⎣ 4πε 0 r0 r0 ⎦ 4πε 0 n
2

Thus, Binding energy of molecule or lattice energy is

Ae 2 Ae2 r0n −1 ⎡ Ae 2 ⎤ ⎡ n − 1 ⎤
U 0 = [U ]r = r =− + = −⎢ ⎥⎢ ⎥
0
4πε 0 r0 4πε 0 nr0n ⎣ 4πε 0 r0 ⎦ ⎣ n ⎦
K
Given repulsive term of the potential is (∴ n = 9 )
r9
Also binding energy per molecule is U 0 = 7.95 eV

4πε 0 r0 n
The Modelung constant is A = U 0 × ×
e 2
n −1
Given, the lattice parameter is a = 0.563 nm , thus, the interatomic separation is
a
r0 = = 0.82 nm
2

⇒ A = 7.95 ×1.67 × 0 −19



(
4 × 3.14 × 8.85 × 10−12 C 2 N −1m −2 × 0.282 × 10−9 ) ( )×9
(1.67 ×10 )
2
−19
J 8

7.95 × 4 × 3.14 × 8.85 × 0.282 × 9


⇒ A= ×10−2 ⇒ A = 1.68
1.67 × 8
Q33. Match the phrases in Group I and Group II and identify the correct option.
Group I Group II
(P) Electron spin resonance (ESR) (i) radio frequency
(Q) Nuclear magnetic resonance (NMR) (ii) visible range frequency
(R) Transition between vibrational states of a molecule (iii) microwave frequency
(S) Electronic transition (iv) far-infrared range
                                                                                
Head office  Branch office 
 
fiziks, H.No. 40 D, G.F, Jia Sarai,  Anand Institute of Mathematics, 
 
Near IIT, Hauz Khas, New Delhi‐16  28‐B/6, Jia Sarai, Near IIT 
 
Phone: 011‐26865455/+91‐9871145498 Hauz Khas, New Delhi‐16 
                                                   
                                             Website: www.physicsbyfiziks.com                                                                                          
                                                            Email: fiziks.physics@gmail.com                                                                     414 
fiziks
Institute for NET/JRF, GATE, IIT‐JAM, JEST, TIFR and GRE in PHYSICAL SCIENCES 
 
(a) (P-i), (Q-ii), (R-iii), (S-iv) (b) (P-ii), (Q-i), (R-iv), (S-iii)
(c) (P-iii), (Q-iv), (R-i), (S-ii) (d) (P-iii), (Q-i), (R-iv), (S-ii)
Ans.: (d)
Solution: (P) Electron spin resonance (ESR) is achieved by Microwave frequency (iii)
(Q): Nuclear magnetic resonance (NMR) is achieved by Radio frequency (i)
(R): Transition between vibrational states of a molecule is achieved by radiation of far
infrared range (iv)
(S): Electronic transition is achieved by visible radiation (ii)
Q34. The excitation wavelength of laser in a Raman effect experiment is 546 nm . If the
Stokes’ line is observed at 552 nm , then the wavenumber of the anti-Stokes’ line (in

cm −1 ) is ___________
Ans.: 18514
Solution: Raman displacement is
1 1 1 1
Δν = ν AS −ν 0 = ν 0 −ν S or Δν = − = −
λAS λ0 λ0 λS
where λ AS , λ0 , λ S are wavelength of anti-stoke, exciting & stoke line.
From above relation we can write
1 1 1 1 1 2 1 1 2λ S − λ 0 λ0 λ S
− = − ⇒ = − ⇒ = ⇒ λ AS =
λ AS λ0 λ0 λS λ AS λ0 λS λ AS λ0 λ S 2λ S − λ 0

∴ λAS =
( 546 ×10 m )( 552 ×10 m )
−9 −9

=
546 × 552
×10−9 m
( 2 × 552 ×10 −9
m − 546 ×10 m ) −9
558

⇒ λAS = 540.129 ×10−9 m = 540.129 × 10−7 cm

1 1
Anti-stoke wavenumber is ν AS = = = 18514 cm −1
λAS −7
540.129 × 10 cm

                                                                                
Head office  Branch office 
 
fiziks, H.No. 40 D, G.F, Jia Sarai,  Anand Institute of Mathematics, 
 
Near IIT, Hauz Khas, New Delhi‐16  28‐B/6, Jia Sarai, Near IIT 
 
Phone: 011‐26865455/+91‐9871145498 Hauz Khas, New Delhi‐16 
                                                   
                                             Website: www.physicsbyfiziks.com                                                                                          
                                                            Email: fiziks.physics@gmail.com                                                                     415 
fiziks
Institute for NET/JRF, GATE, IIT‐JAM, JEST, TIFR and GRE in PHYSICAL SCIENCES 
 
Q35. The number of permitted transitions from 2 P3 / 2 → 2 S1 / 2 in the presence of a weak
magnetic field is ________________
Ans. : 6
Solution: Zeeman splitting of 2 P3 / 2 and 2 S1/ 2 is shown below
MJ
+3/2
2
P3/2 +1/2

−1/2
−3/2
2
S1/ 2 +1/2

−1/2

The selection rule for Zeeman transactions are


ΔM J = 0, ± 1 (0 →/ 0Δ if J = 0)
There are total six transition in accordance with above selection rules.

GATE-2016
17
Q36. The molecule O2
(a) Raman active but not NMR (nuclear magnetic resonance) active.
(b) Infrared active and Raman active but not NMR active.
(c) Raman active and NMR active.
(d) Only NMR active.
Ans.: (c)
17
Solution: (i) Molecule O2 can not absorb infrared as there is no change in dipole moment
17
during vibration. Thus O2 is infrared inactive.
17
(ii) Molecule O2 shows change in polaraziability during rotation. Thus it is Raman

active molecule.
17 5
(iii) The nucleus of O has spin , therefore it is NMR active.
2

                                                                                
Head office  Branch office 
 
fiziks, H.No. 40 D, G.F, Jia Sarai,  Anand Institute of Mathematics, 
 
Near IIT, Hauz Khas, New Delhi‐16  28‐B/6, Jia Sarai, Near IIT 
 
Phone: 011‐26865455/+91‐9871145498 Hauz Khas, New Delhi‐16 
                                                   
                                             Website: www.physicsbyfiziks.com                                                                                          
                                                            Email: fiziks.physics@gmail.com                                                                     416 
fiziks
Institute for NET/JRF, GATE, IIT‐JAM, JEST, TIFR and GRE in PHYSICAL SCIENCES 
 
Q37. There are four electrons in the 3d shell of an isolated atom. The total magnetic moment
of the atom in units of Bohr magneton is ________.
Ans.: 0
Solution: The configuration leads to S = 2 and L = 2
Since it is the case of less than half filled sub shell, thus according to Hund’s rules, lower
J will be in ground state. M L = −2 −1 0 +1 +2
∴ J = L − S = 0 . Thus μ = 0 3d 4 =
Q38. Which of the following transitions is NOT allowed in the case of an atom, according to
the electric dipole radiation selection rule?
(a) 2 s − 1s (b) 2 p − 1s (c) 2 p − 2s (d) 3d − 2 p
Ans.: (a)
Solution: In electron dipole transition, Δl = ±1 . Thus in transition 2s → 1s, Δl = 0 . It violate the
selection rule and hence not allowed.
Q39. The number of spectroscopic terms resulting from the L.S coupling of a 3 p electron and
a 3d electron is _______.
Ans.: 12
1 1
Solution: For 3 p1 3d 1 : s1 = , s2 = ∴ S = 0,1
2 2
l1 = 1, l2 = 2 ∴ L = 1, 2,3

S = 0, L = 1 ⇒ J = 1 ∴ Term = 1P1

S = 0, L = 2 ⇒ J = 2 ∴ Term = 1D2

S = 0, L = 3 ⇒ J = 3 ∴ Term = 1F3

S = 1, L = 1 ⇒ J = 0,1, 2 ∴ Terms = 3P0 , 3P1 , 3 P2

S = 1, L = 2 ⇒ J = 1, 2,3 ∴ Terms = 3D1 , 3D2 , 3 D3

S = 1, L = 3 ⇒ J = 2,3, 4 ∴ Terms = 3F2 , 3F3 , 3 F4

Thus total number of spectroscopic terms are 12.

                                                                                
Head office  Branch office 
 
fiziks, H.No. 40 D, G.F, Jia Sarai,  Anand Institute of Mathematics, 
 
Near IIT, Hauz Khas, New Delhi‐16  28‐B/6, Jia Sarai, Near IIT 
 
Phone: 011‐26865455/+91‐9871145498 Hauz Khas, New Delhi‐16 
                                                   
                                             Website: www.physicsbyfiziks.com                                                                                          
                                                            Email: fiziks.physics@gmail.com                                                                     417 
fiziks
Institute for NET/JRF, GATE, IIT‐JAM, JEST, TIFR and GRE in PHYSICAL SCIENCES 
 
SOLID STATE PHYSICS SOLUTIONS

GATE-2010

Q1. The valence electrons do not directly determine the following property of a metal
(a) Electrical conductivity (b) Thermal conductivity
(c) Shear modulus (d) Metallic luster
Ans: (c)
Q2. Consider X-ray diffraction from a crystal with a face-centered-cubic (fcc) lattice. The
lattice plane for which there is NO diffraction peak is
(a) (2, 1, 2) (b) (1, 1, 1) (c) (2, 0, 0) (d) (3, 1, 1)
Ans: (a)
Q3. The Hall coefficient, RH, of sodium depends on
(a) The effective charge carrier mass and carrier density
(b) The charge carrier density and relaxation time
(c) The charge carrier density only
(d) The effective charge carrier mass
Ans: (c)

Q4. The Bloch theorem states that within a crystal, the wavefunction, ψ( r ), of an electron has
the form

() ()
(a) ψ r = u r e i k .r where u( r ) is an arbitrary function and k is an arbitrary vector

(b)ψ (r ) = u (r )e i G ⋅r
where u( r ) is an arbitrary function and G is a reciprocal lattice vector

(c) ψ (r ) = u (r )e i G ⋅r
() ( )
where u r = u r + Λ , Λ is a lattice vector and G is a reciprocal
lattice vector

() () () ( )
(d) ψ r = u r e i k .r where u r = u r + Λ , Λ is a lattice vector and k is an arbitrary
vector
Ans: (d)

                                                                                
Head office  Branch office 
 
fiziks, H.No. 40 D, G.F, Jia Sarai,  Anand Institute of Mathematics, 
 
Near IIT, Hauz Khas, New Delhi‐16  28‐B/6, Jia Sarai, Near IIT 
 
Phone: 011‐26865455/+91‐9871145498 Hauz Khas, New Delhi‐16 
                                                   
                                             Website: www.physicsbyfiziks.com                                                                                          
                                                            Email: fiziks.physics@gmail.com                                                                     418 
fiziks
Institute for NET/JRF, GATE, IIT‐JAM, JEST, TIFR and GRE in PHYSICAL SCIENCES 
 
Q5. In an experiment involving a ferromagnetic medium, the following observations were
made. Which one of the plots does NOT correctly represent the property of the medium?
(TC is the Curie temperature)
(a) (b)

1 / TC 1/ T

(c) (d)

TC T TC T
Ans: (c)
Q6. The thermal conductivity of a given material reduces when it undergoes a transition from
its normal state to the superconducting state. The reason is:
(a) The Cooper pairs cannot transfer energy to the lattice
(b) Upon the formation of Cooper pairs, the lattice becomes less efficient in heat transfer
(c) The electrons in the normal state lose their ability to transfer heat because of their
coupling to the Cooper pairs
(d) The heat capacity increases on transition to the superconducting state leading to a
reduction in thermal conductivity
Ans: (d)

                                                                                
Head office  Branch office 
 
fiziks, H.No. 40 D, G.F, Jia Sarai,  Anand Institute of Mathematics, 
 
Near IIT, Hauz Khas, New Delhi‐16  28‐B/6, Jia Sarai, Near IIT 
 
Phone: 011‐26865455/+91‐9871145498 Hauz Khas, New Delhi‐16 
                                                   
                                             Website: www.physicsbyfiziks.com                                                                                          
                                                            Email: fiziks.physics@gmail.com                                                                     419 
fiziks
Institute for NET/JRF, GATE, IIT‐JAM, JEST, TIFR and GRE in PHYSICAL SCIENCES 
 
Q7. For a two-dimensional free electron gas, the electronic density n, and the Fermi energy
EF, are related by

(2mE F ) 2 (mE F ) 3
3 1 1
mE F mE F 2 3
(a) n = (b) n = (c) n = (d) n =
3π 2 3
π 2
2π 2 π
Ans: (b)
Solution: For two dimensional gas, the number of possible k-states between k and k+dk is

2 2
⎛ L ⎞ ⎛ L ⎞
g (k )dk = ⎜ ⎟ 2π k dk = 2⎜ ⎟ 2π k dk it is multiplied by 2 for electron gas
⎝ 2π ⎠ ⎝ 2π ⎠

2mE 2m 2πm
Since k 2 = 2
∵ 2k dk = 2
dE ⇒ 2π k dk = 2
dE

2
⎛ L ⎞ 2πm
∴ g (E )dE = 2⎜ ⎟ ⋅ 2 dE
⎝ 2π ⎠
The total number of electrons at T = 0 0 K is
EF EF 2 EF
2m ⎛ 1 ⎞ 2m L2
N= ∫ g (E )dE × F (E ) =
0
∫ g (E )dE = 2π ⋅
0
2 ⎜ ⎟
⎝ 2π ⎠
∫ dE = 2π ⋅
0
2

4π 2
⋅ EF

m π ⎛N⎞ π
2 2
mE F
⇒N= ⋅ L2 E F ⇒ E F = ⎜ 2⎟ = ⋅n ⇒ n =
π 2
m ⎝L ⎠ m π 2
Q8. Far away from any of the resonance frequencies of a medium, the real part of the
dielectric permittivity is
(a) Always independent of frequency (b) Monotonically decreasing with frequency
(c) Monotonically increasing with frequency (d) A non-monotonic function of frequency
Ans: (a)
dipolar

∈ −1 ionic
∈ +2
electronics

frequency

                                                                                
Head office  Branch office 
 
fiziks, H.No. 40 D, G.F, Jia Sarai,  Anand Institute of Mathematics, 
 
Near IIT, Hauz Khas, New Delhi‐16  28‐B/6, Jia Sarai, Near IIT 
 
Phone: 011‐26865455/+91‐9871145498 Hauz Khas, New Delhi‐16 
                                                   
                                             Website: www.physicsbyfiziks.com                                                                                          
                                                            Email: fiziks.physics@gmail.com                                                                     420 
fiziks
Institute for NET/JRF, GATE, IIT‐JAM, JEST, TIFR and GRE in PHYSICAL SCIENCES 
 
GATE-2011

Q9. The temperature (T) dependence of magnetic susceptibility (χ) of a ferromagnetic


substance with a Curie temperature (Tc) is given by
C C
(a) , for T < Tc (b) , for T > Tc
T − Tc T − Tc

C C
(c) , for T > Tc (d) , for all temperatures
T + Tc T + Tc

where C is constant .
Ans: (b)
Q10. The order of magnitude of the energy gap of a typical superconductor is
(a) 1 MeV (b) 1 KeV (c) 1 eV (d) 1 meV
Ans: (d)
Q11. For a three-dimensional crystal having N primitive unit cells with a basis of p atoms, the
number of optical branches is
(a) 3 (b) 3p (c) 3p – 3 (d) 3N – 3p
Ans: (c)
Q12. For an intrinsic semiconductor, me* and mh* are respectively the effective masses of
electrons and holes near the corresponding band edges. At a finite temperature the
position of the Fermi level
(a) depends on me* but not on mh* (b) depends on mh* but not on me*
(c) depends on both me* and mh* (d) depends neither on me* nor on mh*
Ans: (c)
Ec + Ev 3 ⎛ mh* ⎞
Solution: The Fermi level for intrinsic semicondutor is E F = + k B T ln⎜⎜ * ⎟⎟
2 4 ⎝ me ⎠
Q13. A metal with body centered cubic (bcc) structure show the first (i.e. smallest angle)
diffraction peak at a Bragg angle of θ = 30o. The wavelength of X-ray used is 2.1 Ǻ. The
volume of the PRIMITIVE unit cell of the metal is
(a) 26.2 (Ǻ)3 (b) 13.1(Ǻ)3 (c) 9.3 (Ǻ)3 (d) 4.6 (Ǻ)3
Ans: (b)
                                                                                
Head office  Branch office 
 
fiziks, H.No. 40 D, G.F, Jia Sarai,  Anand Institute of Mathematics, 
 
Near IIT, Hauz Khas, New Delhi‐16  28‐B/6, Jia Sarai, Near IIT 
 
Phone: 011‐26865455/+91‐9871145498 Hauz Khas, New Delhi‐16 
                                                   
                                             Website: www.physicsbyfiziks.com                                                                                          
                                                            Email: fiziks.physics@gmail.com                                                                     421 
fiziks
Institute for NET/JRF, GATE, IIT‐JAM, JEST, TIFR and GRE in PHYSICAL SCIENCES 
 
a
Solution: According to Bragg’s law 2d sin θ = λ where d =
h2 + k 2 + l 2
For BCC structure the first diffraction peak appear for (110) plane.
a 2a
∵d = ⇒ sin 30 0 = λ ⇒ 2a sin 30 0 = 2.1A 0
2 2
1
⇒ 2 a× = 2.1A 0 ⇒ a = 2 × 2.1A 0 ⇒ a = 2.97 A 0 .
2

The volume primitive unit cell of BCC is volume =


a 3 26.2 0
2
=
2
A ( ) 3
( )
= 13.1 A 0
3

Common Data for Questions 14 and 15:


The tight binding energy dispersion (E-k) relation for electrons in a one-dimensional
array of atoms having lattice constant a and total length L is
E = E0 – β – 2γ cos (ka),
where E0, β and γ are constants and k is the wave vector.
Q14. The density of states of electrons (including spin degeneracy) in the band is given by
L L L L
(a) (b) (c) (d)
πγ a sin (ka ) 2πγ a sin (ka ) 2πγ a cos(ka ) πγ a cos(ka )
Ans: (b)
⎛ L ⎞ 1 ⎛ L ⎞ 1 L
Solution: D (E ) = 2⎜ ⎟⋅ = 2⎜ ⎟⋅ =
⎝ 2π ⎠ dE / dk ⎝ 2π ⎠ 2aγ sin (ka ) 2πγa sin (ka )
Q15. The effective mass of electrons in the band is given by
2 2 2 2
(a) (b) (c) (d)
γ a 2 cos(ka ) 2γ a 2 cos(ka ) γ a 2 sin (ka ) 2γ a 2 sin (ka )
Ans: (b)
2 2 2
Solution: Effective mass m * = = =
⎛ d 2E ⎞ 2a 2 γ cos(ka ) 2γa 2 cos(ka )
⎜⎜ 2 ⎟⎟
⎝ dk ⎠

                                                                                
Head office  Branch office 
 
fiziks, H.No. 40 D, G.F, Jia Sarai,  Anand Institute of Mathematics, 
 
Near IIT, Hauz Khas, New Delhi‐16  28‐B/6, Jia Sarai, Near IIT 
 
Phone: 011‐26865455/+91‐9871145498 Hauz Khas, New Delhi‐16 
                                                   
                                             Website: www.physicsbyfiziks.com                                                                                          
                                                            Email: fiziks.physics@gmail.com                                                                     422 
fiziks
Institute for NET/JRF, GATE, IIT‐JAM, JEST, TIFR and GRE in PHYSICAL SCIENCES 
 
GATE-2012

Q16. For an ideal Fermi gas in three dimensions, the electron velocity VF at the Fermi surface
is related to electron concentration n as,
(a) V F ∝ n 2 / 3 (b) V F ∝ n (c) VF ∝ n1 / 2 (d) V F ∝ n1 / 3
Ans: (d)

Solution: V F =
m
(3π n ) 2 1/ 3

Q17. The total energy, E of an ideal non-relativistic Fermi gas in three dimensions is given by
N 5/3
E∝ where N is the number of particles and V is the volume of the gas. Identify the
V 2/3
CORRECT equation of state (P being the pressure),
1 2 5
(a) PV = E (b) PV = E (c) PV = E (d) PV = E
3 3 3
Ans: (b)
Q18. Which one of the following CANNOT be explained by considering a harmonic
approximation for the lattice vibrations in solids?
(a) Deby’s T3 law (b) Dulong Petit’s law
(c) Optical branches in lattices (d) Thermal expansion
Ans: (d)
Solution: Thermal expansion in solid can only be explained if solid behave as a anharmonic
oscillator.
Q19. A simple cubic crystal with lattice parameter a c undergoes transition into a tetragonal

structure with lattice parameters at = bt = 2ac and ct = 2a c , below a certain


temperature. The ratio of the interplanar spacing of (1 0 1) planes for the cubic and the
tetragonal structure is

1 1 3 3
(a) (b) (c) (d)
6 6 8 8
Ans: (c)
                                                                                
Head office  Branch office 
 
fiziks, H.No. 40 D, G.F, Jia Sarai,  Anand Institute of Mathematics, 
 
Near IIT, Hauz Khas, New Delhi‐16  28‐B/6, Jia Sarai, Near IIT 
 
Phone: 011‐26865455/+91‐9871145498 Hauz Khas, New Delhi‐16 
                                                   
                                             Website: www.physicsbyfiziks.com                                                                                          
                                                            Email: fiziks.physics@gmail.com                                                                     423 
fiziks
Institute for NET/JRF, GATE, IIT‐JAM, JEST, TIFR and GRE in PHYSICAL SCIENCES 
 
a a
Solution: For Cubic Lattice d c = = c
h2 + k 2 + l 2 2

a 2a c
For Tetragonal lattice d t = =
h +k
2
l 2 2
3
2
+ 2
a c
dc 3
Therefore the ratio is =
dt 8
Q20. Inverse susceptibility (1/χ) as a function of temperature, T for a material undergoing
paramagnetic to ferromagnetic transition is given in the figure, where O is the origin. The
values of the Curie constant, C, and the Weiss molecular field constant, λ, in CGS units,
are
1
600 K
χ
(a) C = 5 × 10 −5 , λ = 3 × 10 −2

(b) C = 3 × 10 −2 , λ = 5 × 10 −5 O T

(c) C = 3 × 10 −2 , λ = 2 × 10 4
− 2 × 10 4
(d) C = 2 × 10 , λ = 3 × 10
4 −2
(CGS unit)

Ans: (c)
1 T − TC 1
Solution: = and TC = λC . Here TC = 600 K and = −2 × 10 4
χ C χ
Thus C = 3 × 10 −2 and λ = 2 × 10 4 .

Common Data for Questions 21–22


The dispersion relation for a one dimensional monatomic crystal with lattice spacing a,
which interacts nearest neighbour harmonic potential is given by
Ka
ω = A sin
2
where A is a constant of appropriate unit.
Q21. The group velocity at the boundary of the first Brillouin zone is

                                                                                
Head office  Branch office 
 
fiziks, H.No. 40 D, G.F, Jia Sarai,  Anand Institute of Mathematics, 
 
Near IIT, Hauz Khas, New Delhi‐16  28‐B/6, Jia Sarai, Near IIT 
 
Phone: 011‐26865455/+91‐9871145498 Hauz Khas, New Delhi‐16 
                                                   
                                             Website: www.physicsbyfiziks.com                                                                                          
                                                            Email: fiziks.physics@gmail.com                                                                     424 
fiziks
Institute for NET/JRF, GATE, IIT‐JAM, JEST, TIFR and GRE in PHYSICAL SCIENCES 
 
Aa 2 1 Aa 2
(a) 0 (b) 1 (c) (d)
2 2 2
Ans: (a)
Solution: At the first Brillouin zone the frequency is maximum and the group velocity which is
the derivative of the angular frequency is zero.
Q22. The force constant between the nearest neighbour of the lattice is (M is the mass of the
atom)
MA 2 MA 2
(a) (b) (c) MA2 (d) 2MA2
4 2

Ans: (a)

4K MA 2
Solution: A = ⇒K=
M 4
GATE-2013

Q23. A phosphorous doped silicon semiconductor (doping density: 1017/cm3) is heated from
100°C to 200°C. Which one of the following statements is CORRECT?
(a) Position of Fermi level moves towards conduction band
(b) Position of dopant level moves towards conduction band
(c) Position of Fermi level moves towards middle of energy gap
(d) Position of dopant level moves towards middle of energy gap
Ans: (c)
Solution: Phosphorous doped silicon semiconductors behave as a n-type semiconductor. In
n-type semiconductor Fermi level lies near conduction band and moves toward middle of
the band gap upon heating. At a very high temperature the Fermi level is near the middle
of the band gap and semiconductor behaves as intrinsic semiconductor.
Q24. Considering the BCS theory of superconductors, which one of the following statements is
NOT CORRECT? ( h is the Plank’s constant and e is the electronic charge)
(a) Presence of energy gap at temperature below the critical temperature
(b) Different critical temperature for isotopes
                                                                                
Head office  Branch office 
 
fiziks, H.No. 40 D, G.F, Jia Sarai,  Anand Institute of Mathematics, 
 
Near IIT, Hauz Khas, New Delhi‐16  28‐B/6, Jia Sarai, Near IIT 
 
Phone: 011‐26865455/+91‐9871145498 Hauz Khas, New Delhi‐16 
                                                   
                                             Website: www.physicsbyfiziks.com                                                                                          
                                                            Email: fiziks.physics@gmail.com                                                                     425 
fiziks
Institute for NET/JRF, GATE, IIT‐JAM, JEST, TIFR and GRE in PHYSICAL SCIENCES 
 
⎛h⎞
(c) Quantization of magnetic flux in superconduction ring in the unit of ⎜ ⎟
⎝e⎠
(d) Presence of Meissner effect
Ans: (c)
⎛ h⎞
Solution: Quantization of magnetic flux in superconduction ring in the unit of ⎜ ⎟
⎝ 2e ⎠
Q25. Group I contains elementary excitations in solids. Group II gives the associated field with
these exciations. MATCH the excitations with their associated field and select your
answer as per codes given below.
Group I Group II
(P) phonon (i) photon + lattice vibration
(Q) plasmon (ii) electron +elastic deformation
(R) polaron (iii) collective electron oscillations
(S) polariton (iv) elastic wave
Codes
(a) (P − iv ), (Q − iii ), (R − i ), (S − ii ) (b) (P − iv ), (Q − iii ), (R − ii ), (S − i )
(c) (P − i ), (Q − iii ), (R − ii ), (S − iv ) (d) (P − iii ), (Q − iv ), (R − ii ), (S − i )
Ans: (b)
Solution: Phonon: Quantum of energy of the elastic wave in solid, produced due to the vibration
of atoms in solid.
Plasmon: Quantum of energy of the wave produced due to the oscillation of plasma,
which contains charged particles (positive ions and negative electrons or ions).
Polaron: A charge placed in a polarizable medium will be screened. The induced
polarization will follow the charge carrier when it is moving through the medium. The
carrier together with the induced polarization is considered as one entity, which is called
a polaron.
Polariton: A polariton is a quasiparticle resulting from the mixing of a photon with
phonon.

                                                                                
Head office  Branch office 
 
fiziks, H.No. 40 D, G.F, Jia Sarai,  Anand Institute of Mathematics, 
 
Near IIT, Hauz Khas, New Delhi‐16  28‐B/6, Jia Sarai, Near IIT 
 
Phone: 011‐26865455/+91‐9871145498 Hauz Khas, New Delhi‐16 
                                                   
                                             Website: www.physicsbyfiziks.com                                                                                          
                                                            Email: fiziks.physics@gmail.com                                                                     426 
fiziks
Institute for NET/JRF, GATE, IIT‐JAM, JEST, TIFR and GRE in PHYSICAL SCIENCES 
 
Q26. (
A lattice has the following primitive vector (in Å ) : a = 2 ˆj + kˆ , b = 2 kˆ + iˆ , c = 2 iˆ + ˆj . ) ( ) ( )
The reciprocal lattice corresponding to the above lattice is
⎛π ⎞
(a) BCC lattice with cube edge of ⎜ ⎟Å -1
⎝2⎠
(b) BCC lattice with cube edge of (2π )Å -1

⎛π ⎞
(c) FCC lattice with cube edge of ⎜ ⎟Å -1
⎝2⎠
(d) FCC lattice with cube edge of (2π )Å -1
Ans: (a)
Solution: The reciprocal lattice vectors are

a∗ = 2π
b ×c
a × (b × c ) 2
(- î + ĵ + k̂ )Å
=
π -1

= (î − ĵ + k̂ )Å
c×a π
b ∗ = 2π -1

a × (b × c ) 2

c ∗ = 2π
a ×b
a × (b × c ) 2
(î + ĵ − k̂ )Å
=
π -1

α e2 B
Q27. The total energy of an ionic solid is given by an expression E = − + 9 where α
4πε 0 r r
is Madelung constant, r is the distance between the nearest neighbours in the crystal and
B is a constant. If r0 is the equilibrium separation between the nearest neighbours then
the value of B is
α e 2 r08 α e 2 r08 2α e 2 r010 α e 2 r010
(a) (b) (c) (d)
36πε 0 4πε 0 9πε 0 36πε 0
Ans: (a)
dE α e2 9B α e 2 r08
Solution: At r = r0 , =0= − 10 ⇒ B=
dr r = r0 4πε 0 r0 r0
2
36πε 0

                                                                                
Head office  Branch office 
 
fiziks, H.No. 40 D, G.F, Jia Sarai,  Anand Institute of Mathematics, 
 
Near IIT, Hauz Khas, New Delhi‐16  28‐B/6, Jia Sarai, Near IIT 
 
Phone: 011‐26865455/+91‐9871145498 Hauz Khas, New Delhi‐16 
                                                   
                                             Website: www.physicsbyfiziks.com                                                                                          
                                                            Email: fiziks.physics@gmail.com                                                                     427 
fiziks
Institute for NET/JRF, GATE, IIT‐JAM, JEST, TIFR and GRE in PHYSICAL SCIENCES 
 
GATE-2014
Q28. The Miller indices of a plane passing through the three points having coordinates (0, 0, 1)

(1, 0, 0) ⎛⎜ 1 , 1 , 1 ⎞⎟ are
⎝2 2 4⎠
(a) (212) (b) (111) (c) (121) (d) (211)
Ans: (a)
Q29. The plot of specifies heat versus temperature across the superconducting transition
temperature (Tc ) is most appropriately represented by
(a) (b)

Cp Cp

TC T TC T

(c) (d)

Cp Cp

TC T TC T
Ans: (a)
⎛ Δ ⎞
−⎜ ⎟
Solution: CV ∝ e ⎝ 2kT ⎠

                                                                                
Head office  Branch office 
 
fiziks, H.No. 40 D, G.F, Jia Sarai,  Anand Institute of Mathematics, 
 
Near IIT, Hauz Khas, New Delhi‐16  28‐B/6, Jia Sarai, Near IIT 
 
Phone: 011‐26865455/+91‐9871145498 Hauz Khas, New Delhi‐16 
                                                   
                                             Website: www.physicsbyfiziks.com                                                                                          
                                                            Email: fiziks.physics@gmail.com                                                                     428 
fiziks
Institute for NET/JRF, GATE, IIT‐JAM, JEST, TIFR and GRE in PHYSICAL SCIENCES 
 
Q30. The energy ε k for band electrons as a function of the wave vector k in the first Brillouin

⎛ π π⎞
zone ⎜ − ≤ k ≤ ⎟ of a one dimensional monatomic lattice is shown as ( a is lattice
⎝ a a⎠
εk
constant)

k
− π /a O π /a

The variation of the group velocity vg is most appropriately represented by

(a) vg (b) vg

− π /a k − π /a k
O π /a O π /a

(c) (d)
vg vg

− π /a k − π /a k
O π /a O π /a

Ans: (b)
⎛ ⎛ ka ⎞ ⎞
Solution: E = ⎜ E0 − γβ ⎜ cos ⎟ ⎟
⎝ ⎝ π ⎠⎠
1 dE ka
Vg = = γβ sin
dk π

                                                                                
Head office  Branch office 
 
fiziks, H.No. 40 D, G.F, Jia Sarai,  Anand Institute of Mathematics, 
 
Near IIT, Hauz Khas, New Delhi‐16  28‐B/6, Jia Sarai, Near IIT 
 
Phone: 011‐26865455/+91‐9871145498 Hauz Khas, New Delhi‐16 
                                                   
                                             Website: www.physicsbyfiziks.com                                                                                          
                                                            Email: fiziks.physics@gmail.com                                                                     429 
fiziks
Institute for NET/JRF, GATE, IIT‐JAM, JEST, TIFR and GRE in PHYSICAL SCIENCES 
 
Q31. For Nickel the number density is 8 × 10 atoms / cm 3 and electronic configuration is
23

1s 2 2 s 2 2 p 6 3s 2 3 p 6 3d 8 4s 2 . The value of the saturation magnetization of Nickel in its

ferromagnetic state is _____________ × 10 9 A / m .

(Given the value of Bohr magneton μ B = 9.21 × 10 −21 Am 2 )


Ans: 4.42
Solution: Component of magnetic dipoles in a solid material are in the direction of external field.
M S = (Magnetic dipole moment) μ B N ,

0.6 × 9.21× 10−21 ( Am 2 ) N (For N i :magnetic dipole moment = 0.6

Fe :2.22, For Cu :1.2 )


ρ NA
N= = 8 × 1029 / m3 μ B = 9.21×10−21 A / m
AN

M S = 0.6 × 9.21×10−21 × 8 ×1029 = 4.42 × 109 A / m

GATE-2015
Q31. The energy dependence of the density of states for a two dimensional non-relativistic
electron gas is given by, g (E ) = CE n , where C is constant. The value of n
is____________
Ans.: 0
Solution: We know that
g ( E ) ∝ E1/ 2 for 3 − D , g ( E ) ∝ E 0 for 2 − D , g ( E ) ∝ E −1/ 2 for 1 − D

⇒ n = 0 for 2 − D
Q32. The lattice parameters a, b, c of an orthorhombic crystal are related by a = 2b = 3c . In
units of a the interplanar separation between the (110 ) planes is ____________. (Upto
three decimal places)
Ans.: 0.447

                                                                                
Head office  Branch office 
 
fiziks, H.No. 40 D, G.F, Jia Sarai,  Anand Institute of Mathematics, 
 
Near IIT, Hauz Khas, New Delhi‐16  28‐B/6, Jia Sarai, Near IIT 
 
Phone: 011‐26865455/+91‐9871145498 Hauz Khas, New Delhi‐16 
                                                   
                                             Website: www.physicsbyfiziks.com                                                                                          
                                                            Email: fiziks.physics@gmail.com                                                                     430 
fiziks
Institute for NET/JRF, GATE, IIT‐JAM, JEST, TIFR and GRE in PHYSICAL SCIENCES 
 
1 1 a
Solution: d hkl = ⇒ d110 = = = 0.447 ∵ a = 2b = 3c
2
h k 2
l 2
1 1 5
+ + + +0
a 2 b2 c2 a 2 ⎛ a ⎞2
⎜ ⎟
⎝2⎠
Q33. The dispersion relation for phonons in a one dimensional monatomic Bravais lattice with
lattice spacing a and consisting of ions of masses M is given by

2c
ω (k ) = [1 − cos(ka )] , where ω is the frequency of oscillation, k is the wavevector
M
and C is the spring constant. For the long wavelength modes (λ >> a ) , the ratio of the
phase velocity to the group velocity is_________
Ans.: 1

2C
Solution: ω ( k ) = ⎡1 − cos ( ka ) ⎤⎦
M ⎣
For long wavelength modes (λ >> a )

( ka ) 2C ⎡ ( ka ) ⎤
2 2
C
∵ cos ( ka ) ≅ 1− ⇒ ω (k ) = ⎢1 − 1 + ⎥ =a k
2 M ⎣⎢ 2 ⎦⎥ M

ω C dω C
Phase velocity vP = =a and Group velocity vg = =a
k M dk M
vP
⇒ =1
vg

Q34. In a Hall effect experiment, the hall voltage for an intrinsic semiconductor is negative.
This is because (symbols carry usual meaning)
(a) n ≈ p (b) n > p (c) μ * > μ h (d) mθ* > mn*
Ans.: (c)
Solution: The Hall voltage is V H = RH JB
where J : current density, B : magnetic field and R H : Hall constant

                                                                                
Head office  Branch office 
 
fiziks, H.No. 40 D, G.F, Jia Sarai,  Anand Institute of Mathematics, 
 
Near IIT, Hauz Khas, New Delhi‐16  28‐B/6, Jia Sarai, Near IIT 
 
Phone: 011‐26865455/+91‐9871145498 Hauz Khas, New Delhi‐16 
                                                   
                                             Website: www.physicsbyfiziks.com                                                                                          
                                                            Email: fiziks.physics@gmail.com                                                                     431 
fiziks
Institute for NET/JRF, GATE, IIT‐JAM, JEST, TIFR and GRE in PHYSICAL SCIENCES 
 
1 p μ p − nμ n + ( p − n ) μ n μ p B
2 2 2 2 2

RH =
e ( nμ + p μ )2 + ( p − n ) 2 μ 2 μ 2 B 2
n p n p

1 μ p − μn
For intrinsic semiconductor (n = p = ni ) RH =
eni μ p + μ n

In Intrinsic semiconductor μ n > μ p , therefore Hall voltage is negative.

Q35. Which one of the following represents the electron occupancy for a superconductor in its
normal and superconducting states?

Normal Superconducting Normal Superconducting


state state state state
(a) (b)
f (E ) f (E ) f (E ) f (E )

E E E E

Normal Superconducting Normal Superconducting


(c) state state (d) state state

f (E ) f (E ) f (E ) f (E )

Ans. : (d) E E E E
Solution: In normal slide, some states below Fermi levels are empty and equal number of states
above Fermi levels are filled. If material converts into a superconductor, electrons above
the Fermi Level makes cooper pair and they fall back below level Fermi level as same
energy released during cooper pair formation. Therefore, correct option is (d).
Q36. Given that the Fermi energy of gold is 5.54 eV , the number density of electrons is

__________ ×10−28 m−3 (upto one decimal place)


(Mass of electron = 9.11 × 10 −31 kg ; h = 6.626 × 10 −34 j ⋅ s; 1 eV = 1.6 × 10 −19 j )
Ans.: 5.9
Solution: Relation between electron density (n) and Fermi energy (E F ) is
                                                                                
Head office  Branch office 
 
fiziks, H.No. 40 D, G.F, Jia Sarai,  Anand Institute of Mathematics, 
 
Near IIT, Hauz Khas, New Delhi‐16  28‐B/6, Jia Sarai, Near IIT 
 
Phone: 011‐26865455/+91‐9871145498 Hauz Khas, New Delhi‐16 
                                                   
                                             Website: www.physicsbyfiziks.com                                                                                          
                                                            Email: fiziks.physics@gmail.com                                                                     432 
fiziks
Institute for NET/JRF, GATE, IIT‐JAM, JEST, TIFR and GRE in PHYSICAL SCIENCES 
 
( 2m )
2 3/ 2

EF = (3π n) 2 2/3
⇒n=
1
3π 2 3
⋅ EF3/ 2
2m

( 2 × 9.1×10 kg )
−31 3/ 2
1
× ( 5.54 × 1.6 × 10−9 J )
3/ 2
⇒n= ×
3 × ( 3.14 ) (1.0546 × 10 J − s)
2 −34

1 2.45 × 10−45 × 8.35 ×10−28 −3


⇒n= × m = 0.59 × 10 29 m −3
29.61 1.17 ×10−102
⇒ n = 5.9 × 1028 m −3
GATE-2016
Q37. Consider a metal which obeys the Sommerfeld model exactly. If E F is the Fermi energy
of the metal at T = 0 K and R H is its Hall coefficient, which of the following statements
is correct?
3 2

(a) R H ∝ E 2
F (b) R H ∝ E 3
F

−3

(c) RH ∝ E F2 (d) R H is independent of EF .


Ans.: (c)
1
Solution: RH =
ne
2 3/ 2 3/ 2

( ) ⎛ 2m ⎞ ⎛ EF ⎞
2/3
where EF = 3π 2 n ⇒n=⎜ 2 ⎟ ⋅⎜ 2 ⎟
⇒ RH ∝ EF−3 / 2
2m ⎝ ⎠ ⎝ 3π ⎠
Q38. A one-dimensional linear chain of atoms contains two types of atoms of masses m1 and
m2 (where m2 > m1 ), arranged alternately. The distance between successive atoms is the
same. Assume that the harmonic approximation is valid. At the first Brillouin zone
boundary, which of the following statements is correct?
(a) The atoms of mass m 2 are at rest in the optical mode, while they vibrate in the
acoustical mode.
(b The atoms of mass m1 are at rest in the optical mode, while they vibrate in the
acoustical mode.

                                                                                
Head office  Branch office 
 
fiziks, H.No. 40 D, G.F, Jia Sarai,  Anand Institute of Mathematics, 
 
Near IIT, Hauz Khas, New Delhi‐16  28‐B/6, Jia Sarai, Near IIT 
 
Phone: 011‐26865455/+91‐9871145498 Hauz Khas, New Delhi‐16 
                                                   
                                             Website: www.physicsbyfiziks.com                                                                                          
                                                            Email: fiziks.physics@gmail.com                                                                     433 
fiziks
Institute for NET/JRF, GATE, IIT‐JAM, JEST, TIFR and GRE in PHYSICAL SCIENCES 
 
(c) Both types of atoms vibrate with equal amplitudes in the optical as well as in the
acoustical modes.
(d) Both types of atoms vibrate, but with unequal, non-zero amplitudes in the optical as
well as in the acoustical modes.
Ans.: (a)
Solution: In optical mode, at Brillouin zone boundary atom of heavier mass ( m2 ) is at rest,

whereas in Acoustic mode, atoms of lighter mass ( m1 ) is at rest.

⎛ 1 1 ⎞
2β ⎜ + ⎟
optical mode m
⎝ 1 m2 ⎠

ω2 = 2 β / m1
Acoustic ω1 = 2 β / m2
mode
−π π
2a k→ 2a
Q39. A solid material is found to have a temperature independent magnetic susceptibility,
χ = C . Which of the following statements is correct?
(a) If C is positive, the material is a diamagnet.
(b) If C is positive, the material is a ferromagnet.
(c) If C is negative, the material could be a type I superconductor.
(d) If C is positive, the material could be a type I superconductor.
Ans.: (b)
μ
Solution: Susceptibility is defined as χ = − 1 , where μ → μ0 is permeability of medium and
μ0
vacuum respectively.
(i) For Diamagnet; μ < μ0 , thus χ < 0 i.e. χ is negative

(ii) For Ferromagnet, μ >> μ0 , thus χ >> 0 i.e. χ is positive

(iii) For superconductor, μ = 0 , thus χ = −1


Thus best answer is (b)
                                                                                
Head office  Branch office 
 
fiziks, H.No. 40 D, G.F, Jia Sarai,  Anand Institute of Mathematics, 
 
Near IIT, Hauz Khas, New Delhi‐16  28‐B/6, Jia Sarai, Near IIT 
 
Phone: 011‐26865455/+91‐9871145498 Hauz Khas, New Delhi‐16 
                                                   
                                             Website: www.physicsbyfiziks.com                                                                                          
                                                            Email: fiziks.physics@gmail.com                                                                     434 
fiziks
Institute for NET/JRF, GATE, IIT‐JAM, JEST, TIFR and GRE in PHYSICAL SCIENCES 
 
Q40. Atoms, which can be assumed to be hard spheres of radius R , are arranged in an fcc
lattice with lattice constant a , such that each atom touches its nearest neighbours. Take
the center of one of the atoms as the origin. Another atom of radius r (assumed to be
⎛ a ⎞
hard sphere) is to be accommodated at a position ⎜ 0, ,0 ⎟ without distorting the lattice.
⎝ 2 ⎠
r
The maximum value of is ________. (Give your answer upto two decimal places)
R
Ans.: 0.41
⎛ a ⎞
Solution: The new atom location is ⎜ 0, , 0 ⎟ i.e. it is on the middle of y - axis.
⎝ 2 ⎠
z
a

2a

Position of new r
x
⎛ a ⎞ ⎛ a ⎞
atom ⎜ 0, , 0 ⎟ a new atom at ⎜ 0, ,0 ⎟
figure (i) ⎝ 2 ⎠ ⎝ 2 ⎠
2
figure (ii)
If new atom of radius r fit without distorting the original lattice, then from figure (ii) we get
a
= R+r (i)
2
4
whereas for FCC 2a = 4 R ⇒ a = R = 2 2R (ii)
2
Thus from (i) and (ii)

2 2
2
R = R+r ⇒ ( )
2 −1 R = r ⇒
r
R
= 2 − 1 = 1.414 − 1 = 0.414

                                                                                
Head office  Branch office 
 
fiziks, H.No. 40 D, G.F, Jia Sarai,  Anand Institute of Mathematics, 
 
Near IIT, Hauz Khas, New Delhi‐16  28‐B/6, Jia Sarai, Near IIT 
 
Phone: 011‐26865455/+91‐9871145498 Hauz Khas, New Delhi‐16 
                                                   
                                             Website: www.physicsbyfiziks.com                                                                                          
                                                            Email: fiziks.physics@gmail.com                                                                     435 
fiziks
Institute for NET/JRF, GATE, IIT‐JAM, JEST, TIFR and GRE in PHYSICAL SCIENCES 
 
Q41. The energy vs. wave vector (E − k ) relationship near the bottom of a band for a solid can

be approximated as E = A(ka ) + B(ka ) , where the lattice constant a = 2.1 A . The


2 4 0

values of A and B are 6.3 × 10 −19 J and 3.2 ×10 −20 J , respectively. At the bottom of
the conduction band, the ratio of the effective mass of the electron to the mass of free
electron is _______. (Give your answer upto two decimal places)
(Take = 1.05 × 10 −34 J − s , mass of free electron = 9.1×10 −31 kg )
Ans.: 0.22

Solution: E = A ( ka ) + B ( ka )
2 4

∂E ∂2 E
∴ = 2 Aa 2 k + 4 Ba 4 k 3 and = 2 Aa 2 + 12 Ba 4 k 2
∂k ∂k 2

At the bottom of the band k = 0


2 2
Thus effective mass m* = =
∂ 2 E / ∂k 2 2 Aa 2
(1.05 × 10 )
−34 2
J ⋅s 1.1025 × 10−68
⇒m = *
= = 0.01984 × 10−29
( )
2 −39
2 × 6.3 × 10−19 J × 2.1 × 10−10 m 55.57 × 10

m* 19.84 × 10−32 kg
= 19.84 × 10−32 kg ∴ = −31
= 2.18 × 10−1 = 0.218 ≅ 0.22
m 9.1 × 10 kg
Q42. The Fermi energies of two metals X and Y are 5 eV and 7 eV and their Debye
temperatures are 170 K and 340 K , respectively. The molar specific heats of these
metals at constant volume at low temperatures can be written as
(CV ) X = γ X T + AX T 3 and (CV )Y = γ Y T + AY T 3 where γ and A are constants. Assuming
that the thermal effective mass of the electrons in the two metals are same, which of the
following is correct?
γ X 7 AX γ X 7 AX 1
(a) = , =8 (b) = , =
γ Y 5 AY γ Y 5 AY 8
γ X 5 AX 1 γ X 5 AX
(c) = , = (d) = , =8
γ Y 7 AY 8 γ Y 7 AY

                                                                                
Head office  Branch office 
 
fiziks, H.No. 40 D, G.F, Jia Sarai,  Anand Institute of Mathematics, 
 
Near IIT, Hauz Khas, New Delhi‐16  28‐B/6, Jia Sarai, Near IIT 
 
Phone: 011‐26865455/+91‐9871145498 Hauz Khas, New Delhi‐16 
                                                   
                                             Website: www.physicsbyfiziks.com                                                                                          
                                                            Email: fiziks.physics@gmail.com                                                                     436 
fiziks
Institute for NET/JRF, GATE, IIT‐JAM, JEST, TIFR and GRE in PHYSICAL SCIENCES 
 
Ans.: (a)
Solution: Heat capacity is defined as CV = γ T + AT 3

3 2 1 12π 4 1 1
where γ = Nk B ⋅ and A = Nk B ⋅ 3 ≅ 234 Nk B ⋅ 3
2 EF 5 θD θD
3 2 1
Nk B ⋅
γx 2 EFx EF y 7eV 7
Thus, = = = =
γ y 3 Nk 2 ⋅ 1 EFx 5eV 5
B
2 EFy

1
234 Nk B 3
Ax θ D3 x ⎛ θ Dy ⎞ ⎛ 340 ⎞3
= ( 2) = 8
3
and = =⎜ ⎟ =⎜
Ay 234 Nk 1 ⎜ θD ⎟ ⎝ 170 ⎟⎠
B 3 ⎝ x ⎠
θ Dy

γx 7 A
Thus, = and x = 8
γy 5 Ay

                                                                                
Head office  Branch office 
 
fiziks, H.No. 40 D, G.F, Jia Sarai,  Anand Institute of Mathematics, 
 
Near IIT, Hauz Khas, New Delhi‐16  28‐B/6, Jia Sarai, Near IIT 
 
Phone: 011‐26865455/+91‐9871145498 Hauz Khas, New Delhi‐16 
                                                   
                                             Website: www.physicsbyfiziks.com                                                                                          
                                                            Email: fiziks.physics@gmail.com                                                                     437 
fiziks
Institute for NET/JRF, GATE, IIT‐JAM, JEST, TIFR and GRE in PHYSICAL SCIENCES 
 
NUCLEAR AND PARTICLE PHYSICS SOLUTIONS

GATE-2010
Q1. The basic process underlying the neutron β-decay is
(a) d → u + e − + ν e (b) d → u + e −
(c) s → u + e − + ν e (d) u → d + e − + ν e
Ans: (a)
Q2. In the nuclear shell model the spin parity of 15N is given by
1− 1+ 3− 3+
(a) (b) (c) (d)
2 2 2 2
Ans: (a)
Solution: Z = 7 ; (s1 / 2 ) ( p3 / 2 )4 ( p1 / 2 )1
2
and N = 8

1 ⎛1⎞
⇒ parity = (− 1) = −1,
1
l = 1, J = spin - parity = ⎜ ⎟
2 ⎝2⎠
Q3. Match the reactions on the left with the associated interactions on the right.
(1) π+ → μ+ +ν μ (i) Strong

(2) π0 → γ + γ (ii) Electromagnetic


(3) π0 + n → π- + p (iii) Weak
(a) (1, iii), (2, ii), (3, i) (b) (1, i), (2, ii), (3, iii)
(c) (1, ii), (2, i), (3, iii) (d) (1, iii), (2, i), (3, ii)
Ans: (a)
Q4. The ground state wavefunction of deuteron is in a superposition of s and d states. Which
of the following is NOT true as a consequence?
(a) It has a non-zero quadruple moment
(b) The neutron-proton potential is non-central
(c) The orbital wavefunction is not spherically symmetric
(d) The Hamiltonian does not conserve the total angular momentum
Ans: (d)

                                                                                
Head office  Branch office 
 
fiziks, H.No. 40 D, G.F, Jia Sarai,  Anand Institute of Mathematics, 
 
Near IIT, Hauz Khas, New Delhi‐16  28‐B/6, Jia Sarai, Near IIT 
 
Phone: 011‐26865455/+91‐9871145498 Hauz Khas, New Delhi‐16 
                                                   
                                             Website: www.physicsbyfiziks.com                                                                                          
                                                            Email: fiziks.physics@gmail.com                                                                     438 
fiziks
Institute for NET/JRF, GATE, IIT‐JAM, JEST, TIFR and GRE in PHYSICAL SCIENCES 
 
228
Q5. The first three energy levels of Th90 are shown below

4+ 187 keV
2+ 57.5keV
0+ 0keV

The expected spin-parity and energy of the next level are given by
(a) (6+; 400 keV) (b) (6+; 300 keV) (c) (2+; 400 keV) (d) (4+; 300 keV)
Ans: (a)
E 2 J 2 ( J 2 + 1) E 6(6 + 1)
Solution: = ⇒ 6 = ⇒ E 6 = 393keV
E1 J 1 ( J 1 + 1) E 4 4(4 + 1)

GATE-2011

Q6. The semi-empirical mass formula for the binding energy of nucleus contains a surface
correction term. This term depends on the mass number A of the nucleus as
(a) A-1/3 (b) A1/3 (c) A2/3 (d) A
Ans: (c)
Q7. According to the single particles nuclear shell model, the spin-parity of the ground state
of 178 O is
− − + +
1 3 3 5
(a) (b) (c) (d)
2 2 2 2
Ans: (d)
Solution: Z = 8 and N = 9; (s1 / 2 ) ( p3 / 2 )4 ( p1 / 2 )2 (d 5 / 2 )1
2

+
5 ⎛5⎞
l = 2, J = ⇒ parity = (− 1) = +1, spin - parity = ⎜ ⎟
2

2 ⎝2⎠

                                                                                
Head office  Branch office 
 
fiziks, H.No. 40 D, G.F, Jia Sarai,  Anand Institute of Mathematics, 
 
Near IIT, Hauz Khas, New Delhi‐16  28‐B/6, Jia Sarai, Near IIT 
 
Phone: 011‐26865455/+91‐9871145498 Hauz Khas, New Delhi‐16 
                                                   
                                             Website: www.physicsbyfiziks.com                                                                                          
                                                            Email: fiziks.physics@gmail.com                                                                     439 
fiziks
Institute for NET/JRF, GATE, IIT‐JAM, JEST, TIFR and GRE in PHYSICAL SCIENCES 
 
Q8. In the β-decay of neutron n→ p + e +ν e , the anti-neutrinoν e , escapes detection. Its
-

existence is inferred from the measurement of


(a) energy distribution of electrons (b) angular distribution of electrons
(c) helicity distribution of electrons (d) forward-backward asymmetry of electrons
Ans: (a)
Q9. The isospin and the strangeness of Ω − baryon are
(a) 1, -3 (b) 0, -3 (c) 1, 3 (d) 0, 3
Ans: (b)

GATE-2012

Q10. Deuteron has only one bound state with spin parity 1+, isospin 0 and electric quadrupole
moment 0.286 efm2. These data suggest that the nuclear forces are having
(a) only spin and isospin dependence
(b) no spin dependence and no tensor components
(c) spin dependence but no tensor components
(d) spin dependence along with tensor components
Ans: (d)

Q11. The quark content of Σ + , K − , π − and p is indicated:

Σ + = uus ; K − = su ; π − = u d ; p = uud .

In the process, π − + p → K − + Σ + , considering strong interactions only, which of the


following statements is true?
(a) The process, is allowed because ∆S = 0
(b) The process is allowed because ∆I3 =0
(c) The process is not allowed because ∆S ≠ 0 and ∆I3 ≠ 0
(d) The process is not allowed because the baryon number is violated

                                                                                
Head office  Branch office 
 
fiziks, H.No. 40 D, G.F, Jia Sarai,  Anand Institute of Mathematics, 
 
Near IIT, Hauz Khas, New Delhi‐16  28‐B/6, Jia Sarai, Near IIT 
 
Phone: 011‐26865455/+91‐9871145498 Hauz Khas, New Delhi‐16 
                                                   
                                             Website: www.physicsbyfiziks.com                                                                                          
                                                            Email: fiziks.physics@gmail.com                                                                     440 
fiziks
Institute for NET/JRF, GATE, IIT‐JAM, JEST, TIFR and GRE in PHYSICAL SCIENCES 
 
Ans: (c)
Solution: π − + p → k − + ∑ +

S: 0 0 −1 −1 (not conserved)
1 1
I3 : −1 + − +1 (not conserved)
2 2
For strong interaction S and I3 must conserve. Therefore this process is not allowed under
strong interaction
Q12. Which one of the following sets corresponds to fundamental particles?
(a) proton, electron and neutron
(b) proton, electron and photon
(c) electron, photon and neutrino
(d) quark, electron and meson
Ans: (a)
Q13. In case of a Geiger-Muller (GM) counter, which one of the following statement is
CORRECT?
(a) Multiplication factor of the detector is of the order of 1010
(b) Type of the particles detected can be identified
(c) Energy of the particles detected can be distinguished
(d) Operating voltage of the detector is few tens of Volts
Ans: (c)
Q14. Choose the CORRECT statement from the following
(a) Neutron interacts through electromagnetic interaction
(b) Electron does not interact through weak interaction
(c) Neutrino interacts through weak and electromagnetic interaction
(d) Quark interacts through strong interaction but not through weak interaction
Ans: (d)

                                                                                
Head office  Branch office 
 
fiziks, H.No. 40 D, G.F, Jia Sarai,  Anand Institute of Mathematics, 
 
Near IIT, Hauz Khas, New Delhi‐16  28‐B/6, Jia Sarai, Near IIT 
 
Phone: 011‐26865455/+91‐9871145498 Hauz Khas, New Delhi‐16 
                                                   
                                             Website: www.physicsbyfiziks.com                                                                                          
                                                            Email: fiziks.physics@gmail.com                                                                     441 
fiziks
Institute for NET/JRF, GATE, IIT‐JAM, JEST, TIFR and GRE in PHYSICAL SCIENCES 
 
GATE-2013
Q15. The decay process n → p + + e − + v e violates
(a) Baryon number (b) lepton number (c) isospin (d) strangeness
Ans: (c)
Q16. The isospin (I ) and baryon number (B ) of the up quark is
(a) I = 1, B = 1 (b) I = 1, B = 1 / 3
(c) I = 1 / 2, B = 1 (d) I = 1 / 2, B = 1 / 3
Ans: (d)
Q17. In the β decay process, the transition 2 + → 3 + , is
(a) allowed both by Fermi and Gamow-Teller selection rule
(b) allowed by Fermi and but not by Gamow-Teller selection rule
(c) not allowed by Fermi but allowed by Gamow-Teller selection rule
(d) not allowed both by Fermi and Gamow-Teller selection rule
Ans: (c)
Solution: According to Fermi Selection Rule:
ΔI = 0, Parity = No Change
According to Gammow-Teller Selection Rule:
ΔI = 0,±1, Parity = No Change

In the β decay process, the transition 2 + → 3 + ,


ΔI = ±1, Parity = No Change .
GATE-2014

Q18. Which one of the following is a fermions’?


(a) α -particle (b) 4 Be7 nucleus
(c) Hydrogen atom (d) deuteron
Ans: (b)

                                                                                
Head office  Branch office 
 
fiziks, H.No. 40 D, G.F, Jia Sarai,  Anand Institute of Mathematics, 
 
Near IIT, Hauz Khas, New Delhi‐16  28‐B/6, Jia Sarai, Near IIT 
 
Phone: 011‐26865455/+91‐9871145498 Hauz Khas, New Delhi‐16 
                                                   
                                             Website: www.physicsbyfiziks.com                                                                                          
                                                            Email: fiziks.physics@gmail.com                                                                     442 
fiziks
Institute for NET/JRF, GATE, IIT‐JAM, JEST, TIFR and GRE in PHYSICAL SCIENCES 
 
Q19. Which one of the following three-quark states ( qqq ) denoted by X CANNOT be a

possible baryon? The corresponding electric charge is indicated in the superscript.


(a) X + + (b) X + (c) X − (d) X − −
Ans: (d)
Solution: X = qqq
2 2 2 6
X ++ ( uuu ) + + = = 2 ( two unit positive charge )
3 3 3 3
2 2 1 4 1
X + ( uud ) + − = − = 1( single unit positive charge )
3 3 3 3 3
1 1 1
X − ( ddd ) = − − − = −1( single unit negative charge )
3 3 3
X −− [ Not possible with qqq ] . So the correct option is (d)

Q20. Consider the process μ + + μ → π + + π − . The minimum kinetic energy of the muons
(μ ) in the centre of mass frame required to produce the pion (π ) pairs at rest is
______ MeV .
Ans: 81.7
Solution: Use conservation of energy and momentum in relativistic form.
Q21. A nucleus X undergoes a first forbidden β -decay to nucleus Y . If the angular
7−
momentum (I ) and parity (P ) , denoted by I as P
for X , which of the following is a
2
possible I P value for Y ?
1+ 1− 3+ 3−
(a) (b) (c) (d)
2 2 2 2
Ans: (b)

GATE-2015
Q22. The decay μ + → e + + γ is forbidden, because it violates
(a) momentum and lepton number conservations
(b) baryon and lepton number conservations
(c) angular momentum conservation
(d) lepton number conservation
                                                                                
Head office  Branch office 
 
fiziks, H.No. 40 D, G.F, Jia Sarai,  Anand Institute of Mathematics, 
 
Near IIT, Hauz Khas, New Delhi‐16  28‐B/6, Jia Sarai, Near IIT 
 
Phone: 011‐26865455/+91‐9871145498 Hauz Khas, New Delhi‐16 
                                                   
                                             Website: www.physicsbyfiziks.com                                                                                          
                                                            Email: fiziks.physics@gmail.com                                                                     443 
fiziks
Institute for NET/JRF, GATE, IIT‐JAM, JEST, TIFR and GRE in PHYSICAL SCIENCES 
 
Ans.: (d)
Solution: μ + → e + + γ . In this decay lepton number is not conserved.
Q23. A beam of X - ray of intensity I 0 is incident normally on a metal sheet of thickness

2 mm . The intensity of the transmitted beam is 0.025 I 0 . The linear absorption

( )
coefficient of the metal sheet in m −1 is _______________ (upto one decimal place)
Ans.: 1844.4
μx 1 ⎛ Ι0 ⎞ 1 ⎛ Ι ⎞ 1
Solution: I = I 0 e ⇒μ= ln ⎜ ⎟ = −3
ln ⎜ ⎟= −3
ln ( 40 )
x ⎝ Ι ⎠ 2 × 10 ⎝ 0.025 I 0 ⎠ 2 × 10
2.303
−3 [
⇒μ= log10 40] = 1.151× 103 [ 2 × 0.3010 + 1] = 1844.4 m −1
2 × 10
Q24. The mean kinetic energy of a nucleon in a nucleus of atomic weight A varies as An ,
where n is____________(upto two decimal places)
Ans.: -0.67
R =2 ⎛ d 2 2 d ⎞ =2 =2
∫ − ⎜ 2+ ⎟ 4π r dr
2 R
0 2m ⎝ dr r dr ⎠ − 4π ∫ ( 2 + 2 ) dr − 4π × 4 R
Solution: < T >= = 2m 0
= 2 m
R R
4π R 3 / 3
∫ 4π r 2 dr ∫ π 2
4 r dr
0 0

2
R 1 1 1 − 2
⇒< T >∝ 3
∝ 2
= 2
= 2
= A 3
⇒ n = − = −0.667 = −0.67
R R ⎛ 1
⎞ 3
R A
⎜ 0 ⎟
3 A3
⎝ ⎠
152 152
Q25. The atomic masses of 63 Eu, 62 Sm, 11 H and neutron are 151.921749, 151.919756,

1.007825 and 1.008665 in atomic mass units (amu), respectively. Using the above
−3
information, the Q - value of the reaction 152
63 62 Sm + p is ___________ × 10
Eu + n →152
amu (upto three decimal places)
Ans.: 2.833
Solution: Q = 152.930414 − (152.927581) = 2.833 × 10−3 a.m.u.

                                                                                
Head office  Branch office 
 
fiziks, H.No. 40 D, G.F, Jia Sarai,  Anand Institute of Mathematics, 
 
Near IIT, Hauz Khas, New Delhi‐16  28‐B/6, Jia Sarai, Near IIT 
 
Phone: 011‐26865455/+91‐9871145498 Hauz Khas, New Delhi‐16 
                                                   
                                             Website: www.physicsbyfiziks.com                                                                                          
                                                            Email: fiziks.physics@gmail.com                                                                     444 
fiziks
Institute for NET/JRF, GATE, IIT‐JAM, JEST, TIFR and GRE in PHYSICAL SCIENCES 
 
1 G G
Q26. In the nuclear shell model, the potential is modeled as V (r ) = mω 2 r 2 − λL ⋅ S , λ > 0 .
2
The correct spin-parity and isospin assignments for the ground state of 13 C is
1− − 1 1+ − 1 3+ 1 3− − 1
(a) ; (b) ; (c) ; (d) ;
2 2 2 2 2 2 2 2
Ans.: (a)
Solution: 13C6 , N = 7, Z = 6
2 4 1
⎛ ⎞ ⎛ ⎞ ⎛ ⎞ 1
For N = 7 ; ⎜1S 1 ⎟ ⎜1P3 ⎟ ⎜ P1 ⎟ ⇒ j = and l = 1
⎝ 2⎠ ⎝ 2⎠ ⎝ 2⎠ 2

⎛1⎞
Thus spin- parity is ⎜ ⎟ .
⎝2⎠

                                                                                
Head office  Branch office 
 
fiziks, H.No. 40 D, G.F, Jia Sarai,  Anand Institute of Mathematics, 
 
Near IIT, Hauz Khas, New Delhi‐16  28‐B/6, Jia Sarai, Near IIT 
 
Phone: 011‐26865455/+91‐9871145498 Hauz Khas, New Delhi‐16 
                                                   
                                             Website: www.physicsbyfiziks.com                                                                                          
                                                            Email: fiziks.physics@gmail.com                                                                     445 
fiziks
Institute for NET/JRF, GATE, IIT‐JAM, JEST, TIFR and GRE in PHYSICAL SCIENCES 
 
GATE-2016
Q27. In the SU (3) quark model, the triplet of mesons (π + , π 0 , π − ) has
(a) Isospin = 0 , Strangeness = 0 (b) Isospin = 1 , Strangeness = 0
1 1
(c) Isospin = , Strangeness = +1 (d) Isospin = , Strangeness = −1
2 2
Ans.: (b)
Solution: π + , π 0 , π − are not strange particle thus strangness = 0
Since meson group contain 3 particles, thus I = 1

25 Mn + e → 24 Cr + X . The particle X is
Consider the reaction 54 54
Q28.

(a) γ (b) ν e (c) n (d) π 0


Ans.: (b)
Q29. Which of the following statements is NOT correct?
(a) A deuteron can be disintegrated by irradiating it with gamma rays of energy 4 MeV .
(b) A deuteron has no excited states.
(c) A deuteron has no electric quadrupole moment.
(d) The 1 S 0 state of deuteron cannot be formed.
Ans.: (c)
Q30. According to the nuclear shell model, the respective ground state spin-parity values of
15 17
8 O and 8 O nuclei are

1+ 1− 1 5+ 3− 5 + 3 − 1−
(a) , (b) , (c) , (d) ,
2 2 2 2 2 2 2 2
Ans.: (d)
N = 7 : ( s1/ 2 ) ( p3/ 2 ) ( p1/ 2 )
2 4 1
8 O ; Z = 8 and N = 7 ;
Solution: 15

1 ⎛1⎞
⇒ j= and l = 1 . Thus spin and parity = ⎜ ⎟
2 ⎝2⎠

N = 9 : ( s1/ 2 ) ( p3/ 2 ) ( p1/ 2 ) ( d5/ 2 )


2 4 2 1
17
8 O ; Z = 8 and N = 9 ;
+
5 ⎛5⎞
⇒ j = and l = 2 . Thus spin and parity = ⎜ ⎟
2 ⎝2⎠

                                                                                
Head office  Branch office 
 
fiziks, H.No. 40 D, G.F, Jia Sarai,  Anand Institute of Mathematics, 
 
Near IIT, Hauz Khas, New Delhi‐16  28‐B/6, Jia Sarai, Near IIT 
 
Phone: 011‐26865455/+91‐9871145498 Hauz Khas, New Delhi‐16 
                                                   
                                             Website: www.physicsbyfiziks.com                                                                                          
                                                            Email: fiziks.physics@gmail.com                                                                     446 

S-ar putea să vă placă și